Visualizing the "Greenhouse Effect" – Light and Heat

Guest Post by Ira Glickstein

Solar “light” radiation in = Earth “heat” radiation to Space out! That’s old news to those of us who understand all energy is fungible (may be converted to different forms of energy) and energy/mass is conserved (cannot be created nor destroyed).

My Visualizing series [Physical Analogy, Atmospheric Windows, Emission Spectra, and Molecules/Photons] has garnered almost 2000 comments, mostly positive. I’ve learned a lot from WUWT readers who know more than I do. However, some commenters seem to have been taken in by scientific-sounding objections to the basic science behind the Atmospheric “Greenhouse Effect”. Their objections seemed to add more heat than light to the discussion. This posting is designed to get back to basics and perhaps transform our heated arguments into more enlightened understanding :^)

Solar "light" energy in is equal to Earth "heat" energy out.
[Click on image for larger version]

As I’ve mentioned before, during my long career as a system engineer I’ve worked with many talented mathematical analysts who always provided precise results, mostly correct, but some precisely wrong, usually due to mistaken assumptions. I got into the habit of doing a “back of the envelope” calculation of my own as a “sanity check” on their results. If their results matched within reasonable limits, I accepted them. If not, I investigated further. In those days my analysis was really done using a slide rule and scrap paper, but I now use spreadsheets.

The graphic above is based on an excellent spreadsheet from http://serc.carleton.edu/files/introgeo/models/mathematical/examples/XLPlanck.xls. It uses Planck’s Law to calculate the black body radiation spectrum from the Sun, as observed at the top of the Earth’s Atmosphere. It also may be used to calculate the radiation spectrum from the Earth System (Atmosphere and Surface, see below for explanation) at any assumed temperature. (I will refer to this spreadsheet as “Carleton” in this posting.)

I modified the Carleton spreadsheet to compute the mean Solar radiation per square meter absorbed by the Earth System, which turns out to be 240 Watts/m^2. I then used the spreadsheet to determine the effective mean temperature of the Earth System that would emit an equal amount of energy to Space, and that turned out to be 255 Kelvins (-18ºC which is 1ºF).

Since the mean temperature at the surface of the Earth is 288 Kelvins (+15ºC which is 59ºF), that leaves 33 Kelvins (33ºC which is 58ºF) to be accounted for. Guess how we acount for it?

The yellow curve (above left) shows that Solar radiation is in a tall, narrow “shortwave” range, from about 0.1μm (microns, or millionths of a meter) to about 4μm, which we call ultra-violet, visual, and near-infrared. The vertical axis is Intensity of the radiation, measured in Watts/m^2/μm, and the horizontal axis is Wavelength, measured in μm. If you divide the area under the yellow curve into vertical strips, and add up the total area, you get 240 Watts/m^2.

Since we humans sense the visual portion of this radiation as “light”, that is the name we give it, and that has led to the false assumption that it contains no “heat” (or “thermal”) energy.

The violet curve (above right) shows that, assuming a mean temperature of 255 K, Earth System radiation to Space is in a squat, wide “longwave” range, from about 5μm to beyond 40μm, which we call mid- and far-infrared. If you divide the area under the violet curve into vertical strips, and add up the total area, you get the same 240 Watts/m^2 as is under the yellow curve.

DETAILED EXPLANATION

Left: Actual Solar radiation spectrum observed at top of Atmosphere, compared to black body model. Right: Black body Earth System radiation spectrum out to Space.

The graph on the left shows the actual observed Solar radiation spectrum (in red) as measured at the top of the Atmosphere. It is superimposed on a black body model (in blue) showing very good correlation. Thus, while the Sun is not exactly a black body, it is OK to assume it is for this type of “sanity check” exercise.

If you calculate the area under the curve you get about 1366 Watts/m^2. That means that a square meter of perfect black body material, held perpendicular to the Sun, would absorb 1366 Watts.

However, the Earth is not a perfect black body, neither is it a flat surface perpendicular to the Sun! So, to plot the yellow curve at the top of this posting, I had to adjust that value accordingly. There are two adjustments:

  • The Earth may be approximated as a sphere, with the Sun shining on only half of it at any given time. The adjustment factor for this correction is 0.25.
  • The albedo (reflectiveness) of the Earth system, primarily clouds and light-colored areas on the Surface such as ice, causes some of the Solar radiation to be reflected back out to Space without contributing any energy to the Earth System. The adjustment factor for this correction is 0.7.

After applying these adjustments, the net Solar energy absorbed by the Earth System is 240 Watts/m^2.

The graph on the right shows the black body model for an Earth System at a mean temperature of 255 K, a temperature that results in the same 240 Watts/m^2 being emitted out to Space.

Of course, the Earth System is not a perfect black body, as shown by the graph in the upper panel of the illustration below, which plots actual observations from 20 km looking down. (Adapted from Grant Petty, A First Course in Atmospheric Radiation, Figure 8.2, http://www.sundogpublishing.com/AtmosRad/Excerpts/index.html.)

The actual measured radiation is the dark squiggly curve. Note that it jigs and jags up and down between the topmost dashed curve, which is the black body spectrum for a temperature of 270 K and a lower dashed curve which is the black body spectrum for 230 K. This data was taken over the Arctic, most likely during the daytime. The Petty book also has a graph looking down from over the Tropical Pacific which ranges from 300 K down to 210 K. Observations will vary by tens of degrees from day to night, summer to winter, and Tropical to Polar.

However, it is clear that my result, based on matching 240 Watts/m^2, is within a reasonable range of the true mean temperature of the Earth System as viewed from Space.

NOTE ABOUT THE ABOVE ILLUSTRATION

WUWT readers will notice some apparent inconsistencies in the graphs above. The top and bottom panels, from Petty, peak at 15μm to 20μm, while the purple, blue, and black curves in the middle panel, and the Earth System curves from the Carleton spreadsheet I used (see above) peak in the 9μm to 11μm range. Also, the Petty black body curves peak at a “Radiance” around 100 mW/m^2/sr cm^-1 while the black body curves from Carleton peak at an “Intensity” of around 14 W/m^2/μm. Furthermore, if you look closely at the Petty curves, the labels on the black body curves are mirror image! What is going on?

Well, I know some of the reasons, but not all. (I hope commenters who are more fluent in this than I am will confirm my explanations and provide more information about the differences between “Radiance” and “Intensity”.) I have Googled and Wikied the Internet and am still somewhat confused. Here is what I know:

  • The horizontal axis in Petty’s plots are what he calls “Wavenumber”, increasing from left to right, which is the number of waves that fit into a cm (centimeter, one hundredth of a meter).
  • This is proportional to the frequency of the radiation, and the frequency is the inverse of the wavelength. Thus, his plots are the mirror image of plots based on wavelength increasing from left to right.
  • The spreadsheet I used, and my previous experience with visual, and near-, mid-, and far-IR as used in military systems, always uses wavelength increasing from left to right.
  • So, when I constructed the above illustration, I reversed Petty’s curves, which explains why the labels on the black body curves are mirror image.
  • Fortunately, Petty also included a wavelength legend, which I faithfully reproduced, in non-mirror image, at the top of each plot.

But, that still does not explain why the Petty black body curves peak at a longer wavelength than the Carleton spreadsheet and other graphics on the Internet. I tried to reproduce Petty’s blackbody curves by multiplying the Carleton values by the wavelength (μm) and that did not move the peak to the right enough. So, I multiplied by the wavelength again (μm^2) and, voila, the peaks agreed! (I hope some WUWT reader will explain why the Petty graphs have this perverse effect. advTHANKSance!)

ANSWERING THE OBJECTIONS TO BASIC ATMOSPHERIC “GREENHOUSE EFFECT” SCIENCE

First of all, let me be clear where I am coming from. I’m a Lukewarmer-Skeptic who accepts that H2O, CO2 and other so-called “greenhouse gases” in the Atmosphere do cause the mean temperature of the Earth Surface and Atmosphere to be higher than they would be if everything was the same (Solar radiation, Earth System Albedo, …) but the Atmosphere was pure nitrogen. The main scientific question for me, is how much does the increase in human-caused CO2 and human-caused albedo reduction increase the mean temperature above what it would be with natural cycles and processes? My answer is “not much”, because perhaps 0.1ºC to 0.2ºC of the supposed 0.8ºC increase since 1880 is due to human activities. The rest is due to natural cycles and processes over which we humans have no control. The main public policy question for me, is how much should we (society) do about it? Again, my answer is “not much”, because the effect is small and a limited increase in temperatures and CO2 may turn out to have a net benefit.

So, my motivation for this Visualizing series is not to add to the Alarmist “the sky is falling” panic, but rather to help my fellow Skeptics avoid the natural temptation to fall into an “equal and opposite” falsehood, which some of those on my side, who I call “Disbelievers”, do when they fail to acknowledge the basic facts of the role of H2O and CO2 and other gases in helping to keep temperatures in a livable range.

Objection #1: Visual and near-visual radiation is merely “light” which lacks the “quality” or “oomph” to impart warmth to objects upon which it happens to fall.

Answer #1: A NASA webpage targeted at children is sometimes cited because they say the near-IR beam from a TV remote control is not warm to the touch. Of course, that is not because it is near-visual radiation, but rather because it is very low power. All energy is fungible, and can be changed from one form to another. Thus, the 240 Watts/m^2 of visible and near-visible Solar energy that reaches and is absorbed by the Earth System, has the effect of warming the Earth System exactly as much as an equal number of Watts/m^2 of “thermal” mid- and far-IR radiation.

Objection #2: The Atmosphere, which is cooler than the Earth Surface, cannot warm the Earth Surface.

Answer #2: The Second law of Thermodynamics is often cited as the source of this falsehood. The correct interpretation is that the Second Law refers to net warming, which can only pass from the warmer to the cooler object. The back-radiation from the Atmosphere to the Earth Surface has been measured (see lower panel in the above illustration). All matter above absolute zero emits radiation and, once emitted, that radiation does not know if it is travelling from a warmer to a cooler surface or vice-versa. Once it arrives it will either be reflected or absorbed, according to its wavelength and the characteristics of the material it happens to impact.

Objection #3: The Atmospheric “Greenhouse Effect” is fictional. A glass greenhouse works mainly by preventing or reducing convection and the Atmosphere does not work that way at all.

Answer #3: I always try to put “scare quotes” around the word “greenhouse” unless referring to the glass variety because the term is misleading. Yes, a glass greenhouse works by restricting convection, and the fact that glass passes shortwave radiation and not longwave makes only a minor contribution. Thus, I agree it is unfortunate that the established term for the Atmospheric warming effect is a bit of a misnomer. However, we are stuck with it. But, enough of semantics. Notice that the Earth System mean temperature I had to use to provide 240 Watts/m^2 of radiation to Space to balance the input absorbed from by the Earth System from the Sun was 255 K. However, the actual mean temperature at the Surface is closer to 288 K. How to explain the extra 33 K (33ºC or 58ºF)? The only rational explanation is the back-radiation from the Atmosphere to the Surface.

0 0 votes
Article Rating
958 Comments
Oldest
Newest Most Voted
Inline Feedbacks
View all comments
JT
May 7, 2011 8:34 pm

Ira, the basic bookkeeping of the simplest greenhouse calculations assumes that the re-emission of infra-red radiation by the greenhouse gas molecules is spatially symmetrical. I am well aware that spontaneous emission is equally likely to be in any direction so that approximately 1/2 will head downward to the surface. However, there is another kind of emission – stimulated emission – and it is massively biased in the same direction of travel as the direction of travel of the stimulating photon. Thats what creates laser light, but stimulated emission can occur without lasing. IF there were any significant amount of stimulated emission happening in the atmosphere then the basic 1/2 back down calculation would likely be wrong because most of the IR through the atmosphere comes from the ground up and, if it is stimulating atmospheric emission, would enhance outgoing IR. Has anyone ever looked for evidence of stimulated emission of outgoing IR in the atmosphere?

Cherry Pick
May 7, 2011 8:38 pm

Let’s add two more objections to this partial visualization of the Earth System.
1. Where are the storages? Oceans store and release huge amounts of energy. Elaborate your views of the energy/radiation balance.
2. Energy is not just radiation. Earth System is more complex than that. For example, absorbed radiation energy may result in increased cloudiness that has impact on who much radiation reaches the earth. Ignoring major part of the system does not help in believing your conclusions.

charles nelson
May 7, 2011 8:58 pm

CO2 vs Earth’s Atmosphere Screen Saver!
1. Find out how many pixels there are on your computer screen. Using basic math calculate the proportional size in pixels of an image that represents 380 and 400 ppm.
For example if your screen had exactly 1million pixels the image would be exactly 380 or 400.
2. (Sticking with the above example) Go to Google images and search for images that are…380…400..or 20 pixels. Download and place in the centre of an empty screen.
3. Show the images to people who Believe and ask them if they still think that CO2 is anything other than a trace gas and ask them if the eensy teensy baby image; 20 pixels (the increase in CO2) looks sufficient to destabilize the other 999,980 parts of the atmosphere!

Roger Carr
May 7, 2011 9:15 pm

This posting is designed to get back to basics and perhaps transform our heated arguments into more enlightened understanding :^)
A little humour goes a long way… Nice, Ira!

martin mason
May 7, 2011 9:23 pm

Ira, the Second Law of thermodynamics works at every level and you cannot transfer heat from a colder body to a warmer body. A black body cannot also reabsorb and re-emit lower intensity radiation that it has already emited. I’d say that it is also possible to arrive at the temperature difference between the black body radiating temperature and the average surface temperature without any recourse to radiant heating (adiabatic compression by gravity) and same for the surface temperature on the surface of venus. Greenhouse heating by back radiation is surely wrong because it says that if you put a frozen steak inside a vacuum flask with reflective interior that the steak would cook, that if you stand in front of a mirror you will heat up from the reflected rays or that you can save on your heating bills by filling the loft with CO2?
I agree that we can shoot down the dire predictions of the AGW whilst acknowledging that the basic greenhouse theory is correct but there are many now refuting that there is any greenhouse effect.

Charles Higley
May 7, 2011 9:33 pm

It should be pointed out that the 33K difference referred to is the effect of the heat-trapping gases in the atmosphere. In the case of each gas, the first small amount has the greatest effect, with diminishing effect as more is added, as according to Beer’s Law. For CO2, it’s effect is 90-95% spent and, thus, the hypothetical doubling of atmospheric CO2 would have little effect, possibly 0.10–0.01 K.
It should also be pointed out that, as CO2 partitions 50 to 1 into the oceans, we would have to emit 50 times more CO2 that required to simply double atmospheric CO2. There is not enough carbon available to do this. If we really tried, we might be able to do 20%.
So, where is the majority of atmospheric CO2 coming from? Outgassing from the oceans with warming, as according to Henry’s Law. And, of course, with cooling, the oceans will soak up CO2. There is a lag period before increases reverse to decreases, but with 30–50 years of cooling ahead, we will be able to see it happen.

Jeff Norman
May 7, 2011 9:46 pm

Ira is just doing a gross energy balance, so it seems to me that JT’s and Cherry Pick’s objections all occur within the black box of the Earth-black-body emitter.
I don’t like that Ira treats albedo as a constant. That sounds like a fudge factor you would throw into a GCM.
“The adjustment factor for this correction is 0.7.”
What is that? 70%? I don’t think so but I wait to be illuminated after Ira reflects on this.
How much heat energy gets transferred to the Earth black box by the electromagnetic interactions between the Sun and the Earth?
240 Watts/m^2. What’s the error margin on that for the Sun (solar “constant” anyone?) and for the Earth? Seems you could hide a great deal of heat in there.

Henry
May 7, 2011 9:50 pm

3. Show the images to people who Believe and ask them if they still think that CO2 is anything other than a trace gas and ask them if the eensy teensy baby image; 20 pixels (the increase in CO2) looks sufficient to destabilize the other 999,980 parts of the atmosphere!
Yes! and then replace “CO2” with Cyanide and “atmosphere” with your body. If they don’t believe you, just ingest 2mg of cyanide for each kilogram of your body weight (1:500,000) and show those iteyucktuals what for!

May 7, 2011 10:02 pm

Ira; The surface air temperature is higher then theoretical because of the insulative mass of the air of the atmosphere. Just like the insulation in the wall of a cooler or a kiln.
Energy out equals energy in. The air is heated at the ground level and looses energy to space. So the air at ground level has more energy per unit then the theoretical black body temperature. The thicker and less dense the insulation the slower the energy flow out. The thinner more dense the insulation (atmosphere) the faster energy lose.
The solar radiation incoming is of wave lengths that are mostly transparent to the atmosphere and heat the oceans and land. Thermal energy as heat and that carried in water vapor is transported through the atmosphere towards space. The surface air temperature is just a single point of measurement in this energy flow. Solar radiation that energizes the atmospheric constituents change the density and thickness of the insulation and therefore the rate of energy flow to space. Ultraviolet and shorter wavelengths are more effective at energizing gas molecules to effect changes in the insulative value of the atmosphere. pg

Ted Dooley
May 7, 2011 10:06 pm

The basis for the adjustments to the total absorption due to geometric and reflectivity are based on???

Gnomish
May 7, 2011 10:10 pm

“The only rational explanation is the back-radiation from the Atmosphere to the Surface.”
aren’t you overlooking the obvious? ever fry an egg on the sidewalk from back radiation of the atmosphere?

Geoff Sherrington
May 7, 2011 10:15 pm

One often meets the assertions that (a) the sea in polar and sub-polar regions is commonly several degrees warmer than the air above it; and (b) in these regions, incoming solar energy has heated the air which in turn heats the sea.
Please expand?

May 7, 2011 10:17 pm

I still have issues with down welling or back radiation in the troposphere. It needlessly complicates the simple fact that clouds, air have a temperature that varies. Increased CO2 can change that temperature but doesn’t beam back little phaser blasts of photons. The layering of temperatures in the atmosphere just complicates the radiative cooling like walls of differing layers of insulation thickness. Without the cutesy back radiation into a warmer surface invention, it might be easier to explain the potential radiative impact of CO2 and with a better model of the atmospheric layers, understand why it is not living up to its potential. Other than that, nice post.

Mooloo
May 7, 2011 10:27 pm

Show the images to people who Believe and ask them if they still think that CO2 is anything other than a trace gas
Remove that “trace” gas and all photosynthetic life of earth dies. And us with it.
Small concentrations can have powerful effects.

gbaikie
May 7, 2011 10:28 pm

“I modified the Carleton spreadsheet to compute the mean Solar radiation per square meter absorbed by the Earth System, which turns out to be 240 Watts/m^2. I then used the spreadsheet to determine the effective mean temperature of the Earth System that would emit an equal amount of energy to Space, and that turned out to be 255 Kelvins (-18ºC which is 1ºF).”
When you say absorbed by earth system, this mostly means amount the surface warms up and the atmosphere itself is only absorbing a small percentage of the incoming solar flux?
And assuming this is the case wouldn’t different material absorb different amounts. Or has this been simplified to mostly account for how much the oceans absorb energy, and since ocean absorb the vast majority of energy that reaches the earth surface, anything else can be mostly be ignored?

Günter Heß
May 7, 2011 10:37 pm

Ira,
Thanks for your visualization.
I’d like to comment on your sentence and adress some of the comments here:
“The only rational explanation is the back-radiation from the Atmosphere to the Surface.”
To start, here is my view that I already posted on Climate etc awhile ago..
As I understand it: “backradiation” is the downwelling infrared radiation integrated over all directions that go into the surface.
I am an experimentalist. For me a physical cause is something that I can switch on and off, at least in a thought experiment, and get an intended effect.
If I switch off the sun in a thought experiment, I get cooling of the atmosphere and the surface. If I switch on the sun again, I get warming of the atmosphere and the surface.
If I switch off ”back radiation” in a thought experiment on the night side I get a higher cooling rate.
If I switch on “backradiation” on the night side I get a lower cooling rate.
If I switch on “backradiation” on the day side I get a higher heating rate. Combining my thought experiments I deduct that if I switch on “back radiation” on the day side I get a higher heating rate due to a lower cooling rate because of the “back radiation”
Moreover, I can deduct that with “back radiation” the earth system gets or stays warmer than a reference system without “back radiation”, because of a lower cooling rate.
One might also say “back radiation” leads to a warmer surface compared with a reference system.
However, I do consider sentences as incorrect with respect to physics that state only: “back radiation” is warming the surface.
Of course in any private conversation we state things like: my new coat is warming me nicely. But in a scientific discussion, we need to employ the correct wording, especially in lectures or papers.
For my opinion sentences like: “back radiation” is warming the surface
cause therefore a lot of misunderstanding in the blogosphere. The Gerlich and Tscheuschner discussion is based on it.
Thanks again for your article.
Best regards
Günter

dr.bill
May 7, 2011 10:38 pm

Ira, the radiance formula involves either a “df” or “dλ” at the end. Converting from frequency to wavelength thus involves replacing df by -cdλ/λ², which explains why multiplying the frequency data by λ² puts the peak back in the right place.
This same issue arises in Wien’s Law. The “peak color” is different, depending on whether you use wavelength or frequency as a variable. Intensity, however, is integrated over all colors, and gives the same result for either variable.
/dr.bill

Dishman
May 7, 2011 10:54 pm

In re Objection #2:
Gas molecules follow ballistic trajectories between collisions…
… unless all the work on Gravity since Galileo is wrong, and doesn’t apply at the molecular level, of course.

May 7, 2011 10:57 pm

Would it make any difference if we doubled the average insolation for half a day, then dropped it to zero for the other half (we’ll call that half night)? We’re radiating outward the entire time, day and night. Messes up the simplicity of the model, but since the temperature varies on a 24 hour clock, the outward radiation does, too.

Steeptown
May 7, 2011 11:14 pm

“As I’ve mentioned before, during my long career as a system engineer I’ve worked with many talented mathematical analysts who always provided precise results, mostly correct, but some precisely wrong, usually due to mistaken assumptions. I got into the habit of doing a “back of the envelope” calculation of my own as a “sanity check” on their results. If their results matched within reasonable limits, I accepted them. If not, I investigated further. In those days my analysis was really done using a slide rule and scrap paper, but I now use spreadsheets.”
During my long career as a nuclear engineer, I got into the habit of doing a “back of the envelope” calculation of my own as a “sanity check” on other’s results. If their results matched within reasonable limits, I accepted them. If not, I investigated further. I also encouraged new engineers to do the same.

RJ
May 7, 2011 11:51 pm

What happened to your previous thread. It was getting interesting until no more comments could be posted
http://wattsupwiththat.com/2011/03/29/visualizing-the-greenhouse-effect-molecules-and-photons/
This above thread convinced me even more that backradiation does not further heat the planet. Luke warmers are as scientifically wrong as alarmists if they believe this
The average temperature might slightly increase due to more CO2. But the highest temperature is due entirely to the Sun. In no way can CO2 and backradiation further heat the surface. It can only slow the cooling rate and by so doing might raise the average temperatures
But a colder object can never further heat a warmer one. And saying this time and time again will not make it true. So this objection is correct IMHO.
“Objection #2: The Atmosphere, which is cooler than the Earth Surface, cannot warm the Earth Surface”.

Gilles
May 7, 2011 11:51 pm

I’d like to comment the “However, it is clear that my result, based on matching 240 Watts/m^2, is within a reasonable range of the true mean temperature of the Earth System as viewed from Space.”
Actually for an inhomogeneous body, and even more if it is not a perfect black body, there is nothing like “the true mean temperature”. You can define different averages, but not a single one; the important thing (never clearly stated by climate scientists, although they probably know it very well), is that the average surface temperature (the integral of TdS divded by the surface), is different from the effective temperature Teff which is such that Prad = S.sigma.Teff^4 , where sigma is the Stefan constant. And these temperatures would also be different from a “best fit Black Body temperature”.
The important thing is that you can easily make one of the temperature vary , holding another one constant ! for instance, the average temperature can vary without varying the effective temperature, that is with the same energy budget – and conversely. This can be demonstrated very simply by noticing that the local temperature can be written as +∆T where is the average surface temperature and ∆T a surface (not temporal) “anomaly”. But the only condition is that = 0. Any repartition with a vanishing surface average gives the same average temperature, but NOT the same effective temperature, since on average the won’t vanish.
This lets a lot of room for spontaneous variation even with no change of forcings. This is carefully hidden under the name of “unforced variability”. After all during El Niño/La Niña cycles, the average temperature changes by several tenths of °C (= several decades of observed trend ! ) without any significant change in the energy budget.
The gospel of climate science is that the unforced variability is restricted to 30 years, which would mean that its power spectrum is cut-off for longer variations. But it can be easily seen that this cannot be true. Natural variability is necessary to explain that long term, secular variations, at century and millenium scales. If it were restricted to 30 years, for instance, the paleoclimate data should show that the average temperature should follow very closely the forcing (Milankovitch) curve, which is obviously not the case. And there are obvious physical reason why the Earth climate could oscillate naturally on millenium scale – for instance that’s the characteristic timescale of thermo-haline circulation and it can be expected to give cycles at this frequency. This is much overlooked by climate scientists, in my sense.

Gilles
May 7, 2011 11:52 pm

(sorry for the italic tags, can you fix it Anthony. Sorry also for the possible bad english, that’s not my mother language 🙂 ).

Ronaldo
May 7, 2011 11:54 pm

Ira
Congratulations, you have provided a good, visual explanation of the basic energy inputs and outputs for the earth. The devil is, of course, as I am sure you would agree, in the detail.
When calculating surface temperatures, variations in the assumed constants in the equations need to be considered. To me the most obvious ‘variable’ constant is that assumed for the earth’s albedo. Spatial and temporal variations due, amongst other things, to the ability of H2O to exist as all three of its phases within the surface and atmospheric regions of the earth, coupled with the large amounts of energy emitted or absorbed at the transition temperatures result in variations in cloud and ice cover, affecting both incoming and outgoing radiation and provide major feedbacks.
When these are coupled with the vast energy storage capabilities of the oceans and more subtle long-term variations of energy input imposed on the earth and solar system from ‘external’ sources known and unknown, the prediction of the earth’s surface temperature variation over an extended period is not within our present capabilities.
In my view the attempts by the powers that be to convince us that the science is settled are the only significant source of ‘hot air’ in the debate!

Alcheson
May 7, 2011 11:54 pm

“How to explain the extra 33 K (33ºC or 58ºF)? The only rational explanation is the back-radiation from the Atmosphere to the Surface.”
Since the earth’s core is molten (very hot), there must also be convection of heat from the core to the surface. How much of the extra 33K heat can be attributed to earth’s own heat source? Is it none?… a little?…. or perhaps a significant amount of the 33K? Just wondering.

Gilles
May 7, 2011 11:54 pm

grrr I should’nt use > for average. The correct thing is that
T = Taverage + ∆T
with the condition ∆Taverage =0

wayne
May 7, 2011 11:56 pm

Mike McMillan, you seem to need something like these:
Here are a couple of energy budgets using KT97 and TFK09′s data for the overall averages of reflection (albedo) per the papers, irradiance at the surface, and measured TOA LW upwelling radiation. These are somewhat rough, a first stab at a cosine weighted across 24 hour view but they give you a closer idea of what actually occurs with Trenberth and Kiehls’s numbers. You will notice the totals all match what you see on their graphics.
http://i56.tinypic.com/avc5g.jpg : KT97
http://i53.tinypic.com/ir6lie.jpg : TFK09
They need some small adjustment I can see already:
• Need a better estimate of the actual evaporation rate at nighttime compared to evaporation in the daytime.
• The exact time that the minimum and maximum temperatures are reached, on the average globally, might need to be moved 30 minutes or one hour sooner.
• Window radiation is strictly in those spreads gauged by the surface temperature and I don’t know if this is strictly correct since the cloud layer over 62% of the earth needs to be considered, but, the average should be close.
• …
These are built upon what is actually happening in our atmosphere. You will notice the lack of the mention of back radiation for back radiation is a mirage. A figment of you imagination brought to life by the way the Stefan-Boltzmann equation is applied. In a layered gravity held atmosphere there is no real backward movement of net radiation or any other energy for that matter of fact. When the Stefan-Boltzmann equation is applied correctly to each thin layer you will find this to be true.
Also, radiation in the CO2 and H2O main bands which lie outside the radiative window frequencies cannot pass downward to any great distance to any great degree for the atmosphere is nearly completely “black” to these frequencies. Another way to put it is the mean free path (mean distance a radiation will travel before being absorbed) is simply to short in the lower troposphere to allow this imaginary backward propagation from high in the atmosphere to the surface to ever really occur. Both of these GHGs are simply fast and longer reaching conduction of parcels of energy just like conduction but conduction has to move molecule to molecule, this radiation can jump many meters. Above the tropopause these limits are no longer present enhancing the movement of radiation to space.
So, that is why the familiar 390 & 396 W/m^2 huge red arrows of upward LW radiation and the 324 & 333 W/m^2 of downward IR radiation are missing. They do not belong there. However, they are real and can be measured by radiometers at the surface but this radiation that is being measured there is in the bottom few hundred meters or so except within the window frequencies. It is local, very local as to compared to vastness of the atmosphere. They deserve to be mentioned on those famous budget diagrams but would have more honestly portrayed as a footnote.
Mike, fell free to use these to make you contention a bit stronger.

May 8, 2011 12:04 am

Alcheson;
Since the earth’s core is molten (very hot), there must also be convection of heat from the core to the surface. How much of the extra 33K heat can be attributed to earth’s own heat source? Is it none?… a little?…. or perhaps a significant amount of the 33K?>>>
When I first became interested in climate science that was one of my questions. After much reading and research and my own back of the envelope calculations, the answer came out “insignificant”. Same for radioactive decay of elements in the earth, and I even calculated the energy released by burning of fossil fuels… in the end, all insignificant. Solar rules.

May 8, 2011 12:16 am

wayne says:May 7, 2011 at 11:56 pm . . .
Thanks, Wayne. Seems to me the atmosphere is more of a blanket than greenhouse, just slowing the outward movement of energy. The temperature rises down here to the point where the increased outward movement once again roughly equals the insolation, not that they’re ever exactly equal.

dr no
May 8, 2011 12:19 am

“The only rational explanation is the back-radiation from the Atmosphere to the Surface.”
Has anyone actually calculated how much energy can be absorbed by the GHG molecules in the air? Radiation is not the only heat transfer mechanism.
I have difficulty rationalizing how 2% of the molecules in the air (H2O and CO2) can heat themselves, the other 98% of the air molecules, and the earth surface, by 33K only through back radiation of IR. The GHGs absorb and emit IR but the other air molecules can’t so the heat transfer has to be through collisions. What is the probability of a non-GHG molecule to collide with an excited GHG molecule compared to the probability of colliding with the earth surface and have direct energy transfer from there? (Equally reasonable, we could have “back-collisions” as well, when an air molecule with higher energy than the earth surface, transfer its excess energy back when colliding). If the energy can be inhibited from instantly going out into space by being absorbed by 2% air molecules then it must also be inhibited by being absorbed by e.g. oceans, and released at a later moment. (Some of the energy is also stored in vegetation through the photosynthesis which uses energy from the sun to convert H2O and CO2 into oxygen and cellulose, but that is a longer time perspective.) As has been pointed out in previous postings, we can’t gain more energy than we receive from the sun, but we can delay its disappearance so the 33K energy that is missing has to originate from the sun anyway. Earth is not at instant equilibrium (never at equilibrium at all), and there are many variables that can affect the energy budget at any instant.

gbaikie
May 8, 2011 12:23 am

“However, the actual mean temperature at the Surface is closer to 288 K. How to explain the extra 33 K (33ºC or 58ºF)? The only rational explanation is the back-radiation from the Atmosphere to the Surface.”
Suppose you cover all the ocean of the world with plywood floating “floor”.
You can paint the plywood any color- a nice blue, perhaps.
Would this cause any significant change of global temperature?
And would average global temperature increase or decrease?
This “plywood ocean” could be hotter [though depends on type of paint- not necessarily on simply what color pigment it has] during the day. Since we are not sealing the ocean just covering it, the ocean water would still have some evaporation, but still the nite should be cooler.
Of course such thing would cause a massive extinct event and have all kinds wild effects.
But the point is, would it warm the planet or cool it?
And my answer is it would cool it, significantly.
It could increase global average daytime temperatures- especially if you included the area of the plywood oceans and have somewhat lower global average nite time temperatures. But the northern hemisphere [where most earth land mass is] would get colder- mostly nite time temperature. And other land masses would get colder, and the ocean [beneath the plywood] would get colder.
And looking from space and measuring earth’s radiation would indicate that earth has lost a significant part of it’s “greenhouse affect”
[And if painted the “plywood ocean” black, or put asphalt on it, the earth would still cool.]

May 8, 2011 12:26 am

Ira,
Very detailed explanation. Hard to argue with the fact that when one breaks the data down into that much detail, one gets only one possible conclusion. Either that or all the physics we know that engineers use every day to make planes fly, internal combustion engines that work, air conditioners that cool, furnaces that heat, are wrong. Since the planes they design do fly, the engines work as expected, the air conditioners cool exactly as designed and furnaces heat right to specification, either the physics is right or a few billion calculations based on the physics got it right by pure coincidence. I’ll put my money on the physics!
That said, you can break this down to a much simpler calculation. The sun’s radiance doesn’t get absorbed 100% by earth surface. Some of it is absorbed by the atmosphere. Similarly, the earth’s radiance is not released 100% from the earth’s surface, some of it is released by the atmosphere itself. So the earth surface might be 288K, but as seen from space, any given outward bound photon could have come from the earth’s surface or from the atmosphere. Seen from space, earth is not a “surface” but a sort of fuzzy/murky/semi-transparent sphere. Take any “area” that you can “see” from space, and measure the w/m2 being emitted, and you get a total that is mostly originating from the surface, and some that originated from the atmosphere.
If we assume that your 240 w/m2 is correct, we can use Stefan-Boltzman Law to calculate the “effective” black body temperature of the earth, which if you accept “average” as an argument, would occur not at earth surface, but somewhere between earth surface and the top of the atmosphere.
SB Law being P=5.67*10^-8*T^4
If we use P of 240 watts/m2 we get T of 255.07 degrees K or -17.93 degrees C.
The same logic still applies. Since we can measure what’s going into the system at TOA (top of atmosphere) as well as what is coming out, and the various satellite measurements and experiments like ERBE and others all arrive at a MEASURED value of arounf 240 w/m2, then there is only one possible conclusion. Earth surface is 288K or +15C on average because the atmosphere retains heat that keeps the surface warmer than it otherwise would be. Call it “less cooling” or “back radiation” or “invisible pajamas” but the measurements and the math arrive at the same conclusion. 240 watts/m2 going in =240 watts/m2 going out = -18C. Since the earth surface is +15C, it can only be 33 degrees warmer because the invisible pajamas are making it warmer.

Mike Bromley the Kurd
May 8, 2011 12:33 am

Cherry Pick says:
May 7, 2011 at 8:38 pm
“Energy is not just radiation. Earth System is more complex than that. For example, absorbed radiation energy may result in increased cloudiness that has impact on who much radiation reaches the earth. Ignoring major part of the system does not help in believing your conclusions.”

What you have just used as an example is one that would cause a cooling effect. I’m not sure if that was your intent, while trying to educate Dr. Glickstein.
Good nickname, by the way.

Alcheson
May 8, 2011 1:09 am

Thanks David. Seems the radioactive decay of elements is estimated to be ~30TW (thought to be ~80% of the earth’s internal heating based on Wikipedia) which would result in about 0.1K of the 33K difference. Wonder how well that value is actually known? Off hand, 30TW doesn’t seem like a lot to keep the earth’s core molten. I might have to spend some more time investigating how that 30TW was derived, and see if it makes sense given the thermal conductivity and temperature gradient of the earth to convince myself as well that it is totally insignificant.

May 8, 2011 1:09 am

Energy in = Energy out
Only if integrated over period spanning centuries !

Bryan
May 8, 2011 1:13 am

Ira
You should read the article below.
It will answer all of your misconceptions about the “greenhouse theory” which in fact is pure fiction.
1. It is true that the radiative equilibrium temperature of the Earth is -18C.
2. This figure is confirmed by Satellite measurement from space.
3. It is NOT true that all the radiation emitted from the Earth surface.
4. The emission is mostly from the cloud level.
5. If we pick say around 5km as the average emission height to space we will be near enough.
6. Now use thermodynamics expressed in the form of the lapse rate.
7. An average moist lapse rate is -6.6K/km.
8. The lapse rate has nothing to do with back radiation as even scienceofdoom admits.
9. Work back down from -18C at 5km to the Earth surface and we get 5×6.6 = 33K.
So now you have an Earth surface temperature of 15C without any “greenhouse effect”.
http://www.ilovemycarbondioxide.com/pdf/Understanding_the_Atmosphere_Effect.pdf

Don K
May 8, 2011 1:48 am

Alcheson says:
May 7, 2011 at 11:54 pm
Since the earth’s core is molten (very hot), there must also be convection of heat from the core to the surface. How much of the extra 33K heat can be attributed to earth’s own heat source? Is it none?… a little?…. or perhaps a significant amount of the 33K? Just wondering.
=====
From memory … about a tenth of a watt per meter squared. Big enough that it should probably be explicitly counted in really serious models, but probably not enough to really affect anything. As I recall, the number is in Wikipedia somewhere.

Hector Pascal
May 8, 2011 1:50 am

I’d like to run a little thought experiment here, and ask a question. I’m not pushing an agenda. It’s a very long time since I had the maths to follow the physics above, and I’m too idle to re-learn.
Given a warm radiating surface, and cooler atmosphere of, say, 1 molecule. The surface radiates 10 units of energy out. Nine escape, one unit is trapped by the molecule, and re-radiated back to the surface. Net result is a loss of nine radiation units from the surface. The 2nd law of thermodynamics is satisfied: the cool body is not heating the warm body, merely slowing the cooling rate. Effectively “insulating” the surface (but not in the sense of a “blanket” preventing convection).
Isn’t this the effect of “back-radiation”?

Bob_FJ
May 8, 2011 1:56 am

At a quick flick through I don’t think there was much discussion on the fact that the earth is a rotating sphere.
The solar constant for Earth is commonly given at 1366 W/m^2, and to make a simple average of what is absorbed over the surface and atmosphere, and absorption rates etc, (and the outgoing emissions, spread over the entire sphere), when the wattage is proportional to the fourth power of T, introduces a few complications. In short, might I suggest that the effective radiative T of Earth as is popularly touted is uhm questionable.

charles nelson
May 8, 2011 1:57 am

Gotcha Henry!
He thinks CO2 is like a toxin in the atmosphere!
An amazing toxin which is NOT TOXIC at 380ppm but suddenly BECOMES TOXIC at 400ppm. There in a single post is the fuzzy thinking of the Warmists! Fantastic.
Textbook example of ‘Carbonaphobia’!

charles nelson
May 8, 2011 2:17 am

Oh and by the way Henry, do bear in mind that every time you breath out you are breathing out that toxic gas…be careful not to inhale too quick after you breath out!!
Intellectual…ha…like shooting fish in a barrel.

MartinGAtkins
May 8, 2011 2:22 am

JT says:
I am well aware that spontaneous emission is equally likely to be in any direction so that approximately 1/2 will head downward to the surface. However, there is another kind of emission – stimulated emission – and it is massively biased in the same direction of travel as the direction of travel of the stimulating photon.
This is I believe wrong in that you are assigning some kind of kinetic impact of the photon even though you call it stimulated emission. The photon has no mass, so when it is absorbed it is the electromagnetic properties of the photon that is acting on the molecular structure. In an instant the electromagnetic fields of its components are both attracted and repelled depending on the phase polarity of the photon at the time of absorption.
I find wave theory a better example of the action. At rest the molecule is in balance with its competing fields. The magnetic wave is an oscillation of both positive and negative fields. Dependant on it’s phase, the approaching wave will attract all components of the molecule that are the opposite polarity and repel all those the are the same polarity. As the wave progresses, the polarity is reversed and the components that were repelled are now attracted and those that were attracted are now repelled.
The energy of the wave has now been absorbed and for a brief time the molecule has kinetic energy as, notionally the competing fields struggle to reach equilibrium. In doing so the molecules components that are now vibrating disturb the electromagnetic balance of all the surrounding molecules electromagnetic fields which due to their state of equilibrium with the wider body of gas offer resistance but must acquiescent to the force of the field by propagating the energy though out medium.
The above thought bubble only applies to an ideal gas.

Martin Lewitt
May 8, 2011 2:24 am

The average global surface albedo is about 0.121 to 0.124. The .3 figure includes the cloud contribution. The AR4 model albedos averaged about 0.14 per Roesch (2006), which amounts to about 3W/m^2 globally and annually averaged. This 3W/m^2 of absorbed radiation that the models currently under represent, chiefly from earlier observed spring snow melts while they allegedly “match” the 20th century climate, will be added as the models project the next 100 years, catching up with the snow melt, an addition of heat comparable to the projected CO2 forcing.
Another way to think of the significance of the CO2 concentration is that it is about 2.66 grams per square inch, that is about the mass of a penny. Imagine that penny flattened into a square inch plate, and contiguous over the surface of the earth with all the other pennies. Instead of having the optical properties of copper, this contigous shell is opague to certain characteristic frequencies of CO2. Obviously this is thick enough for several absorption of photons in that range and to allow practically no chance of these photons escaping directly from the surface to space. I don’t see how one can just hand wave this away with no deeper analysis than a dismissive mention of the concentration.

John Marshall
May 8, 2011 2:44 am

You state that the back radiation has been measured. Well it might, or at least some long wave IR that is more probably short wave IR that has lost energy traveling through the atmosphere. It would be impossible to tell the difference from measurements at the surface. It is also true that if a molecule of any so called ‘greenhouse gas’ adsorbs energy it must, by 2nd law rules, share this energy with the surrounding non ‘greenhouse’ gasses, ie it loses heat through conduction and radiation.
I do not see in your calculations any adiabatic effect which is important. Taking this into account raises the temperature at the surface. Also, when any gas is heated it will convect. Not an easy effect to measure in the laboratory compared to the real atmosphere. Convecting air will cool.
Jupiter, with an atmosphere of mainly hydrogen, has warming at the 1 atmosphere level similar to Earth. So there it must be an adiabatic temperature rise not ‘greenhouse effect’. Solar input out there is much lower than Earth’s.
True, glass does not transmit IR energy but a greenhouse, transmits visible light and so doing allows energy losses inside, warming the interior by visible light. These losses mean that the lower wave light converts to IR which cannot be transmitted out through the glass. This, together with lack of mixing with cooler external air raises the internal temperature. At night the temperature falls rapidly. 2nd law dictates that the higher the temperature difference of two bodies the greater the heat flow between the two bodies. Entropy must increase.
Taking into account the adiabatic effect, which is real and measurable, there is no need for the GHG effect.
The greenhouse effect also has one fatal flaw. The warming in the mid troposphere in the tropics shown by all models has yet to be found in real observable data. I do not care what your calculations show (in reality another model), if this warm area shown by models is not there then the theory falls flat on its face!

Bryan
May 8, 2011 3:02 am

Martin Lewitt says:
……Another way to think of the significance of the CO2 concentration is that it is about 2.66 grams per square inch………
Its an odd unit, but it seems much too high;
Ive worked it out in g/cubic inch as 0.000074!

May 8, 2011 3:05 am

Bob_FJ says:
The solar constant for Earth is commonly given at 1366 W/m^2, and to make a simple average of what is absorbed over the surface and atmosphere, and absorption rates etc, (and the outgoing emissions, spread over the entire sphere), when the wattage is proportional to the fourth power of T, introduces a few complications. In short, might I suggest that the effective radiative T of Earth as is popularly touted is uhm questionable.>>>>
One of… OK, my BIGGEST pet peave. There is no such thing as an average temperature or w/m2 that is all that meaningfull. The rough estimate commonly used in climate circles is 1366 x .5 (for day/night) = 683. 683 x .5 (cuz its a sphere) = 360. 360 x .7 (for albedo) = 250, pretty close to Ira’s 240.
But that of course would be around 500 w/m2 at high noon at the equator, and pretty much 0 for six months at a time at the poles. So despite being relatively cold, the poles actually beam way more energy into space than they recieve from the sun, and the equator, despite getting very high insolation year round (in daytime at least) is a net absorber (the excess being moved away by wind and water currents).
So trying to come up with an average insolation value and an average temperature value is meaningful in terms of the total of each. But since insolation varies with w/m2 and w/m2 varies with T^4… trying to track “warming” by following “temperature” is silly. 1 watt of extra insolation results in a much larger temperature anomaly at the south pole than it does at the equator. So when the IPCC says that the “average” temperature of the earth has increased about 1 degree “on average” since 1880, that is highly misleading.
But if they said about 0.3 degrees at the equator and about 1.5 degrees at the poles, it would sound a lot less alarming. And before anyone starts screaming about the ice caps melting, let’s keep in mind that T^4 applies by latitude as well as season. So to get an even more reasonable expression of that “one degree average” they would need to say something like:
Equator +0.3 degrees all year round.
Poles +0.5 degrees in summer
Poles +2 degrees in winter
In other words, that scary “one degree” is mostly confined to the dead of winter in high latitudes. Sorry “the ice caps are melting we’re all going to drown” crowd, but if the poles go from -40 in the dead of winter to -38, the extra melting would be…. between almost zero and zero. And neither the polar bears nor the seals will notice. Or mind.

May 8, 2011 3:09 am

I’m surprised that a “skeptical” site such as WUWT would print such nonsense:-
“Answer #2: The Second law of Thermodynamics is often cited as the source of this falsehood. The correct interpretation is that the Second Law refers to net warming, which can only pass from the warmer to the cooler object. The back-radiation from the Atmosphere to the Earth Surface has been measured (see lower panel in the above illustration). All matter above absolute zero emits radiation and, once emitted, that radiation does not know if it is travelling from a warmer to a cooler surface or vice-versa. Once it arrives it will either be reflected or absorbed, according to its wavelength and the characteristics of the material it happens to impact.”
[b]No![/b] The earths surface is already emitting radiation in the Infra Red. It’s atoms and molecules are ALREADY excited sufficiently to be emitting IR. Therefore that IR scattering back to the surface CANNOT warm the surface any further because the atoms and molecules of the ground are already kinetically excited to this level. The IR will merely be scattered back to the atmosphere again- and eventually espace to space. The radiation from a body can ONLY transfer heat to a cooler body. The second law applies also to radiation.
This sentence is true however:-
” Once it arrives it will either be reflected or absorbed, according to its wavelength and the characteristics of the material it happens to impact.”
Yes, and temperature of that surface is one of those characteristics. The radiation, does not need to “know” anything! If the surface is ALREADY emitting radiation that is “hotter” , i.e. more energetic then the “back radiation ” will in effect just be reflected again.
Heat always passes from hot to cold, never the other way round, THERE ARE NO EXCEPTIONS!
Ira Glickstein does not seem to understand how matter interacts with radiation.

May 8, 2011 3:18 am

“I modified the Carleton spreadsheet to compute the mean Solar radiation per square meter absorbed by the Earth System, which turns out to be 240 Watts/m^2. I then used the spreadsheet to determine the effective mean temperature of the Earth System that would emit an equal amount of energy to Space, and that turned out to be 255 Kelvins (-18ºC which is 1ºF).
Since the mean temperature at the surface of the Earth is 288 Kelvins (+15ºC which is 59ºF), that leaves 33 Kelvins (33ºC which is 58ºF) to be accounted for. Guess how we acount for it?”
We can account for the 33C difference without any need for an imaginary greenhouse effect. I urge everyone at WUWT to read this brilliant paper by physicist Joe Postma ;
http://www.globalwarmingskeptics.info/forums/attachment.php?aid=327
In this paper, Joe accounts for the 33C. The truth is that the radiative temperature of the Earth is measured from space IS -18C. And it is true that the average air temperature near the surface of the earth is 15C. However, if one includes the whole of the atmosphere right up to the edge of space and the surface of the earth in calculating the average radiative temperature of the Earths surface [i]and[/i] atmosphere, then we get to the -18C figure.
There is no discrepancy, there is no need for an imaginary “greenhouse effect”.
Surely the Phlogiston of our times!

May 8, 2011 3:19 am

….and just for the “cold things can’t heat warm things” crowd, here is a link to Denmark’s DMI which publishes average temperature in the arctic going back to 1958. The green line is the average and the red line is “this year”:
http://ocean.dmi.dk/arctic/meant80n.uk.php
Now take a close look at days 1 to about 60. That’s roughly January 1 to the end of February. The average is flat at about 247 K or -26 degrees C. Think about that for a moment. ITS DARK THE WHOLE TIME! NO SUNSHINE! NO INSOLATION!
Despite which it drops down to around -26C by December… and then just stays there. Why doesn’t it keep cooling off? After all, the atmosphere above the arctic is even colder than the arctic itself… Heck, at 25,000 feet its colder than that even at the equator! So… with 0 watts/m2 from the sun, the arctic nonetheless sticks right around -26 degrees. Hmmmmm…..wonder what keeps it from cooling off more….

May 8, 2011 3:43 am

Alcheson;
Off hand, 30TW doesn’t seem like a lot to keep the earth’s core molten.>>>
You’re thinking about it wrong way round. The 30 TW (actually I think it is 44 TW but no matter) is what ESCAPES from the molten core to the surface. OK correction, the outer core is molten, the inner core is actually solid. Fact is that dirt and rocks are pretty good insulators. In high latitudes where winter temps might be as low as -40, houses have to be constructed on a foundation that goes below the “frost line”. That line is a few feet below the surface and in even the harshest of winters, the dirt only reaches the freezing temperature by no more than an inch or so deeper than average. Get about 10 feet down and the temperature varies by almost nothing all year round. So figure that molten outer core is insulated by kilometers of dirt and rock…the heat gets out…but slowly.
There’s a decent summary at wikipedia (not the best source for climate info but good enough in this case) http://en.wikipedia.org/wiki/Earth's_energy_budget
Geothermal about 0.08 w/m2
Tidal about 0.006 w/m2
Fossil Fuel consumption about 0.025 w/m2.
Compared to 250 w/m2 from solar… pretty minor.

Allan M
May 8, 2011 4:05 am

All matter above absolute zero emits radiation and, once emitted, that radiation does not know if it is travelling from a warmer to a cooler surface or vice-versa. Once it arrives it will either be reflected or absorbed, according to its wavelength and the characteristics of the material it happens to impact.
But will be reflected by a body of a higher energy level.
http://www.vermonttiger.com/content/2008/07/nasa-free-energ.html
In fig. 3 a box states:
CO2 & H2O ~15µ absorption; re-emit 7µ, 10µ, 15µ.
But re-emission at a higher frequency (shorter wavelength) than the original absorption is not possible.

Richard111
May 8, 2011 4:28 am

Notice that the Earth System mean temperature I had to use to provide 240 Watts/m^2 of radiation to Space to balance the input absorbed from by the Earth System from the Sun was 255 K. However, the actual mean temperature at the Surface is closer to 288 K. How to explain the extra 33 K (33ºC or 58ºF)? The only rational explanation is the back-radiation from the Atmosphere to the Surface.
255K is the mean earth radiative temperature at around 4,000 metres altitude and above. Escaping IR radiation is from a volume of the atmosphere, not a flat surface.
The mean surface temperature of 288K is simply the compression factor of gravity on the air. Air at the surface has the highest density and returns highest temperature. Think deserts, polar regions, equitorial regions. All very different suface temperatures but always the same adiabatic lapse rate, water vapour content allowing.

Allan M
May 8, 2011 4:43 am

http://www.vermonttiger.com/content/2008/07/nasa-free-energ.html
………
Figure 3 has a box which states:
CO2 & H2O ~ 15µ absorption; re-emit 7µ, 10µ, 15µ.
a body cannot re-emit at a higher frequency (energy level) than it absorbs.

David
May 8, 2011 5:17 am

There is a bathtub which can hold energy. (the earth’s atmosphere/surface/oceans)
There is a energy tap flowing into it. (the sun)
There is a hole in the tub where energy flows out. (radiative emission into space)
Inflow = outflow, energy in = energy out.
The quantity of energy the black-box “bathtub” is able to hold (wants to hold) can go up and down and is indicated by the temperature. When the variables which alter the level stored in the tub are changed (knobs turned) there will be an imbalance between inflow and outflow until the new equalibrium is reached – a equilibrium at which a different level of energy is stored in the tub. We expect a change in the total level of energy stored to affect the temperature at the bottom of the tub.
I expect the IR interference to be one of those variables but I’m sure there are many others. The claim that 30 degrees celsius at the surface is all due to the greenhouse effect is rubbish.
3 scenarious.
A. Black body earth (-30 % albido).
B. Earth with atmosphere and no IR interference.
C. Earth with atmosphere and IR interference.
A is cooler than B is cooler than C.
C – B = greenhouse warming.

Fred Souder
May 8, 2011 5:35 am

Davidmhoffer,
The ground and ocean warm the lower levels of the atmosphere in the polar winters.
Does anyone else wonder what the rate of heat transfer to the surface is from conduction through the crust from the mantle?

RJ
May 8, 2011 5:46 am

We can account for the 33C difference without any need for an imaginary greenhouse effect. I urge everyone at WUWT to read this brilliant paper by physicist Joe Postma ;
http://www.globalwarmingskeptics.info/forums/attachment.php?aid=327
Agree it was a brilliant paper. But this Postma paper was posted on Ira’s previous thread and Ira and others criticised the paper or would not read it all.
http://wattsupwiththat.com/2011/03/29/visualizing-the-greenhouse-effect-molecules-and-photons/
Ira Glickstein, PhD says March 30, 2011 at 2:02 pm
This post by Ira has not really moving forward at all with Ira still posting this for example
“How to explain the extra 33 K (33ºC or 58ºF)? The only rational explanation is the back-radiation from the Atmosphere to the Surface.”
This is just total nonsense. (The only rational explanation). It is one explanation and not a very good one. Unfortunately Ira is just ignoring comments on previous thread so this series is not moving forward. He could summarise the main areas of contention without giving a viewpoint rather than posting again his beliefs that he knows many disagree with

Cementafriend
May 8, 2011 5:55 am

Engineering covers a considerable range of displines such as chemical, civil, electrical, mechanical etc. Heat transfer, thermodynamics, fluid dynamics are basic subjects for some of these disciplines but are not taught in all the disciplines. It is not surprising that some engineers do not understand the fundamental theory and experimental knowledge in these subjects. It is surprising, however, that qualified professional engineers who supposedly comply with a code of ethics which includes the requirement of competence should express opinions about a subject which they do not understand.
Ira, you have been sucked in by climate scientists who have no understanding of heat transfer, fluid dynamics and thermodynamics. You make no mention of heat transfer by convection and phase change (evaporation and condensation). One can only assume that you have no knowledge of this subject. For a quick overview I suggest that you look at Perry’s Chemical Engineering Handbook. Learn about Nusselt and Reynolds numbers. Also, take note of the definition of a black body -it has a surface, and all absorbs energy of all wavelengths. The atmosphere is not a surface and the trace gas CO2 only absorbs radiant energy in very narrow wavelengths ie it is not a black body. The basic Stefan Boltzman equation is not applicable to the atmosphere. Engineers have carried out a huge amount of research on radiation involving gases in heat exchangers.
Then read the following http://climategate.nl/wp-content/uploads/2011/02/CO2_and_climate_v7.pdf

May 8, 2011 5:59 am

Dishman says:

In re Objection #2:
Gas molecules follow ballistic trajectories between collisions…
… unless all the work on Gravity since Galileo is wrong, and doesn’t apply at the molecular level, of course.

Ballistic trajectories are trajectories under the action of gravity. Note, he didn’t say they travel in straight lines.
However: the approximate speed of molecules in the atmosphere is about 1,700 km/h. The path isn’t very curved, in other words. And the mean free path of a molecule in air is about 65 nanometres. You’d be pretty amazing if you could measure the deviation from a straight line of a path segment with that length and curvature.

David
May 8, 2011 6:19 am

I’m reading the post by Joe Postma now. He appears to be saying my energy tub can only be filled to a maximum level and can’t hold more after that.
It already occurred to me that climate scientists didn’t really have any firm understanding of the mechanism of action of the greenhouse “theory”, hence this very subject has been at the front of my mind lately, hence my bathtub theorizing. I’m glad to see this particular issue is slowly but surely coming to a head.
If Joe is right they’ll just start saying that energy is being redistributed in a way to make the surface warmer.

Leonard Weinstein
May 8, 2011 6:22 am

Bryan,
I read the link you gave, and the guy got it partially wrong. He stated that a black body near a light bulb in equilibrium would not heat up if a reflector was then introduced to reflect back onto the black body some of the radiated heat. This is wrong. The black body would absorb the reflected radiation, and thus the equilibrium temperature would be increased. However, this is not related to the back radiation issue. The main remainder of yours and his points are correct, i.e., it is the effective average altitude of outgoing radiation and lapse rate that heats the surface, and backradiation is a consequence rather than cause of the heating. The reason is that buoyancy overcomes absorbed radiation heat transfer to prevent stagnant heating from the radiation.

May 8, 2011 6:36 am

John of Kent says:

I’m surprised that a “skeptical” site such as WUWT would print such nonsense:-
“Answer #2: The Second law of Thermodynamics is often cited as the source of this falsehood. The correct interpretation is that the Second Law refers to net warming, which can only pass from the warmer to the cooler object. The back-radiation from the Atmosphere to the Earth Surface has been measured (see lower panel in the above illustration). All matter above absolute zero emits radiation and, once emitted, that radiation does not know if it is travelling from a warmer to a cooler surface or vice-versa. Once it arrives it will either be reflected or absorbed, according to its wavelength and the characteristics of the material it happens to impact.”
[b]No![/b] The earths surface is already emitting radiation in the Infra Red. It’s atoms and molecules are ALREADY excited sufficiently to be emitting IR. Therefore that IR scattering back to the surface CANNOT warm the surface any further because the atoms and molecules of the ground are already kinetically excited to this level. The IR will merely be scattered back to the atmosphere again- and eventually espace to space. The radiation from a body can ONLY transfer heat to a cooler body. The second law applies also to radiation.

Firstly, I do wish that people who think they know something would just be polite about it; this is not a peer-reviewed journal (thank goodness). Anthony is offering a wonderful service here. The way I look at it is: the entire article, including comments, is the publication. If an article contains fallacies, we will learn from the more knowledgeable commenters.
Having said that, you are not one of those more knowledgeable commenters. All radiators are absorbers. Contrary to your assertion, anything capable of emitting a certain wavelength at a certain temperature can absorb ditto. So yes, a photon radiated back towards the surface can indeed be absorbed by the surface. What this absorption will do, however, is raise the temperature slightly so that the surface now emits at a higher rate. Heat still travels on average from hot to cold. The temperature increase of the hotter surface, whilst happening only if the back radiation is happening, still comes about because of the impinging heat from the sun.
Think of it this way: focus a flame upon a single cubic inch of steel that happens to be part and parcel of a massive steel girder. The flame’s heat will flow into the steel all around and you will find it very hard to increase the temperature of the cubic inch by very much. Now focus your flame on a cubic inch embedded in another surrounding cubic inch, embedded in a very poor conductor. Your heat will heat the first cubic inch, which will heat the second. The poor conductor will prevent escape of much heat and the molecules of steel will bump upon each other. Molecules in your first cubic inch will bump (heat) those in the second, those in the second will bump those in the first. The entire two cubic inches will quickly increase in temperature. The outer cubic inch will never become hotter than the cubic inch being directly heated, and yet the directly heated part will rise in temperature. So there is such a thing as back flow of heat and it is by no means contrary to the 2nd law.
But having said that, the idea of back radiation as formulated by some warmists doesn’t hold much water. The most likely thing to happen to a CO2 molecule that has absorbed a photon and risen to a higher energy state is that it will collide with another molecule and transfer the energy into general heat of the whole gas. When that heats the gas enough, then the number of CO2 molecules undergoing the reverse phenomenon (getting knocked into a higher state by an impinging molecule – remember, these processes are always reversible) will rise enough to provide enough radiation that back radiation becomes significant. But there is a huge heat capacity to fill before that point is reached, since the CO2 is a trace gas heating a vastly huger bulk of N2, O2 etc. In the meantime, the heating of the gas starts convection, limiting the rise in lower air temperature to the adiabatic lapse rate. So once again, the back radiation has its effect auto-limited. Since all planets we see (Earth, Venus, Jupiter) are at close to maximum lapse rate, this is clearly a phenomenom that is quickly reached by almost any trace amounts of greenhouse gas in the atmosphere, and so the mechanism required for the AGW theory to work is already exhausted to within a few percent, and thus the phenomenon is basically harmless.

Leonard Weinstein
May 8, 2011 6:36 am

Ira,
I enjoyed your clear presentation of detail until I saw the kicker “How to explain the extra 33 K (33ºC or 58ºF)? The only rational explanation is the back-radiation from the Atmosphere to the Surface.”
There have been several good presentations of the correct reason. The presence of IR absorbing gases acts as a partial radiation insulator. However, the atmosphere is unbounded, so if there were even a small amount of local heating occurs due to absorbing IR, the warmer air would rises and then cool by adiabatic expansion. In fact, the atmosphere mixes and rises so strongly, that noticeable radiation warming does not occur. The absorbed radiation passes energy into the surrounding air, but the more energetic air molecules (from the velocity distribution) impacting the absorbing molecules cause radiation, and the average is so in balance that the air is considered to be in LTE. However, the fact of the partial radiation insulation (i.e., prevention of direct radiation to space) results in the radiation to space occurring at higher altitude (average about 5 km), and it is the adiabatic lapse rate combined with the equilibrium temperature at 5 km that results in the ground temperature. The back radiation and ground up radiation are a result of the increased temperature, not the cause!

David
May 8, 2011 6:48 am

I didn’t think he was right with the mirror example. If you shine a bright torch and a dim torch at a object it will get hotter than if you shone only the bright torch. If you shine the bright torch and reflect a mirror at the object it’s like adding another dimmer torch.

gbaikie
May 8, 2011 6:53 am

“Heat always passes from hot to cold, never the other way round, THERE ARE NO EXCEPTIONS!”
What about the sun corona? It’s a million degree and sun surface is cooler.
Perhaps you referring to solids. Such as warm air can not heat up a warmer surface via radiant energy.
Say, you had an object in space and it’s temperature was 100 K. This is a cold object, -173 C but compared to background temperature of Space 2.7 K it’s fairly warm. And it is certainly radiating energy into space. No part of earth is as cold as 100 K, if colder can not pass to warmer this means that with infrared telescope on Earth you could never detect such a cold object in space. It also means that your telescope mirrors would need to as cold or colder than any cool object you wanted to observe.

Dave in Delaware
May 8, 2011 7:00 am

* Blackbody – As you have stated, the inbound radiation spectrum from the Sun can be approximated as black body (at it’s temperature) and from the surface (at it’s temperature). The implication being, a blackbody emits in a radiant distribution dependant on it’s temperature. But you incorrectly indicate that Atmosphere is also a black body and it is NOT.
“It also may be used to calculate the radiation spectrum from the Earth System (Atmosphere and Surface, see below for explanation) at any assumed temperature. ”
If CO2 absorbs at 15 micron, it also emits at 15 micron (not a blackbody distribution of all wavelengths based on atmospheric temperature). So where your chart says CO2 ‘Re-Emit 7, 10, 15 micron’ – that is not correct. Similarly for H2O. The purple, blue and black curves in your atmosphere graphic have no meaning.
The ‘looking down’ emission spectrum tells you where the energy was emitted if you look at the temperature curves. The atmospheric window wavelengths are like the surface temperature, cold from the antarctic, hot from the Sahara, with no emission from the intermediate atmosphere. The CO2 wavelengths always look like the tropopause temperatures no matter if they are measured over the desert or the poles, which tells us we are seeing emissions from high in the atmosphere, not re-emitted near the surface. And the H2O wavelengths seem to be in between, more like upper atmosphere and perhaps indicative of cloud tops or ice crystals.
But ‘looking down’ distribution doesn’t look like a one-temperature blackbody distribution. Using a one size fits all blackbody emission from the atmosphere at any one temperature is probably not a good assumption.

Dave in Delaware
May 8, 2011 7:03 am

Radiant energy accounts for all of the outbound energy at the top of atmosphere, but only a fraction of the energy transfer at the surface. Understanding how the energy moves up through the atmosphere via non-radiative pathways is key in estimating potential GHG effects.
If we use values from Kiehl & Trenberth 1977, they show 60% of the surface energy transfer to the atmosphere is due to Thermals (convection) plus Evapo-transpiration (water evaporation and release as latent heat). Leaving no more than 40% as Radiant Energy transfer, with 24% of that 40% as direct loss to Atmospheric Window. Whatever the GHG affects may be, they are happening in the 16% of the total that is left. (And I read somewhere that Trenberth used 40 W / m-2, knowing that the actual ‘window’ value was more like 66 W/m-2. That would increase the ‘atmospheric window’ from 24% to about 39%, leaving only about 1% of the total for the net of all other radiant emissions).
How can that be, since we know the surface emits a blackbody distribution dependent on the surface temperature? One possible explanation is to look at what the energy ‘pathway’ might be. Sort of by definition, the radiant energy that is NOT in the atmospheric window wavelengths, is susceptible to being absorbed by one of the GHG, mostly CO2 and H2O. So the surface emits radiant energy, and the GHG susceptible wavelengths are absorbed fairly close to the surface. Next, we think that the absorbed photons are more likely to be ‘thermalized’ rather than re-emitted as photons (10,000 : 1 more likely thermalized). Thermalized means the GHG molecule collides with a non-GHG molecule in the air, converting the photon to kinetic energy, which can now be seen on a thermometer. Now all of the non-radiative pathways take over. In essence, the radiant emissions from the surface have ‘speeded up’ the energy transfer to get the energy into the other pathways. Hot air rises, clouds and thunderstorms happen when conditions are right, and generally energy moves up through the atmosphere at something like the wet or dry Lapse rate, as appropriate to conditions. When the energy gets high enough, the radiant pathways again become significant, and eventually take over, sending the energy out to space.
Of course the devil is in the details, but that description seems to fit with what others have said earlier. It is not all about just the radiant component.

Nullius in Verba
May 8, 2011 7:06 am

“The only rational explanation is the back-radiation from the Atmosphere to the Surface.”
There’s an objection you haven’t addressed yet – that if extra heat tried to build up near the surface, convection would immediately carry it away again because warm air rises.
In an atmosphere without convection, it would indeed work as you suggest, and the Earth’s surface temperature would be about 60 C on average. When we stop convection artificially – with greenhouses and solar ponds – we do get much higher temperatures. But the mechanism doesn’t apply in a convective fluid, and the 60 C prediction (and other things like an exponential relationship of temperature with altitude) conflicting with observation falsify it.
The back-radiation explanation (or the “shells model” as I sometimes call it) was the original one developed by De Saussure, Fourier, etc. and was overturned in the 1960s by the work of Manabe, Moller, Strickler, and others. The actual explanation is that extra greenhouses gases raise the average altitude of emission to space, raising the altitude of the level that equilibrates to -20 C, and the warmth of the surface relative to this level is due primarily to the adiabatic lapse rate (adjusted for water vapour latent heat).
The surface temperature is the effective radiative temperature (-18 C) plus the average height of emission (5.5 km) times the (moist) adiabatic lapse rate (6 C/km).
T_surf = T_eff + z_emiss * MALR.
The CO2 greenhouse works by increasing z_emiss.
The tropical hotspot, incidentally, is supposed to be caused by also decreasing MALR, through water vapour feedback.
I’m very pleased to see a lot of people now picking up on the convection/lapse rate connection. Things have much improved over a few years ago.

Bryan
May 8, 2011 7:35 am

Leonard Weinstein
……”The black body would absorb the reflected radiation, and thus the equilibrium temperature would be increased.”……
Yes I agree, he could have been clearer on that point.
This aspect causes a great deal of confusion and some intellectual mischief.
See if you would agree with this formulation.
The reflected radiation is the reflective insulation component.
If increased it will slow down the heat loss from the black body.
If the black body has an unchanged energy input its equilibrium temperature would be increased.
However if the energy input to the black body was cut off the temperature of the black body would drop despite the backradiation.

DirkH
May 8, 2011 7:40 am

Ira, explaining the 33K temp difference between surface and top of atmosphere in terms of LWIR backradiation AKA greenhouse gas effect is misleading; as it is already entirely explained by the adiabatic lapse rate.
(see for instance William C. Gilbert here
http://bit.ly/fIhWPS
)
This leaves us with the question – if the lapse rate explains the temp difference, why does the purported greenhouse effect contribute nothing – to which my answer would be: As Absorption and re-emission of LWIR is an extremely fast process the re-radiation process can only delay heat transport via LWIR radiation to space by minutes at best – it’s a ping pong game of photons, but no heat is “trapped”, no matter how often Dessler et. al. use the term “heat-trapping gases” ( see here:
http://www.chron.com/disp/story.mpl/editorial/outlook/6900556.html
)
If this delay rises by a few percent due to increased CO2 levels then it is still only a short term delay.
And that is why we see no warming related to CO2 (if there were such warming, it would correlate to the Keeling curve, which it doesn’t).

DirkH
May 8, 2011 7:43 am

And i see that others have mentioned Postma. A very good explanation of the lapse rate.

Alistair
May 8, 2011 7:56 am

The 33K is the result of the warming by gravitational potential energy of the atmosphere below the -18°C isotherm determined by equality of radiation out to radiation in. It’s about 5.5 km. Because it would be the same for an atmosphere of the same average Cp but without GHGs, it is unconnected to greenhouse warming.
All that does is to provide extra heat in the lower atmosphere and it’s convected to the top where it can radiate away. In the absence of convection, we’d get another 44K warming so the convection/negative feedback is extremely efficient.
If you increase [CO2], [H2O] ust decrease in the upper atmosphere, as observed in 61 years’ radiosonde data. The mechanism is whatever is consistent with the 2nd Law’s requirement of maximum rate of entropy production.
Climate science’s confusion over greenhouse warming comes from the mistake made by Arthur Milne in 1922 when he solved a differential equation for IR absorption in the atmosphere using as boundary condition infinite thickness. The term it generated, extra ‘back-radiation’ is imaginary.
This is not to say there is no back-radiation: that is essential to match exactly the IR radiation from the atmosphere below that point in specific spectral ranges consistent with the local deviation from the lapse rate, e.g from clouds. So, the spectral curve will vary depending on whether there are clouds for example.
The problem is that most scientists haven’t a clue about practical conductive, convective, radiative heat transfer so are easy meat for the charlatans who have made a good career out of pretending there’s an effect of CO2. As an exercise they need to understand the UHI effect is caused by convective heat transfer. You get higher temperature because to get the required thermal equilibrium, you need more radiative flux from the Earth’s surface. Yet the sum of all the heat transfer components, equal to the short wave heating, remains the same once you have exhausted thermal storage effects.
Similarly, at night, you can freeze water by digging a hole in the desert to restrict convection even though air temperatures can be 20°C+.
Before anyone else contributes, they need to know some chemical engineering: http://rpaulsingh.com/teaching/LectureHandouts/Convection%20Heat%20Transfer_handout.pdf

DirkH
May 8, 2011 7:56 am

davidmhoffer says:
May 8, 2011 at 3:19 am
“Why doesn’t it keep cooling off? After all, the atmosphere above the arctic is even colder than the arctic itself… Heck, at 25,000 feet its colder than that even at the equator! So… with 0 watts/m2 from the sun, the arctic nonetheless sticks right around -26 degrees. Hmmmmm…..wonder what keeps it from cooling off more….”
David, if the atmosphere above the arctic is colder than the arctic surface, do you want to imply here that this cold atmosphere is nevertheless capable of WARMING the surface? Are you sure you wanted to say that?

Dave Springer
May 8, 2011 8:08 am

Ira:
As a sanity check you should do this calculation for the moon which receives exactly the same amount of solar radiation as the earth system and there’s no atmosphere or water to complicate the situation. Further, the average surface temperature of the moon was experimentally measured by two separate Apollo experiments where a borehole 3 meters deep was made and a probe with thermocouples at various depths was put into the boreholes. These experiments transmitted temperature data back for several years. The average temperature of the surface is that depth at which there is no daily, seasonal, or annual variation i.e. a constant year-round temperature. At two mid-latitude locations that temperature is -23C or -9F at all depths of 1 meter or more. The experiments were actually designed to measure the thermal conductivity of lunar regolith.
So it would appear on the face of it that your theoretical figure for the earth of 33C warmer than it would be for two approximate black bodies is somehow in disagreement with experimental reality by about 5C.
So before talking about the earth as an approximate black body, which we can’t measure because it ain’t one with a dynamic ocean/atmosphere wrapping it, you have to do a sanity check and see if your black body calculations agree with the measured temperature of the moon which actually is an approximate black body.
I can’t possibly take seriously any discussion which hinges on this commonly number that the earth is 33C warmer than it would be sans ocean and atmosphere when it appears to be 5C smaller than experimental observations. Near as I can tell from the facts that number should be 38C. Normally when two numbers like that differ by something near 10% I shrug it off because it’s still pretty close to agreement. But in this case 5C of disagreement five times greater than the commonly claimed 1C of anthropogenic warming to date.
This goes to the heart of the problem. Anthropogenic effects are so tiny as to easily buried in the noise of the real world and in the error margins of all these toy models of the real world.

Retired Engineer
May 8, 2011 8:22 am

This makes sense. Absent CO2 and water vapor, we would freeze. How much warming does each contribute? If CO2 did it all, and mankind producses about 3% of the CO2, then we are responsible for 3% of the 33C warming or about 1C. Thus AGW is real. Except for water vapor which may cause 90% of the 33C, in which case we cause 0.1C warming. Big deal. Given the daily and seasonal variations in temperature, I find it very hard to believe that 0.1C will have any measureable effect. Or be measureable at all.

ferd berple
May 8, 2011 8:22 am

“How to explain the extra 33 K (33ºC or 58ºF)? The only rational explanation is the back-radiation from the Atmosphere to the Surface.”
Are we suggesting that the earth’s surface is one black body and the atmosphere another? And thus the atmosphere radiates to the surface? However, the atmosphere and the surface are part of the same black body at TOA. Treating them separately below TOA may lead to nonsensical results.
A more reasonable explantion is that the missing 33 K is due to the weight of the atmosphere which compresses the gas nearer the surface, which we see as an increase in temperature.
We know this by comparing the atmospheres of Venus (CO2), Earth(N2O2) and Mars (CO2). All three planets show that temperature varies as the pressure of the atmosphere and the distance from the sun, independent of the composition of the atmosphere.
Venus is totally covered in clouds. If albedo has a 30% blocking effect on earth, then it should have a near 100% blocking effect on venus and the surface should be cold. But it is not.
If CO2 has a warming effect, the mars, which has a greater partial pressure of CO2 at the surface than does earth, should be warm, but it is not.
Climate science continues to build models. Why? Because the argument goes that their is only one earth, so we need models to tell us the effects of CO2 on climate.
This is a rubbish argument. Our two nearest neighbors have CO2 rich atmosphere. These are perfect models for us to judge if the GHG theory is correct, but climate science continues to ignore this more accurate alternative.
We hear about “run-away” greenhouse effect on Venus, without any explanation of how this is possible with 100% cloud cover if albedo and aerosols works as we think they do.
The facts are that the surface pressure of Venus is 90 times. Yes 90 times greater than on earth. This is why the surface temperature is high, even though according to climate theories about clouds and albedo, it should be low.
Why are the temperatures at the bottom of the oceans on earth not higher than the surface? After all this happens in the atmosphere, should it not happen in the oceans.
The reasons is compression. The oceans do not compress, so a cubic meter of surface water has the same number of molecules as a cubic meter of water from the deep oceans. However, the atmosphere does compress. So, a cubic meter of atmosphere at the surface has more molecules than a cubic meter of atmosphere higher up.
It is these molecules that carry the heat. If the energy of the molecules is unchanged, then the more you have per cubic meter, the greater the energy per cubic meter, which we will see as increased temperature.

Theo Goodwin
May 8, 2011 8:31 am

What is so bothersome about this topic is that people who take it up do nothing but rehearse the Great Assumptions, such as the claim that Earth should be treated as a black body. It’s like listening to discussions among Ptolemy and his colleagues. The solution is always the same: more epicycles and maybe an eccentric or two. Yet in setting forth the Great Assumptions, you mention known facts which contradict them. Earth has albedo. Suppose for a moment that Earth’s albedo became 100%, would that motivate you to change the Great Assumptions. Is an object with 100% albedo properly treated as a black body? Can anyone state a plausible account of Earth’s radiation budget that does not treat Earth as a black body? I don’t believe anyone can and I believe that fact reveals a limitation of imagination, not the firmness of accepted science. I guess we will just have to wait for the science to advance. Anyone know somebody named Kepler?

Dave Springer
May 8, 2011 8:36 am

Ira (con’t):
The first pass you need to make in accounting for this discrepancy is to take albedo into account. The moon’s albedo is constant and accurately measured at some 16% so that amount of insolation doesn’t get absorbed the surface. That will account for the theoretical difference between black-body absorption and reality. The moon would be about 5C warmer if the surface albedo was close to 0%.
The problem doesn’t go away however because the earth has a non-zero albedo too and it isn’t known nearly as well as the moon’s with, depending on who you ask, an average albedo in the range of 32% which is primarily the result of some 70% being shrouded by clouds of some sort at any given instant. The problem here is there is no satisfactory agreement between experiments attempting to measure the earth’s average albedo and estimate range from as little as 30% to as much as 40%. All the experimental attempts to measure the earth’s average albedo do agree on one thing – it isn’t constant and varies by as much 1% year over year.
One percent doesn’t seem like much variation but once again it’s a very large number when compared to anthropogenic warming. A 1% change in the amount of insolation actually reaching the surface and not being reflected directly back out into space is some 2.5 watts per square meter. The IPCC third assessment is “95%” confident that anthropogenic forcing lies in the range of 0.6 to 2.4 w/m2. Yet measured variations in earth’s albedo over just several years has it changing by more watts/m2 than the highest estimate of alleged anthropogenic warming. Worse yet, the actual albedo can’t be pinned down and estimates vary by about 7% which is 7 times greater than anthropogenic forcings.
So how can we possibly start talking about anthropogenic forcings and surface temperature changes wrought by same when we don’t even know to +-5C what the average temperature of the earth should be due to our albedo measurements being so imprecise and having no bloody idea how, when, and why the earth’s average albedo varies.
The earth being a dynamic water world makes all attempts to model it exercises in futility. The models are toys and should be regarded as precisely that – toys with zero credibility.

ferd berple
May 8, 2011 8:53 am

6. Now use thermodynamics expressed in the form of the lapse rate.
7. An average moist lapse rate is -6.6K/km.
8. The lapse rate has nothing to do with back radiation as even scienceofdoom admits.
9. Work back down from -18C at 5km to the Earth surface and we get 5×6.6 = 33K.
So now you have an Earth surface temperature of 15C without any “greenhouse effect”.
Exactly, the earth and atmosphere are part of the same black body. Considering them as two different black bodies, based on observation of their combined effect at the TOA without consideration of lapse rate, convection and condensation seems unlikely to provide an accurate answer.

Alan D McIntire
May 8, 2011 8:56 am

Bryan stated that using clouds, you get a 33 C temperature difference between clouds and surface “without any greenhouse effect”. I’d say inseead, that a significant fraction of the greeenhouse effect is due to clouds.
Ira Glickstein stated that the warming due to CO2 would only be about 0.2C. Here’s a way to get a rough estimate of the figure:
We get an average of 342 watts/M^2 from the sun.
As others have pointed out, excluding clouds, the average reflectivity of earth’s surface is about 0.124
Earth is not a blackbody, most of the earth is covered by ocean, which has an emissivity between 0.92 and 0.96- I’ll use a 0.94 average. This non- blackbody emissivity means earth actually radiates away less than a blackbody at the same temperature.
A non-greenhouse earth surface would receive an average of
342 *(1-.124reflectivity)/0.94 emissivity would give an average surface flux of
342*0.876/.94 = 318.7 watts. All of the positive and negative feedbacks to the
greenhouse effect give an average temperature of 288K, or effectively 390.7 watts/m^2, for an effective magnification of 1.226
From Trenbeth’s figures,
http://content.imamu.edu.sa/Scholars/it/net/trenbert.pdf
161 watts /m^2 hit the earth’s surface, with back radiation we get a total of 492 watts, not all in sensible heat, for a magnification of
493/161 = 3.06,
Putting the two formulas together, a theoretical additional factor of 2.06 multiplier due to the greenhouse effect becomes an actual multiplier of 0.226, thanks to negative factors like increases in albedo due to clouds, decreases in lapse rate, etc.
A doubling of CO2 would supposedly increase the surface flux by something less than 3.7 watts/m^2, for a total of 496.7 watts, an increase to 335.7 from 332 over the original 161 watt/m^2 hitting the surface.
Multiply that 335.7/332 * 0.226 and we get 1.2285* 318.7 for an effective
wattage increase from 390.7 to 391.52 watts, or a temperature increase to
(391.52/390.7)^0.25 = 288.15K from 288 K – Close to Ira Glickstein’s estimate of 0.2K- a drop in the bucket.

jae
May 8, 2011 8:57 am

Ira:
“The only rational explanation is the back-radiation from the Atmosphere to the Surface.”
This is the crux of the issue. It is the point of contention. You and others keep repeating the same GHE theory over and over again, with slightly different words. By now, I doubt that there are many people with any scientific background who do not understand what you are saying. The problem is that it has not been demonstrated empirically AND there are other ways to explain why the surface of the Earth is higher than -18C!
The -18 C represents the average radiation temperature which comes from an altitude of about 5 km, NOT the surface. Due to the lapse rate, it is much warmer on the surface (viola, about 33C higher!).
Now, that 33C “increase” may well be due to the GHE that you describe. However, it could also be explained simply by heat storage and the ideal gas law. So I’m still not sure.
BTW, I hope you are not still trying to say that the surface is HEATED by the backradiation from the atmosphere. The atmosphere may manage to keep the surface from cooling so fast (due to the backradiation), but it is certainly NOT heating it. A colder body cannot heat a warmer one. Period.

Dave Springer
May 8, 2011 8:59 am

Ira Glickstein, PhD says:
May 8, 2011 at 8:35 am
You say the earth’s average is pretty close to 70%.
First of all that’s the reciprocal of albedo. Albedo is a number ranging from 0-1 with 0 being dead black and 1 being a perfect mirror. Multiply by 100 for a percentage.
Now I know wikipedia is frowned on as a reference but when it comes to global warming their distinct bias is always on the warmist side so if you find something in wikipedia that is contrary to warmist dogma you can bet your bottom dollar it’s a painful admission.
I quote:
http://en.wikipedia.org/wiki/Black_body

Estimates of the Earth’s average albedo vary in the range 0.3–0.4, resulting in different estimated effective temperatures. Estimates are often based on the solar constant (total insolation power density) rather than the temperature, size, and distance of the sun. For example, using 0.4 for albedo, and an insolation of 1400 W m−2, one obtains an effective temperature of about 245 K.[25] Similarly using albedo 0.3 and solar constant of 1372 W m−2, one obtains an effective temperature of 255 K.[26][27]

This is exactly the figure I gave you of 30% to 40% for estimated average albedo. However I didn’t get it from wiki. I got it from much more in depth reading of the experimental attempts to measure it. Wikipedia in this instance is accurate and for the warmista it is a fatal blow with regard to the credibility of their propaganda. We don’t know the average insolation that reaches the surface of earth any closer than +-25w/m2. To start talking about anthropogenic forcings in the range of -0.6 to 2.5w/m2 is ludicrous when the error margin in experimental attempts to measure average insolation is 25 watts.
Before any toy model of the earth can begin to be given any credibility it must first have a accurate number for the earth’s average albedo down to a fraction of percent and then it must further model the observed variation in albedo down to a fraction of a percent. If it can’t do that it can’t even begin to seperate anthropogenic from natural forcing.

DirkH
May 8, 2011 9:11 am

Ira Glickstein, PhD says:
May 8, 2011 at 8:35 am
“The point of my simplified calculation was to show that, absent back radiation from the Atmosphere, there is no good explanation for the approximately 33ºC (58ºF) temperature difference between the “light” input from the Sun and the “heat” output from an Earth that would have to be cooler than it is for the heat balance to work.”
Again: The Earth radiates from the top of the atmosphere and that’s where you find exactly the temperature needed to radiate enough. See also
Harry Dale Huffman : No Greenhouse effect on Venus
http://theendofthemystery.blogspot.com/2010/11/venus-no-greenhouse-effect.html
where Mr. Huffman does the same computations for Venus.

jae
May 8, 2011 9:27 am

Probably should have read more comments before posting. fred burple already said about the same thing that I did. I think he is correct.

May 8, 2011 9:27 am

jae –
Someone who would benefit an understanding of ‘traveling waves’; incident and reflected waves (energy), S-Parameters (Scattering Matrix Parameters).
Falling back on the simple ‘laws of thermodynamics’ does you a disservice …

May 8, 2011 9:37 am

Ira Glickstein, PhD says:
“The only rational explanation is the back-radiation from the Atmosphere to the Surface.”
“absent back radiation from the Atmosphere, there is no good explanation for the approximately 33ºC (58ºF) temperature difference”

As many comments above have pointed out, the adiabatic lapse rate alone can completely explain this. Thus, Ira you need to explain why in effect you consider this to be an irrational or poor explanation.

May 8, 2011 9:44 am

I disagree with answer three. As a fellow systems engineer I appreciate and support your back of the envelope checks. But remember, we also have to explore all feasible answers, and the difference between black body and the actual measured radiation back to space does NOT require a ‘greenhouse’ effect.
It simply requires a long time constant between heating and radiation, which is where heat sinks and non-solar heat sources come into play.
The 33° K difference could simply be a result of a time delay between solar heating and thermal cooling. The heat capacity of the Earth could require the system to bank 33°K of heat before the there is sufficient flow of energy to the upper atmosphere to radiate.
Moreover, the inner core and the active underwater volcanoes could easily be dumping a extra wedge of gravity-friction generated heat (akin to what drives the Io system). What our massive oceans provide is a heat store, not just a heat sink. Gravity-friction (which produce wave action, which transfers kinetic energy into heat) is a source of energy transfer at the surface, as is any flowing water and its passage to the sea.
How much kinetic energy is in a river flowing to the sea? How was that energy stored? It was stored as water vapor evaporating from ground and bodies of water from solar heat, which rise to be COOLED, where the kinetic energy from gravity is then brought back to Earth with the rain. Friction supplies the transfer.
You can identify numerous transitions where heat is captured, converted, released, transferred and held for days, weeks, months and years.
What is wrong with ALL these models is they lack a time constant from which a joule of solar energy can travel through our system – over any one of hundreds of energy transformation paths – before it is made available for radiation. And we never remember to include the yet still unquantified heat source that is at the core of our planet. There is NO dissipation path for that heat except through radiation. It is a massive heat source which could EASILY account for the 33°K difference.
We know so little, yet act as if we have all the answers. The minute that happens we stop being scientists and become zealots.

May 8, 2011 9:49 am

DirkH says:
May 8, 2011 at 7:56 am
davidmhoffer says:
May 8, 2011 at 3:19 am
“Why doesn’t it keep cooling off? After all, the atmosphere above the arctic is even colder than the arctic itself… Heck, at 25,000 feet its colder than that even at the equator! So… with 0 watts/m2 from the sun, the arctic nonetheless sticks right around -26 degrees. Hmmmmm…..wonder what keeps it from cooling off more….”
DirkH;
David, if the atmosphere above the arctic is colder than the arctic surface, do you want to imply here that this cold atmosphere is nevertheless capable of WARMING the surface? Are you sure you wanted to say that?>>>>
Dirk, I didn’t say the atmosphere made the surface warmer. I said, what keeps it from cooling off more? Why does the temp drop from 275K in July, to 240K in December, a drop of 35 degrees over 5 months, and then over the next 2-3 months, despite having 0 insolation, it drops by perhaps two degrees more. Since it isn’t getting any energy from the sun, where is the energy to keep it from cooling any further come from?
There are several possibilities. The arctic is mostly ocean covered by ice, so warm water currents could be bringing in heat. Wind from warmer latitudes could be bringing in warm air. Reflection from clouds that are high enought in altitude to catch some sunlight cold contribute. All of these have been measured in various ways, all are part of the picture, but combined they are insufficient to keep the surface at -26.
So why doesn’t it cool off more? Answer is (in part) that some of the outward bound radiance is in fact returned to the surface. In fact, the atmospheric window starts to close pretty fast right around that temperature, and by -40 it is shut tight. That’s why the very coldest nights at 80 N latitude are about -40 and the very coldest nights at 50 N latitude, where the “average” temp is MUCH higher, are around -40.
Once the temp hits -25 not only is the “colder” atmosphere above the artic backradiating, reradiating, simulating pajamas, what ever term you want, it get’s more efficient at it. So it can’t EVER cause an INCREASING temperature trend, but it can certainly recycle, reradiate, pajamas, the surface radiance to the point where continued decreases in temp slow to a crawl.

Dave Springer
May 8, 2011 9:54 am

ferd berple says:
May 8, 2011 at 8:22 am
I don’t want to rain on your parade but the temperature at the top of the atmosphere is in the thousands of degrees. The TOA boundary for energy budget calculations is the top of the thermosphere. So the temperature is indeed much higher at TOA than it is at BOA.
Your argument based on lapse rate is only valid to the top of the middle atmosphere and falls apart in the upper atmosphere where temperature increases with altitude.
It’s very thin atmosphere but it’s still atmosphere and its temperature can be measured. So the temperature at the bottom of the atmosphere is indeed much colder than at the top.

Gilles
May 8, 2011 10:02 am

In my view, the explanation of the 33°C difference is somewhat tricky.
If the atmosphere were totally transparent to radiation, the effective temperature of 280 K would be that of the ground (although, as I mentioned above, this effective temperature does not coincide with the average surface temperature). So the atmosphere must be opaque to thermal IR radiation to produce an effective temperature at the TOA different from the ground one. But absorption of IR radiation IS greenhouse effect. So “some ” greenhouse effect must be present to make the atmosphere partly opaque.
BUT the open question is to compute the average difference between ground and TOA and this is much less obvious. In the absorption lines, the outgoing LWR intensity is determined by the local temperature at the last diffusion surface – so the upper atmosphere temperature. But the heat transfer in the troposphere is a complicated mixture of convection and radiation, and is indeed dominated by convection. So the difference temperature is mainly controlled by the adiabatic law between ground and and tropopause – and this can NOT be computed only with radiative physics. So in some sense, those claiming that the difference between the ground and the TOA is mainly due to gas thermodynamics are also right – this is a very complicated mixture of convective transport and radiative opacity.
This can only be solve through detailed modeling – and it is quite possible that this modeling is inaccurate.
A further complication is that as I mentioned , average ground temperature is NOT the same as effective temperature, so there is no clear relationship between the two: The Earth is NOT an isothermal copper sphere ! actually the average temperature depends strongly on meridional circulation that transports a lot of heat towards the high latitudes – especially with oceans that are, to say the least, not very well described and understood. Any variability in oceanic circulation could have strong effects on local, and hence average temperature, even with a fixed energy budget. This is again totally overlooked in “simple” isothermal , radiative arguments. I would say IMHO that this is actually the less known feature of current GCM – and the main reason for doubting about their accuracy and predictive power. And I am not really surprised that they struggle so much with the “lack of warming” .. 🙂

Dave Springer
May 8, 2011 10:04 am

ferd (con’t)
The reason the middle and lower atmosphere get colder with height isn’t because of compression. It’s because in the middle and lower atmosphere the sun doesn’t do any appreciable heating of it.
One must always begin with the big picture and work down to the smaller details not try to add up the details into a big picture.
In the big picture the sun heats the ocean, the ocean heats the atmosphere, and the frigid cold of the cosmic background (3K) cools the atmosphere.
The lower atmosphere is warmer than the middle atmosphere because the lower atmosphere is closer to the source of the ocean that heats it.

Massimo PORZIO
May 8, 2011 10:15 am

Ira said:
“Notice that the Earth System mean temperature I had to use to provide 240 Watts/m^2 of radiation to Space to balance the input absorbed from by the Earth System from the Sun was 255 K. However, the actual mean temperature at the Surface is closer to 288 K. How to explain the extra 33 K (33ºC or 58ºF)? The only rational explanation is the back-radiation from the Atmosphere to the Surface.”
No, by my point o f view there is another explanation. The incoming radiation from the Sun is truly measured from the radiometers because of the distance from the source we get all the parallel energy rays in the SW, while the emission of the Earth in the LW range is scattered in a spheric space around the planet, so the satellites’ radiometers can’t really measure all that energy because those rays are not parallel each other and many of them will never be perpendicular to the radiometer input slit.

Dave Springer
May 8, 2011 10:23 am

I suppose this is my segue into talking about the temperature of the ocean since it is responsible for the temperature of the lower atmosphere where we live and breathe.
The average temperature of the ocean is 4C. 90% of the volume of the global ocean is at a near constant temperature of 3C. Only the surface layer comprising 10% of its volume gets any warmer (or colder) than that. It’s not a temperature/density thing either because seawater, due to its saline content, increases in density all the way down to its freezing point of about -2C.
The only possible explanation for why the average temperature of the ocean is 4C is because that is the average surface temperature of the earth taken over a period of time long enough for convection and conduction to equilibrate the entire volume.
Ostensibly I should think that period of time is about 120,000 years or once complete glacial/interglacial cycle.
Any sane person armed with the facts should find their source of angst to be that huge volume of icewater lying just below the warm thin surface layer of the ocean. The current interglacial period is getting long in tooth so again by any sane measure we should be worried about an impending ice age and if there is any merit whatsoever in anthropogenic global warming we should be glad for it and try to get as much of it as we can in the hope that it might delay the inevitable return of glaciers a mile thick covering everything north of Washington, D.C.
I wonder how many of the people I share the planet with are sane in this regard. The facts on the ground are incontrovertable. The only thing we have to fear is more ice not less ice. For this reason I call the warmista’s “ice huggers” in fond remembrance of their predecessors the “tree huggers”. The tree huggers morphed into ice huggers. Ain’t that a hoot? Trees don’t grow well in ice. They are lucky to survive where ice prevails much of the year much less thrive in such conditions. A warmer earth is a greener earth and a greener earth is what we all want, innit?

jae
May 8, 2011 10:25 am

Dave Springer:
Please “debunk” the article at the link given by DirkH: http://theendofthemystery.blogspot.com/2010/11/venus-no-greenhouse-effect.html
You folks are, in essence, saying that the lapse rate is CAUSED by the GHE. That is nonsense. The lapse rate is dependent upon only two variables, gravity (g) and the heat capacity of the air (Cp) and can easily be calculated from these two variables. GHE has no “place” in this formula. The lapse rate for ANY atmosphere (even pure nitrogen or oxygen) would be calculated the same way–without “allowance” for any GHE.

Anthony Zeeman
May 8, 2011 10:58 am

No mention is made of ocean and atmospheric heat transport. With one side of the Earth exposed to near absolute zero temperatures and the other exposed to the Sun; any theory has to take into account the movement of vast amounts of heat energy from hot to cold and vice versa. Although the Earth may lose heat at night, there is still substantial heat transfer from the day side to the night side keeping the night side warmer than it would be otherwise. The true “greenhouse effect” is a perfect example. Without convective heat transport a greenhouse becomes much hotter than its unenclosed environment. Simplistic models with lots of fudge factors to make the numbers fit are insufficient to describe the climate.

Dave Springer
May 8, 2011 11:02 am

More on heating and cooling of gases due to compression and expansion.
Heat and cooling only takes place as the gas pressure is changing. When the pressure stops changing so does the temperature. There is very little dynamic variation of atmospheric pressure at any constant altitude so there is very little heating or cooling taking place due to pressure variation.
In a real world example I have an air compressor in my shop. If the air in the tank is not compressed and I run the compressor to bring up to 10 atmospheres the tank will get much warmer than the air in my shop and if I quickly bleed it back to 1 atmosphere the tank will get colder than the air in my shop.
However (this is the key) if I leave the tank full of compressed air it won’t stay warmer than the air in my shop for long. That’s because the pressure is constant.
Ferd’s assertion that atmospheres get warmer as you go deeper into them because of compression is quite wrong. If there were some mechanism that were taking volumes of air high in the atmosphere and transporting them lower in the atmosphere his thesis would have some merit but there is no such mechanism at work on Venus, Earth, or Mars. The pressure gradient is nearly static and if an actual volume of gas is neither expanding nor contracting there is no associated change in temperature.

ferd berple
May 8, 2011 11:07 am

“A warmer earth is a greener earth and a greener earth is what we all want, innit?”
It certainly is surprising that people fear the earth getting warmer, yet when it comes to spending their hard earned money to go on vacation, few it any head towards the poles. If warming is something to be feared, where did the term “tropical paradise” come from? Why isn’t it “polar paradise”?

DirkH
May 8, 2011 11:10 am

davidmhoffer says:
May 8, 2011 at 9:49 am
“Once the temp hits -25 not only is the “colder” atmosphere above the artic backradiating, reradiating, simulating pajamas, what ever term you want, it get’s more efficient at it. So it can’t EVER cause an INCREASING temperature trend, but it can certainly recycle, reradiate, pajamas, the surface radiance to the point where continued decreases in temp slow to a crawl.”
David, this means that at -25 deg the upper atmosphere is no more colder than the ground; an equilibrium is reached. Of course, nothing happens from there; but your premise was to show that the people who say a cold object cannot warm a warm object are somehow wrong – your example shows that two equally cold objects are in equilibrium and does not contradict those people at all. In your example, the upper atmosphere is no more colder than the ground.

ferd berple
May 8, 2011 11:13 am

I don’t want to rain on your parade but the temperature at the top of the atmosphere is in the thousands of degrees.
Then it must be back radiating a fantastic amount of energy to the earth. At thousands of degrees this radiation would be in the visible or even UV.

Gnomish
May 8, 2011 11:14 am

” there is no good explanation for the approximately 33ºC (58ºF) temperature difference between the “light” input from the Sun and the “heat” output from an Earth”
temperature is not heat – you can’t measure heat with a thermometer. degrees can not be converted to watts.
the sidewalk doesn’t get hot because the air above it warmed it.
the sidewalk stores heat and warms the air above it.
incidentally, phase change from liquid to gas has no temperature change – but loads of heat is involved.
incidentally, avogadro’s ‘laws’ trump co2 fetish fantasy – co2 molecules don’t get to be hotter than everything else around them – all molecules in a local region are basically at the same temperature via kinetic transfer.
/me sick of co2 shrimpers. it’s like an old ed wood flick with fat ladies in a playpen and some old fart claiming to be the walrus. or maybe it’s like divine following that pooch.

ferd berple
May 8, 2011 11:16 am

“It was stored as water vapor evaporating from ground and bodies of water from solar heat, which rise to be COOLED, where the kinetic energy from gravity is then brought back to Earth with the rain.”
I recall years ago seeing a calculation that the amount of heat energy delivered to the earth by rainfall significantly exceed that delivered by bright sunlight.

Dave Springer
May 8, 2011 11:17 am

Ira:
We seem to be out of synchronization. In a series of comments I left the second in the series explained why the moon is not at its theoretical black-body temperature i.e.. because it isn’t a black-body. It’s a gray body with an albedo around 12%. I found the missing heat for you in that comment. I then went on to explain that the earth is also a gray body with an albedo of 30% to 40% and pointed out the crux of the issue – the earth’s average albedo isn’t know to any better certainty than +-5% and +-5 uncertainty in albedo translates into 25 watts per square meter of uncertainty in how much insolation is actually absorbed at the surface. Moreover all actual attempts to measure albedo, while disagreeing by 10% (I actually only found 7% disagreement in actual studies but still…) all do agree that the average in any given year is not static and varies from year to year. One experiment using earthshine as a measure that went on for about five years found a 1.5% difference between year one and year five. Their methodology in that one was measuring the brightness of the new moon which is illuminated solely by light reflected from the earth. It’s a bit surprising how difficult it is to get an accurate measure of the earth’s average albedo in any one year but on the other hand it isn’t difficult to measure change from year to year. The earthshine measure might not be accurate but it is precise and consistent. Precision and consistency is all you need to identify trends.
[Thanks Dave Springer and I appologize for not remembering that detail of your earlier attempts to clarify the issues. Please continue to share your special knowledge of climate science with me and others at WUWT. Your comments are most welcome here in my threads, even when they are corrections to my mistakes. In fact, especially when they help me improve my understanding and learn more about this topic. THANKS! – Ira 9:25PM EST]

Joel Shore
May 8, 2011 11:19 am

jae says:

You folks are, in essence, saying that the lapse rate is CAUSED by the GHE. That is nonsense. The lapse rate is dependent upon only two variables, gravity (g) and the heat capacity of the air (Cp) and can easily be calculated from these two variables. GHE has no “place” in this formula. The lapse rate for ANY atmosphere (even pure nitrogen or oxygen) would be calculated the same way–without “allowance” for any GHE.

The lapse rate is not caused by GHE’s per se. Your statement about the lapse rate is more or less correct…except that this sets a maximum lapse rate. That is, it is possible for an atmosphere to have a smaller lapse rate than you get from the stability calculation that you allude to. However, it won’ t have a steeper lapse rate because such an atmosphere becomes unstable to convection, with then brings the lapse rate down to the marginal stability value.
So, the best way to put it is that the lapse rate is caused by the fact that the atmosphere is heated from below in concert with stability arguments based on buoyancy, adiabatic expansion, and the existence of convection.
What the GHEs determine is where in the atmosphere the emitted radiation is able to escape to space. Radiative balance of the earth system then sets the temperature at this level in the atmosphere and the temperature at the surface basically follows from the lapse rate.

DirkH
May 8, 2011 11:29 am

Dave Springer says:
May 8, 2011 at 10:23 am
“For this reason I call the warmista’s “ice huggers” in fond remembrance of their predecessors the “tree huggers”.”
Good idea. They really do love their Greenland ice and all that.

Anton Eagle
May 8, 2011 11:32 am

Ira,
I have been reading many posts here that discuss the blackbody radiation of the earth. You have claimed that the 33K difference is due to GHG back radiation. I don’t want to argue that with you… but I think it would be worthwhile to be very very clear on exactly what you are claiming.
So… I can boil this down to a simple yes or no question…
Are you claiming that if the atmosphere were replaced with a different set of gasses that do not contain GHG molecules, but still had all the other macro effects such as clouds etc in the exact same amount as our current atmosphere… are you claiming that this atmosphere would NOT cause (directly or indirectly) the surface of the earth to warm at all?
You see… it does seem as if the answer to the above question is yes. It does seem as if you are making that claim. Are you? Or, are we just misunderstanding your argument?
-Anton Eagle

Joel Shore
May 8, 2011 11:33 am

John of Kent says:

We can account for the 33C difference without any need for an imaginary greenhouse effect. I urge everyone at WUWT to read this brilliant paper by physicist Joe Postma ;
http://www.globalwarmingskeptics.info/forums/attachment.php?aid=327

Bryan says:

You should read the article below.
It will answer all of your misconceptions about the “greenhouse theory” which in fact is pure fiction.
1. It is true that the radiative equilibrium temperature of the Earth is -18C.
2. This figure is confirmed by Satellite measurement from space.
3. It is NOT true that all the radiation emitted from the Earth surface.
4. The emission is mostly from the cloud level.
5. If we pick say around 5km as the average emission height to space we will be near enough.
6. Now use thermodynamics expressed in the form of the lapse rate.
7. An average moist lapse rate is -6.6K/km.
8. The lapse rate has nothing to do with back radiation as even scienceofdoom admits.
9. Work back down from -18C at 5km to the Earth surface and we get 5×6.6 = 33K.
So now you have an Earth surface temperature of 15C without any “greenhouse effect”.
http://www.ilovemycarbondioxide.com/pdf/Understanding_the_Atmosphere_Effect.pdf

Okay…So it is time to debunk the Postma nonsense again, even though we already did it in the last thread that Ira wrote.
The problem with the Postma argument is that the “average emission height” depends on…you guessed it…the IR-absorbing substances, namely greenhouse gases and clouds, in the atmosphere. So, the Postma argument does not get rid of the greenhouse effect…He merely sweeps it under the rug by obfuscating the fact that this average emission height being above the earth’s surface is due to the greenhouse effect.
Think about it. If the atmosphere contained no IR-absorbing substances, then all the IR emitted by the earth’s surface would escape into space and radiative balance would dictate that the earth’s average surface temperature (or really the average of emissivity*T^4 where T is the absolute temperature and the emissivity of most terrestrial materials in the wavelength range of interest is very close to 1) is set by the condition that the earth must radiate as much energy as it absorbs from the sun.
Now…Slowly add IR-absorbing substances. What happens is that some of the radiation from the earth’s surface is absorbed in the atmosphere, which subsequently radiates it again. This absorption can even occur multiple times but once the radiation occurs high enough up in the atmosphere that the remaining IR-absorbing substances above that layer are unlikely to absorb the radiation, then it can successfully escape to space. Hence, the average emission height is what is determined by the IR-absorbing substances in the atmosphere.
As the greenhouse gases in the atmosphere increase, this average emission height rises…and because of the lapse rate…this means less radiation is emitted back out into space. As a result, the earth system heats up until radiative balance is restored.

Anton Eagle
May 8, 2011 11:33 am

Ira,
I have a second question regarding the two emission diagrams you show.
The upper graph, showing down-looking readings at the top of the atmosphere shows that approximately 1/2 of the 15 um radiation is absorbed. It looks as if it would be about 90 mW/m^2 without absorption, and instead is about 45 mW/m^2 or so with absorption.
Then you show the lower graph showing the up-looking readings at the surface… showing about 90-100 mW/m^2 of radiation (at 15 um) down to the surface from the atmosphere.
So, my question is… if the GH gasses are absorbing about 45 mW/m^2… how are the re-radiating almost 100 mW/m^2 back to the surface?
It doesn’t look like the sun can be the source, since according to these curves the sun doesn’t radiate hardly anything at those wavelengths. So… where is the extra energy coming from?
Thanks,
-Anton Eagle

Bryan
May 8, 2011 11:36 am

Ira Glickstein
Do you know what the adiabatic lapse rate is?.
On page 21 of the Postma paper it is derived.
This is not a controversial point, serious IPCC advocates include this derivation as part of their narrative.
If you like I can give you a couple of such references.
The clouds at 5Km are far better radiators than gaseous CO2 or H2O .
Yet strangely enough it is the Latent Heat and high specific heat capacity of water in clouds that accounts for cloudy warmer nights rather than radiative effects.
The convection mechanism is reduced if there is a smaller temperature difference between Earth Surface and clouds.

MartinGAtkins
May 8, 2011 11:40 am

John of Kent says:
No! The earths surface is already emitting radiation in the Infra Red. It’s atoms and molecules are ALREADY excited sufficiently to be emitting IR.
Yes..The molecules of the earth surface have kinetic energy because they are excited or are being exited with the absorption of radiation.
This excitement distorts the magnetic field of the molecules components in a way that they are not in balance as they would be if the molecule was at rest. This leads the electromagnetic field of the molecule to send out waves of electromagnetic energy into space at decreasing rates if the source of the energy is turned of.
When the source energy is turned off the emitting waves from the molecule don’t cease immediately because the molecules components have mass and it is the movement of these parts that cause the decreasing waves of electromagnetic disturbance as energy is lost.
Similarly when the molecule is hit by a magnetic wave it will not reach its full kinetic and radiative potential until all its parts are moving in such a way as to propagate a field of disturbance equal to the constant of the energy input over time. At this point it is at radiative equilibrium with the incoming energy and no more work can be done.
Let’s say we are on the moon so there is no overlaying gas and pick an arbitrary incoming value of 100Watts/m^2. The mass of the particles at the surface will eventually increase in temperature until the moving parts of the molecules are so excited they cause a electromagnetic disturbance that is equal to the incoming electromagnetic energy from the sun. At this point the waves of energy from the surface propagate outward and are lost to open space. 100Watts/m^2 in, 100Watts/m^2 out.
This however is not the case with our planet. The over laying gas doesn’t impede the inflow of energy to the surface but the principle remains the same as the moons surface in that in its self the surface will move toward radiative equilibrium with the incoming energy.
This time however the outgoing radiation is now blocked by a molecule of gas which of course also has mass and a state of rest.
The propagating electromagnetic wave from the earths surface is now absorbed by the molecule which in turn adopts kinetic and thereby electromagnetic properties as the state of rest is now disrupted and its components vibrate, sending out electromagnetic waves as it struggles to balance its competing internal forces.
Although free floating the rules for the gas molecule are the same as the molecules at the earths surface. It will absorb all the incoming energy from the earths surface until it radiates away the same amount of radiation it is receiving. It will do this in all directions, so as it moves to reach radiative equilibrium with the surface it will radiate half toward the surface and half away. The energy being radiated back to the surface is not kinetic, it is electromagnetic. It would be 50 Watts/m^2 in the first instance of time with the other 50 Watts radiating away. The surface doesn’t know where the 100 Watts or the 50 Watts are coming from it only knows it is receiving 150 Watts in the second instance of time.
Think of this. If we had a closed system where no energy is lost and we were to put 100 watts a second into the system we would have a system with 100 joules after the first second. Now in the next second we only put 50 Watts a second into the system we get 150 joules in the system. The system doesn’t care that the that the last second had a lower value than the first second it only knows that over two seconds it has received 150 Watts of energy.

Joel Shore
May 8, 2011 11:49 am

John Marshall says:

Taking into account the adiabatic effect, which is real and measurable, there is no need for the GHG effect.

No…It does not as I have explained above.

The greenhouse effect also has one fatal flaw. The warming in the mid troposphere in the tropics shown by all models has yet to be found in real observable data. I do not care what your calculations show (in reality another model), if this warm area shown by models is not there then the theory falls flat on its face!

The prediction of tropical tropospheric amplification has nothing to do with the greenhouse effect. It has to do with the fact that in the tropics the temperature decrease with height is expected to closely follow the moist adiabatic lapse rate. Hence, this amplification is predicted for warming due to any mechanism.
It is also worth noting that tropical tropospheric amplification is well-verified by the satellite and balloon data for the temperature fluctuations that occur on monthly to yearly timescales, e.g., due to ENSO. Where the data is more ambiguous is with the long-term multidecadal trends, where such data is easily contaminated by artifacts due to changes in instrumentation over time, switches from one satellite to another, etc. The different data sets…and various analyses of the data sets tend to differ not only from the model predictions (depending on which data set you look at) but also from one another.
Even Richard Lindzen agrees that tropical tropospheric amplification follows from basic physics and has nothing to do with the greenhouse effect mechanism…and that the fact that is a problem with the observational data, not the models (and, at any rate, nothing to do with the mechanism causing the warming). His only point of departure is that he thinks the data more likely to be wrong is the surface data in the tropics whereas most others think it is the data at altitude.
The specific fingerprint of warming due to an increased greenhouse effect (at least relative to, for example, the mechanism of heating by increased solar radiation) is that the greenhouse effect is predicted to cause cooling in the stratosphere. And, this is indeed what is seen. (Some of the cooling is due also to the decrease in stratospheric ozone but the amount and altitude distribution of the cooling apparently cannot be explained solely by this mechanism.)

Dave Springer
May 8, 2011 11:53 am

Ira Glickstein, PhD says:
May 8, 2011 at 10:41 am
“I hasten to add that engineers are not as picky about precision of estimates as scientists and mathematical analysts are about their data. If we get estimates within 15% (33ºC vs 28ºC) that is usually close enough, since other uncertainties are likely to be in the mix. ”
Acceptable margins of error are established in context. Sometimes being within an order of magnitude is good enough and other times the proverbial nine nines isn’t good enough. But I understand your point. It’s illustrated by a trite expression that +-10% is good enough for government work. Left unsaid is that the government screws up a lot because what’s considered “good enough” by them often isn’t.

Dave Springer
May 8, 2011 12:06 pm

Ira:
A chain is no stronger than its weakest link. I think we can agree on that.
When it comes to trying to sort out natural forcings from unnatural forcings we run into a link in the chain where we have a 25w/m2 range of uncertainty in surface forcing from natural factors. We might reliably know that unnatural forcings fall in the range 0.6 – 2.5w/m2. So that’s a strong link but it’s strength is meaningless when there’s a link in the chain with only a tenth of that strength. See what I’m saying?

Joel Shore
May 8, 2011 12:06 pm

Martin Mason:
Ira, the Second Law of thermodynamics works at every level and you cannot transfer heat from a colder body to a warmer body. A black body cannot also reabsorb and re-emit lower intensity radiation that it has already emited.
You clearly do not know the basis for the Second Law, which is statistical physics. You seem to believe in some sort of magical version of the Second Law. The actual Second Law is based on the fact that although energy transfers occur in both directions, by simple statistics it becomes essentially astronomically improbable that the net flow of energy, what we call “heat” will be from the hotter body to the colder body for any macroscopic bodies.
In particular, applied to two radiating objects at different temperatures, the Second Law simply states that the amount of radiation from the hotter body that is absorbed by the colder body is always larger than the amount of radiation from the colder body that is absorbed by the hotter body. The laws of radiative transfer automatically satisfy the Second Law when applied correctly and all models of the greenhouse effect, be they toy models or advanced radiative-convective codes, satisfy the Second Law.

Greenhouse heating by back radiation is surely wrong because it says that if you put a frozen steak inside a vacuum flask with reflective interior that the steak would cook, that if you stand in front of a mirror you will heat up from the reflected rays

No…That doesn’t happen because of simple conservation of energy. The steak is not producing thermal energy or receiving it from a hotter object. So, all that the reflective interior can do is slow the cooling of the steak.
The earth is receiving lots of thermal energy from a hotter object, namely the radiation that it receives from the sun. Its steady-state temperature is set by the balance of what it receives and what it emits back out into space. The greenhouse effect, by affecting the rate at which the earth emits radiation back out into space for a given surface temperature, causes the earth’s temperature to warm in order to maintain radiative balance.

or that you can save on your heating bills by filling the loft with CO2?

Insulation in general does help you to save on heating bills. The reason that CO2 in particular does not is for a variety of reasons, one being that the heat loss from a house is due to many mechanisms besides radiation. (For the earth system as a whole, the only way it can lose heat is via radiation, although heat can be transferred between different parts of the system, e.g., surface to atmosphere by convection and evaporation/condensation.) Another is that, as discussed above, the full picture of how adding CO2 affects the greenhouse effect relies on the fact of the lapse rate in the troposphere, something that is not applicable in your example.

Tim Folkerts
May 8, 2011 12:08 pm

Dave Springer says: May 8, 2011 at 10:23 am
The only possible explanation for why the average temperature of the ocean is 4C is because that is the average surface temperature of the earth taken over a period of time long enough for convection and conduction to equilibrate the entire volume.

Dave,
Wouldn’t the explanation be that the surface temperature where there are down-welling currents must be around 4C?
Currently (no pun intended) the major down-welling current is the gulf stream in the north Atlantic. As long as the north Atlantic is ~4 C, the currents along the bottom will be ~ 4 C. It is reasonable to speculate that down-welling currents would always be in cold regions, since that is where the surface water is most dense. (Add in the fact that evaporation of warm water as it heads pole-ward will increase salinity and increase density and we have a stronger reason for down-welling currents are in the polar regions.)

Charlie Foxtrot
May 8, 2011 12:11 pm

Several commenters maintained that a colder body cannot heat a warmer body. They may have confused heat with temperature. Heat can be transferred from a cold body to a hotter body, but a colder body cannot increase the temperature or heat content of a hotter body. However, a colder body can decrease the rate of cooling of the hotter body by radiation to the hotter body. Electrons jumping orbitals and producing radiation don’t know if there is a warmer or colder body out there somewhere, therefore the total instantaneous radiation from any body is independent of the temperature of the surroundings.
Something I did not see in the comments was mention of emissivity, which would have a direct impact on the sensible temperature of the air and surface of the earth. Good emitters (high emissivity) both absorb and emit energy better (grass, leaves, black bodies) than low emitters (most light colors, stainless steel). Emissivity will vary greatly from year to year depending on snow cover, clouds, crops, deforestation, etc.
Dr. Glickstein’s discussion is interesting, but I think is too simplified to convince any warmist that CO2 is not cooking the earth.
A couple of things that I am curious about that directly affect the emissions balance, and which have not appeared in any discussion I have read, are:
1. Energy is absorbed and converted to chemical energy through photosynthesis. I have no idea how much energy that would be (probably not much), but with higher CO2 levels, vegetation is growing faster so it is logical that more radiation is being converted to plant material than in the past, which is stored energy that will not be re-emitted until the material is burned or otherwise reverts to it’s former state.
2. The earth might correct temperature variations by moderating emissions, especially in the Arctic. When the northern oceans are warmer than normal, they freeze later in the season, and since they emit to an almost perfect black body (deep space with no sunlight) they release much more heat and take up very little, esp. in winter, helping to correct the global temperature. Ice and snow are poor emitters and also insulate the warmer ocean beneath whereas a dark ocean is a very good emitter. A change in ocean currents, for example diversion of a warm pacific current to the Arctic for a prolonged period, could cause global cooling by eliminating ice in the Arctic all year. I have no idea how significant this cooling would be, but suspect it is substantial. Perversely, an abnormally warm current to the Arctic would cause warming of the atmosphere while reducing heat content in the ocean, making it appear as if the earth is warming when in fact it is cooling. Do the GCM’s address this?

William
May 8, 2011 12:21 pm

As there are multiple periods in the geological record of tens of millions years in duration when CO2 levels were high and the planet was cold and when CO2 levels were low and the planet was warm, it appears there is a basic fundamental assumption in the model of atmospheric radiation that is incorrect or there is an omission of another mechanism from the standard models of atmosphere radiation.
One possibility is the greenhouse mechanism saturates such that the initial CO2 causes most of the warming.
Gas molecules transfer energy from molecule to molecule by collisions in addition to radiation.
As one moves higher in the atmosphere there are more ions due to galactic cosmic rays striking the atmosphere. An ion radiates continuously due to motion of the ion in addition to the band specific radiation that is emitted when an ion captures an electron. It is possible that radiation from ions provides the leak to cause the CO2 mechanism to saturate.
Atmospheric carbon dioxide levels for the last 500 million years
http://www.pnas.org/content/99/7/4167.full
Using a variety of sedimentological criteria, Frakes et al. (18) have concluded that Earth’s climate has cycled several times between warm and cool modes for roughly the last 600 My. Recent work by Veizer et al. (28), based on measurements of oxygen isotopes in calcite and aragonite shells, appears to confirm the existence of these long-period (_135 My) climatic fluctuations. Changes in CO2 levels are usually assumed to be among the dominant mechanisms driving such long-term climate change (29).
Superficially, this observation would seem to imply that pCO2 does not exert dominant control on Earth’s climate at time scales greater than about 10 My. A wealth of evidence, however, suggests that pCO2 exerts at least some control [see Crowley and Berner (30) for a recent review]. Fig. 4 cannot by itself refute this assumption. Instead, it simply shows that the ‘‘null hypothesis’’ that pCO2 and climate are unrelated cannot be rejected on the basis of this evidence alone.
http://www.nature.com/uidfinder/10.1038/nature01087
Despite these successes in linking variations in greenhouse gas concentrations to climate change in the geologic past, the oxygen isotope palaeotemperature record from 600 Myr ago to the present displays notable intervals for which inferred temperatures and pCO2 levels are not correlated1. One of these occurred during the early to middle Miocene (about 17 Myr ago), a time well established as a warm interval (relative to today), but with proxy evidence for low atmospheric pCO2 (ref. 2). Moreover, whereas climate models predict tropical warming in response to elevated pCO2, geologic data — in particularly the oxygen isotope record — indicate muted warming or even cooling at low latitudes while higher latitudes warm (the ‘cool tropicsparadox’10–11).

Martin Lewitt
May 8, 2011 12:24 pm

Bryan,
The 2.66 grams per square inch figure is considering the whole air column.
http://www.wolframalpha.com/input/?i=14.7+lbs+*+400%2F1000000
Alan D. McIntire,
The solar radiation hitting the earths surface is 184 W/m^2 per Trenberth 2009 or 198 W/m^2 per Trenberth 1997. The surface albedo must be applied to this to get the net gain. Back radiation adds another 333 or 324 W/m^2 respectively. Since Trenberth’s diagram don’t show any reflection at these wavelengths , it is unclear whether he has applied the albedo that corresponds to these wavelengths. The albedo of surfaces varies with wavelength, and for some surfaces like snow the variation is quite significant.
If the emission from a surface is blocked from escaping in some parts of its spectral emission range, then, everything else being equal, it must increase in temperature until enough of its spectrum is in the ranges that do allow the energy to escape. That would be its equilibrium temperature. Of course, everything else isn’t equal, it will never have to reach that temperature, because conduction, evaporation, convection, poleward heat transport, etc. will assist in transporting the heat away.

stumpy
May 8, 2011 12:42 pm

Energy in = energy out is not technically true though, the heat that reaches the surface is attenuated as the surface warms and cools, for example heating of the ocean surface and ground surface. When the sun first hits them in the morning they are not instantly at thermal equilibrium and radiating what they recieve straight back upwards, plus the deeper ocean can store and release energy – I know over time energy in = energy out, but it doesnt actually apply to any moment during a day, its why the coldest temps are night are the early morning, and why its warming in the afternoon, not when the sun is highest. I think the diurnal pattern of energy release is also something worth considering, as it does not match the incoming diurnal pattern.
I think to further peoples understanding the impact of surface attenuation and the affect on the ocean on the global energy flow need to be discussed, as the greenhouse effect doesnt quite hold if these affects are not considered when looking at a point in time on the earth

Super Turtle
May 8, 2011 12:50 pm

The issue at hand here is not that co2 can absorb energy, but what occurs and how does it contributes to AGW, and specific man’s co2 output.
If I suspend in the middle of my room a 4 foot by 6 ft by 2 inch thick lead plate in the middle of my room (not touching the walls), then while that lead plate has huge ability to absorb energy (1000’s of times more than co2), that introduction of the lead plate will NOT cause the room to warm up. So, any energy absorbed by this lead plate will be re-emitted back into the room.
If the above lead plate could heat up the room, the we would all purchasing big lead plates to suspend in the middle of the room for free heat.
On the other hand, increasing the insulation of the WALLS of the room is a different matter.
So, at the end of the day is not that co2 can absorb energy, but what occurs with that effect. As the lead plate example shows, just because physics says something can absorb energy does not necessary mean it affects the inflows and outflows of energy in a given system.
And in fact it not the co2 that holding this energy anway, but the other substances such as ground and ocean and other parts of atmosphere that represents the bulk of the “mass” of the atmosphere to hold that heat energy.
In fact, this explains why engine blocks and especially motor cycle heads are often painted black, not silver in color. The choice of black paint on the engine can cause the engine to cool better! In other words, placing black paint on an engine block INCREASES cooling effect yet black paint is known to absorb energy better!
In effect, since black is a good absorber of energy it’s also thus becomes a good emitter of energy. Black objects emit more radiant energy (cools faster) than a white or silver to its cooler surroundings.
The issue here never been that co2 can absorb energy, but the issue is not the slam dunk as to what effect this has in terms warming the overall system, or even as some claimed can in fact cause cooling of the system based on the same reasons why black paint on engines can help cooling!
Super Turtle

Nullius in Verba
May 8, 2011 12:53 pm

“If there were some mechanism that were taking volumes of air high in the atmosphere and transporting them lower in the atmosphere his thesis would have some merit but there is no such mechanism at work on Venus, Earth, or Mars. The pressure gradient is nearly static and if an actual volume of gas is neither expanding nor contracting there is no associated change in temperature.”
Hadley cells transport volumes of atmosphere from high up to lower down. Don’t confuse the motion of the fluid with its pressure distribution. The fluid can move across a pressure gradient, expanding and contracting as it goes, while the pressure at every point remains constant.
If there were no convection loops on Venus, it wouldn’t be so windy there.

HankHenry
May 8, 2011 1:20 pm

It seems that for incoming sunlight there is one surface (where sunlight strikes) but for the outgoing radiation there is another surface (at the top of the atmosphere for some wavelengths). One might say that the heat that consumes time migrating from the incoming surface to the outgoing surface represents the heat of greenhouse warming.
I always wonder about the 15 degree C number quoted as the earth’s surface temperature. It seems that this is the just the air temp nearest the surface struck by sunlight (assuming I don’t misunderstand the source of the number). Why wouldn’t it be more appropriate to attempt to average all temps (air, sea, and land) inside the regions somehow touched by solar radiation (but excluding deep earth temps influenced by heat of radioactivity).

Dishman
May 8, 2011 2:12 pm

Ron House wrote:

Dishman says:
In re Objection #2:
Gas molecules follow ballistic trajectories between collisions…
… unless all the work on Gravity since Galileo is wrong, and doesn’t apply at the molecular level, of course.
Ballistic trajectories are trajectories under the action of gravity. Note, he didn’t say they travel in straight lines.
However: the approximate speed of molecules in the atmosphere is about 1,700 km/h. The path isn’t very curved, in other words. And the mean free path of a molecule in air is about 65 nanometres. You’d be pretty amazing if you could measure the deviation from a straight line of a path segment with that length and curvature.

Whether or not it’s directly measureable doesn’t matter. What matters is whether or not it’s happening.
If molecules follow ballistic trajectories, the minimum KE will occur at zenith and be higher at all other points.
A short MFP just means that equipartitioning will be rebalanced (on average) every MFP.
Assuming air (a diatomic gas with 5 degrees of partitioning), for a given delta-h, the thermal energy will be reduced by 1/5th of the energy required to lift molecules by that delta-h.
My point is that Objection #2 is not, in fact, correct.

ferd berple
May 8, 2011 2:24 pm

The solar radiation hitting the earths surface is 184 W/m^2 per Trenberth 2009 or 198 W/m^2 per Trenberth 1997. The surface albedo must be applied to this to get the net gain. Back radiation adds another 333 or 324 W/m^2 respectively.
There seems to be a whole lack of common sense going around these days.
So, according to T, the earth receives 184 w from the sun, radiates some of this back outwards, where some of it is absorbed by the atmosphere and re-radiated back to earth, where it provides 333 w.
In other words, 184 w ends up as 666 w, because the back radiation is omni-directional at at most 50% reaches the earth.
Of course, it is like two mirrors pointing in towards each other. Start out with 184w and after it reflects back and forth a bunch of times, you end up with 333w.
Only one problem. If you start out with 184w from the sun, you can NEVER absorb more that that. Where is the excess energy coming from? The sun is the energy source.
The atmosphere cannot manufacture 333 w of back radiation for a 184 w power source. The back radiation from the atmosphere must be less than the original 184 w from the sun.
Otherwise we could put an ingenious array of one-way mirrors around a 100 watt light bulb, and generate millions of watts of power. What climate science has invented is a perpetual motion device, where the energy out is more than the energy in.

ferd berple
May 8, 2011 2:31 pm

“If there were some mechanism that were taking volumes of air high in the atmosphere and transporting them lower in the atmosphere his thesis would have some merit but there is no such mechanism at work on Venus, Earth, or Mars.”
There is massive movement of air from high altitudes to low altitudes, as a result of an equally massive movement of air from low altitudes to high altitudes.
http://en.wikipedia.org/wiki/Hadley_cell

wayne
May 8, 2011 3:01 pm

Here is an experiment to disprove the existence of back radiation.
CAUTION: Only perform with parental supervision.
CAUTION: Treat any large Fresnel lens as if it were an open flame!
CAUTION: Since this experiment is indoors, pay close attention to ANY light or infrared (IR) source, which could be focused by the lens. That includes sunlight through windows, hot kitchen stove filaments, etc.

The only thing great expense in performing this experiment with be the Fresnel lens itself. One square meter Fresnel lenses are available through science supply sites. The lens need to be a plano-convex radial lens to focus parallel rays onto a one square centimeter spot (or even down to one square millimeter). I have heard an F-stop near one is best. This gives a one ten-thousand multiplication though that will to be reduced by up to ~50% due to refraction, scattering and optical imperfections in the facets.
You will need a very small black metal cap to hold one cm³ (1 gram) of water or ice. This will be placed at the focal point of the lens. You will need an infrared (IR) heat source, a space heater, large wattage hair drier, something to add additional warmth to a wall later in the experiment. You will later need an ice cube.
Similar to:
http://www.modulatedlight.org/optical_comms/fresnel_lens_comparison.html
http://www.scientificsonline.com/large-fresnel-lens.html?&cm_mmc=Mercent-_-Google-_-NULL-_-3052833&mr:trackingCode=CC6E1735-DB81-DE11-8C0A-000423C27502&mr:referralID=NA
Now, ready? If back-radiation proponents are correct, we should be able to use this huge lens to focus back-radiation from any warm wall within your house, at the ambient room temperature of say 20 ºC (68 ºF), in some room to boil water in seconds.
Use a small flashlight to illuminate the wall and so the exact focal point can be located. At this point place the black cap with the one gram of water. Wait. If they are correct the water should evaporate quickly, even boil, for that 70 ºF wall is giving off 5.67e-8*(273K+20K)^4 or 418 Watts per square meter that is going to focus down to 400 W x 5,000 multiplication factor or 2,000,000 Watts per square centimeter and we only need 2250 Joules to vaporize that gram of water. Wait, maybe there are some loses within your setup so be patient. Wait. Don’t know about your experiment results but mine is not working. Half an hour later I gave up, something mystical is blocking all of those back radiation photons from entering that lens. Watts up with that?
Now replace the water and cap with an ice cube. it should develop a hole slowly at the focus. Here the temperature at the focus is less than the temperature of the wall and radiation flows.
But I want to see the water boil, so cover the lens and warm up that wall in front of the Fresnel lens to a much higher temperature, let’s say 104 ºF (40 ºC). The watt flux should now be something near 5.67e-8*(313K-293K)^4 = 0.009 W/m^2 or 50% efficiency of 0.009 W/m^2 x 5,0000x = 45 W/cm^2. Uncover the lens. Well, it probably won’t boil but you should now evaporate that gram of water in about 2250 J/g ÷ 45 J/s or 50 seconds if you have a perfect setup with no loses. Let’s just say in a couple of minutes.
Careful, but now fill the cap and point that heat source, directly into the lens from five or six feet away, this is why Fresnel lenses can be dangerous, the water should now boil and quickly in seconds.
Thermal radiation heat going backwards does not exist in reality, at all, if you are a scientist type and not merely a AGWChurch member, run the experiment, prove it in reality.
Sorry back-radiation lover’s, you love is just in your minds (and wrong application of Stefan-Boltzmann equation).
That is also why Fresnel lenses are not dangerous in a very warm dark room at the focus. That room is at radiative equilibrium due to everything being at the same temperature and there is, in reality, no radiation at all, or you could focus it.

Martin Lewitt
May 8, 2011 3:11 pm

Ferd Berple,
You are just looking at the downward radiation, the upward radiation balances it, there is no energy problem. The mirrors on two sides of the bulb has some merits as an analogy, you wouldn’t doubt that the total flux of photons going back and forth between the mirrors sums to more than the 100 watts being input, but at equilibrium, only 100 watts escapes. A completely enclosed bulb is not a good analogy, because the energy is allowed to accumulate, the filament’s resistance will increase with temperature, reducing the amount of energy input, but still the bulb components will eventually melt.
In the climate case, the system is very complex, because the increase in temperature in response to the back radiation, increases a number of other responses to increased temperature before equilibrium is reached.
The idea that the CO2 effect is insignificant, and adiabatic explains all, fails for the common observation of another greenhouse gas, H2O. If it isn’t back radiation, how do you explain the difference in heat loss, on a clear calm humid night vs a clear calm dry night?

May 8, 2011 3:17 pm

DirkH;
David, this means that at -25 deg the upper atmosphere is no more colder than the ground; an equilibrium is reached. Of course, nothing happens from there; but your premise was to show that the people who say a cold object cannot warm a warm object are somehow wrong – your example shows that two equally cold objects are in equilibrium and does not contradict those people at all. In your example, the upper atmosphere is no more colder than the ground.>>>>
Read more carefully my friend. I said that at at 25,000 feet it is colder than that even at the equator. Two points:
1. According to AMSU-A http://discover.itsc.uah.edu/amsutemps/ satellite data the global average at that altitude is in the range of -38C. It would be warmer than that over the equator, and colder… much colder, over the arctic.
2. 25,000 feet isn’t “upper atmosphere”. In fact, it is only part way up the troposphere which is the lowest layer of the atmosphere. Troposphere ends at 11Km, then Stratosphere about 50Km, Mesosphere about 88Km and then the thermosphere about 500km and then the Exosphere.
Lot’s of other examples by the way. Build a house with no insulation and measure the temperature in the house and between the inner and outer walls with the furnace running at a constant rate. Then insulate the wall cavity. With the furnace running at the same rate the inside of the house will become warmer. The thermometer between the inner and outer walls will also read warmer, but still cooler than inside the house. The insulation absorbs outbound heat and reradiates it back into the house. Pile snow up around a house, and despite being colder than the inside of the house, the house gets warmer with the exact same heat sources inside, that’s how the early pioneers in NA insulated their sod shacks in the winter. That’s how the Inuit survived centuries in the far north by living the winter in igloos where the cold ice walls nonetheless kept them warmer than they would have been in the open…It’s even the same principle that a freezer works on as does an air conditioner. A kiln gets much hotter than an open fire, but put a thermometer inside the wall of the kiln and you will find that it is cooler than the temperature inside the kiln. Same with a forge. And no, these examples have nothing to do with convection, you can build a forge or a kiln with thin walls or thick walls and have the exact same air circulation, but the thick walled ones have a higher temperature inside, and the temperature of the walls is cooler.
I am a raging skeptic. An angry, they are misleading us with how they represent the results, producing results from tragically flawed and sometimes outright fraudulant “science” and are being used by lobbyists who are more interested in lining the pockets of their lobby than they are in what’s best for humanity, and I’m disgusted by the whole thing. But cold things radiate photons carrying energy and if they bump into something warmer, there are any number of factors that determine if they will be absorbed or not. But the relative temperature of the two bodies has zippo to do with it. The relative temperature of the two bodies only defines the magnitude and direction of the NET energy exchange, which can only be from warmer to cooler. But take the cooler body away, and the warmer body’s temperature must fall because it is no longer recieving any heat from the colder body.

wayne
May 8, 2011 3:38 pm

Martin Lewitt says:
May 8, 2011 at 3:11 pm
The idea that the CO2 effect is insignificant, and adiabatic explains all, fails for the common observation of another greenhouse gas, H2O. If it isn’t back radiation, how do you explain the difference in heat loss, on a clear calm humid night vs a clear calm dry night?
——
Simply the difference in the temperatures of one layer to the other (clouds in this case). The air above the cloud tops will be cooler than normal shielded by the water mass in the clouds and the air between the surface and the cloud base warmer than normal. Please, pick up a calculator and calculate these layer’s flux, the overall flux from surface to TOA when all added remains the same, but, the inter-atmosphere profile of the temperatures causing the flux at each layer will be different in the two cases.

Martin Lewitt
May 8, 2011 3:46 pm

Wayne,
For how much of the infrared black body frequency range is your Fresnel lens able to focus at that one spot? Is your lens material transparent to infrared?

Bryan
May 8, 2011 3:57 pm

Joel Shore says:
…”Okay…So it is time to debunk the Postma nonsense again, even though we already did it in the last thread that Ira wrote.”……
Joel, in case any readers are unaware, is one of the authors of one of the most imaginative pieces of fiction ever to have made its way into a science publication.
Readers of Ira’s last thread will know that Joel postulated that;
1. Heat can move from a colder surface to a warmer surface spontaneously.
2. Invented statements and then proceeded to attack them.
3. Has the Stephan Boltzmann equation applying to gases.
Perhaps Joel is still a bit “rusty” about when and where the Stephan Boltzmann Equation applies.
It applies to black bodies issuing a continuous spectrum centred around a characteristic temperature.
It is the full integration of the Planck function.
It does not apply to the line spectra issued by H2O and CO2 making up less than 1% of the atmosphere.
Joels proof of the “greenhouse theory” involves getting rid of the Oceans and atmosphere.
Postma is absolutely correct in stating that the long wave radiation produced by the Earth system is released at an average height.
Some radiation from the ground leaves directly through the “window”.
Radiation also leaves from different heights above the surface.
The average height is about 5Km.
The lapse rate which is completely independent of the radiative effects then determines the surface temperature .
The greenhouse theory is like Joel’s paper, …… pure fiction.
Bare Rock Earth is Joel’s favorite topic and he rarely strays from it.

May 8, 2011 4:02 pm

Charlie Foxtrot: exactly. Well-said.
Dr. Glickstein: I appreciate your efforts to draw analogies to explain the CO2-induced warming of the Earth. However, I’m suspect the effort is doomed from the start because one cannot apply a steady-state model to a dynamic process. The Earth does indeed receive energy from the Sun, and radiates energy into space. But, the system is not at steady state due to Earth’s rotation around its axis, slight eccentricity of its orbit around the sun, greater or lesser cloudiness over time, absorption of energy into oceans or loss of energy from the oceans, and likely a host of other factors not mentioned in this list.
I’ve done a fair bit of outdoor cooking, over a campfire with a chunk of meat rotating on a spit above the fire. One would have a tough time creating a valid model of how the meat cooks using a steady-state model. Same thing applies to the Earth’s energy balance.

Bill Illis
May 8, 2011 4:21 pm

1 Watt = 1 Joule/second
1 Watt/m2 = 1 Joule/second/m2
Now convert the whole concept into Joules and bring time into the equation – say a 24 hour period. It will look much different.
The problem with all the calculations is that the definition of “Watt” has a time dimension already built into it so we tend to forget that all this has to happen in time.
The Sun is beating down at 960 Joules/second/m2 at the height of the day but there is almost no energy coming at night. How come the average Earth temperature is only the equivalent of 418 Joules/second/m2 in the daytime and 364 Joules/second/m2 at night.

wayne
May 8, 2011 4:40 pm

Martin Lewitt, it’s an experiment for you to perform, look on the specifications when your lense arrives, or better, mid-IR lens are also available, specify your choice when ordering. Let us know your results. ☺

May 8, 2011 4:46 pm

“However, the actual mean temperature at the Surface is closer to 288 K. How to explain the extra 33 K (33ºC or 58ºF)? The only rational explanation is the back-radiation from the Atmosphere to the Surface.”
Maybe the effect of compressing the atmosphere might cause an increase in of 33 degrees C.

wayne
May 8, 2011 4:49 pm

I wish my fingers would learn that ‘lens’ is singular, ‘lenses’ are the plural. ☺

May 8, 2011 4:49 pm

“Yes, ferd berple, when a gas is compressed its temperature increases. However, once compressed, that increase in temperature will radiate away.
For example, when you pump up your bicycle tire, the pump and the air in the tire will be warmer than the ambient (uncompressed) air in the room. However, the air in the tire, although it is compressed to several times the normal ambient pressure, will not remain warmer for very long. The heat energy will radiate and convect and conduct away until the tire reaches equilibrium with the ambient temperature of its surroundings.”
A tire is an enclosed space where the air pressure is the same in all locations. The atmosphere is not. The air circulates. As it warms, it rises and cools, and as it cools it falls and warms. Go ahead and bat away a fact of science and try and shove your less than fully thought out process down some other people’s throats.

May 8, 2011 4:59 pm

And continuing.
I would like to see the back radiation proponents do the following.
Take a spherical black body object with a small heat source in its center and shine a light on it and read the temperature.
Take the same spherical object and enclose it with a black body enclosure that does not touch the surface except at some required points to center it around it. use the same lamp at the same distance, and measure the temperature at the surface of the original sphere. Now make another black body enclosure and repeat the procedure. Do this until you have 12 layers of black body. If your argument is correct, then the temperature should sky rocket to the point of starting a fire. According to back radiation proponents statement of how it works.

DirkH
May 8, 2011 5:04 pm

Ira Glickstein, PhD says:
May 8, 2011 at 4:26 pm
“The same is true of the Atmosphere. If we transported an Atmosphere’s worth of air to a planet that had no Atmosphere, the force of gravity of that planet would indeed compress the Atmosphere and warm it. However, before long, the Atmosphere would cool down to the temperature of the planet. If we then placed that planet in an Earth-like orbit around the Sun, short-wave Solar radiation would be absorbed by the surface of the planet”
You’re right so far.
” and the surface would re-emit long-wave radiation to its Atmosphere where some would pass through and out to Space, while some would be absorbed by the H2O and CO2 and other “greenhouse” gases in the Atmosphere.”
Here it starts going downhill. There are three modes of energy transport from the surface upward: LWIR Radiation, conduction and convection, LWIR radiation being the weakest. IOW, the oceans get warmed by visible and UV radiation and will give the energy to the atmosphere mostly by surface conduction; kinetically.
” The heated gas molecules would bump into other air molecules and warm them, and like any material above absolute zero, the Atmosphere would emit radiation at a variety of long-wave wavelengths in random directions, some of which would be absorbed by the surface of the planet, warming it further. Just like our Earth.”
The gas molecules who get excited by LWIR photons would reradiate some immediately; in some cases they would give the energy to neighbouring molecules, thermalizing the energy. But Kirchhoff’s Law states that for any number of thermalization events there must be an equal number of dethermalization events as long as the gas is in local thermal equilibrium, which it is in the lower atmosphere. So no heat is trapped there.
See
http://wattsupwiththat.com/2010/08/05/co2-heats-the-atmosphere-a-counter-view/
So, the consequence is that LWIR radiation is transporting energy to space. Some is re-emitted back to Earth from where it is re-emitted back upwards. Whether there is 0.03 % or 0.04 % of CO2 in the atmosphere only influences how often the photons get absorbed and re-radiated on their way to space – an increase in CO2 delays the process a little but does not change it fundamentally and *Does* *Not* *Trap* *Heat* any more than a sieve traps water.

May 8, 2011 6:16 pm

Ira,
Please detail exactly why you think Swedish climatologist Dr. Hans Jelbring and chemical engineer William Gilbert are “foolish” “non-scientific Disbelievers” and “irrational” in their explanation of the so-called “greenhouse effect” based simply upon the adiabatic lapse rate:
http://www.tech-know.eu/NISubmission/pdf/Politics_and_the_Greenhouse_Effect.pdf
as well as physicist Joe Postma who explains the same
http://www.globalwarmingskeptics.info/forums/attachment.php?aid=327

Joel Shore
May 8, 2011 6:36 pm

Bryan says:

Readers of Ira’s last thread will know that Joel postulated that;
1. Heat can move from a colder surface to a warmer surface spontaneously.
2. Invented statements and then proceeded to attack them.
3. Has the Stephan Boltzmann equation applying to gases.

Actually, what readers of that thread will know is that Bryan is a purveyor of pseudo-science. He has absolutely no serious desire to discuss the science and, like all purveyors of pseudo-science, tries to distract people by nitpicking words…even after the wording has been corrected to everyone’s satisfaction. (See here for more discussion specifically on the tactics that Bryan, Gerlich and Tscheuschner and other purveyors of such pseudo-science employ: http://wattsupwiththat.com/2011/03/29/visualizing-the-greenhouse-effect-molecules-and-photons/#comment-649274 ) The existence of people like Bryan within the skeptic movement, willing to actively engage in such deception, certainly helps to keep you guys marginalized in the scientific realm.

3. Has the Stephan Boltzmann equation applying to gases.

Joels proof of the “greenhouse theory” involves getting rid of the Oceans and atmosphere.

More nonsense. Simple toy models of the greenhouse effect illustrate the concept and give one a qualitatively understanding of what is going on. Line-by-line radiations codes do the full calculations for the actual absorption lines as they exist in the atmosphere in order to nail down the effects quantitatively. Yes, the simple models don’t have all the gory details in them…That is the whole point.

Postma is absolutely correct in stating that the long wave radiation produced by the Earth system is released at an average height.
Some radiation from the ground leaves directly through the “window”.
Radiation also leaves from different heights above the surface.
The average height is about 5Km.
The lapse rate which is completely independent of the radiative effects then determines the surface temperature .

Fine…Everything you say is basically correct. The one thing that you and Postma are leaving out is what I said in my last post…that the average height of 5km is determined by the IR-absorptivity of the atmosphere, i.e., by the greenhouse gases in the atmosphere and clouds. (The technical statement is something like this: At any given wavelength, the height at which most of the radiation escapes to space is given by the height at which the “optical depth” [ http://en.wikipedia.org/wiki/Optical_depth ] of all of the atmosphere above that point to radiation of this wavelength is of order 1. More quantitative details can be found in books on atmospheric radiation, such as Ray Pierrehumbert’s book “Principles of Planetary Climate”.)

John Runberg
May 8, 2011 6:41 pm

I haven’t had time to read all of the comment but I haven’t seen any comment on land-use/botany. Even in the oceans there are plants. Much of the earth is a GREEN body that is converting solar energy into some form of biomass. How many watts does it take to produce a bushel of wheat or a 1,000 board feet of pine lumber or produce a nice turf (or Veg. Garden) at the White House? It isn’t being factored in!

Joel Shore
May 8, 2011 6:55 pm

Ira has explained it here: http://wattsupwiththat.com/2011/05/07/visualizing-the-greenhouse-effect-light-and-heat/#comment-655793
I have explained it here: http://wattsupwiththat.com/2011/05/07/visualizing-the-greenhouse-effect-light-and-heat/#comment-655686
The only way that adiabatic compression could possibly explain the earth’s surface temperature would be if the atmosphere (and / or the earth itself) were undergoing continual gravitational collapse, which we know is of course not the case. Otherwise, such a hypothesis does not even satisfy the First Law of Thermodynamics (basically, conservation of energy): Without substances in the atmosphere that absorb terrestrial radiation, the earth’s surface at its present temperature would be emitting back out into space way more energy than it receives from the sun and hence would rapidly cool down.
The temperature structure as a function of altitude, i.e. the lapse rate, in the troposphere is set by the considerations discussed regarding adiabatic expansion and compression (basically because a lapse rate higher than the appropriate adiabatic lapse rate for the given water vapor content is unstable and leads to convection until marginal stability is restored).
However, this alone does not determine the temperature at the surface. That is determined by consideration of the absorption of the atmosphere of terrestrial radiation (and radiation emitted by the atmosphere), which essentially ends up determining at what altitude the temperature has to be determined via radiative balance between the Earth system (earth + atmosphere) and the sun and space [which for the earth system with its current albedo is ~255 K]. Then, by using the lapse rate, you can get the temperature at the surface.
However, as one adds greenhouse gases, one raises this altitude and hence the required surface temperature. (In the simplest case, this is under the assumption that the lapse rate doesn’t change, whereas the more complicated calculations assume that there is somewhat of a decrease in lapse rate, by the well-understood physics of the moist adiabatic lapse rate in the tropics. This decrease in lapse rate is a negative feedback that is included in all of the climate models.)

jae
May 8, 2011 6:59 pm

IRA: It is getting more and more conspicuous that you are completely ignoring the alternative explanations for the Earth’s temperature. You have completely ignored me and many others who have suggested some other theories to this point, unless I missed something. You need to respond. With some facts. Otherwise, my advice is to shut up.

wayne
May 8, 2011 7:06 pm

Thank you Hockey Schtick !!…
William C. Gilbert’s words are much better than the ones I have been using and now I have someone to attribute them to. He’s right, it is merely not so simple, but understandable, physics.

Joel Shore
May 8, 2011 7:40 pm

wayne says:

William C. Gilbert’s words are much better than the ones I have been using and now I have someone to attribute them to. He’s right, it is merely not so simple, but understandable, physics.

It is clear that Gilbert has never actually read any elementary textbook on atmospheric physics or climate science or he would be aware that the argument about adiabatic lapse rate that he presents is not new to anyone in the field…It is discussed in all of these books. (Alternatively, maybe Gilbert is aware of this but is confident that the readers who he wants to convince have not.)
However, far from his claim that this temperature profile explains the 33 deg temperature difference between what is observed and what radiation balance requires, the fact is that it alone doesn’t explain any of it. You need to consider not only how the temperature varies with height but what then sets the constant that tells you what the absolute temperature is at some height in the troposphere. In the absence of absorption of terrestrial radiation by the atmosphere (and with the other caveats about still having the same albedo and such), that average temperature would have to be 255 K at the surface because of radiative balance and then the temperature would decrease with height at the lapse rate from there.

Joel Shore
May 8, 2011 7:43 pm

jae says:

IRA: It is getting more and more conspicuous that you are completely ignoring the alternative explanations for the Earth’s temperature. You have completely ignored me and many others who have suggested some other theories to this point, unless I missed something. You need to respond. With some facts.

The errors in your explanations have now been explained to you multiple times in this thread. There is no excuse for you to continue to cling to them.

Stu N
May 8, 2011 8:03 pm

Joel:
“The only way that adiabatic compression could possibly explain the earth’s surface temperature would be if the atmosphere (and / or the earth itself) were undergoing continual gravitational collapse, which we know is of course not the case.”
Thank you! I’ve had big trouble trying to explain this to people before. I tried using the example of a bike tire, which heats up when you compress the air but then doesn’t stay hot (even though it stays compressed, but the elasticity of the innertube, analgous to gravity, is not reducing the volume of gas). Success rate with this explanation is surprisingly low.

jae
May 8, 2011 8:10 pm

Joel:
“The errors in your explanations have now been explained to you multiple times in this thread. There is no excuse for you to continue to cling to them”
?? Can you please point me to those explanations, sir? I must have missed them, or else, perhaps, probably, most likely, they were balogne? Come on, fella, provide some tangible thing that I can relate to, instead of your Orwellian insistance that there is, “out there,” something that proves your case/argument/status/being/etc..
WTF?

May 8, 2011 8:32 pm

“The only way that adiabatic compression could possibly explain the earth’s surface temperature would be if the atmosphere (and / or the earth itself) were undergoing continual gravitational collapse, which we know is of course not the case.”
Typical CAGW-practitioner gobbledygook
And the atmosphere does not behave as an enclosed bicycle tire. No wonder the “success rate” with that explanation is so low.
Here’s Dr. Jelbring’s more detailed explanation:
http://www.google.com/url?sa=t&source=web&cd=1&ved=0CBYQFjAA&url=http%3A%2F%2Fruby.fgcu.edu%2Fcourses%2Ftwimberley%2FEnviroPhilo%2FFunctionOfMass.pdf&ei=9V3HTbH4IIP4sAOLzuTqAQ&usg=AFQjCNHTux_zuvY-oHVCehfC6OOmfzUafQ&sig2=p-LjGfmce1A-Vqk5ybrjZg
Show me the exact error’s you claim, citing specific excerpts from his paper.

richard verney
May 8, 2011 8:39 pm

ferd berple says:
May 8, 2011 at 2:24 pm
The solar radiation hitting the earths surface is 184 W/m^2 per Trenberth 2009 or 198 W/m^2 per Trenberth 1997. The surface albedo must be applied to this to get the net gain. Back radiation adds another 333 or 324 W/m^2 respectively.
There seems to be a whole lack of common sense going around these days.
So, according to T, the earth receives 184 w from the sun, radiates some of this back outwards, where some of it is absorbed by the atmosphere and re-radiated back to earth, where it provides 333 w.
In other words, 184 w ends up as 666 w, because the back radiation is omni-directional at at most 50% reaches the earth……………………………………
///////////////////////////////////////////////////////
There does appeart to be a complete lack of commonsense in these figures.
If these figures were correct (ie., back radiation is approximately twice the power of solar), why are we wasting time with trying to capture solar energy rather than back radiation energy?
Why don’t we construct a large array of mirrors collecting and focusing this back radiation much like the Californian Solar power station: see
http://greenwombat.files.wordpress.com/2010/01/esolar-power-plant.jpg
This not only would produce more energy (viz about 330 cf about 190 W/m^2; back radiation cf solar), it would additionally solve the storage problems inherent with present day green energy projects since back radiation is available day and night etc.
The answer is simple, either this back radiation does not exist, or if it does exist it is incapable of doing any sensible work (ie., it lacks sensible energy).
The lack of any serious research into collecting and utilising the back radiation energy source strongly suggests that leading energy scientists regard the back radiation set out in Trenberth’s energy diagrams to be complete and utter nonsence!
Anyone who supports Trenberth’s diagram and considers that the back radiation theory is correct should be asked to explain why we are not utilising this fanastic energy source. After all, if it existed it should be the main focus of green energy projects.

May 8, 2011 8:45 pm

“The only way that adiabatic compression could possibly explain the earth’s surface temperature would be if the atmosphere (and / or the earth itself) were undergoing continual gravitational collapse, which we know is of course not the case.”
Typical CAGW-practitioner gobbledygook
And the atmosphere does not behave as an enclosed bicycle tire. No wonder the “success rate” with that one is so low.
Here is a more detailed explanation from Dr. Jelbring:
http://www.google.com/url?sa=t&source=web&cd=1&ved=0CBYQFjAA&url=http%3A%2F%2Fruby.fgcu.edu%2Fcourses%2Ftwimberley%2FEnviroPhilo%2FFunctionOfMass.pdf&ei=9V3HTbH4IIP4sAOLzuTqAQ&usg=AFQjCNHTux_zuvY-oHVCehfC6OOmfzUafQ&sig2=p-LjGfmce1A-Vqk5ybrjZg
Show me exactly where he is in error, citing exact quotes from the paper, without any hand waving.

jae
May 8, 2011 8:45 pm

Luuuucy?????? Ira?????? we are missing some comments here. WTF? Are you THAT desperate????

May 8, 2011 9:05 pm

While you’re at it, Dr. Noor van Andel’s paper also explains the “GHE” based upon the adiabatic lapse rate. Again, show me exactly where he is in error, citing exact quotes from the paper, without any hand waving.
http://climategate.nl/wp-content/uploads/2011/02/CO2_and_climate_v7.pdf

UncertaintyRunAmok
May 8, 2011 9:13 pm

Wow. Once again, misconceptions abound on both sides.
Ira, just because it is the only seemingly rational suggestion that has been posited publicly does not mean that it actually describes the physical process accurately. If it did, the term “backradiation” would not have had to have been coined in the first place.
It is a simple matter to increase the absorption in a gas above the level of the incident energy, without ever having to increase the relative concentration of the gas, and is done on a daily basis in my field, and has been for many decades, but we do not call it “backradiation”.
Some (no, I don’t know exactly how much) of the reflected solar energy has already been absorbed by the atmosphere before being reflected. The statement that it contributes nothing to the atmospheric temperature profile is thus at least partly incorrect.
So Ira, if you really are sincere about wanting to know where the 33K difference comes from, you could start by calculating the Mie backscatter cross section of a 10um dielectric sphere, and I would suggest calculating it with a 9.25um or 10.59um em wave incident on it. I suggest those 2 wavelengths because measurements have already been done with them, and you will have something to compare the calculated values to, although the measured values were normally given in steradians, which can also be calculated.
You might also consider how it is possible to treat two entities that are physically coupled (think about it) as if they were two separate “blackbodies” radiating “at” each other.
Cheers.
BTW, N2 DOES have resonance lines in the solar UV region.

David
May 8, 2011 9:21 pm

A warm atmosphere will emit radiation. Some of it will be backradiation.
Even without IR absorption the atmosphere will warm.
http://earthobservatory.nasa.gov/Features/EnergyBalance/page6.php
Convection.
Evaporation.
Incoming energy absorption (visible light).
With this in mind, and accepting the backradiation mechanism, how can the “greenhouse IR” component be ascribed the full 33 C difference?
Surely it can’t be?

martin mason
May 8, 2011 10:01 pm

From what I see the proponents of the Greenhouse theory simply keep stating it as fact without explaining the observed phenomena that seemingly falsify it such as.
– You cannot heat a hot body from a colder body which you have to be able to do.
– The temperature at the surface and at height can be explained without recourse to radiative theory which to me means that the temperature is determined by the fact that we have an atmosphere not necessarily because we have an atmosphere with GHGs. The atmospheric column is nothing like a bicycle tyre in physical terms, that is a red herring.
– In the past when we have had multiples of current CO2 levels we have not had runaway warming.
– There is patently no runaway warming now while CO2 levels ramp up inexorably. There is no runaway warming on venus.
– If back radiation is so powerful and it is a heat source, why don’t we use it?
It’s the answers to the simple things that are important not the circular academic arguments. The consensus view is that AGW is happening because of a trace GHG, the only problem is that there is no unequivocally correct theoretical or observational data that supports it.

Phil.
May 8, 2011 10:39 pm

John of Kent says:
May 8, 2011 at 3:09 am
Ira Glickstein does not seem to understand how matter interacts with radiation.

On the contrary he seems to understand it fairly well, you however appear not to!

Martin Lewitt
May 8, 2011 10:58 pm

Wayne,
“The air above the cloud tops will be cooler than normal shielded by the water mass in the clouds and the air between the surface and the cloud base warmer than normal.”
The problem is for those doubting back radiation to explain the greater night time head loss in dry clear skies vs humid clear skies. You don’t seem to be able to do it without clouds, are you doubting the humid vs dry clear sky difference exists?
BTW, no huge lens covering a wide enough range of the infrared spectrum exists to conduct your hypothetical experiment.

Darren Parker
May 8, 2011 11:27 pm

How does this fit in with the Holographic Universe theory?

tallbloke
May 9, 2011 12:07 am

According to the more reasonable reconstructions of past temperature (Moberg, Loehle) the surface temperature has been rising since 1700. This is 230 years longer then co2 has been rising to any great degree.
What is Ira’s explanation for this earlier warming?
The Earth is a big heat engine. Most of the effective solar heating of the ocean occurs in the tropics, whereas most of the effective radiation of heat to space occurs near the poles. Most of the energy between the equator and temperate latitudes is shifted by the ocean. From the temperate latitudes to the poles, most of the energy is shifted by the atmosphere. This movement of energy is achieved mainly by convection of one sort or another, and since the temperature of the ocean is little affected by back radiation, changes in co2 concentration can’t have much effect.
Why does Ira refer simply to the Earth’s surface as an amorphous entity instead of taking into consideration the fact that 70% of it doesn’t absorb much energy from back radiation?
over 90% of downwelling radiation striking the ocean surface has been emitted a kilometre or less above it, because of re-absorption and re-emission. Since this re-emitted radiation is travelling at any random angle wrt to the ocean surface, most of it is travelling at angles which will either miss the surface or be reflected by it. The back radiation which is entrained into the ocean bulk by wave action is minimal, because wave action subducts water which is well below the level back radiation can penetrate to (little more than its own wavelength). Most of the radiation measured as coming from the ocean surface is actually either been reflected or is being emitted by molecules evaporated from the ocean surface.
Since the ocean surface temperature changes precede surface air temperature changes by several months, and since the top two metres of ocean contain as much heat capacity as the entire atmosphere above it, it is clear that surface temperature and atmospheric temperature is strongly influenced by the ocean, which is heated by the sun, not by back radiation.
Solar variation affects albedo, and that affects how much solar energy the ocean absorbs. This is why the number of sunshine hours correlates with surface temperature over the last 130 years much more closely than co2 level.
http://tallbloke.files.wordpress.com/2010/06/soon2009.jpg
In my opinion, surface temperature variation is due to the variation of the Sun. Co2 is along for the ride.

May 9, 2011 12:26 am

Why is “the Petty black body curves peak at a longer wavelength than the Carleton spreadsheet”?
As dr.bill said, this is because the Planck equation is actually an integral. When you change the x-axis, the peak actually does shift. I have written a full explanation at
http://mc-computing.com/Science_Facts/Blackbody/Blackbody_Equations.html
There is also a plot showing the difference in the shape of the two 300K curves and their peaks.
biological evolution has “tuned” our visual system to be most sensitive where the light energy is maximized
Not quite, that is where the intensity per wavelength interval is brightest, not where the energy per frequency interval is largest.
This demonstrates why people who believe in Global Warming always plot spectra versus wavelength. When you want to analyze the energy in the system, all plots should be versus frequency (normally expressed as a wavenumber) to show an honest plot. In that case, most of the energy from the Sun is in the IR. When plotted versus wavelength, the plots incorrectly imply that most of the energy is in the visible and UV spectra.

Bryan
May 9, 2011 12:52 am

The one big attractive feature of the “greenhouse theory” is its amazing elasticity.
The climate getting warmer is a sure proof of CAGW
But then so is the climate getting colder, say some IPCC advocates.
Joel Shore has refined this even further to say Postema is a believer, only Postemo hasn’t quite realised it.
Perhaps with the same elasticity Joel will prove that Gerlich and Tscheuschner also are really IPCC advocates.
Joel goes on to say of me;
…..”He has absolutely no serious desire to discuss the science”…..
Quite the opposite Joel;
On page 9 of your co-written Halpern et al comment paper you state that the Stephan Boltzman Law can be used to work out the thermal energy between atmospheric shells.
I brought your attention to it in my post above;
….. ” 3. Has the Stephan Boltzmann equation applying to gases.”……
Its no good trying to weasle your way out of it.
Your comment was printed in a specialist Journal with readership almost entirely of Physicists.
They would have no problem understanding your point.
Do you still maintain this position or will this be your third major repudiation of this notorious document.
In fact you should perhaps repudiate the document in its entirely!

May 9, 2011 12:52 am

Ira, I’d really like some of the “back radiation” now. Just stepped outside my house in Kamloops BC and pointed an IR thermometer straight up into the sky and got a temperature reading of -10 F. When I pointed the thermometer horizontally down the street, got 34 F. Pointing it at the outside of my house gave a temperature of 44 F as did pointing it at my driveway. The airport temperature now is given as 48 F. The altitude of the airport is about 200′ lower than where I live. I should note that the sky is perfectly clear at this time with no clouds in sight (just wish it would do this in the daytime).
The surface of the earth heats up considerably during the day and this heat is released during the night. Of course having heat buffers of variable capacity greatly complicates models. The -10 F temperature that I’m seeing with my IR thermometer is the average temperature of the atmosphere from 1600′ above sea level to space. What portion of this would be related to “greenhouse gasses” and what portion is the result of heat flux from sun-warmed surface and air to space?

wayne
May 9, 2011 1:16 am

Martin Lewitt, you are right, I do not “believe” in “back radiation”, just plain radiation as taught in physics.
Clouds make warmer nights because the clouds are usually warmer than the normal air temperature at that altitude and therefore the surface’s rate of loss by radiation upward will be less leaving you with a warmer than normal night. It is also that same radiation keeps those clouds warmer than te air at that altitude would normally be. Just using Stefan-Boltzmann will get you very close if you know the temperatures and emissivities involved. There are even rare cases when the clouds are actually warmer than the surface and then the clouds are radiating downward to the surface and also to space but as you should see, this condition is short lived since it has cooler on both sides unless continually replenished with warmth to maintain the temperature differential.
BTW, that experiment does not require all frequencies to be valid. I’ll just leave it for each here to decide it’s merits, sounds like most here can handle high-school science correctly.

Vince Causey
May 9, 2011 1:29 am

From the comments here, a lot of people seem to believe that the greenhouse effect cannot exist because it violates the 2nd law of thermodynamics. To my shock, I am finding myself agreeing with Joel Shore on the elementary physics. I have just finished reading the Postma essay, and it contains the same strawman argument that crops up every time this matter is discussed.
Postma described a black body warmed by a heat source and claimed that you cannot make it warmer by reflecting that emitted radiation back. Whilst acknowledging back radiation exists, he asserts that this energy cannot cause the black body to increase in temperature. He states that a body cannot raise its own temperature. Well of course not. The reason this is a strawman fallacy is because nobody is making that claim. The claim is that the back radiation reduces the rate of cooling of the earths surface because the net outflow in watts is slightly less than it would have been without the back radiation. It is the sun that causes the warming because energy is now coming in faster than it is leaving, so the temperature will rise until a new equilibriumis reached.
An analogy is a tub with a ‘V’ cut near the top. The higher the water level is above the ‘V’, the faster it flows out. This is an analogy of the SB law of blackbody radiation. Suppose the sun is represented by a running tap. The water will rise above the ‘V’ just enough so that the rate of input is equal to the rate of overflow. Let us now represent the back radiation by blocking some of the ‘V’ which represents the reduced outflow of radiation from the earths surface. The water level (temperature) will now rise such that the outflow increases to balance the inflow. If you turned the tap off (remove the sun from the climate system), the water level (temperature) would start to decline just as you would expect from thermodynamics.

Geoff Sherrington
May 9, 2011 1:34 am

Ira,
Here is a schematic from a search, nothing special, merely from the first site that had a lapse curve, month of June, 45 degrees North latitude.
http://i260.photobucket.com/albums/ii14/sherro_2008/Lapse.png?t=1304929624
Source was Bigg 2005, blog was http://scienceofdoom.com/2010/04/24/tropospheric-basics/
The coldest temperature measured on earth is about -89 deg c at Vostok. This is colder than any part of the lapse curve below about 75 km above Earth.
How can it get so cold? What stops heat getting to Vostok?

RJ
May 9, 2011 1:41 am

“Anyone who supports Trenberth’s diagram and considers that the back radiation theory is correct should be asked to explain why we are not utilising this fanastic energy source. After all, if it existed it should be the main focus of green energy projects.”
“- If back radiation is so powerful and it is a heat source, why don’t we use it?”
Good questions. Although what about this free energy oven. Does it qualify?
http://www.slayingtheskydragon.com/en/blog/111-a-pictures-worth-a-1000-words#comments

Allan M
May 9, 2011 2:04 am

Joel Shore says:
You clearly do not know the basis for the Second Law, which is statistical physics.
The basis for the second law (and the first) is careful and accurate measurement, mostly paid for by wicked 19th century capitalists, who wanted to know how to get more from their steam engines, and also how to avoid being sold snake oil.
There is no need for a statistical ‘fudge’ unless you wish to introduce special pleading for “IPCC Approved” photons.
If I were to dump a billion microscopic ice particles into a tank of hot water (di-hydrogen monoxide), your statistical approach would insist that a few of them may have warmed the water (hydrogen hydroxide), especially using a computer model.

Robert Stevenson
May 9, 2011 2:16 am

I use spreadsheets to calculate Earth IR emissions and absorption by CO2. Using mean beam lengths and CO2 partial pressures, I use PcL charts by Hoyt C hottel to obtain the emissivity of CO2 in the atmosphere. For current CO2 partial pressure or concentration I calculate a distance of 3600 m of traverse through the earth’s atmosphere by ‘IR radiation’ before the absorbable IR is ‘filtered out’ and reduced to zero. I then calculate that by doubling the atmospheric CO2 to 700ppm or so the ‘absorption distance’ is reduced to 2000 m but with no additional IR absorption by CO2. Therefore I conclude no global warming from increasing atmospheric CO2.

Bomber_the_Cat
May 9, 2011 2:53 am

Ira,
“But, that still does not explain why the Petty black body curves peak at a longer wavelength than the Carleton spreadsheet and other graphics on the Internet”
Maybe I could help, or perhaps just confuse you further.
If you plot the radiant energy against wavelength for blackbody around 287K, then you get a peak intensity at about 10 micron. The graphs reproduced from Petty show the peak intensity at about 18 micron. This apparent anomaly arises because the vertical axes of the 2 graphs use different units.
The Plank distributions plotted against wavelength (the way I like to see them) have a vertical scale of ‘Watts per Square metre PER MICRON’. The graphs shown by Petty are plotted against wavenumber (not wavelength) and the vertical axes have units of ‘Watts per Square Metre per Steradian PER WAVENUMBER’.Note: graphs plotted in these different ways are NOT simply mirror images of each other. [Forget the Steradian bit, this is just a multiplying factor which doesn’t effect the peak or shape]
Say that the peak radiation on the wavelength plot occurs at 10.5 micron. Because the energy is plotted per micron it represents the total energy between 10 micron and 11 micron. In terms of wavenumber, 10 to 11 micron is equivalent to a range of 909 to 1000 per cm. So the energy for the 10.5 micron peak has to be dispersed over 91 different wavenumbers on the wavenumber plot. If we consider another point on the wavelength plot, say between 18 and 19 microns, this is represented by a wavenumber range from 526 to 555. So the energy around 18 microns has only to be shared between 29 wavenumbers whilst the energy around 10 microns has to be shared over 91 wavenumbers. As a consequence, the peak intensity no longer occurs at 10 micron, it now occurs at 18 micron. That is why the two methods of plotting give different peak intensities.
Did that help, or just confuse everyone further?

Martin Lewitt
May 9, 2011 3:02 am

Boris Gimbarzevsky,
Thanks for sharing your thermometer readings It would be interesting to know if on clear nights, sky readings taken at the same ambient temperature but different absolute humidities varied due to increased water vapor greenhouse effect.

May 9, 2011 3:25 am

Dishman says:

If molecules follow ballistic trajectories, the minimum KE will occur at zenith and be higher at all other points.
A short MFP just means that equipartitioning will be rebalanced (on average) every MFP.
Assuming air (a diatomic gas with 5 degrees of partitioning), for a given delta-h, the thermal energy will be reduced by 1/5th of the energy required to lift molecules by that delta-h.
My point is that Objection #2 is not, in fact, correct.

That may be your point, but nothing you say backs it up. You say “the thermal energy will be reduced by 1/5th of the energy required to lift molecules by that delta-h.”
A quick calculation shows that the average molecular speed, assuming constant g, is sufficient to lift a molecule about 11 km. The mean free path of a molecule in air is about 65 nanometres. Dividing these, we see the energy needed to lift the molecule by your delta-h is about 1.7*10^-11 of its total energy. In other words, diddly squat. Gravitational effects make no difference to the result that could possibly be measured.

Smoking Frog
May 9, 2011 4:52 am

Charles Nelson said: Gotcha Henry! He thinks CO2 is like a toxin in the atmosphere! An amazing toxin which is NOT TOXIC at 380ppm but suddenly BECOMES TOXIC at 400ppm. There in a single post is the fuzzy thinking of the Warmists! Fantastic. Textbook example of ‘Carbonaphobia’!
I don’t read him that way. I think he’s making the point that the tiny size of the fraction, by itself, doesn’t tell us anything. If he’s making the point that you think he’s making, he’s doing a poor job of it.

May 9, 2011 5:10 am

Ira,
you do not consider conduction and convection, nor non-GHG absorbers/reflectors of radiation nor spatial issues. Simply saying that the difference in temperature between a perfect blackbody and the incoming radiation is due to “greenhouse” effects is disingenious. You can convince yourself by a thought experiment: would the atmosphere really be at zero degree Kelvin were it not for GHGs? I’d say obviously not and hence your simple difference doesn’t apply.
best regards,

Smoking Frog
May 9, 2011 5:21 am

Martin Mason said – If back radiation is so powerful and it is a heat source, why don’t we use it?
Because the earth’s surface is already at the temperature to which the back-radiation has raised it.

Dave Springer
May 9, 2011 5:35 am

Allan M says:
May 9, 2011 at 2:04 am
“If I were to dump a billion microscopic ice particles into a tank of hot water (di-hydrogen monoxide), your statistical approach would insist that a few of them may have warmed the water (hydrogen hydroxide), especially using a computer model.”
Some actually will warm the water. Things get more than passing strange when you leave to domain of bulk matter and look at what’s going on with individual atoms and molecules. Some of the water molecules in your tank of hot water are colder than ice and some of the water molecules in the ice are more than boiling hot. Those molecules are few and far between so for most practical considerations they are ignored. Temperature is an artifact of motion. In bulk matter where the molecules are all bumping into each other chaotically some small fraction will happen to be moving very fast and some very slow. A good analogy would be what happens to a set of billiard balls after the break. Initially all the motion is in the cue ball but when it hits the rack the energy of its motion is distributed amongst all the balls but not equally. The cue ball could stop dead in its track only to get whacked back into motion by one of the other balls a moment later or if it’s a glancing blow the cue ball might remain the fastest mover for a few moments. But because energy can be neither created nor destroyed the total kinetic energy in all the balls on the table will always equal the initial energy in the cue ball. Temperature is analogous to the total kinetic energy of all the billiard balls.

Dave Springer
May 9, 2011 5:49 am

Robert Stevenson says:
May 9, 2011 at 2:16 am
“Therefore I conclude no global warming from increasing atmospheric CO2.”
This was disproven by expermental physicist John Tyndall over 150 years ago. Back radiation from gases that absorb IR is quite real. Suggest you read the original work here (it’s free):
http://www.archive.org/details/heatconsideredas00tyndrich

Dave Springer
May 9, 2011 5:57 am

RJ says:
May 9, 2011 at 1:41 am
“Anyone who supports Trenberth’s diagram and considers that the back radiation theory is correct should be asked to explain why we are not utilising this fanastic energy source. After all, if it existed it should be the main focus of green energy projects.”
“- If back radiation is so powerful and it is a heat source, why don’t we use it?”
Good questions. Although what about this free energy oven. Does it qualify?

We do use it all the time. One example is the smudge pots used to limit freeze damage in orange and avocado groves. The heat of the smoke from the smudge pots is inconsequential. The effect comes from the black carbon particles in the smoke which absorb upwelling IR from the ground and send a portion of it downwards and sideways. It works as an insulator. This is the same mechanism by which CO2 molecules insulate. Anyone who contests this is simply wrong and needs to take an introductory class in physics.

ferd berple
May 9, 2011 6:12 am

“The water will rise above the ‘V’ just enough so that the rate of input is equal to the rate of overflow. Let us now represent the back radiation by blocking some of the ‘V’ which represents the reduced outflow of radiation from the earths surface. ”
The problem is that Mr “T” says the energy from the sun is 184 watts and the amount being back radiated is 333 w. In other words, you are blocking more water than the maximum flow of the outlet, which is physically impossible in you analogy.

Dave Springer
May 9, 2011 6:31 am

Boris Gimbarzevsky says:
May 9, 2011 at 12:52 am
Ira, I’d really like some of the “back radiation” now. Just stepped outside my house in Kamloops BC and pointed an IR thermometer straight up into the sky and got a temperature reading of -10 F.

You’d like more back radiation. You’re getting quite a bit of it now. The temperature of the cosmos is 3K and if it weren’t for back radation when you point your IR thermometer at the sky on a clear night you’d get a reading of -454F.
Back radiation doesn’t come from just CO2. When a CO2 molecule absorbs upwelling IR it thermalizes surrounding non-GHG molecules via kinetic transfer. A common fallacy is that CO2 molecules emit a photon of the same frequency as that absorbed. That’s true in a very thin gas but in a cold dense gas the excited CO2 molecule almost instantly bumps into a neighboring molecule (likely a nitrogen molecule in the case of the earth’s atmosphere) and transfers some of its newly acquired energy to the neighbor.

ferd berple
May 9, 2011 7:05 am

Ira Glickstein, PhD says:
May 8, 2011 at 4:26 pm
“If we then placed that planet in an Earth-like orbit around the Sun”
Thanks Ira, what I read you to be saying is that if the atmosphere was pure N2O2, without any H2O, that it would have no effect upon the surface temperature of the earth. That since N2O2 do not absorb longwave radiation, then they cannot keep the earth warm.
I’m saying that this is simply wrong. I’m saying that an atmosphere of pure N2O2 would change the surface temperature of the earth significantly as compared to an earth without an atmosphere.
This is the calculation that must be undertaken first. How much of a difference would we see in the surface temperature of the earth due to N2O2.
The next calculation is the effect of phase change of H2O in the atmosphere. Not the GHG effect of H2O, but the effects of evaporation and condensation in transporting heat vertically.
Then, when you have completed those two calculations, what you have left is a candidate for the GHG effect.
From what I’ve seen, when you apply these calculations to real models, such as venus, earth and mars, then the first calculation, the effects of the atmosphere without regard to the composition provides nearly the same amount of heating on venus, earth and mars.
These are real observations, not theoretical models. And, observation trumps models, no matter how correct we think our models are. I don’t see this calculation in your work. I don’t see where you calculate the effects of a N2O2 atmosphere that does have any H2O or CO2.
As to why the atmosphere of venus, earth and mars show the same amount of heating regardless of composition, that is a matter of theory. Put 10 experts in a room and you will have 20 opinions. None of these opinions will change the facts.
The atmosphere of venus, earth and mars show a significant heating effect that is independent of the composition of GHG and this is not accounted for in your model.
As to why this is, my theory is that it is due to compression. That the molecules i the atmosphere all have roughly the same amount of energy due to convection, but since there are many more of them per cubic meter at lower altitudes, there is more energy per cubic meter, which we register as increased temperature.
For example, we could have 1 air molecule per cubic meter. When we went to measure the temperature of this meter of air, we would average out the energy of this one molecule over the volume and end up with a low temperature.
Now, if we were to have a whole lot more molecules per cubic meter, all will the same energy as our first example, we would see this as an increase in temperature. However, in both examples the molecules are at the same energy level, yet the samples taken from lower down in the atmosphere will appear to have a higher temperature.
However, this may not be the reason for what we observe. And it doesn’t really matter. What does matter is that FROM OBSERVATION the atmospheres of venus, earth and mars show similar warming at similar pressure, independent of their levels of CO2.

May 9, 2011 7:11 am

Ira,
Here’s why your bicycle tire analogy fails: you instead need a leaky tire that has to be inflated continuously. Would the air in that situation remain hot? Yes
The open atmosphere essentially acts as a continuous ‘air conditioner’ or pump. Cold air continuously descends to the surface and is heated by the surface as well as from compression. That air then ascends and expands and cools until the process repeats. Thus, adiabatic compression is able to maintain a higher surface temperature on a continuous basis, the so-called GHE.

Bomber_the_Cat
May 9, 2011 7:14 am

We seem to have an unusually large number of comments from the anti-science brigade today who seem to have invented a whole new branch of previously unknown physics.
Imagine an ice cube, at zero degrees Celsius, alone in the vacuum of space (which is at, say, absolute zero). What happens to that ice cube? Well, it radiates away heat (as everything above absolute zero does) and eventually cools to absolute zero itself, asymptotically. Now imagine an another ice cube, this time at -10 degrees Celsius, being placed near to the original ice cube. This colder ice cube also radiates heat; some of it will impact the first ice cube and ‘warm’ it. The original ice cube still continues to cool, but not as fast as it was doing before because it now has a heat input which was not there in the first case. The colder ice cube therefore helps to keep the original ice cube warmer. Of course, it never gets above its starting value of zero degrees Celsius, but that is how the presence of cold objects can keep hot objects warmer i.e. they are still warmer than what would be in their place if they were not there (in this case the cold vacuum of space at absolute zero).
The 2nd Law of Thermodynamics? – Yes – it fully supports this – no contradictions. .

Joel Shore
May 9, 2011 7:31 am

martin mason says:

From what I see the proponents of the Greenhouse theory simply keep stating it as fact without explaining the observed phenomena that seemingly falsify it such as.
– You cannot heat a hot body from a colder body which you have to be able to do.

In any model of the greenhouse effect, the net heat flows are from the warmer earth to the colder atmosphere. The atmosphere is not heating the earth itself…what it is doing is slowing down the cooling of the earth (for a given surface temperature). Since the steady-state temperature of the earth is determined by the balance between what it receives from the sun and what it emits back out into space, an IR-absorbing atmosphere will in fact cause the earth’s steady-state temperature to be higher than it would be if the atmosphere did not absorb IR. In a colloquial sense, one might call this “heating” the earth, although it is best to avoid such language since it leads to confusion and to instead state clearly what the effect of the atmosphere is, as I have explained above.

– The temperature at the surface and at height can be explained without recourse to radiative theory which to me means that the temperature is determined by the fact that we have an atmosphere not necessarily because we have an atmosphere with GHGs. The atmospheric column is nothing like a bicycle tyre in physical terms, that is a red herring.

No…Such a hypothesis does not even satisfy energy conservation! What can be explained without recourse to GHGs is the lapse rate in the troposphere. What can’t be explained is what the temperature at any altitude is. It is as if you told me the slope m of a line of the form y = mx + b and claimed that I could now compute y for any x. In fact, I would need the value of b to do so. The value of b is in essence what is determined by the absorption of IR radiation by the atmosphere.

– In the past when we have had multiples of current CO2 levels we have not had runaway warming.
– There is patently no runaway warming now while CO2 levels ramp up inexorably.

This is a strawman argument. (Almost) noone is claiming that there will be “runaway warming”. [Hansen is talking about the possibility under certain circumstances and has specific arguments as to why the current case may be different from what the earth has experienced in the past, so let’s leave him out of this.]
What we are talking about is a climate that is quite sensitive, but not unstable, to perturbations. See, for example, here for a discussion of what the paleoclimate record tells us: http://www.sciencemag.org/content/306/5697/821.summary

There is no runaway warming on venus.

Really? You think the temperature on Venus is very pleasant? Did the runaway go on forever? No…An object that is linearly unstable will tend to find a point where stability is restored. However, a runaway did occur to produce Venus’s current very hot climate. Fortunately for earth, we are not as close to the sun and thus not subject to the runaway scenario that occurred on Venus.

– If back radiation is so powerful and it is a heat source, why don’t we use it?

How are you proposing that we harness it? As has already been noted by Smoking Frog, the back-radiation is already heating the earth. (Because most objects tend to be almost perfect blackbodies in the mid- and far-IR, almost all of the back-radiation that is received by the earth is absorbed.) Also, the back-radiation that we receive is diffuse (coming from all different angles) and so it cannot be focused in the same way that solar radiation (which is approximately from a point source) can. Furthermore, since the energy of an individual photon is proportional to its frequency, the back-radiation…unlike solar radiation…cannot be harnessed by photovoltaic devices, which rely on having photons of sufficient energy to cause electronic transitions (in particular, energies much larger than thermal energy of ~kT where K is Boltzmann’s constant). Finally, although the back-radiation we receive from the atmosphere is, in the global average (e.g., day and night at all latitudes), larger than the radiation we receive directly from the sun, the amount of radiation that we receive from the sun when it is actually shining and close to overhead is a fair bit larger.

Dave in Delaware
May 9, 2011 7:37 am

Ira – regarding your summary comment 4) at May 8, 2011 at 7:51 pm
my comment – NO, the atmosphere does NOT emit LWIR across a distribution of wavelengths like a blackbody , see my earlier comment at Dave in Delaware says: May 8, 2011 at 7:00 am
Ira Glickstein, PhD says:
“4) As I understand it, the ~15μm radiation from the Surface to the Atmosphere is absorbed by H2O and CO2 molecules which, when excited, bump into nitrogen and oxygen and other air molecules, and heat the air. The result is emission of long-wave radiation at a range of wavelengths (not just in the ~15μm region, but including the ~7μm and ~10μm regions) in random directions, some of which finds its way back down to the Surface and is re-absorbed. That conversion of a portion of the ~15μm radiation into other wavelengths may explain why there is a “bite” out of the ~15μm radiation that seems to be missing from the radiation out to Space. ”
——————————————
This is the part of your comment I disagree with –
“The result is emission of long-wave radiation at a range of wavelengths (not just in the ~15μm region, but including the ~7μm and ~10μm regions) in random directions, …”
My reply – If CO2 absorbs at 15μm, it emits at 15μm. The Nitrogen and Oxygen in the atmosphere neither absorb nor emit in the LWIR wavelengths of interest. That is why there is an Atmospheric Window – the atmosphere neither absorbs nor emits in that 10μm region.
Think about the satellite-looking-down readings – the 10μm atmospheric window corresponds to the surface temperature. Over the Sahara, the 10μm readings show a surface temperature around 320 K. Over the Mediterranean, the 10μm readings show a surface temperature around 280 K. Over the Antarctic, the 10μm readings show a surface temp just below 220 K. If the atmosphere (mostly N2 and O2) emitted LWIR like a blackbody, the atmospheric window readings would look like some middle atmosphere temperature – not like the surface. There is an Ozone band in the middle of the atmospheric window, I expect from ozone in the upper atmosphere; if N2 was emitting blackbody LWIR there should be other N2 bands in the middle of the window, and there are not.
———————————-
Ira comment – “That conversion of a portion of the ~15μm radiation into other wavelengths may explain why there is a “bite” out of the ~15μm radiation that seems to be missing from the radiation out to Space. ”
My reply – Recall that the satellite-looking-down readings are Emissions; their temperature tells us something about where those emissions originated. Looking at the CO2 band satellite 15μm readings, the temperatures are around 220 K and nearly the same for the Sahara, the Mediterranean and the Antarctic. That looks to me like the Tropopause. It does not necessarily tell me how the energy got to the Tropopause, so I am not sure I could estimate a ‘bite’ value. That emitted energy could have arrived from lower atmosphere CO2 emissions, or from atmospheric lapse type convection, or possibly from the tops of thunderstorms.
My comments are referenced to these satellite IR spectra at
http://mensch.org/5223/IRspectra.pdf
Slide (c) Western Tropical Pacific is particularly interesting because it compares ‘clear’ with ‘Thunderstorm Anvil’. Note that the CO2 band temperature is identical in the clear and thunderstorm curves. Also note that the atmospheric window 10μm region temperatures are MUCH DIFFERENT for clear vs thunderstorm anvil. This suggests to me that while the atmosphere does not emit like a black body, the thunderstorm (liquid water) does.
Best regards to Ira – I hope my comments are constructive and instructive, and will help your ‘mental models’ and descriptions.

Joel Shore
May 9, 2011 7:56 am

Allan M says:

The basis for the second law (and the first) is careful and accurate measurement, mostly paid for by wicked 19th century capitalists, who wanted to know how to get more from their steam engines, and also how to avoid being sold snake oil.
There is no need for a statistical ‘fudge’ unless you wish to introduce special pleading for “IPCC Approved” photons.

Experiment did provide the evidence for the Second Law. However, our modern (20th century) understanding of where the law comes from is based on statistical physics. To dismiss an entire field of physics that has successfully explained a huge wealth of empirical observations as “a statistical ‘fudge'” because one implication of it does not line up with your ideology is bizarre.
This is essentially the project that Claes Johnson, one of the “Slaying the Sky Dragon” authors, has embarked upon. He wants to base things on the bizarre proposition that an artifact of numerical calculations of the differential equations actually governs the universe instead. The irony is that although he claims to have gotten rid of “back radiation” by coming up with a different interpretation of the terms in the equation governing radiative transfer between two bodies, he has in fact done nothing to change any numerical result using those equations…including all of the numerical results that support the existence of the greenhouse effect! What he has done is snookered a few people who want to believe the nonsense that he is peddling.

If I were to dump a billion microscopic ice particles into a tank of hot water (di-hydrogen monoxide), your statistical approach would insist that a few of them may have warmed the water (hydrogen hydroxide), especially using a computer model.

It depends what you mean by “microscopic”. It doesn’t take a very large particle size before the statistics make it vastly improbable for such a thing to occur. But, yes, if you particles were truly only a few molecules large, then you could detect behavior contrary to the 2nd Law. (Of course, the distinction between “ice” and “water” begins to lose meaning at such a small length scale since phases of matter are themselves a macroscopic concept.)

Dave Springer
May 9, 2011 7:57 am

Circa 1850 Tyndall experiments with greenhouse gases in a nutshell.
Radiation flow:
heat_lamp -> -> adjustable shade -> salt crystal window -> mirrored tube filled with test gas -> salt crystal window -> adjustable shade -> galvanometer
Tyndal ran literally thousands of experiments with different gases and mixtures at different pressures, different infrared light sources, and different lengths of tubes. His setup took up a lot of space but I marvel at the cleverness he used to increase accuracy and precision using the technology of the day. His analog gear was so sensitive he read the reading from the galvinometer through a telescope because getting a warm body anywhere near the experiment mucked it up.
His best most stable lamp was a black painted face of a vessel filled with boiling water. His galvanometer response is not linear so the primary purpose of the adjustable shades was keep the radiative energy input to the face of the galvanometer in a range where you got maximum needle deflection from minimal change in radiation.
In lectures he used to take his galvanometer to the podium in the hall. He’d point it at a blank wall on the other side of the hall and ask someone from the audience to walk over to that wall. The galanometer needle would move dramatically when a person entered the scene.
Anyhow Tyndall found a great many gasses, most famously water vapor, that would dramatically lower the galvanmeter reading when present in the tube versus the same mixture without it. He also used some rather ingenious ways of processing his gasses prior to filling the tube with them including means of completely drying a sample of the atmosphere.
If one wants to dispute the concept of back radiation one must first explain why Tyndall’s galvanometer would read progressively lower as the absolute humidity of the atmospheric sample in the tube rose. No one will. Every attempt is fatally flawed in some way. Tyndall didn’t discover anything that wasn’t already predicted by the theoretical physics of the day. He confirmed the theory via experiment or in other words he was doing science the way it’s supposed to be done.
Tyndall did discover some things that no one had worked through on paper at the time. One notable thing he discovered was the non-linear absorptive response as the partial pressure of an IR-absorbing gas went up. He found it to be linear at the lowest partial pressures then progressing to exponential or in other words when it comes to IR absorbing gasses increasing the amount of it is a case of diminishing returns after a certain amount.
This is why the climate boffins talk about CO2 doublings and temperature rise. At the current atmospheric CO2 partial pressure its absorptive response is in the exponential range. The linear response range is all below the first 100ppm. It’s sort of like people picking low hanging fruit in an apple orchard. They can all pick as fast as they can for a while but as the number of people picking increases at some point they’ll be competing with each other for the same low hanging fruit. So putting more pickers in the orchard will always increase the rate at which fruit is picked but at some point it becomes a case of diminishing returns.

Robert Stevenson
May 9, 2011 7:59 am

Dave Springer
You did not read what I had written – no additional heat absorbed- no temperature increase- no global warming. 150 years ago Tyndall new nothing about CO2 absorption bands

izen
May 9, 2011 8:02 am

Charles Higley says:
May 7, 2011 at 9:33 pm
“It should also be pointed out that, as CO2 partitions 50 to 1 into the oceans, we would have to emit 50 times more CO2 that required to simply double atmospheric CO2. There is not enough carbon available to do this. If we really tried, we might be able to do 20%.”
It could also be pointed out that CO2 partitions 100,000 to 1 between the rocks and the air, so we would have to add 100,000 times more CO2 to just double atmospherics CO2…
This is wrong, as the ocean comparison is wrong because of time-scales. It takes the oceans centuries if not millenia to absorb additional CO2 so that partitioning will take effect eventually, for rcok/air the timescales are more like millions of years…

Philip Peake (aka PJP)
May 9, 2011 8:02 am

One little point about assumptions this is all based upon.
Incoming energy is spread over HALF the earth’s surface.
Outgoing energy is spread over the FULL surface of the earth, with an uneven distribution, probably (?) higher rate of outgoing radiation on the night side of the planet.
So its entirely incorrect to say that incoming = 240W/m^2 and outgoing = 240W/m^2.
Get the energy distributions right, then try again.

ferd berple
May 9, 2011 8:11 am

Here is a reference showing that Jupiter has 60K higher temperature than expected.
33K higher than expected on earth. 60K higher than expected on Jupiter. Co-incidence?
http://burro.astr.cwru.edu/stu/advanced/jupiter.html
•As seen in the table below, the average temperature of Jupiter is approximately 160 K. However, due to the equation for thermal equilibrium (below), it should only be about 100 K.

Dave Springer
May 9, 2011 8:15 am

Also one disputing back radiation must explain how many instruments that measure CO2 concentration manage to work. The basic design of these:
15um infrared light source -> beam splitter
one side of beam splitter -> sealed sample of air with known CO2 concentration -> IR photo transister
other side of beam splitter -> sample of ambient atmosphere -> IR photo transister
CO2 concentration is obtained by comparing the output of the two photo transisters.
Pretty much the same as Tyndall’s experimental setup only tweaked for one particular gas in a narrow range of partial pressures and put into a space the size of a thimble using modern technology.

wayne
May 9, 2011 8:24 am

Dave Springer says:
May 9, 2011 at 5:57 am
RJ says:
May 9, 2011 at 1:41 am
“Anyone who supports Trenberth’s diagram and considers that the back radiation theory is correct should be asked to explain why we are not utilising this fanastic energy source. After all, if it existed it should be the main focus of green energy projects.”
“- If back radiation is so powerful and it is a heat source, why don’t we use it?”
Good questions. Although what about this free energy oven. Does it qualify?
We do use it all the time. One example is the smudge pots used to limit freeze damage in orange and avocado groves. The heat of the smoke from the smudge pots is inconsequential. The effect comes from the black carbon particles in the smoke which absorb upwelling IR from the ground and send a portion of it downwards and sideways. It works as an insulator. This is the same mechanism by which CO2 molecules insulate. Anyone who contests this is simply wrong and needs to take an introductory class in physics.
———–
Dave, you and I structurally disagree, but end at the same conclusion. To me, and I am sure it is correct by physics, is the effect comes from the black carbon particles in the smoke which absorb upwelling IR from the ground increases the temperature at that altitiude (10 m) and by SB using delta T (LBL if you want) between the surface and these particles decreases the upward flux making it warmer than it would have been without the carbon particles in this thin layer where the smoke is.
You do not need a special kind of “back radiation” that flows from cold to hot to explain it, it only scrambles minds away from what is actually happening. If you care about people’s minds at all, stop wording it like you just did. There is no special radiation called “back radiation” or “backward radiation”.

ferd berple
May 9, 2011 8:28 am

“It takes the oceans centuries if not millenia to absorb additional CO2 ”
This does not appear correct. CO2 absorbtion at the surface of the ocean is limited by the surface area. However, the surface area of H2O in the clouds is fantasitically greater than the surface of the ocean. In addition, the water in the clouds is cold, which increases its capacity to hold CO2.
This is where CO2 is absorbed and returned to the oceans. High in the couds, not at the ocean surface. Near the equator, where the oceans are warm this CO2 will be released back into the atmosphere as the rainwater warms. In higher lattitudes where the oceans are cold this CO2 will not be released. The rain water will mix with the cold seawater and sink to the bottom of the ocean, carrying the CO2 with it. The polar oceans should be less caustic, more Ph neutral as a result, as compared to the tropical oceans. Which has been reported.
Eventually this cold water will upwell in the tropics and release its CO2. Thus the CO2 being released during La Nina years was deposited at higher latitudes many years before. This accounts for the cause and effect lag observed between warming and CO2 release, which Al forgot to mention in his movie. This is strong evidence that temperature drives CO2. There is no similar observation that temperature lags CO2 in the paleo record, which is evidence that CO2 is not a driver of temperature.

Dave Springer
May 9, 2011 8:30 am

ferd berple says:
May 9, 2011 at 8:11 am

Here is a reference showing that Jupiter has 60K higher temperature than expected.
33K higher than expected on earth. 60K higher than expected on Jupiter. Co-incidence?
http://burro.astr.cwru.edu/stu/advanced/jupiter.html
•As seen in the table below, the average temperature of Jupiter is approximately 160 K. However, due to the equation for thermal equilibrium (below), it should only be about 100 K.

“This extra heat is generated due to gravitational contraction – the planet is slowly shrinking in diameter. This way, by compressing by only a few millimeters every year, it can generate heat by increasing the pressure of its constituent gas.”
So Ferd, is it your assertion that the earth and venus are shrinking? I already explained to you that temperature only increases during the process of compression and the earth, Venus, and Mars (unlike Jupiter) aren’t shrinking.
What part of that don’t you understand?
Let me fix that for you by adding the explanation for the extra warmth on Jupiter from the article you quote:

wayne
May 9, 2011 8:38 am

Robert Stevenson says:
May 9, 2011 at 2:16 am
For current CO2 partial pressure or concentration I calculate a distance of 3600 m of traverse through the earth’s atmosphere by ‘IR radiation’ before the absorbable IR is ‘filtered out’ and reduced to zero.
—-
Robert, I come up with a close conclusion, not exact, but close. However, the 3600 m is more than 36 times that is given in other conversations I have read speaking of it’s absorption very near the surface. Do you have an average absorption coefficient you are using, or a mean optical thickness for CO2 from which that is calculated? Maybe Hottel specifies emissivity direct then what equation are you using to get the 3600 m extinction distance?

Dave Springer
May 9, 2011 8:40 am

Wayne
If you want a bit deeper understanding of the radiative energy flow one needs to understand that all matter above absolute zero radiates and where there are two bodies at different temperatures there’s a net transfer of energy between the two from warmer to colder. The net transfer rate is all the matters for most practical concerns but at the deeper level the radiative transfer goes two ways. The warm object doesn’t stop the cold object from radiating so energy from the cold object is indeed reaching the warm object just as energy from the warm object reaches the colder one. It’s simply a matter of the colder object receiving more energy from the warmer than the warm object receives from the colder.
That doesn’t seem like proverbial rocket science to me but you may perhaps be right that it might as well be rocket science for many people.

ferd berple
May 9, 2011 8:46 am

Explain why the surface of venus, which only receives some 2.5% of the sun’s energy due to the albedo effect of the clouds can have a temperature of 500C.
Why do we use a figure of .7 when calculating the solar energy reaching the earth’s surface? If we did this on venus, then the surface temperature should be very cold indeed.
The missing 33K comes from the assumption that only 70% of the solar radiation is available at the surface due to albedo. This is an incorrect assumption, as is clearly demonstrated by the surface temperature of venus.
97.5% of the solar radiation never reaches the surface of venus, yet it has a high temperature. Thus, the .7 figure used when calculating solar radiation at the surface is refuted by observation.
Consider 240k / 288K. This is .8. Thus, the missing 33K may not be missing at all. It may simply result from our asusmption that albedo is .7. If albedo is not what we assume it to be, for reasons that are apparent on venus, then the problem is not where is the missing 33K, but rather why temperatures are not hotter than they are.
fully available in our assumption

ferd berple
May 9, 2011 8:48 am

So Ferd, is it your assertion that the earth and venus are shrinking?
Earth’s Shrinking Atmosphere Baffles Scientists
An increase in CO2 could be one reason why a layer of Earth’s upper atmosphere went through its biggest contraction in 43 years.
http://news.discovery.com/earth/earth-atmosphere-shrinking.html

ferd berple
May 9, 2011 9:14 am

“This extra heat is generated due to gravitational contraction – the planet is slowly shrinking in diameter.”
What is the mechanism by which this is possible? Why has the planet not reached equilibrium? Why does the temperature of all the planets and the sun increase towards the center? If the temperature increase is a result of continued shrinkage, then this should long ago have ended on some, if not all planets.
If this efect is unique to Jupiter, why is it so similar to what we see on other planets. Is is reasonable to suggest that a similar observation has unique and different causes on each different planet? Or is it more reasonable to assume that these unique explantions are a result of a lack of understanding. That they are modern day epicycles.
We see a surface temperature of venus that is higher than expected if we used the same methodology for albedo as used on earth. We see a surface temperature for Earth that is higher than expected if we use .7 for albedo, but NOT higher than expected if we use for example .8 or .9. We do not have to increase this above 1, so it is possible the missing 33k is simply due to an error in our understanding about how albedo actually works.
We see a surface temperature for Jupiter that is higher than we expect. This follow a similar pattern as with venus and earth. On each of the three planets science explains this by using 3 different methods, and proposes that these are unique to each planet.
That may well be, but occam tells us that this is not the likely explanation. Since a simpler explanation exists, namely that we are not properly accounding for the effects of clouds in our calculations. The observations of venus suggest that clouds do not block solar radiation the way we think, or the surface of venus would be much cooler. When we apply this logic to earth, the”missing” 33 K is no longer missing. It is simply a result of us using .7 when a more accurate number would be .8 or .9. Jupiter is also covered with clouds. It could well be that the missing 60K on jupiter is due to the poorly understood clouds, not due to continued shrinkage.

Dave Springer
May 9, 2011 9:16 am

ferd berple says:
May 9, 2011 at 8:28 am

“It takes the oceans centuries if not millenia to absorb additional CO2 ”
This does not appear correct. CO2 absorbtion at the surface of the ocean is limited by the surface area.

The effective surface area is greatly increased or diminished by wind driven waves. It’s also limited by how far out of equilibrium it is. It’s also limited by how fast the surface water mixes with the deeper water which is governed by both winds and convective currents. The amount of CO2 that can be held in water increases with depth as well. In deep ocean trenches CO2 has been observed in liquid droplets. So the rate at which surface water is sequestered at depth is also very important.
It’s been a bit of a surprise for the climate boffins that no matter how much faster humans introduce CO2 into the atmosphere only about half of it remains in the atmosphere. The mechanisms that serve as carbon sinks for the atmosphere (we didn’t even start talking about biological and chemical sequestration) are diverse and not well enough understood to predict whether this will remain true going forward.

wayne
May 9, 2011 9:21 am

Dave Springer,
You see, I know radiation in many cases can and does radiate in any and all directions, but that depends on the temperature and radiation from the surrounding matter. To radiate all directions the matter all around must be colder than the source that is radiating. The main difference you and I have is that radiation never actually flows from cold to hot matter, it doesn’t even flow from equal temperature pieces of matter or you could burn yourself with a Fresnel lens in a dark warm room as my little experiment should show you, if you will get curious and perform it yourself. Planets and stars radiate against basically nothing so you can let the other temperature in the Stefan-Boltzmann equation be zero, but always subtracting 2.7K would be more proper. You are one tiny step from moving your rather unrealistic view into what actually happens.
Here is what happens because of the Stefan-Boltzmann equation that screws so many people up (sigma=5.6704r-8, epsilon=1):
ε σ ( 288K^4 – 280K^4 ) = 41.572 W/m^2
or
ε σ ( 288K^4 ) – ε σ ( 280K^4 ) = 41.572 W/m^2
The top one will always show you the correct and actual flow of radiation between two pieces of matter. The latter one implies that matter always radiates at all times in all directions even if the temperatures are identical and the last term gets this quirky term applied to it, “back radiation”. You will end up with the same numeric answer but it leaves you with the wrong impression that two things have occurred when in reality only one thing has occurred, radiation from the warmer to the cooler or not at all if the two temperatures are the same.
This same logic applies to your split beams and it is why such instruments must be constructed to tightly control the temperatures inside. By the second set of equations temperatures of different pieces of the instrument do not matter at all, by the top equation surrounding temperatures of the pieces of the measuring instrument matters greatly. Go lookup an operation manual and see if I am not correct.
Please, drop the “back radiation”, it really does not exist, use the top equation.

May 9, 2011 9:23 am

Dave Springer says “I already explained to you that temperature only increases during the process of compression ”
What you and Ira and Joel fail to consider is that there is continuous compression of cold descending air which heats that air until it becomes hot enough to then ascend, expand, and cool until the process repeats. The atmosphere thus acts as a continuous compressor (and expander).
In addition to the 4 links I provided above, here’s 2 more peer-reviewed papers that explain this as well:
http://www.climatephysics.com/PDFs/Chilingar%20-%20Cooling%20due%20to%20CO2.pdf
http://docs.google.com/fileview?id=0B74u5vgGLaWoYTAyYTc4NmUtZDFkMC00ODg3LTgwYzAtOTg3OTJhZjI5MDVk&authkey=CKyD1r8H&hl=en

Dave Springer
May 9, 2011 9:26 am

ferd berple says:
May 9, 2011 at 8:48 am

So Ferd, is it your assertion that the earth and venus are shrinking?
Earth’s Shrinking Atmosphere Baffles Scientists
An increase in CO2 could be one reason why a layer of Earth’s upper atmosphere went through its biggest contraction in 43 years.
http://news.discovery.com/earth/earth-atmosphere-shrinking.html

Does this change air pressure at sea level significantly?
That’s what’s happening on Jupiter. Pressure at depth is constantly rising. That isn’t happening on the earth. Atmospheric pressure at sea level rises and falls to some degree as horizontal pressure ridges sweep along but the average pressure remains the same at 1 bar.

Dave Springer
May 9, 2011 9:36 am

Jupiter is still undergoing gravitational contraction that began when the solar system was born. This process stopped for the earth billions of years ago. The sun stopped its gravitational contraction when temperature increase from ongoing compression lit off a sustained fusion reaction. The outward pressure from fusion counteracts the inward pressure from gravity and an equilibrium point was reached where they are equal and opposite in force and the sun has a relatively stable diameter that will persist until it starts running out of hydrogen fuel to sustain the outward pressure.

Joel Shore
May 9, 2011 9:43 am

Hockey Schtick says:

What you and Ira and Joel fail to consider is that there is continuous compression of cold descending air which heats that air until it becomes hot enough to then ascend, expand, and cool until the process repeats. The atmosphere thus acts as a continuous compressor (and expander).

We consider that…but it can’t change the result. You are talking about things that can only redistribute energy within the earth-atmosphere system. You still must satisfy conservation of energy when you look at the interaction of the Earth System with the sun and space. And, the combination of a surface temperature above the 255 K blackbody temperature and an atmosphere transparent to terrestrial IR radiation doesn’t do this: There is much more energy going out of the system than coming into it. The result of this would be rapid cooling.

richard verney
May 9, 2011 9:53 am

Ira
I always enjoy reading your articles and the comments that they generate.
I am sceptical as to how useful it is to consider just one element of how the atmosphere may work ignoring other factors which are known to exist and which in some circumstances are definitely more important factors (I have in mind convection, evaporation etc). Leaving that aside, I have problems in envisaging that the back radiation plays any significant role in the workings of the atmosphere.
I have no problem with the argument on net energy flow and that if the atmosphere is warmer it slows down the heat loss from the warmer earth (or ocean). The problem I have is whether the atmosphere is warm simply because of the absorption of solar energy (a combination of absorption of incoming solar radiation and absorption of reflected and radiated solar energy from the bottom up) and adiabatic lapse, or whether in addition back radiation from GHGs plays a significant role.
You will see the point I made at (richard verney says) at May 8, 2011 at 8:39 pm
Smoking Frog sought to answer the point. He comments at May 9, 2011 at 5:21 am:
“Martin Mason said – If back radiation is so powerful and it is a heat source, why don’t we use it?
Because the earth’s surface is already at the temperature to which the back-radiation has raised it.”
However, with respect that explanation cannot be correct. We know as fact that solar (possibly with the help of back radiation) heats the earth’s surface to the temperature to which solar energy has raised it, yet not withstanding this we can still extract work from the incoming solar energy. We can collect and focus the 184 w/m^2 of incoming solar energy and use it to melt salts etc in solar power plants. That being the case, it begs the question why can’t we collect and focus the 333 w/m^2 of back radiation and extract useful work from it.
The fact that we cannot do this suggests that the ground is not being bomb barded with 333 w/m^2 of (back) radiation.
I have a black and white marble path. On sunny days, this path is warm to walk on. The black marble slabs are considerably warmer than the white marble slabs. Not surprising since black is a better absorber than white and therefore the black marble stones absorb more of the incoming 184 w/m^2 of solar energy. IF back radiation existed and if there was some 333 w/m^2 of back radiation beating down on the ground, the black marble slaps should be significantly warmer than the white marble slaps at night or on cloudy days. However, they do not appear to be. In particular when left over night to bask in the supposedly 333 w/m^2 of back radiation, the black marble slaps do not heat up whereas they do heat up with only 184 w/m^2 of incoming solar energy!!
Ira, I would like to see someone conduct a very simple experiment along the following lines;
1. Get two identical metal slabs (same dimensions for area and thickness) save that one is painted brilliant white and the other matt black,
2. Pre heat the slabs to the forecasted daily temperature (say picking a day when the estimated temperature is going to be 20degC).
3. On a clear sunny day (ie the day estimated to be 20 degC) place the slaps in the sun and measure there respective temperatures every half hour.
4. On a mild summer night (one estimated to be 20 degC), pre heat the slaps to the forecasted night temp of 20C and then leave them out after the sun has fully set and measure there respective temperatures every half hour.
5. Compare the results.
6. In particular does the matt black slab gain heat at night from the down welling back radiation. If it does not, why not?
Ira, I would appreciate your comments

Joel Shore
May 9, 2011 9:54 am

Bryan says:

Joel Shore has refined this even further to say Postema is a believer, only Postemo hasn’t quite realised it.
Perhaps with the same elasticity Joel will prove that Gerlich and Tscheuschner also are really IPCC advocates.

No…What I am saying is that Postma and G&T mix in a lot of correct science that is already known with some incorrect science…or just glossing over things…to reach completely incorrect and absurb conclusions. That tends to be how peddlers of pseudo-science operate.

On page 9 of your co-written Halpern et al comment paper you state that the Stephan Boltzman Law can be used to work out the thermal energy between atmospheric shells.
I brought your attention to it in my post above;
….. ” 3. Has the Stephan Boltzmann equation applying to gases.”……

It is sad to see that the pull of pseudo-scientific arguments are just too strong for you to resist making them again and again. The shells that we consider are blackbody shells, so yes, the S-B equation applies by definition. This is a model. Is it a good model? Well, yes, it is quite a fine model to illustrate the basic features of the greenhouse effect. When you want to do detailed quantitative calculations, you adopt a better model. Is that concept so difficult to understand?
It is not science to attack a model for being to simple when it is meant to be a simple representation and where more refined models are available and can be used to show that the qualitative predictions of the simpler models for which they are used are correct. That is instead the realm of pseudo-science, a realm that is apparently just too irresistible to you. That you cannot even distinguish the difference between scientific arguments and the pseudo-scientific nonsense that you peddle is very very sad for you, the people reading this thread, anybody who respects the scientific enterprise, and even AGW skeptics who don’t want to be considered complete crackpots. I actually pity people like Ira and David M. Hoffer for having to put up with people like you on their side of the AGW debate.

JAE
May 9, 2011 9:54 am

Heh, and LOL. Unless I missed it, nobody has yet tackled the “venus thing” presented above by a commenter (which suggests that the GHE theory isn’t valid):
http://theendofthemystery.blogspot.com/2010/11/venus-no-greenhouse-effect.html
BTW, it holds for other planetoids with an atmosphere (I can provide links if anyone cares).
Despite the Orwellian chants about “multiple lines of evidence,” the case for a GHE threat is really hurting these days, due to the total lack of any such lines of evidence. In fact the multiple lines of evidence appear to go the other way:
1. No warming for 10-15 years, despite all the increases in the terrible GHGs (and not just OCO).
2. Ice core records show no correlation between GHGs and temperature.
3. The temperature swings during the past couple of millenia indicate the climate changes naturally without changes in GHGs. (We don’t need Mann’s proxies to know what happened during the MWP and LIA; it’s recorded in the history books, for crying out loud!)
4. No “hot spot” in the mid-troposphere in the tropics, as predicted by ALL the climate models. (So much for treating their output as “data.”)
5. No laboratory demonstrations or other real-world empirical demonstrations of any GHE effects.
6. Areas with the most greenhouse gases (tropics) have maximum temperatures that are no where near as high as some desert areas with the lowest amount of GHEs (yeah, yeah, it’s the water evaporation and clouds; but that is precisely the point on Earth).
7. Articles such as the one cited above, which show clearly that temperature in a gaseous atmosphere has nothing to do with the “greenhouse effect.”

JAE
May 9, 2011 10:02 am

Dave Springer:
“We do use it all the time. One example is the smudge pots used to limit freeze damage in orange and avocado groves. The heat of the smoke from the smudge pots is inconsequential. The effect comes from the black carbon particles in the smoke which absorb upwelling IR from the ground and send a portion of it downwards and sideways. It works as an insulator. This is the same mechanism by which CO2 molecules insulate. Anyone who contests this is simply wrong and needs to take an introductory class in physics.”
Well, I question this. Since the backradiation is coming from cooler air above, it cannot be warming the orange grove.
The Clean Air Act killed the smudge pots, so they now just blow air through the orchards or add heat by spraying water. Maybe they didn’t ever really need that backradiation?

Dave Springer
May 9, 2011 10:05 am

Hockey Schtick says:
May 9, 2011 at 9:23 am
“What you and Ira and Joel fail to consider is that there is continuous compression of cold descending air which heats that air until it becomes hot enough to then ascend, expand, and cool until the process repeats. The atmosphere thus acts as a continuous compressor (and expander).”
Not in my case. I’ve discussed it in depth many times at WUWT. The following search reveals some of it.
http://www.google.com/search?hl=&q=tunderstorm+heat+engine+springer+site%3Awattsupwiththat.com&sourceid=navclient-ff&rlz=1B3GGGL_enUS290US290&ie=UTF-8
The big picture:
The sun heats the ocean, the ocean heats the atmosphere, the cosmic void cools the atmosphere.
This thread is about radiative transfer in and out which is a detail in the big picture. Convective transfer is a different detail but also very important. The water cycle in general is exceedingly important in basic understanding of atmospheric physics. Vast amounts of energy are lifted from the surface by evaporation in what’s called “latent heat of vaporization”. It’s called “latent” because it won’t register on a thermometer. When the vapor condenses the latent heat is released. As the earth gets warmer (for whatever reason) the speed of the water cycle increases. This is a negative feedback mechanism and it’s the most glaring flaw in the whole warmist narrative. There has never been a runaway greenhouse on the earth. Never. The hypothetical amplification of anthropogenic CO2 forcing by increased water vapor is a wholesale fabrication without a shred of evidence to support it and with mountains of contrary evidence. The water cycle is a negative feedback not a positive feedback. Most of the more informed global warmists and skeptics know that this amplification is the heart of the controversy. None dispute that when everything else is equal more CO2 will result in a warmer surface and that this is would be about 1.0C for each CO2 doubling starting from a base point of 280ppm. They also acknowledge that a warming surface will speed up the water cycle. The controversy is over whether a faster water cycle is a positive or negative feedback. The evidence says it’s negative. The alarmist must have it as a positive feedback because otherwise the only warming that’s gonna happen is a modest welcome amount that is of great net benefit to the biosphere. Living things don’t tolerate ice very well so the less of it there is the more life can flourish on this third rock from the sun.

Dave Springer
May 9, 2011 10:25 am

Climate boffins have pretty much conceded the debate over water vapor amplification. As this concession ocurred it manifested to the unwashed masses as “global warming” was first replaced by “climate change” and now is morphing yet again into “climate disruption”.
The climate does nothing if not change and it has been doing so since long before humans entered the equation so “climate change” failed to elicit the needed level of fear response in the masses. Climate disruption has more fear factor in it.
An inconvenient amount of climate disruption is a distinct possible result of anthropogenic activity but once again the evidence is all stacked against it. By the same token radically modifying anthropogenic activity to lessen any imagined disruption has an immediate negative consequence of slowing down the global economy which is driven in large part by fossil fuel consumption.
Thus we have a risk/reward decision to make and in my view trying to reduce CO2 emissions is essentially all risk and no reward. Real pollutants that immediately and directly effect our health and well being are a much greater concern. Heavy metals in our water, particulate pollutants in our air, ozone, and lots of other things are legitimate red flag events but CO2 is plant food and the more of it we have the better off we are.

ferd berple
May 9, 2011 10:27 am

“In deep ocean trenches CO2 has been observed in liquid droplets. So the rate at which surface water is sequestered at depth is also very important.”
That is intersting. The ocean pressure goes up 1 atmosphere per 30 feet of depth. At 300 K CO2 becomes a liquid at 100 atmospheres. The deep oceans are all below 300K and 100 atmospheres is only 3000 feet – much less the the depth of the average ocean basins.
So, what this means is that CO2 can exist as a liquid across most of the ocean bottoms. As liquid CO2 should weigh more than H2O, the question really should be why the bottom of the oceans is not all liquid CO2? Except that the oceans are not saturated, and thus liquid CO2 would be absorbed.
http://en.wikipedia.org/wiki/File:Carbon_dioxide_pressure-temperature_phase_diagram.svg
http://oceanservice.noaa.gov/facts/oceandepth.html

ferd berple
May 9, 2011 10:37 am

http://theendofthemystery.blogspot.com/2010/11/venus-no-greenhouse-effect.html
well worth the read, as well as the follow on explanation. what I find fascinating is that albedo has no effect on the temperature comparison between earth and venus, which accounts fully for the missing 33K in Ira’s analysis.
quote from the article:
For example, in the analysis, not only does the amount of CO2 not enter in (Earth has 0.04%, Venus a whopping 96.5%), but the albedo (from either cloud tops or the planetary surface) does not either (Venus has dense clouds that reflect much of the incident visible radiation, while Earth does not, and Earth’s surface is 70% deep ocean, while Venus is solid crust). The real atmospheres don’t care at all about these great differences in the two atmospheres and planetary surfaces, they only care, and quite precisely, about their distances from the Sun.

ferd berple
May 9, 2011 10:45 am

http://2.bp.blogspot.com/-4R16sH96XBk/TX5hIW8MADI/AAAAAAAAAD4/lhiu503gY58/s1600/venus%2Bearth%2Bcomparison.JPG
This cannot simply be coincidence. How can two planets with completely different atmospheres, albedo’s and surfaces show almost identical temperatures as a function of pressure if the standard climate model is correct? Quite simply they can’t. These observations are either false or climate science has it fundamentally wrong.

MartinGAtkins
May 9, 2011 10:47 am

JAE says:
May 9, 2011 at 10:02 am
Well, I question this. Since the backradiation is coming from cooler air above, it cannot be warming the orange grove.
Even without knowing about the exact situation you are discussing, I can tell you don’t understand the concept or the reality of back radiation.
The air above is not warming the orange grove but slowing the heat loss by radiating some of the LW radiation back. If no more energy enters the grove from the outside it will continue cool over time. It is only the law of conservation of energy at work, so the back radiation is not extra energy but only energy that has had it’s ability to leave the system delayed.

George E. Smith
May 9, 2011 10:56 am

“”””” JT says:
May 7, 2011 at 8:34 pm
Ira, the basic bookkeeping of the simplest greenhouse calculations assumes that the re-emission of infra-red radiation by the greenhouse gas molecules is spatially symmetrical. I am well aware that spontaneous emission is equally likely to be in any direction so that approximately 1/2 will head downward to the surface. However, there is another kind of emission – stimulated emission – and it is massively biased in the same direction of travel as the direction of travel of the stimulating photon. Thats what creates laser light, but stimulated emission can occur without lasing. “””””
I don’t believe that is true. Stimulated emission is still isotropic. The highly directional output of a laser beam, is a consequence of the resonant Optical cavity that the emission takes place in. Only wave modes, that are aligned with the axis of the resonator, can continue to propagate in the system. The off-axis modes are suppressed. The concept is quite simple. Take two parallel mirrors, and launch a geometrical ray normal to one of the mirrors, to wards the other. It will in the geometrical limit, bounce back and forth along the same line indefinitely. If the angle is even slightly off normal, the ray will walk over to the edge of one mirror, and get reflected into the side walls. Parallel flat mirrors are a completely unstable resonator configuration.
Only certain mirror configurations produce stable multiple reflections that retain the energy within the resonator, except for small transmission losses that let light leak out of the business end of the laser. The other mirror is usually made as high a reflectance as technology allows, including the use of TIR mirrors.

Allan M
May 9, 2011 11:09 am

Joel Shore says:
May 9, 2011 at 7:56 am
Allan M says:
The basis for the second law (and the first) is careful and accurate measurement, mostly paid for by wicked 19th century capitalists, who wanted to know how to get more from their steam engines, and also how to avoid being sold snake oil.
There is no need for a statistical ‘fudge’ unless you wish to introduce special pleading for “IPCC Approved” photons.
Experiment did provide the evidence for the Second Law. However, our modern (20th century) understanding of where the law comes from is based on statistical physics.

If you you use statistics properly, the tail doesn’t wag the dog.

JAE
May 9, 2011 11:28 am

Martin:
“The air above is not warming the orange grove but slowing the heat loss by radiating some of the LW radiation back. If no more energy enters the grove from the outside it will continue cool over time. It is only the law of conservation of energy at work, so the back radiation is not extra energy but only energy that has had it’s ability to leave the system delayed.”
Well, maybe so. But then aren’t you saying that the backradiation is affecting the lapse rate? That cannot be, since the lapse rate depends upon only Cp and g. ??

May 9, 2011 11:34 am

wayne said:
Clouds make warmer nights because the clouds are usually warmer than the normal air temperature at that altitude and therefore the surface’s rate of loss by radiation upward will be less leaving you with a warmer than normal night.
This is nonsense. Typically, clouds are much colder than the ground. (When they are the same temperature, we call it fog.) Instead, they are more like styrofoam cups, they scatter IR radiation back toward the ground without affecting their own temperature. This is because the bottom of a cloud is a phase change boundary and, therefore, the temperature has almost no effect on the amount of energy released. Stefan’s equation does not apply.
Bottom line, cold clouds make for warm nights. This is a part of the greenhouse effect.

ferd berple
May 9, 2011 1:27 pm

“Well, maybe so. But then aren’t you saying that the backradiation is affecting the lapse rate? That cannot be, since the lapse rate depends upon only Cp and g. ??”
My question is why we assume that N2 in the atmosphere doesn’t back radiate. All objects radiate depending on their temperature. As the atmosphere is mostly N2, that must be the source of most of the back radiation.
If we increased the atmospherice pressure 90 fold, such as on venus, then there would be 90 times as much N2 per cubic meter, and the back radiation would go up proportionally, as would the surface temperature. Thus, the surface temperature must be a function of atmospheric pressure, which is a function of gravity.
The standard radiative model ignores gravity, which is why it cannot be applied to other planets to see if it is correct. Thus the reliance on computer models, which have the added bonus that there is no way they can be proven wrong in the short term.

JAE
May 9, 2011 1:36 pm

ferd:
“My question is why we assume that N2 in the atmosphere doesn’t back radiate. All objects radiate depending on their temperature. As the atmosphere is mostly N2, that must be the source of most of the back radiation”
N2 doesn’t radiate significally in the IR portions of the spectrum.

May 9, 2011 1:37 pm

Can you provide a reference to a recognized standard text which supports this contention that it is only the net difference which must respect the Second Law of Thermodynamics?
The only instance I know of where a colder object can impart energy to a warmer object is in the entirely theoretical case of two objects being at negative Kelvin temperatures where the colder object must perforce be at a higher energy state the the warmer one. Yet in this case energy still flows from a higher state to a lower state and thus respects the Second Law.
Heat is the manifestation of energy flowing from a higher state to a lower state. It is why heat pumps are efficient and air conditioners aren’t.

Bryan
May 9, 2011 1:40 pm

Joel Shore
You cannot weasel out of your gross error.
You challenged me to discus the science.
I am here to do so.
If you prefer to avoid the science and instead issue smears instead then I will not be surprised.
This item is directly addressed to Ira’s post and deserves an answer from you!
On page 9 of your co-written Halpern et al comment paper you state that the Stephan Boltzmann Law can be used to work out the thermal energy exchange between atmospheric shells.
There is a whole series of statements on pages 8,9,and 10 referring to the atmosphere, Stephan Boltzman Law and indeed including the SB formula.
Gerlich and Tscheuschner specifically said that the Stephan Boltzman Law cannot be used for atmospheric gases.
Do you now agree with them and withdraw your comments?
Or will you persist in this unphysical assumption.
Depending on your answer we can perhaps call on Ira’s spreadsheet to settle the matter.
Your paper is shown below but I dont have a direct link.
Comment on ‘Falsification Of the atmospheric CO2 greenhouse effects within the frame Of Physics’ by Joshua B. Halpern, Chistopher M. Colose, Chris Ho-Stuart, Joel D. Shore, Arthur P. Smith, Jorg Zimmermann.

Joel Shore
May 9, 2011 1:41 pm

Dave Springer says:

The hypothetical amplification of anthropogenic CO2 forcing by increased water vapor is a wholesale fabrication without a shred of evidence to support it and with mountains of contrary evidence. The water cycle is a negative feedback not a positive feedback.

Climate boffins have pretty much conceded the debate over water vapor amplification.

Actually, there is plenty of evidence of the water vapor feedback, both existence and rough magnitude, as discussed here: http://www.sciencemag.org/content/323/5917/1020.summary
The only sense in which your argument for a negative water cycle feedback makes much sense is if you are grouping together cloud and water vapor effects in such a feedback (which I guess is not unreasonable when you refer to it as “water cycle” but becomes confusing when you refer to it as “water vapor feedback”).
The cloud feedback is indeed associated with quite a large degree of uncertainty and basically the only way in which we could be spared from having a fairly significant climate sensitivity is to have a significantly-negative cloud feedback. Alas, besides not being what all of the models show (which admittedly could be a problem with the models), such a negative feedback makes it rather difficult to explain paleo-climate without somehow coming up with much larger forcings than are estimated right now (for. e.g., the glacial – interglacial oscillations).

The controversy is over whether a faster water cycle is a positive or negative feedback.

Limiting the controversy to specifically the feedback due to clouds, I essentially agree with this statement….which is why we both apparently find it so ridiculous to see people here wasting tons of time arguing about well-settled science that they have no hope of possibly overturning. (The most extreme cases being people like Allan M who apparently want to overturn all of modern statistical physics in order to come up with a version of the 2nd Law that is more toward their liking.)
People should realize that, even from a purely tactical point-of-view, it would be wise to pick your battles. When you argue that the greenhouse effect doesn’t exist, the only cause your further, at least within the scientific community, is the cause of those who would like to show how ridiculously ignorant of science the AGW skeptics really are. I would invite those who want to help demonstrate that to continue to make their arguments here; the rest of you should consider carefully reading what Dave, David Hoffer, Ira, myself and others are writing here and actually trying to understand the correct science.

JAE
May 9, 2011 1:42 pm

“Bottom line, cold clouds make for warm nights. This is a part of the greenhouse effect.”
Hmmm. Maybe not. Clouds affect convection and the lapse rate (decrease rates), too. Water (clouds) also store 4 times as much heat as air. Water vapor stores twice as much. How do you separate these effects from the “radiative greenhouse effect?”

Joel Shore
May 9, 2011 1:55 pm

Ira Glickstein says:

Should we interpret your “leave him [Hansen] out of it” as a repudiation of Hansen and his leadership of NASA GISS or as simple embarassment at the antics of a nice old uncle who means well but sometimes leaves his fly down by mistake?

Ira,
I think you may be over-interpreting the statement that I made. As far as I can tell, Hansen’s notion that if we really go to town using fossil fuels, we might trigger a true runaway greenhouse effect just seems rather vaguely-supported by any detailed argument or evidence at this point and does not seem to be the general belief within the climate science community. So, although I have a lot of respect for Hansen, who has the track-record of saying things that seem a bit far-out at the time but later become quite well-accepted by the scientific community (such as when he pronounced back in the late 1980s that the warming occurring was almost definitely due to greenhouse gases), I think it is at best premature to give much credence to this particular prediction of Hansen’s.
So, I was simply saying that the generally-accepted projections for AGW are that positive feedbacks amplify the radiative effects due to greenhouse gases alone but do not lead to an actual “runaway” instability. For some reason that I don’t totally fathom, some people on the skeptic side (like martin mason above or Smokey in other threads) seem to like to group together this prediction of feedbacks amplifying things and producing an actual “runaway” instability, perhaps because it makes a better “strawman” argument to attack.

Joel Shore
May 9, 2011 2:09 pm

Alleyne says:

Can you provide a reference to a recognized standard text which supports this contention that it is only the net difference which must respect the Second Law of Thermodynamics?

Any text on thermodynamics that treats it from a statistical physics perspective ought to do fine. In the context of radiative heat transfer specifically, any text that talks about the exchange of radiation between two objects or any object and its surroundings ought to give you the basic radiative transfer equations that are used in computing the greenhouse effect.

Heat is the manifestation of energy flowing from a higher state to a lower state.

Yes…So, heat (which is a macroscopic concept by its definition) always flows from hotter to colder (in the absence of work). However, note that the interpretation that the radiative energy flows in both directions, while having an abundance of empirical support, is not necessary to show there is a greenhouse effect. All that is really necessary to say is that the heat flow between two objects depends on the temperature of both objects and not just on the temperature of the hotter object.

It is why heat pumps are efficient and air conditioners aren’t.

I have no clue what you are trying to say in this last sentence. The term “heat pump” is sometimes used as a category that includes air conditioners and refrigerators. Other times, it is used to represent the subcategory of the “heat pump” in the above context that are actually used to heat a house rather than cool it. However, regardless of how the term is used, heat pumps used for heating and air conditioners operate by the same basic principle. Both use work to “pump” heat energy from cold to hot, i.e., in the opposite direction from which heat spontaneously “flows”.

MartinGAtkins
May 9, 2011 2:33 pm

JAE says:
May 9, 2011 at 11:28 am
Well, maybe so. But then aren’t you saying that the backradiation is affecting the lapse rate? That cannot be, since the lapse rate depends upon only Cp and g. ??
It’s not a question of back radiation effecting the lapse rate but rather the laps rate reduces the effectiveness of back radiation.
As the air thins then there is an increased possibility that some of the electromagnetic wave may find windows that are not closed by a receptive molecule. This being the case then the wave propagates out into open space and so is not absorbed and not re-radiated.
This is important with regard to H20, as humidity drops rapidly and the wave lengths that are absorbed by this molecule now escape at a greatly increased rate and so back radiation diminishes accordingly.

May 9, 2011 2:33 pm

Ira, Why are leaves green?
It is because chlorophyll reflects green light. Leaves absorb and use the energy that is most abundant .. the near IR.
People also usually ignore the fact that our retinas are highly sensitive to UV light. So sensitive, in fact, that there is a special UV filter associated with the lens to absorb it. This was discovered when people with new artificial lenses were able to see in the UV. Because our eyes use simple lenses, this caused significant chromatic aberration such that they were not able to focus when out doors. I mention this because it argues against the common idea that our eyes are most sensitive to green light because “that is the brightest part of the spectrum”.
You should also consider that many people wear sun glasses because it is too bright near the equator. This implies that it is possible that our eyes developed to block the bright near IR and to use the significantly dimmer visible part of the spectrum.
Therefore, I reject the argument that “using human eyes for a sanity check” makes sense. Using leaves (chlorophyll) makes a lot more sense since they have (it has) been around a lot longer.
Remember, energy is expressed as frequency times Planck’s constant – E=hf. Therefore, when studying energy, frequency is the more appropriate x-axis.
As for physical sense, the distance between 1 and 2 is the same as the distance between 9 and 10. However, the distance between 1/1 and 1/2 (0.5) is NOT the same as the distance between 1/9 and 1/10 (0.011). Since the energy is measured as the area under a curve (an integral), an adjustment has to be made to keep the areas the same when using the wavelength. Remember, energy is linear only in the frequency domain.

Joel Shore
May 9, 2011 2:41 pm

Bryan says:

You challenged me to discus the science.
I am here to do so.

What you (and G&T) are engaging in has ABSOLUTELY nothing to do with science, unless you include undermining and obfuscating science. I have explained the philosophy behind using a hierarchy of models here: http://wattsupwiththat.com/2011/05/07/visualizing-the-greenhouse-effect-light-and-heat/#comment-655880 and here: http://wattsupwiththat.com/2011/05/07/visualizing-the-greenhouse-effect-light-and-heat/#comment-656374 . If you refuse to understand it, I don’t have any more to say to you.
You are adding only useless noise to this thread.

Bryan
May 9, 2011 2:54 pm

JAE says:
Hmmm. Maybe not. Clouds affect convection and the lapse rate (decrease rates), too. Water (clouds) also store 4 times as much heat as air. Water vapor stores twice as much. How do you separate these effects from the “radiative greenhouse effect?”
Your points are valid but clouds should not be appropriated by greenhouse enthusiasts
Clouds, raindrops and hailstones are much better radiators than gases.
Not because they are composed of so called “greenhouse gases”.
ANY solid or liquid can provide a much larger range of radiation than the line spectra of gases.
They don’t need to be composed of greenhouse gases to show this effect.

May 9, 2011 3:02 pm

Ira Glickstein,
You have exactly conveyed my own world view regarding the effect and importance of CO2:

…AGW has a real but minor effect compared to natural cycles and processes and data bias by the official climate Team, and that a bit of warming and higher CO2 levels may turn out to be of net benefit to humanity. Therefore we should do little about it.

# # #
Alleyne,
Heat can flow from a colder to a warmer object; from higher to lower entropy. What is required is work. For example, a refrigerator does reverse entropy work. Work requires energy. Of course, total entropy in the universe only goes in one direction. It’s the exceptions that make it interesting.

wayne
May 9, 2011 3:43 pm

Robert Clemenzi says:
May 9, 2011 at 11:34 am
wayne said:
Clouds make warmer nights because the clouds are usually warmer than the normal air temperature at that altitude and therefore the surface’s rate of loss by radiation upward will be less leaving you with a warmer than normal night.
This is nonsense. Typically, clouds are much colder than the ground. (When they are the same temperature, we call it fog.)
—-
Respectable Robert, what you say I said is not what I said at all. Read what I said again. I never said clouds were warmer than the surface (well, except rarely and even that was pointed out later). I think you jumped the gun a bit. ☺

Richard M
May 9, 2011 4:16 pm

Whew, I’ve read most of the comments over the last couple of days and yet I see nothing discussed about the cooling effect of GHGs.
How can we discuss the temperature of the atmosphere without understanding the transfer of heat by collisions? When I see Trenberth’s energy chart it ignores heat transfer by collisions of gas particles. While I accept radiation flows both ways I understand collisions create a heat flow between the Earth’s surface and the atmosphere that also goes both ways.
Therefore, the Earth loses energy at times to the atmosphere and gains energy at other times. However, just like in the case of radiation, the net flow is most likely to the atmosphere. Has anyone measured this heat flow? I’ve never seen it documented and yet it seems important if we are to understand the big picture.
Now, why is this important? Well, consider the situation without GHGs and ignore the Earth radiating energy for the moment. The Sun would heat the surface and the atmosphere. Energy would move between them but would have no way to escaping. It would get pretty darn hot.
Now, add GHGs to the system. The GHG-less atmospheric gases would start colliding with the newly added GHGs and heating them up. They would start radiating energy. Half would go down which would not change the heat in the system since none was escaping before adding the GHGs. However, half would go up and out of the system … thus cooling the atmosphere.
Of course, we know the GHGs would actually intercept radiation from the surface as well. The so-called GHE. So, we have two conflicting processes. One delays the heat leaving the system and the other increases the heat loss that occurred as a result of heating due by other means (collisions with surface and solar).
Without understanding both of these systems in detail how can we know the overall effect of GHGs? It seems to me only half the problem has been addressed by scientists, what am I missing?

IAmDigitap
May 9, 2011 4:21 pm

“In the past 100 years since the formation of the ‘greenhouse’ theory of atmosphere, we have discovered that the atmosphere is a MILES DEEP, FRIGID, compressible fluid, HEAT CONDUCTIVE, IMMERSION BATH, held in place around the earth, by gravity.
The atmosphere possesses physical attributes like:
A: a big, warm blankie
B: a big, warm, greenhouse
C: a MILES DEEP, FRIGID, HEAT CONDUCTIVE, IMMERSION BATH
Someone who believes in a greenhouse effect will tell you the answer is A, but since it’s too complicated to talk about it as A, you can talk about it like it’s B, but not E.V.E.R. under A.N.Y. circumstances, is the answer C.
T.h.a.t. is who you’re dealing with.

ferd berple
May 9, 2011 4:30 pm

N2 doesn’t radiate significally in the IR portions of the spectrum.
How is that possible? Every object radiates depending on its temperature. It may not absorb at certain frequencies, but how can it avoid radiating in IR if that is its temperature?

ferd berple
May 9, 2011 4:47 pm

“The only sense in which your argument for a negative water cycle feedback makes much sense”
Negative feedback makes sense when one considers how long life has existed on the planet. Under positive feedback anything that increased temperature (not just CO2) would have locked the earth into spiralling temperatures and the extinction of life.
This doesn’t happen because the evaporation of liquid water is a super efficient cooling mechnism. As a result humans are able to survive the hottest conditions on the planet, so long as water is available.
In contrast, humans cannot survive without protection temperatures under 82F for any length of time. Below this temperature we radiate more net energy than can be replaced through food and require technology to survive. Interestingly, 82F is the temperature of the tropical rainforests of the earth, which suggests that humans did not leave the jungles until after we developed technology.

May 9, 2011 4:48 pm

Smokey,
Thanks, that was my point exactly.
Where is the work, the external energy, which causes energy to flow from the atmosphere to the earth, from cold to warm?

May 9, 2011 4:54 pm

Knock Knock,
The Earth is a rotating sphere, which emits radiative power from the surface according to the fourth power of T in all directions from all locations, varying with the time of day and the physical properties of the surface. Sunlight essentially has parallel radiation and is at a very much higher energy level with a “hot spot” notionally rotating around the planet. The outgoing stuff is very weak because it is spread around over the entire surface via atmosphere and ocean dynamics. That naughty T^4 thingy is roaming around everywhere, although it is in the minority in HEAT loss from the surface. (according to Trenberth).
I think there are some meaningless averages touted around.

JAE
May 9, 2011 5:01 pm

ferd:
“How is that possible? Every object radiates depending on its temperature. It may not absorb at certain frequencies, but how can it avoid radiating in IR if that is its temperature?”
The electromagnetic radiation with wavelengths in the IR region is associated with changes in a dipole moment. See here:
http://www.wpi.edu/Academics/Depts/Chemistry/Courses/General/infrared.html
Key paragraph:
“The fact that a molecule vibrates does not in itself insure that the molecule will exhibit an IR spectrum. For a particular vibrational mode to absorb infrared radiation, the vibrational motion associated with that mode must produce a change in the dipole moment of the molecule. HCl, for example, with a center of positive charge at the H atom and a center of negative charge at the Cl atom, has a dipole moment. The magnitude of the dipole moment changes as the HCl bond stretches, so this vibration absorbs IR radiation. We say that the vibration is IR active. The N2 molecule, on the other hand, has no dipole moment. Further, stretching the N-N bond does not produce a change in dipole moment, so the vibration is infrared inactive (i.e., cannot directly absorb IR radiation). It is important to realize that there are many molecules that, although possessing no permanent dipole moment, still undergo vibrations that cause changes in the value of the dipole moment from 0 to some non-zero value. An example is CO2, which has no permanent dipole moment because the individual bond dipoles exactly cancel. However, when CO2 undergoes a bending vibration, its dipole moment changes from zero to some non-zero value. This vibration produces a change in dipole moment and is IR active.”

Tim Folkerts
May 9, 2011 5:18 pm

wayne says: May 9, 2011 at 9:21 am
Here is what happens because of the Stefan-Boltzmann equation that screws so many people up (sigma=5.6704r-8, epsilon=1):
ε σ ( 288K^4 – 280K^4 ) = 41.572 W/m^2
or
ε σ ( 288K^4 ) – ε σ ( 280K^4 ) = 41.572 W/m^2
The top one will always show you the correct and actual flow of radiation between two pieces of matter. The latter one implies that matter always radiates at all times in all directions even if the temperatures are identical and the last term gets this quirky term applied to it, “back radiation”.

Sorry, Wayne, but you are the one who is screwing it up. Either or both equations are correct. The object at 288 K will radiate ε σ ( 288K^4 ) independent of any other objects nearby. The other object at 280 K will radiate ε σ ( 280K^4 ) independent of any other objects nearby.
If the second object was at 278 K, then both would radiate ε σ ( 288K^4 ), for a net transfer of zero. This is perfectly in accord with all the laws of physics.
The first version highlights the correct and actual HEAT (ie net flow of energy).
The second version highlights the correct and actual energy from each object.
[Or more specifically, the net heat or energy per second per square meter].
[And both rates of energy transfer depend on the geometry of the two objects. Just knowing how much each object radiates does not tell us how much is actually intercepted by the second object.]

Steve
May 9, 2011 5:21 pm

“Robert Clemenzi says: Ira, Why are leaves green? It is because chlorophyll reflects green light. Leaves absorb and use the energy that is most abundant .. the near IR.”
Leaves reflect green because leaves usually contain a mix of chlorophylls (mostly types a and b), which predominantly absorb violet, blue, orange and red light. And as winter sets in, leaves become red, orange and yellow because the quantity of red absorbing chlorophyll is dropped in favor of violet absorbing chlorophyll (more bang for your metabolic buck). I have never read of any type of chlorophyll that absorbs in the NIR. In fact, chlorophyll meters specifically use the fact that chlorophyll doesn’t absorb NIR in order to take a measurement!
“ferd berple says: How is that possible? Every object radiates depending on its temperature. It may not absorb at certain frequencies, but how can it avoid radiating in IR if that is its temperature?”
Individual molecules are not black bodies. If nitrogen gas, which makes up 80% of the atmosphere, radiated “according to it’s temperature”, how would anyone be able to take an IR photograph that didn’t look like a photo taken in a thick fog?
Radiation is due to the motions of atomic bonds. Temperature is a measurement of average kinetic energy. The temperature of a mass of nitrogen gas in our atmosphere is primarily due to entire molecules moving, not atomic bonds vibrating. So it predominantly cools by transferring this kinetic energy to neighboring molecules (conduction) or doing work (adiabatic cooling), which is the same way it was heated to that temperature in the first place (not by IR absorption).

wayne
May 9, 2011 5:27 pm

Smokey says:
May 9, 2011 at 3:02 pm
Said to Alleyne:
Heat can flow from a colder to a warmer object; from higher to lower entropy. What is required is work. For example, a refrigerator does reverse entropy work. Work requires energy. Of course, total entropy in the universe only goes in one direction. It’s the exceptions that make it interesting.

Ok Smokey, of course, the refrigerator and air conditioner and similar cases, of course, can and do work (sic ☺).
But when viewing two objects as a whole, the atmosphere and the surface, that the atmosphere is performing work to warm the surface without the atmosphere equally cooling from doing that very work? Warmist tend to believe this, cake and eat it too. I can see isolated cases as compression at the poles and other curiosities but not on the average, and besides, even with those effects the temperature gradient is rarely actually inverted so in a net sense that is only slowing the cooling of the surface at the expense of equal cooling in the atmosphere which ends in a greater temperature gradient therefore a greater flux of energy upward to space. A local effect, may be the whole polar cap, but the net effect is the same.
Do you really think this work by the atmosphere is happening and where, how? (Or am I missing something here?)

May 9, 2011 5:33 pm

Alleyne,
Perfect question. Maybe those energetic little CO2 molecules are busily working away, heating up the planet and making it go all CAGW on us. ☺
OTOH, maybe they just delay the transfer of heat, which would explain why Trenberth’s “missing” heat isn’t there – and never was.

wayne
May 9, 2011 5:45 pm

Tim Folkerts, nothing new, I disagree.

May 9, 2011 5:57 pm

wayne says at 5:27 pm: [ … ]
Wayne, I probably didn’t make myself clear enough in the post you replied to, but as you can see from my reply to Alleyne above [which I posted before seeing your comment], I am in complete agreement with your view that CO2 does no work on its own.

Tim Folkerts
May 9, 2011 5:57 pm

ferd berple says: May 9, 2011 at 4:30 pm
“N2 doesn’t radiate significantly in the IR portions of the spectrum.”
How is that possible? Every object radiates depending on its temperature. It may not absorb at certain frequencies, but how can it avoid radiating in IR if that is its temperature?

Not only is it possible, it is REQUIRED! Any table of IR emissivity applies equally well to the object absorbing IR radiation or to it emitting IR radiation.
Take two identical objects; one with a low emissivity and the second with a high emissivity. Warm them both above room temperature and set them out in the cooler room. The high emissivity object will cool faster because it is emitting more IR than the low emissivity object.
Or do you think the low emissivity object will cool at the same rate because “but how can it avoid radiating in IR if that is its temperature?” In fact, here is someone who did this experiment and posted the results here. http://blog.khymos.org/2007/03/
It is true for metal pots. It is true for gases in the atmosphere. How well they absorb is directly related to how well they emit.

Tim Folkerts
May 9, 2011 6:08 pm

Richard M asks: May 9, 2011 at 4:16 pm
Therefore, the Earth loses energy at times to the atmosphere and gains energy at other times. However, just like in the case of radiation, the net flow is most likely to the atmosphere. Has anyone measured this heat flow? I’ve never seen it documented and yet it seems important if we are to understand the big picture.

The transfer of heat by collisions between the surface and the atmosphere is the “thermals” term. Air is a very excellent insulator and has a low heat capacity, so energy transfer by conduction is very limited. However, when the ground warms a layer of air near the surface, it expands and rises, carrying away energy from the surface.

Tim Folkerts
May 9, 2011 6:10 pm

wayne says: May 9, 2011 at 5:45 pm
Tim Folkerts, nothing new, I disagree.

I agree that I said nothing new — I think it is all very standard, well-established physics. What specifically do you disagree with in the discussion of …
εσ( 288K^4 – 280K^4 ) vs. εσ( 288K^4 ) – εσ( 280K^4 )?

philincalifornia
May 9, 2011 6:12 pm

Joel Shore says:
May 9, 2011 at 1:55 pm
So, I was simply saying that the generally-accepted projections for AGW are that positive feedbacks amplify the radiative effects due to greenhouse gases alone but do not lead to an actual “runaway” instability. For some reason that I don’t totally fathom, some people on the skeptic side (like martin mason above or Smokey in other threads) seem to like to group together this prediction of feedbacks amplifying things and producing an actual “runaway” instability, perhaps because it makes a better “strawman” argument to attack.
—————————————————–
But Joel, you used to argue that on here, and quite vehemently as I recall. In fact I distinctly remember proposing the Joel Shore Uncertainty Principle whereby if what Joel Shore proposed was true, there would be no Joel Shore to observe it.

Tim Folkerts
May 9, 2011 6:36 pm

Alleyne says: May 9, 2011 at 4:48 pm
Where is the work, the external energy, which causes energy to flow from the atmosphere to the earth, from cold to warm?

Work would indeed be required for a NET transfer of energy from cold to warm (eg an air conditioner) and you very correctly deduce this would not happen. However SOME energy can be transferred from cold to warm, as long as MORE energy is transferred from warm to cold (ie as long as the net transfer of energy is from warm to cool; ie as long as the heat is from warm to cool).

Joel Shore
May 9, 2011 6:37 pm

philincalifornia: I am afraid you must be misremembering. What I have argued here is that the most likely range of the climate sensitivity is what the IPCC says the most likely range is…which corresponds to feedbacks amplifying the radiative effect due to CO2 alone by a factor of about 3 +/- 1.
I have also consistently said that I am skeptical of Hansen’s claim that we could trigger a runaway effect. I know that because I have used it as an example to challenge Smokey to display some true skepticism by telling me what arguments on “HIS side” he is skeptical of. (Another example I gave where I remain skeptical is on the effect of AGW on hurricane intensity.) As I recall, he never took me up on it.

Martin Lewitt
May 9, 2011 6:39 pm

Joel Shore,
The water cycle feedback is about more than just the clouds and water vapor, it is about the evaporation and precipitation as well. Wentz (2007) documented that the models represented only one-third to one-half the observed increase in precipitation associated with the recent warming. Lindzen mentioned a paper which I wish I remembered that confirmed the increased precipitation with measurements of the latent heat flux (anyone save the cite?). The turning of the water cycle is like a big air conditioner with the water as the working fluid. At these new levels of relative and absolute humidity associated with the warming the air conditioner or heat pump turns faster. Wentz found it was turning faster in proportion to the humidity increase, while in the models it is one half to one third of that. While the water vapor feedback is positive, the net feedback of the whole water cycle may well be negative. This is the area in dispute in which the science and the models need to progress.

suricat
May 9, 2011 6:58 pm

Hi Ira.
“Also, the Petty black body curves peak at a “Radiance” around 100 mW/m^2/sr cm^-1 while the black body curves from Carleton peak at an “Intensity” of around 14 W/m^2/mm.”
As engineer to engineer. 🙂
A “radiance” is a ‘comparison’ to an accepted ‘standard radiance’ (don’t ask me what frequencies this involves, as that’s defined by ‘the standard’), but an “intensity” is an ‘absolute measure’ of ‘energy’ (don’t ask me how the ‘Planck constant’ is considered there) and seems to be commonly used with a specific spectral wavelength. I hope this helps.
It would be nice if someone were to construct a ‘table of resonant interaction’ for ‘atmospheric molecules/local EM field’. However, this would be a stratospheric altitude + project because LW photon emission hardly occurs in the troposphere (where it’s mostly molecular disturbance), meaning that this would come under the heading of insolation for the ‘tropo’. If only we could have the ‘unit measure’ the same.
Best regards, Ray Dart.

jae
May 9, 2011 7:37 pm

Joel Shore:
“philincalifornia: I am afraid you must be misremembering. What I have argued here is that the most likely range of the climate sensitivity is what the IPCC says the most likely range is…which corresponds to feedbacks amplifying the radiative effect due to CO2 alone by a factor of about 3 +/- 1”
WHAT??
Maybe I misunderstand this?? Is an “ampliyfing factor of MINUS ONE actually an AMPLYFIYING FACTOR?? Just how does this brand of logic work (outside of “climate science?”) AN “AMPLIFYING FACTOR OF NEGATIVE ONE????” WTF??? This reeks of “Kinetic Military Action,” “Overseas Contingency Operation,” “Climate Disruption,” and all the other blatant Orwellian Communistic propaganda crap from the “progressive” sickos.
Are you sayin that IPPC actually says that there may be a …..NEGATIVE…..feedback? If so, just WHERE does that occur in the IPPC narritive? I must have missed that.
BTW, you warmist “guys” are losing it, BIG TIME, IMHO. I detect some kind of desperado talk.
Why don’t you just speak the native tongue?
AND, BTW, it has now been shown that a very high percentage of the “literature” cited by the IPPC is Sierra Club -type political propaganda.
Exit question: can any thinking person trust the IPPC?

Richard M
May 9, 2011 7:45 pm

Tim Folkerts says:
May 9, 2011 at 6:08 pm
The transfer of heat by collisions between the surface and the atmosphere is the “thermals” term. Air is a very excellent insulator and has a low heat capacity, so energy transfer by conduction is very limited. However, when the ground warms a layer of air near the surface, it expands and rises, carrying away energy from the surface.

Thank you, I believe that is the net transfer of energy. However, I suspect the total transfer of energy is much higher. The fact air works like an insulator may be due to energy going both ways easily, not that energy is not transferred. That is, if we measure the transfers in both direction we would see huge w/m2 going both ways but only a small difference which is identified as thermals. In any event the thermals term is bigger than the difference in total IR radiation.
Now, the diagram does show the total IR radiation going both ways instead of just the small difference. Why are these treated differently? Well, most likely it is to highlight the back-radiation. But, think about this, if back-radiation heats the surface then thermal radiation toward the atmosphere should increase. If we don’t understand the total effect of thermal radiation then how can we understand the bigger picture?
Also, I’m still waiting for anyone to explain why the cooling effect of GHGs is always ignored.

jae
May 9, 2011 7:46 pm

Dammit, I screwed up in the last comment. I misunderstood the 3+/-1 figure. Please disregard all but the last half of the comment and the nasty attitude. The morons are now fighting a losing battle and are getting bolder and bolder and stupider and stupider, to their detriment. Brings tears to my eyes.

jae
May 9, 2011 7:58 pm

IRA GLICKSTEIN:
I consider it very strange, even weird, that you have not responded to or even acknowledged the other explanations about why the temperature on the surface of the Earth is 33 C higher than it “should be.” You are supporting the 19th century Ahrennius apostles, you know. Have you read and understood the articles that make mincemeat of your theory?
Anyway, the silence is deafening. Do you not have any opinion about this? Are we beneath your contempt? WHAT? You act like a frigging Democrat.

wayne
May 9, 2011 8:24 pm

Tim Folkerts says:
May 9, 2011 at 6:10 pm
wayne says: May 9, 2011 at 5:45 pm
Tim Folkerts, nothing new, I disagree.
I agree that I said nothing new — I think it is all very standard, well-established physics. What specifically do you disagree with in the discussion of …
εσ( 288K^4 – 280K^4 ) vs. εσ( 288K^4 ) – εσ( 280K^4 )?

Thanks for the very civil reply. I was getting ready for a pie in my face!
I need to let you know where I am now coming from, I keep getting acclimated to these areas of atmospheric physics which I had never spent much time getting in the depths, but it is all coming back slowly.
When you get into spectrometry back comes the E/M waves and all of the associated equations governing the transfer of energy via radiation. The standing waves, traveling waves, Poynting complex vectors that I remember going over years ago. You see, I don’t remember matter being able to continually shed energy (continuous radiation) without a lose of temperature, energy will never transfer unless it should and then it must by temperature gradients always keeping in mind that we are speaking in an LTE environment along with the equipartition of the degrees of available freedom, that I will call these microstates for brevity.
To my older knowledge there are standing waves in the complex plane that do not act as nodes on a vibrating string that most, in including me, tend to visualize it as, but that is flawed without including the imaginary portion. These standing waves, I have them right here in my room where all is at the same temperature, do not transfer energy unless one molecule has less microstates than the other molecules. That IS directly related to the temperature. The molecule with less free microstates will radiate to a molecule only if the destination molecule has more free microstates to accept the energy. Otherwise, you are just in a resonating state with no real energy being transferred as standing wave imply.
Now that example is rather tinker-toy for in zillions of molecules there is always some misbalance of available microstates held by infrared enabled molecules (the GHGs) and so some energy is always being radiating but always in one net direction, here’s where we seem to disagree, to me in reality, always warm to cooler. Many professors at esteemed universities, MIT, Stanford, Cornell, Oxford, etc, do agree with my view of reality in energy transfer by radiation, some do not, they tend to be in the climate and ecology required courses.
So, I have traced one big disagreement backwards to this very important point, does matter actually radiate energy that due to the temperatures does not have to violating the Principle of Minimum Entropy Production while at the same time maintaining the Maximum Energy Dissipation Principle always from warmer to cooler surfaces or matter at a microstate level.
Prove me wrong, I’m having trouble doing that to myself.
So: εσ( 288K^4 – 280K^4 ) vs. εσ( 288K^4 ) – εσ( 280K^4 )? I do see a disconnect at this very point in the way you visualize what is actually happening and that, to me, is why a huge Fresnel lens in a dark room no matter how hot it is will never be able to focus “back radiation” and cause damage. To me it is a manufactured concept and wrong, the traveling wave must manifest itself and they do when effective temperatures differ (the real description is very much deeper, see QED and why photons do not appear in space where logically you would say they should, or even must but don’t).

May 9, 2011 8:38 pm

Maybe I’m missing something but I thought the calculated black body temperature of the earth seemed low. I recalled the moon having extremes of temperature and a bit of Googling revealed peak equatorial temperatures on the moon of 380 K. Of course, given the lack of an atmospheric buffer, the non-sun exposed side of the moon can get close to absolute zero in shaded spots.
So, if we assume that the peak temperatures on the earth would be the same as on the moon, it would appear that the primary role of the atmosphere is to cool the earth. I have to admit it’s been 35 years since I’ve taken a physics course but there appears to be something amiss here.
To follow up with my previous post, daytime straight up in the air temperature -5.5 F when ground temperature was 65 F with readings done about 30 minutes ago.

Charlie Foxtrot
May 9, 2011 8:45 pm

To wayne regarding the Fresnel lens experiment at room temperature.
The critical factor you missed in your thought experiment is that the radiation from the wall is not all approaching the lens in parallel beams. The lens will only focus the heat radiation at the target that came from a spot on the all that is directly perpendicular to the surface of the lens. In order to focus the entire heat radiation from the wall onto the target, all of the radiation from the wall would need to be focused in parallel beams first, before it hits the lens. The best way to accomplish this would be to move the wall a long distance away, and heat it up so that the radiation from it reaching the plane of the lens is equal to the total radiation coming from the wall at room temperature. The water in the target would then heat up nicely, and the laws of thermodynamics are satisfied.

May 9, 2011 8:50 pm

Joel Shore;
the rest of you should consider carefully reading what Dave, David Hoffer, Ira, myself and others are writing here and actually trying to understand the correct science.>>>
Joel, I am going to take that as a compliment, and my hat is off to you, you and Dave Springer have been doing yeoman’s work in this thread. Your explanations have been bang on and I have to admit I’ve gotten a bit ill tempered in this particular topic of late. Your patience and clarity is appreciated to no end by me as a skeptic. Pardon me, a RAGING skeptic. For those skeptics who want to be part of winning the debate with the warmists, I have this suggestion:
On the matter of “backradiation” and “greenhouse gases”, I recommend that you accept Joel Shore’s explanations at face value. CAGW is a farce of gigantic proportions, perpetrated by incompetant and/or fraudulent science and I have firm and specific reasons for saying that. Though a warmist he may be, I detect neither incompetance nor fraud in any of his explanations in this thread. And as I am certain Joel will tell you, if I did, he would hear about.
Joel is explaining physics that is used and verified by design engineers by the thousands every single day, all over the world. As are Ira and Dave Springer and others. Some of the arguments that I’ve seen presented in this thread amount to no more than saying “the earth is flat, just LOOK at it”. Yup, I looked, and yup, it looks flat, and NOPE it isn’t.

Martin Lewitt
May 9, 2011 9:35 pm

Boris Gimbarzevsky,
Your reasoning from the lunar daytime temperature is wrong, that temperature has more to do with the heat capacity of the surface and the insulating properties of regolith. Even here on earth it gets hot enough on the hood of a dark colored automobile to fry eggs, even though the ambient temperature is much lower. See David Springer’s comment above for a better consideration of the lunar analogy.
http://wattsupwiththat.com/2011/05/07/visualizing-the-greenhouse-effect-light-and-heat/#comment-655571
BTW, thanx again for your temperature readings. I think the ones at night will be the most interesting, especially if you can also report the humidity along with the ambient temperature.

Tim Folkerts
May 9, 2011 11:30 pm

wayne says: May 8, 2011 at 3:01 pm
“Now, ready? If back-radiation proponents are correct, we should be able to use this huge lens to focus back-radiation from any warm wall within your house, at the ambient room temperature of say 20 ºC (68 ºF), in some room to boil water in seconds.”
No, Wayne, you are misunderstanding the physics involved. I can’t off-hand remember the name of the theorem involved , but you can’t focus the spot any hotter than the surface creating the light. (Similarly, the spot focused by a magnifying glass aimed at the sun can never be hotter/more intense than the light at the surface of the sun.)
So no matter how thermal EM photons are focused, they can not make the surface they approach any warmer than the surface emitting the photons. The IR from the atmosphere could never (by itself) make the surface any warmer than the atmosphere. And the light from the sun could never make any part of the earth any warmer than 5700 K no matter how it was focused. Of course, the IR from the atmosphere PLUS some energy from the sun could warm the earth above the temperature of the atmosphere.
(PS I don’t have time to address the other question of εσ( 288K^4 – 280K^4 ) vs. εσ( 288K^4 ) – εσ( 280K^4 ) .. i”ll have to see about doing that some other time.
I’ll also have to see about addressing Jae’s concerns about Venus vs Earth vs Mars re the GH effect. Lapse rate is important, but I don’t believe it is the whole story — along the lines that some people have already mentioned.)

May 9, 2011 11:54 pm

JAE says:
May 9, 2011 at 11:28 am
the lapse rate depends upon only Cp and g. ??
The Dry Adiabatic Lapse Rate (DALR) depends upon only Cp and g. The actual (environmental) lapse rate depends on other things and varies through out the day.

May 9, 2011 11:56 pm

ferd berple says:
May 9, 2011 at 1:27 pm
My question is why we assume that N2 in the atmosphere doesn’t back radiate
Because it has been measured and N2 does not radiate at the temperatures normally found at the surface.

May 10, 2011 12:25 am

wayne, I think you are right, I misread what you said.
However, I stand by my comment that the temperature of the clouds has little effect on the surface temperature. On cloudy nights, the back radiation is almost the same as the radiation emitted from the surface. This is either because the clouds are the same temperature as the surface or because clouds simply scatter the surface radiation back toward the surface. Since clouds emit both up and down, they should be able to emit in each direction only half what they receive. In addition, since the water vapor mixing ratio generally decreases with altitude, if the surface temperature was the same as the cloud temperature, then there would be fog. Therefore, since the energy emitted toward the surface equals what is received, and since there is no fog, scattering makes more sense. Unfortunately, I have not found the measurements needed to be certain about this.

David
May 10, 2011 12:54 am

Thanks to the following posters who talked about Latent Heat
Dave in Delaware
Nullius in Verba
Bryan
Dave Springer
Martin Lewitt
They’ve set me at ease for I now know I’m not the only one to have noticed that radiative models are missing a large piece of the puzzle.
Even the climate keepers at Wikipedia have allowed it to give 23 % back to the atmosphere
http://en.wikipedia.org/wiki/Earth's_energy_budget
Sensible heat flux is mentioned too (convection) with 7 %.
None of this has anything to do with greenhouse warming so comparing the blackbody example to earth now as a gauge of greenhouse warming is a pointless exercise.

Bryan
May 10, 2011 1:27 am

Joel Shore
You challenged me to discus the science.
I am here to do exactly that so you cant avoid the direct question posed below.
If you prefer to avoid the science and instead issue smears instead then I will not be surprised, as insulting others, seems to be your default mode.
Tell me which part of the query below is not science.
This item is directly addressed to Ira’s post and deserves an answer from you!
In your co-written Halpern et al comment paper you state that the Stephan Boltzmann Law can be used to work out the thermal energy exchange between atmospheric shells.
There is a whole series of statements on pages 8,9,and 10(0r 1316 to 1318 of pdf) referring to the atmosphere, Stephan Boltzmann Law and indeed including the SB formula(constant x T^4).
(The pdf below will allow readers to verify for themselves)
However there is a major problem for you!
A filtered spectrum does not vary with T^4.
The Stephan Boltzmann Law cannot be used for the filtered spectrum of atmospheric gases.
Do you now agree with me and withdraw your comments?
Or will you persist in this unphysical assumption.
Depending on your answer we can call on Ira’s spreadsheet to settle the matter.
That is, to prove that the reduced Planck function does not vary with T^4.
This has implications not just for your absurd paper but for ANY climate model with the T^4 assumption built in!
Comment on ‘Falsification Of the atmospheric CO2 greenhouse effects within the frame Of Physics’ by Joshua B. Halpern, Chistopher M. Colose, Chris Ho-Stuart, Joel D. Shore, Arthur P. Smith, Jorg Zimmermann.
http://scienceblogs.com/stoat/upload/2010/05/halpern_etal_2010.pdf

May 10, 2011 1:51 am

Charlie Foxtrot, I saw Dave Springer’s comment and it may be that the average temperature of the moon is -23 C, but what I’m interested in is the immediate surface temperature of a planet, not an extremely low pass filtered temperature. I’ve worked in the NWT in areas of permafrost and it was very nice to be able to just dig a hole in the ground to keep our food cool when the outside temperature was 90 F during the longest days of the year. The temperature a couple of feet below the ground is below freezing but what we’re interested in is the surface temperature which is where we live. During a hot summer day I have to open all of my vehicle doors for 5 minutes before I can get into the vehicle and touch the steering wheel without getting burned. What I’m trying to get at is that the extremes of temperature get smoothed out by the low pass filter of the atmosphere and that heat transfer between sections of the atmosphere is probably far more important than any minimal effects of greenhouse gasses. The frequency response of the filters is what we’re interested in — I don’t care about the very long time constant of below ground temperatures when a relatively short pulse of heat will melt plastic items in my vehicle.
As far as temperature of the chunk of air above my head goes, it seems to vary over a fairly small range. Just now stepped outside and found a minimum temperature of -12 F when the IR thermometer was vertical and humidity is given as 52% right now. Sky completely clear without a cloud to be seen and great night for stargazing but I have to work in a few hours:-(. When I’ve played around with this IR thermometer before I always seem to get a low value and it increases as one points the IR thermometer at angles of less than 90 degrees. If I was really obsessive about this I could mount the IR thermometer on a jig where I could precisely vary the angle and measure the temperature as a function of angle. I suspect this data could be used to compute a temperature profile of the atmosphere using similar math that one uses to generate an image with a CT scanner. The forecast is for rain tomorrow and will be interesting to see what kind of reading I get from the base of clouds.

R Stevenson
May 10, 2011 1:58 am

Wayne says:
‘Robert, I come up with a close conclusion, not exact, but close. However, the 3600 m is more than 36 times that is given in other conversations I have read speaking of it’s absorption very near the surface. Do you have an average absorption coefficient you are using, or a mean optical thickness for CO2 from which that is calculated? Maybe Hottel specifies emissivity direct then what equation are you using to get the 3600 m extinction distance?’
Wayne,
I read on a website (Gary Novak – Global Warming – not caused by carbon dioxide) that CO2 in air absorbs to extinction at its 15 micron peak in about 10m (Heinz Zug). I tried to confirm this figure using Plank – Hottel and got 3600m . I found that CO2 was too low at 0.038%. I used an absorptivity of 0.1905 for a hemispherical gas mass of radius L=3600m. Hottel gives CO2 emissivity direct from charts based on direct measurements of total emission. The precise method of calculating the effective absorptivity or emissivity is quite complex and is given in -(1) Trans. Am. Inst. Chem Engrs., 31, 517-549 (1935) Hottel, H.C., and H.G. Mangelsdorf. (2) Trans ASME, 57, 463-470 (1935) Hottel, H. C., and V. C. Smith.

R Stevenson
May 10, 2011 2:11 am

Wayne,
When I said CO2 too low at 0.038% I meant too low for an extinction distance of 10m or so. However using the same technique (Planck- Hottel) for water vapour which has a partial pressure or concentration 100 x that of CO2 the extinction distance is much less at 120m (absorptivity 0.5734).

R Stevenson
May 10, 2011 2:52 am

Ferd berple says:
‘My question is why we assume that N2 in the atmosphere doesn’t back radiate. All objects radiate depending on their temperature. As the atmosphere is mostly N2, that must be the source of most of the back radiation’.
Ferd
The emissive power of N2 is small and insignificant compared with the emissive power of the entire spectrum of Earth radiation .

Tim Folkerts
May 10, 2011 5:53 am

David says: May 10, 2011 at 12:54 am
…I now know I’m not the only one to have noticed that radiative models are missing a large piece of the puzzle [Latent Heat].

David, I’m not sure why this came as a surprise to you. When trying to understand radiative effects (like Ira is doing here), naturally people concentrate on EM radiation. However, more advanced models include many more effects in much more detail.
You note that “Even the climate keepers at Wikipedia have allowed it [latent heat] to give 23 % back to the atmosphere … Sensible heat flux is mentioned too (convection) with 7 %”. But, of course, those same terms were included in the Trenberth energy balance diagrams, so it is pretty clear that they have been considered important for a long time in all serious models.
http://en.wikipedia.org/wiki/Global_circulation_model gives a quick overview of how the more advanced models work.

jae
May 10, 2011 6:00 am

Ira:
Funny that you replied only to the comment which I already realized and acknowledged (next comment) was off-base. How about the other comments? Still that irritating silence from the warmers and luke-warmers.

jae
May 10, 2011 6:01 am

Robert says:
“The Dry Adiabatic Lapse Rate (DALR) depends upon only Cp and g. The actual (environmental) lapse rate depends on other things and varies through out the day.”
Yes. And your point is??

Dave Springer
May 10, 2011 6:32 am

Robert Clemenzi says:
May 9, 2011 at 11:56 pm
“Because it has been measured and N2 does not radiate at the temperatures normally found at the surface.”
ALL matter with a temperature above absolute zero radiates. This is one of the most fundamental laws of physics. Either you are making things up out of thin air or the source you got it from doesn’t know his ass from elbow.

Dave Springer
May 10, 2011 6:49 am

wayne says: May 8, 2011 at 3:01 pm
“Now, ready? If back-radiation proponents are correct, we should be able to use this huge lens to focus back-radiation from any warm wall within your house, at the ambient room temperature of say 20 ºC (68 ºF), in some room to boil water in seconds.”

You can’t heat anything to a higher temperature than the temperature of the photons doing the heating. If you used a parabolic mirror aimed at a warm wall to heat water you could heat it to the temperature of the wall then the heating stops because the object being heated, now at the same temperature as the heat source, emits back as much radiation as it receives. It’s precisely because of back radiation that you can’t heat something hotter than the source of the heat.

Dave Springer
May 10, 2011 7:43 am

Retired Engineer says:
May 8, 2011 at 8:22 am
This makes sense. Absent CO2 and water vapor, we would freeze. How much warming does each contribute? If CO2 did it all, and mankind producses about 3% of the CO2, then we are responsible for 3% of the 33C warming or about 1C. Thus AGW is real. Except for water vapor which may cause 90% of the 33C, in which case we cause 0.1C warming.

Things are more complicated than that. Water vapor is a condensing greenhouse gas. In frigid conditions most of it gets frozen out of the air. Antarctica has by far the dryest air on the planet’s surface. A second complication is that the IR absorption response of CO2 (and water vapor too) isn’t linear except at low concentrations (see my apple orchard vs. number of pickers in the field analogy) which is one of the surprising things John Tyndall discovered 150 years ago. Complicating things yet more is the albedo of water changes drastically between liquid and solid phase. The ocean absorbs almost all incident light while a glacier reflects almost all of it.
So. Given the earth with an albedo similar to the moon (i.e. all rocks) it would have an average surface temperature well below freezing. But even your average rock is fairly dark and absorbs near 90% of incident light. But the earth is a water world so while the global ocean has an unfrozen surface it absorbs about 10% more light than rock. But what happens if the surface ever becomes almost completely frozen over? There would be hardly any water vapor in the atmosphere for a greenhouse effect and the snow & ice would reflect 90% of the sunlight. We would have a very frigid planet with no escape.
But there is an escape hatch. If the earth is covered in snow & ice there are can be no green plants taking CO2 out of the atmosphere nor will there be any chemical processes (formation of carbonate compounds) removing CO2 from the atmosphere, nor will it be mixing with the ocean because the ocean is frozen over. In the meantime volcanoes will continue to vent CO2 into the atmosphere. CO2 level will keep on rising until the greenhouse effect from it kicks in and it starts melting ice first at the lowest latitudes and progressively higher latitudes. Water vapor again enters the atmosphere, ocean albedo changes from 90% to 1%, and the melt accelerates in one big hurry. Green plants bloom like crazy from the CO2-rich air/water and dissolved nutrients in the ocean accumulated over eons of nothing removing it.
So basically CO2 greenhouse effect is what eventually stops a permanently frozen earth. Were we farther from the sun, like Mars, where it gets cold enough for CO2 to condense we wouldn’t have an escape hatch and the earth would be a cold dead rock much like Mars.
Because CO2’s ability to absorb IR increases linearly at low concentrations (under 100ppm) a minimal amount even absent most water vapor serves to keep the earth just warm enough to prevent a snowball earth episode most of the time. It is somewhat controversial but it is generally believe that earth has had a few snowball episodes in the past and the CO2 hypothesis above is the only reasonable explanation for how it ever manages to melt once the ice takes over. The sun’s output has been gradually increasing over the billions of years and is about 10% higher now than in the distant past which is probably why there haven’t been any recent snowball episodes.
My hypothesis taken from the above is that the first 100ppm of CO2 serves as a kind of kindling that fires up the water cycle i.e. it raises the average temperature just enough so that liquid water dominates frozen water. Evidently, at 280ppm, it’s just barely enough because the earth has been in an ice age for the past 3 million years with just some small cyclic variations in eccentricty & inclination being enough to transition between glacial periods and interglacial periods with the glacial ages lasting about 100,000 years and the interglacials about 15,000 years.
Clearly the earth’s climate is at a tipping point but that tipping point is tipping back to a glacial period. Because CO2 at 280ppm is in the exponential part of its IR absorption curve it’s likely we can’t possibly enrich the CO2 content enough to end the 3 million year-old ice age but it would be great for all life on the planet if it did including us.
In any case the indisputable testimony of the geologic column reveals that in the last 500 million years (since the Cambrian explosion when most of the modern phyla appeared and Ediacaran phyla disappeared) and animals started crawling out onto land the CO2 level in the atmosphere has, most of time, been 10 to 20 times greater concentration than today. Today’s atmosphere is CO2-depleted in historic terms and life is not nearly as abundant. The normal state of affairs for the earth is complete absence of any polar icecaps – it’s green from pole to pole. The living world today is a shadow of what it is during the good times with life struggling to survive in the nutrient-poor air and through the long cold winters. We should want to be moving away from the point where the climate tips back over into a glacial period. Evidently as a group we’re too stupid to know what’s good for us.

Dave Springer
May 10, 2011 8:02 am

Actually it appears that Mars too was once a liquid water world but it didn’t last long. Mar’s problem is insufficient gravity. Gravity is what gives us our air pressure of 14.7psi at sea level. This in turn raises the boiling point of water 212F above the freezing point. Mar’s lower gravity lowers the boiling point of water at the surface a lot so its oceans literally boiled away. For a time the excessive water vapor in the atmosphere would have given it an uber-greenhouse effect but slowly but surely it was electrolized by energetic radiation and blown off by solar wind – never to return. Mars is smaller than the earth so its liquid iron core cooled off a lot faster. A liquid iron core generates a magnetic field which deflects high energy radiation. The tipping point for Mars was probably when the iron core cooled off enough to solidify and its radiation shield vanished paving the way for it to be stripped of its water.

Tim Folkerts
May 10, 2011 8:09 am

>>Robert Clemenzi says: May 9, 2011 at 11:56 pm
>>“Because it has been measured and N2 does not radiate at the
>>temperatures normally found at the surface.”
>Dave Springer says: May 10, 2011 at 6:32 am
>ALL matter with a temperature above absolute zero radiates.
>This is one of the most fundamental laws of physics. Either you
>are making things up out of thin air or the source you got it from
>doesn’t know his ass from elbow.
I think the point is that N2 doesn’t radiate APPRECIABLE amounts of energy at typical atmospheric temperatures.
The amount of energy radiated depends on the emissivity. Highly polished metal has an emissivity ~ 0.03 and will not radiate much energy. Rough, oxidized, dark surfaces often have an emissivity above 0.9. So it is easy for one material to emit 30x more radiation than another at the same temperature.
It is easy to confirm that even many meters of N2 have little effect on IR – neither absorbing nor emitting appreciable amounts. The image at the top of the post — showing the IR spectrum looking up — shows quite clearly that for most wavelengths, there is almost no IR emitted by the atmosphere. The bulk of the IR that is seen is clearly attributed to H2O & CO2. Hence the entire column of N2 overhead emits no radiation (or close enough to zero for practical practical).
The original statement is (for all practical purposes in climate modelling that I can imagine) correct.
I presented evidence for my claim. Can you present evidence that shows IR emission from N2 is appreciable in any situations?

Dave Springer
May 10, 2011 8:29 am

David says:
May 10, 2011 at 12:54 am
Thanks to the following posters who talked about Latent Heat
Dave in Delaware
Nullius in Verba
Bryan
Dave Springer
Martin Lewitt
They’ve set me at ease for I now know I’m not the only one to have noticed that radiative models are missing a large piece of the puzzle.
Even the climate keepers at Wikipedia have allowed it to give 23 % back to the atmosphere
http://en.wikipedia.org/wiki/Earth's_energy_budget
Sensible heat flux is mentioned too (convection) with 7 %.
None of this has anything to do with greenhouse warming so comparing the blackbody example to earth now as a gauge of greenhouse warming is a pointless exercise.

The toy global circulation models take latent heat flux into account so it isn’t missing although one might question the accuracy of the number or how it changes with temperature. The problem lies in how they model clouds. They believe that clouds have a net warming effect so as temperature rises from additional CO2 we get more clouds and more clouds cause even more warming. Clearly this is wrong because the end result is a runaway greenhouse and the earth has never in billions of years experienced a runaway greenhouse even when as recently as several million years ago CO2 levels were far higher than we could ever raise them through fossil fuel consumption. There just ain’t enough economically recoverable fossil fuel to get CO2 concentration into undiscovered territory. It’s questionable whether we can raise it high enough to cancel the effect of the Milankovich cycle which triggers glacial periods.
Prior to 3 million years ago Milankovich cycles were not enough to trigger ice ages. It’s thought that the arrangement of continents are the other contributing factor. Depending on how the continents are arranged the global ocean conveyor belt changes and having a land mass over a pole blocks warm water from getting at the ice to melt it. Currently we have one large continent directly over a pole and one pole with none. The water sequestered at the south pole is a large amount which reduces the surface area of the global ocean and because land is higher albedo than water it raises the planetary albedo and thus cools it. If we had a continent over both poles at the same time we’d probably get a snowball earth episode that would last until either CO2 built up in the atmosphere to melt it or the continents drifted off the poles or some combination of both.

Dave Springer
May 10, 2011 8:55 am

CO2 extinction altitude can be seen in the spectrum looking down from 20 kilometers above the arctic ocean. IIRC the spectrum follows a 270K blackbody curve where the atmosphere is IR transparent i.e. it “sees” the temperature of the ocean surface but in the 15um region it drops down to follow a 250K blackbody curve. Going by dry adiabatic lapse rate (arctic air is fairly dry) of 1K per 100 meters the IR sensor is “seeing” the air temperature in the 15um range at a height of 2000 meters. I’m not an optics expert but I believe that altitude represents the optical depth of the atmosphere at 15um or in other words the extinction altitude. In any case there is no way in hell the extinction depth is in tens of meters.
One of the concerns about increasing CO2 even if it doesn’t raise surface temperature all that much is that as it increases the extinction point will be lowered and near-surface adiabatic lapse rate will change as a result potentially causing changes in the weather but again I must go back to the indisputable testimony of the fossil record which reveals that the earth is increasingly friendly to life as CO2 rises even when it rises to 20 times the current level (greater than four doublings) so one might reasonably conclude it doesn’t cause climate disaster.

JAE
May 10, 2011 8:56 am

Dave Springer:
“ALL matter with a temperature above absolute zero radiates. This is one of the most fundamental laws of physics. Either you are making things up out of thin air or the source you got it from doesn’t know his ass from elbow.”
Huh? I hope you are not saying that everything radiates in the IR frequencies. To repeat a comment I made above somewhere:
http://www.wpi.edu/Academics/Depts/Chemistry/Courses/General/infrared.html
The key paragraph:
The fact that a molecule vibrates does not in itself insure that the molecule will exhibit an IR spectrum. For a particular vibrational mode to absorb infrared radiation, the vibrational motion associated with that mode must produce a change in the dipole moment of the molecule. HCl, for example, with a center of positive charge at the H atom and a center of negative charge at the Cl atom, has a dipole moment. The magnitude of the dipole moment changes as the HCl bond stretches, so this vibration absorbs IR radiation. We say that the vibration is IR active. The N2 molecule, on the other hand, has no dipole moment. Further, stretching the N-N bond does not produce a change in dipole moment, so the vibration is infrared inactive (i.e., cannot directly absorb IR radiation). It is important to realize that there are many molecules that, although possessing no permanent dipole moment, still undergo vibrations that cause changes in the value of the dipole moment from 0 to some non-zero value. An example is CO2, which has no permanent dipole moment because the individual bond dipoles exactly cancel. However, when CO2 undergoes a bending vibration, its dipole moment changes from zero to some non-zero value. This vibration produces a change in dipole moment and is IR active.”

May 10, 2011 9:25 am

q = k A dT / s (1)
q = σ T4 A (1)
q = k A dT (1)
Above from engineering tool box.
Here are the basic heat transfer equations. Please note there are no inputs for back radiation. NONE. Your conclusion “The only rational explanation is the back-radiation from the Atmosphere to the Surface.” defies these equations.
There are only two basic requirements for heat transfer 1. path 2. temperature difference. If either ar missing no transfer.
I did not consider shape factor, emissivity differences, etc basics as you wanted to get back to.

Joel Shore
May 10, 2011 9:36 am

Martin Lewitt says:

The turning of the water cycle is like a big air conditioner with the water as the working fluid. At these new levels of relative and absolute humidity associated with the warming the air conditioner or heat pump turns faster.

Okay…Let me explain to you why I think this argument fails…at least from what I understand of the empirical data. Basically, you are saying that the evaporation / condensation increases, transporting more energy from the surface up into the atmosphere where it can be radiated back out into space. However, since the only way that energy can escape to space (in any significant amount) is via radiation and the amount of radiation depends on temperature, what this argument amounts to is the proposition that the upper portion of the atmosphere warms more than the surface.
Now, to the extent that this is indeed expected to be true, it is already incorporated in the models via the lapse rate feedback (a negative feedback)…which says exactly this. The reason that the upper part of the troposphere is expected in the global average to warm more than the surface is that in the tropics is that one expects the lapse rate to closely follow the moist adiabatic lapse rate, which indeed implies more warming at altitude than at the surface. This is the so-called “hot spot” in the tropical troposphere.
So, what you must be proposing is that this amplification of warming at altitude relative to the surface happens to a greater degree than the models predict. However, the data for the tropics currently show, if anything, just the opposite. I.e., people are complaining that the “hot spot” is missing. Whether it is really missing and the degree to which it is missing in fact depend on which analysis of the satellite data and radiosonde data one believes…and there are good reasons to believe there are problems with the data and that the models are basically correct. However, your picture would actually require that the “hot spot” really be more pronounced than the models predict, which seems rather unlikely based on what we do see in the data thus far.
Alternately, I suppose you could argue that the models are correct (or overestimate the hot spot) for the tropics…but that they underestimate the warming at altitude (relative the surface) outside of the tropics by such a degree that the magnitude of the lapse rate feedback is still underestimated globally. However, as far as I know, the empirical satellite and radiosonde data, such as it is, don’t really support that idea either.
So, that is my basic complaint about the argument that a faster water cycle somehow provides a negative feedback: To the extent that this expected to be true, it is incorporated into the models. If you want to argue that the models underestimate the effect, this has testable consequences for the temperature distribution in the atmosphere…And, at this point, the data for the temperature distribution do not seem to support this conclusion, if anything going the other way.

Tim Folkerts
May 10, 2011 9:42 am

mkelly,
You needed to look a little farther and understand what you are reading. This is from the SAME source you quoted:

Net Radiation Loss Rate
If an hot object is radiating energy to its cooler surroundings the net radiation heat loss rate can be expressed as
q = ε σ (Th^4 – Tc^4) Ac (3)
http://www.engineeringtoolbox.com/radiation-heat-transfer-d_431.html

Please note there ARE inputs for back radiation (the Tc^4 term).

Charlie Foxtrot
May 10, 2011 10:00 am

Ref. mkelly:
May 10, 2011 at 9:25 am
There are no inputs for back radiation because it is taken care of by dT. No back radiation, then dT is simply the temperature of the emitter.
It might be better to consider the back radiation from the atmosphere as a reduction in dT in order to use standard nomenclature. Using dT simplifies the calculation, esp. if emissivity is ignored. However, calling it back radiation clarifies the attribution to greenhouse gases, and allows the separate consideration of the back radiation from various concentrations of different gasses. Emissivity, of course, is a critical quality since it is this factor that makes CO2 and H2O significant contributors to the greenhouse effect, whereas N2 is not regardless of temperature (ignoring conduction, of course).

Joel Shore
May 10, 2011 10:31 am

Bryan says:

A filtered spectrum does not vary with T^4.
The Stephan Boltzmann Law cannot be used for the filtered spectrum of atmospheric gases.
Do you now agree with me and withdraw your comments?
Or will you persist in this unphysical assumption.

This has implications not just for your absurd paper but for ANY climate model with the T^4 assumption built in!

Okay, let’s try this one more time:
When you choose what model you use, the model chosen is not independent of the purpose. If I were choosing a model to describe with as much quantitative fidelity as possible the greenhouse effect in the earth’s atmosphere, then the model I would choose would be a state-of-the-art convective-radiative transfer code using the actual composition and empirical absorption / emission lines for the atmospheric constituents. Such models most definitely do not just assume a the Stefan-Boltzmann Equation, as they do the calculation line-by-line (i.e., absorption line by absorption line).
However, our purpose in our reply to G&T was not to model the greenhouse effect with as much quantitative fidelity as possible. Instead, it was just about as far as you could get from that in the other direction: We wanted to have the simplest models possible that illustrate the effect that G&T seemed to imply violated the 2nd Law of Thermodynamics, so simple in fact that nobody could seriously argue about whether or not we had solved them correctly because anybody could solve them on the back-of-an-envelope.
So, that is what we came up with —A few very simple models, such as the one that involves 3 objects: one object A producing thermal energy and radiating energy at a fixed rate, two other objects B and C whose temperature is determined via radiative balance with object A and empty space, with a geometry such that the temperature of object B is higher than that of object C. And, what we wanted to illustrate is that the object C “warms” B in the colloquial sense of the word…i.e., that the presence of object C causes B to be at a higher temperature than if C is absent. By analogy, it then follows that if you choose A to be the sun, B to be the earth’s surface, and C to be the atmosphere, then one can immediately see the fallacy in claiming that “the greenhouse effect violates the 2nd Law because it implies heat flows from the colder atmosphere to the warmer surface” (that’s a paraphrase). In fact, the heat flows in our models are easy to calculate, so it is easy to verify that the 2nd Law is satisfied, but that, nonetheless, the presence of the colder object C causes the temperature of B to be warmer than in its absence.
If you were worried about the issue of whether the T^4 dependence is critical in producing our result, it is not be hard to assume some more general dependence and show that as long as the amount radiated by the object increases with temperature, it doesn’t matter exactly what the temperature dependence is. Such are the advantages of simple models! (Note, by the way, that what is true for a radiating object is that the amount of radiation emitted AT ANY PARTICULAR WAVELENGTH is an increasing function of the temperature, a fact that is not always obvious because people often tend to normalize the emission curves when showing emission curves for different temperatures on the same graph.)

Joel Shore
May 10, 2011 10:41 am

Dave Springer says:

The toy global circulation models take latent heat flux into account so it isn’t missing although one might question the accuracy of the number or how it changes with temperature. The problem lies in how they model clouds. They believe that clouds have a net warming effect so as temperature rises from additional CO2 we get more clouds and more clouds cause even more warming.

This is not correct. The models…and empirical data…show that clouds have a combination of warming and cooling effects (depending on such details as the altitude and optical thickness of the clouds) but that the net effect of clouds on the earth is currently cooling. (I believe the magnitude quoted is on the order of 20 W/m^2.)
Also, what the models show in terms of what happens with clouds is complicated. In general, to the extent that high clouds increase and low clouds decrease in the models, then the cloud feedback will be positive. You can find somewhere or other an analysis of the various effects due to clouds that are predicted by various models. They usually separate it out as the effect of the cloud feedback on shortwave radiation (i.e., radiation from the sun) and the effect of the cloud feedback on longwave radiation (i.e., radiation from the earth).

Clearly this is wrong because the end result is a runaway greenhouse and the earth has never in billions of years experienced a runaway greenhouse even when as recently as several million years ago CO2 levels were far higher than we could ever raise them through fossil fuel consumption.

A positive cloud feedback does not imply a runaway greenhouse effect. Ray Pierrehumbert would say that the reason why we cannot have a runaway effect on the earth with the sun at its current luminosity is actually quite well-understood. As I noted above, Hansen seems to feel differently, although the details of his reasoning remains obscure.

May 10, 2011 10:43 am

R Stevenson says:
May 10, 2011 at 1:58 am
I tried to confirm this figure using Plank – Hottel and got 3600m
To get a good figure, you must use the HITRAN data. At 15 cm-1, 99.7% is absorbed in less than 10 meters. At 14.97 cm-1, 99.7% is absorbed in less than 0.8 meters. Both of those measurements are with bandwidths of about 1 cm-1. With a bandwidth of 30 cm-1, 99.7% is absorbed in under 150 meters. To get 3.6 km you must be using a very wide bandwidth.

Joel Shore
May 10, 2011 10:47 am

[I’ll just quickly add what I do know about why Hansen says that a runaway could potentially occur now while it hasn’t in the past: One point he makes is that the sun has slowly gotten brighter, so CO2 levels back several hundred million years ago or more are not directly comparable to levels now with the current luminosity. Another point he makes regards the speed of the change in CO2 levels and that somehow overwhelming negative feedbacks that would otherwise occur if the change in CO2 levels happened over a longer timescale. I am not saying that he has compellingly shown that these two points materially change things, but just wanted to explain why he feels that something that never happened in the past could happen now.]

R Stevenson
May 10, 2011 10:48 am

Dave Springer says:
May 10, 2011 at 8:55 am
CO2 extinction altitude can be seen in the spectrum looking down from 20 kilometers above the arctic ocean. IIRC the spectrum……
Dave Springer,
What is IIRC?
[Reply: IIRC generally means “if I remember/recall correctly,” or similar. This is worth bookmarking. ~dbs, mod.]

May 10, 2011 11:04 am

Tim Folkerts says:
May 10, 2011 at 9:42 am
mkelly,
You needed to look a little farther and understand what you are reading. This is from the SAME source you quoted:
Mr. Folkerts, you need to read what I wrote regarding the two conditions for heat transfer. Path and temperature difference or gradient. Temperature is not back radiation and trying to name it such does not make it so.
Further please note I said “basic” per Ira’s want to go back to basics. Until you can show back radiation as an input for heat transfer then use the proper fromula and stop renaming things.

richard verney
May 10, 2011 11:08 am

Dave Springer you say at May 10, 2011 at 6:49 am
“You can’t heat anything to a higher temperature than the temperature of the photons doing the heating. If you used a parabolic mirror aimed at a warm wall to heat water you could heat it to the temperature of the wall then the heating stops because the object being heated, now at the same temperature as the heat source, emits back as much radiation as it receives. It’s precisely because of back radiation that you can’t heat something hotter than the source of the heat.”
///////////////////////////////////////////
Dave,
What is the temperature of a CO2 re-radiated photon?
What is the (average) temperature of the photons that supposedly downwell about
330 W/m^2 (according to the Trenberth diagram)

George E. Smith
May 10, 2011 11:28 am

“”””” suricat says:
May 9, 2011 at 6:58 pm
Hi Ira.
“Also, the Petty black body curves peak at a “Radiance” around 100 mW/m^2/sr cm^-1 while the black body curves from Carleton peak at an “Intensity” of around 14 W/m^2/mm.”
As engineer to engineer. 🙂
A “radiance” is a ‘comparison’ to an accepted ‘standard radiance’ (don’t ask me what frequencies this involves, as that’s defined by ‘the standard’), but an “intensity” is an ‘absolute measure’ of ‘energy’ (don’t ask me how the ‘Planck constant’ is considered there) and seems to be commonly used with a specific spectral wavelength. I hope this helps. “””””
Well units of ” mW/m^2/sr cm^-1 ” are a little bit screwy; and need some brackets to properly set up.
Izzat (mW) / (m^2/sr cm^-1); or is it (mW) /(m^2. sr. cm^-1). I think it is more likely to be (mW/cm^-1) / (m^2. sr). And the proper term is “spectral Radiance”, not “Radiance” because of that , /cm^-1 (per wave number increment) . And it represents Radiant energy in a small frequency (wave number) increment emitted from a small element of surface area (/m^2) into a small elemental solid angle (/sr) to some particular direction in space.
And it is NOT equivalent to “Intensity” which is a property of a point source; so there is no per unit area of the source involved.
We can reasonably talk about “intensity” when dealing with radiation for distant stars; but certainly not when referring to our local star. Well from a computational accuracy point of view, the measure of intensity is accurate to better than 1% so long as you are more than 10 emitting surface diameters away from a non-point source; but the point is there is NO per unit area section in an Intensity specification; but there is in Radiance, or Spectral Radiance, which these graphs properly are, and also in “emittance” which is simply W/m^2 without regard for directional properties, or wavelength or frequency properties, which would intorduce the spectral terms at least.
“Radiance” without the “spectral” carries no information regarding frequency or wave number or wavelength or photon energy or any other such parameter, it simply measures total energy (Watts) emittted into some small solid angle (steradians) at any direction in space, from some small elemental surface area of the radiator. It’s photometric analog, for visible light conditions would be Luminance, or some like to use the term “Sterance” (not me). and “Brightness” would be a common colloquial and somewhat terrifying term, that we should all avoid like the plague; but we won’t, so we should only use it among friends; who can understand what it is we reqally mean. But the lay person will have no idea what exactly we mean.
And the shift in the peak of the BB Spectral Radiance when we change from frequency or wave number over to a wavelength scale, is simply due to that switch from

R Stevenson
May 10, 2011 11:29 am

Robert Clemenzi says:
To get a good figure, you must use the HITRAN data. At 15 cm-1, 99.7% is absorbed in less than 10 meters. At 14.97 cm-1, 99.7% is absorbed in less than 0.8 meters. Both of those measurements are with bandwidths of about 1 cm-1. With a bandwidth of 30 cm-1, 99.7% is absorbed in under 150 meters. To get 3.6 km you must be using a very wide bandwidth.
Robert,
I integrate between spectral band wavelengths of 12.5 to 16.5 microns for CO2.

May 10, 2011 11:35 am

Joel Shore says:
“…I am not saying that [Hansen] has compellingly shown that these two points materially change things, but just wanted to explain why he feels that something that never happened in the past could happen now.”
If it were not for these endless “what if” scenarios, climate alarmists wouldn’t have much to say.
And the models predicting a tropospheric “hot spot” have been falsified to the point that the backing and filling action now reverts to the stratosphere. But as Prof Richard Feynman makes crystal clear: if the models are contradicted by observations, the models are wrong.
Joel Shore might want to take a few minutes off from writing his lengthy blog comments throughout the work day, and go look out the window. He would see that the climate is normal, and well within its past parameters. The models are wrong, AGW is wildly exaggerated, and the only effect from increased CO2 is on plant growth.

wayne
May 10, 2011 11:35 am

Robert Clemenzi says:
May 10, 2011 at 10:43 am
R Stevenson says:
May 10, 2011 at 1:58 am
I tried to confirm this figure using Plank – Hottel and got 3600m
To get a good figure, you must use the HITRAN data. At 15 cm-1, 99.7% is absorbed in less than 10 meters. At 14.97 cm-1, 99.7% is absorbed in less than 0.8 meters. Both of those measurements are with bandwidths of about 1 cm-1. With a bandwidth of 30 cm-1, 99.7% is absorbed in under 150 meters. To get 3.6 km you must be using a very wide bandwidth.
—–
Excuse me barging in but thanks Robert. I thought that very short mean path length strictly inc CO2’s frequencies was correct. Probably best just converted to tau which used the mean free path as the units maintaining the logrithmic nature of the absorbtion depths. Thanks again, that shores up everything I have been pointing out earlier in these threads in releation to that subject. If the 3600 m had been correct, I would have had to retract much I had claimed earlier, whew!

Tim Folkerts
May 10, 2011 11:46 am

mkelly,
I’m not sure we are on the same wavelength. I agree with you that “path and temperature difference” are important parameters (especially for conduction). Thermal conductivity is also important, as is geometry. For radiation, emissivity is important, as are the specific temperatures (not just the difference). I think we are in agreement so far.
You said “Until you can show back radiation as an input for heat transfer then use the proper formula and stop renaming things.”
I believe this is indeed the proper formula (give or take a few details like you mention — eg the geometry of the objects and the emissivities of the two different materials.) (And this equation is actually the RATE of energy transfer, which would more commonly be labeled q/t)
q = (ε σ Th^4 A) – ( ε σ Tc^4 A)
or in words
(net energy flow from radiation)
= (“out radiation” energy from the object) – (“back radiation” energy to the object)
I’m not renaming anything. “Back radiation” (which is a function of the temperature of the surroundings) IS an input for finding the heat transfer. It is 1/2 of the equation. I realize you said “basic”, but “basic” cannot involve dropping 1/2 of an equation where both halves are important.

Bryan
May 10, 2011 12:39 pm

Joel Shore
I said …” A filtered spectrum does not vary with T^4.
The Stephan Boltzmann Law cannot be used for the filtered spectrum of atmospheric gases.”…..
You now agree with me and withdraw this unphysical assumption.
This has implications not just for your absurd paper but for ANY climate model with the T^4 assumption built in!
Its of interest that you do not try to defend the Filtered T^4 atmosphere model that you presented in the article.
Your talk of “simplification” and ” using heat in the colloquial sense of the word”.
Your audience were physicists who must have cringed at the naive mistakes in your article.
I would imagine that virtually all of them knew the thermodynamic meaning of heat.
It is quite clear from your article that none of the Halpern et al team had any clue as to what heat meant.
What was more disturbing is that none of the six of you could read properly.
It is an almost unique example of group delusion!
All of you came to the conclusion that G&T were of the opinion that colder objects could not radiate to hotter objects .
Surely at this point one of you who actually read the paper should have said……
“Hold on, there are a number of diagrams showing two way radiative interaction”.
Another might have said “G&T have an extensive discussion on the Earths two way radiative response to the Sun”.
Perhaps it is your view that your articles comments about HEAT and T^4 should not be taken seriously.
I think your physics audience would take your remarks seriously.
They would form a judgement on Halpern et al based on your comments.
However from now on I will keep in mind that you should not be taken too seriously.

ferd berple
May 10, 2011 1:43 pm

Here is a thought experiment. If we hold everything else equal and double the amount of N2 in the atmosphere, because N2 does not participate in the back radiation, the surface equilibrium temperature should remain “unchanged” according to the radiative transfer model.
However, comparisons of earth and venus suggest that the temperature of the atmosphere will only remain unchanged at 1 atmosphere pressure. As we now have more atmosphere, the pressure at the surface will be increased as compared to present. Under the gravitational model, the surface temperature would be expected to increase as a result of doubling the N2 in the atmosphere.
Which of these two is correct? Adding N2 will not increase temperature, or adding N2 will increase temperature (holding all else equal)?
The problem for me is the lapse rate. Doubling the N2 in the atmosphere will raise the height of the atmosphere. If the surface temperature does not increase, then the lapse rate dictates that since the atmosphere extends higher, that further up, past the present altitude it must get colder than it is at present. That seem nonsensical.
It seems more likely that the top of the atmosphere will remain relatively unchanged, and due to the laspe rate the temperature at 1 atmosphere will be largely as it is now, and deeper down in the atmosphere towards the surface, temperatures will increase. This tells me that the surface temperature is a function of the amount of atmosphere, not (just) the specific gasses, as confirmed by comparison of the atmosphere of verus and earth.

May 10, 2011 1:46 pm

Ira,
You haven’t answered so I’ll give it another try.
Your attribution of the missing 33K from 240W radiation from earths system to “green house effect” assumes an atmosphere that is at 0K AND assumes no reflection (as I understand you, please correct me if I’m wrong). But even without GHGs the atmosphere must be rather warm (the dry adiabatic lapse rate is one lower limit, sensible heat and latent heat is another reason). The atmosphere is also reflective, Trenberth accounts close to 38% of longwave radiation from the ground to be reflected. Third, the atmosphere is heated by sunlight. Again according to Trenberth 78 W/m^2 is absorbed, i.e. 78 W/m^2 of SUNLIGHT is heating the atmosphere.
Let’s assume (for the sake of argument) that the average optical depth is 7 km in the GHG free atmosphere (vs ca 5 km in our). DALR gives us a temperature of roughly 203 K at this point.
If Trenberth is right on 38% reflection, 78 W is heating the atmosphere from above of which half will radiate downwards, and pure adiabatic (dry) lapse rate gives us 203K to begin with, we get 240W*(1-0,38)-78W/2- 5,67E-8*203^4=13,5 W.
So, 13,5 W/m^2 is what is left to be explained by green house gases combined (if the above is correct). Do you disagree? Why so?
Side note. Trenberths “Energy budget” http://mensch.org/5223/other/EarthsGlobalEnergyBudget.pdf has an implicit reflection of the atmosphere of 37% from below but 23% from above. Whenever I fly the clouds are bright white from above, but often dark from below. Should it not be the opposite of what Trenberth claims? Please enlighten me if you know…

May 10, 2011 1:53 pm

Ira, you say in your article above:
“Since we humans sense the visual portion of this radiation as “light”, that is the name we give it, and that has led to the false assumption that it contains no “heat” (or “thermal”) energy.”
You are making one assumption there that does not fit in with “Climate Science” at all. It has, ever since the days of Friedrich Wilhelm Herschel and Benjamin Franklin, been known to science that different colors absorb “solar light” at different rates.
Having said all that, I too have been led to conclude that light, and indeed that all kinds of other radiation contains no heat. What light does contain however is “electronic energy”. Or to say it in a different way; light is always “on the move” and is transmitted as a part of the electro-magnetic wave-bands or spectrum, i.e. as radiation. – Only when that radiation is stopped or interrupted by “solids” i.e. atoms and/or molecules is that electronic energy either absorbed (transformed to kinetic energy) or reflected. (We can only see images as reflected light – or it’s source.)
Kinetic energy is used to create molecular movements. – Molecular movements create friction. – Friction creates heat. – Heat can be measured as temperature. – So there it is – as far as I understand it. – Heat is the product of work done by energy provided by an energy source, in this case the Sun’s energy via the curtesy of radiation. Yes, I agree if there is radiation there must also be “back radiation” – Maybe I am not expressing myself very well, but I am hoping to keep my comment as short as possible.
Furthermore, what I find curious with your “workings out” is the same as what I find unexplainable in any other “climate calculations” or models. They all “average” the solar W/m² input over the whole globe, which leads to an unavoidable bias in favor of solar irradiation. In your case, you say:
“The Earth may be approximated as a sphere, with the Sun shining on only half of it at any given time. The adjustment factor for this correction is 0.25.”
This 0.25 (or ¼) comes about because solar irradiation at the top of the atmosphere (TOA) which you say is 1366 W/m² will only retain it’s full force if it falls perpendicular onto a 2 dimensional disk. (Disk size does not matter as we are talking Watts for each individual, or any one square meter.) The difference between the square area of a disc and that of a sphere is ¼ – so if you divide 1366/4 you get 341.5, but the fact that the Sun is shining on only half of it at any given time, as you correctly state, is not taken into consideration. – According to what I once read about it in “Science of Doom” the thinking behind it was that if radiation out happens from the whole sphere’s surface then “to spread the solar constant” over the whole sphere seems –Fair enough.
And maybe it is, – as the most likely probability is that nobody has a hope in hell of finding the exact average Wattage. But looking at it from the other angle: what if we averaged 0 solar radiation in (which is equally provided by the dark side? Then we would probably freeze to death before waking up in the morning. So of course if we have got sunshine 24/7 then we are going to get warmer! – CO2 or no CO2.
Anyway, my fault is I waffle on, and on, and o – – – – .
Your missing 33K could just be hiding in the old saying that this one is the “Goldilocks’ planet”. Your calculations only encompasses radiation in general and Infrared radiation in particular. It does not take into account anything else – like for example the Earth’s rotation, which is crucial if you wish to find the missing 33K. The Earth never warms “from scratch”, it keeps heat in during the night. In fact you would be hard pressed to register an overnight reduction in temperature in 70% of it (the oceans).
Radiation my dear Ira is moving energy from one place to another – and back again – It may be a cause of “local warming – or cooling”, but is no more a cause of “global warming” than is conduction, convection and advection.

May 10, 2011 2:27 pm

My proof-reader has told me:
Man, your last comment to Ira’s article is too heavy and therefore too dumb.
In that case I shall just say; if you believe in AGW (or may it even just be a bit of warming by CO2), please show me an example where the product has reproduced its maker.
Heat is the product of energy use, so please explain how heat can produce energy.
If you believe CO2 is the producer of heat then please explain how CO2 is produced.

Joel Shore
May 10, 2011 2:37 pm

Bryan says:

[a bunch of nonsense and then…]
However from now on I will keep in mind that you should not be taken too seriously.

I would be perfectly happy if you were to just ignore me since you are clearly just interested in peddling your pseudo-science. There are other people here, such as David M Hoffer, who…while we may have strongly different opinions on the importance of AGW…do take me seriously and who I can have serious scientific discussions with. You are off in some make-believe land of your own devising, completely divorced from actual science…and in fact you are actively peddling pseudo-scientific nonsense that I think you are intelligent enough to know is such.

JAE
May 10, 2011 4:03 pm

New calcs of radiative GHE shows the GHE is only 8-9 deg. C:
http://activistteacher.blogspot.com/2011/05/radiation-physics-constraints-on-global.html

suricat
May 10, 2011 4:15 pm

George E. Smith says: May 10, 2011 at 11:28 am
“””And the shift in the peak of the BB Spectral Radiance when we change from frequency or wave number over to a wavelength scale, is simply due to that switch from”””
That looks to be an unfinished statement George. Would this finish it?
“……, is simply due to that switch from” ‘its inverse form’.
My time is short, so I thank you for your ‘expansion’ for the benefit of ‘Jo public’, but I think Ira understood the gist of my comment.
Best regards, Ray Dart.

richard verney
May 10, 2011 4:29 pm

JAE says:
May 10, 2011 at 4:03 pm
New calcs of radiative GHE shows the GHE is only 8-9 deg. C:
http://activistteacher.blogspot.com/2011/05/radiation-physics-constraints-on-global.html
//////////////////////////////////////
Thanks for the heads up.
The comment by Hans Shruder to that article is particularly interesting. I have repeatedly commented that we will be unable to understand how the atmosphere works as long as we continue to use average figures/average scenarios. This practice really disguises what is really going on. It will therefore be interesting to read the forthcoming paper to which Hans Shrudder refers.

richard verney
May 10, 2011 4:32 pm

Further to my last post, I mispelt the name of the commentator. It should have read Hans Schreuder.

Bill Illis
May 10, 2011 5:38 pm

Lots of discussion about the theory.
But nobody is talking about what really happens on the surface of the Earth or what really happens up in the troposphere.
The actual Earth surface and the troposphere does not react the way this theoritical framework indicates.
Show me some actual radiation and temperature measurements of a location on Earth that meets this framework. Your backyard does not follow this framework – something else is happening.

May 10, 2011 5:39 pm

R Stevenson says:
I integrate between spectral band wavelengths of 12.5 to 16.5 microns for CO2.
From 12.5 to 13.3 microns, CO2 absorbs (and emits) almost no radiation. On the other side, the band goes to about 16.98 microns.
When I integrate from 2 micron to infinity, assuming a constant pressure, 15c, and 1 km of thickness, CO2 alone absorbs about 73 of the available 390 W/m2. I suspect that there will always be a little more energy absorbed as the thickness is increased. However, once the amount remaining is “small enough”, it is common to consider the gas to be opaque. It is also important to note that about 54 of the 73 W/m2 overlaps with water vapor. Because of this overlap, doubling CO2 produces a net increase of only 4 W/m2 (according to my calculations).

Phil.
May 10, 2011 6:28 pm

JAE says:
May 10, 2011 at 4:03 pm
New calcs of radiative GHE shows the GHE is only 8-9 deg. C:
http://activistteacher.blogspot.com/2011/05/radiation-physics-constraints-on-global.html

Contains a fundamental error, doesn’t understand the definition of Bond albedo, and misapplies Kirchoff’s Law (doesn’t understand that a and ε are functions of wavelength).

Joel Shore
May 10, 2011 6:41 pm

Smokey:

And the models predicting a tropospheric “hot spot” have been falsified to the point that the backing and filling action now reverts to the stratosphere. But as Prof Richard Feynman makes crystal clear: if the models are contradicted by observations, the models are wrong.

You might want to look at what Richard Lindzen has to say about this. Even you might admit that he knows a little bit more about atmospheric science than you do…and you might even believe him since he agrees with your ideologically-driven point-of-view on AGW in general. His wording that the data is wrong and the models / theory are right on this particular point is even stronger than mine ( wattsupwiththat.com/2011/01/17/richard-lindzen-a-case-against-precipitous-climate-action/ ):

The dominant role of cumulus convection in the tropics requires that temperature approximately follow what is called a moist adiabatic profile. This requires that warming in the tropical upper troposphere be 2-3 times greater than at the surface. Indeed, all models do show this, but the data doesn’t and this means that something is wrong with the data.

(The one difference between Lindzen and the consensus view is that Lindzen seems to think that the tropical surface data is suspect whereas the more standard view is that the problems are most likely mainly with the radiosonde & satellite data / analysis.)

jae
May 10, 2011 6:46 pm

Phil: Maybe so, maybe not. You need to EXPLAIN WHY you disagree. With some kind of evidence, links, etc. Sorry, but I don’t know you and cannot take your word on this.

Martin Lewitt
May 10, 2011 7:17 pm

Joel Shore,
“So, that is my basic complaint about the argument that a faster water cycle somehow provides a negative feedback: To the extent that this expected to be true, it is incorporated into the models. If you want to argue that the models underestimate the effect, this has testable consequences for the temperature distribution in the atmosphere…And, at this point, the data for the temperature distribution do not seem to support this conclusion, if anything going the other way.”
I mentioned the Wentz 2007 article in the journal Science that showed none of the models produced more than half the increase in precipitation seen in the observations, so how can you say the speedup is “incorporated into the models”. The presence or absence of a hot spot was never going to be decisive, even if it was as prominent as the models had it, it wouldn’t indicate whether feedback was positive or negative. The modelers were just scrambling to see the presence of something they claimed would be prominent. The lack of the temperature profile that you claim is expected from more turnover of the water cycle, can possibly be explained by the models under representing the efficiency with which greenhouse gases cool the upper troposphere, there is already good evidence that the stratosphere is cooler, that is attributed to this mechanism. But it is the models that aren’t matching the most basic observations of a faster water cycle.

May 10, 2011 8:06 pm

When Joel Shore says: “your ideologically-driven point-of-view on AGW,” you can be sure it’s merely Joel’s psychological projection. I personally have no such ‘ideological’ view. I simply point out the fact that there is zero evidence of global harm due to CO2, and that CO2 enhances plant growth; therefore CO2 is harmless and beneficial.
Joel Shore always avoids trying to show evidence of global CO2 damage, because there is none. He is blinded by his own ideology. Alarmists like Shore have the onus of showing that their beliefs in CAGW are supported by empirical, testable evidence. Since they have failed, they project their own faults onto scientific skeptics.
Someone please wake me when the alarmist crowd begins to follow the scientific method.

Tim Folkerts
May 10, 2011 8:14 pm

JAE,
Here is one specific flaw in http://activistteacher.blogspot.com/2011/05/radiation-physics-constraints-on-global.html

But in fact, the evaluation of a 33 degree warming is in serious error. It is not physically reasonable to assume the true value of Earth’s average emissivity {ε} to be equal to 1. Indeed, it is physically impossible from usual known radiative processes for any opaque body to have a surface albedo (reflectivity) as high as 0.30 yet have an emissivity of 1. An ideal black body has a reflectivity of zero.
This is not an elusive point in radiation physics. Emissivity has been studied since before the development of quantum mechanics and there exists a first-empirically-established relation between reflectivity (albedo) and emissivity. It is known as Kirchoff’s Law of radiation physics. Basically, it says that the larger the reflectivity the smaller the emissivity, in approximate linear relation to each other.
As an equation, in this context, Kirchoff’s Law is:
1 – {a} = {ε}. … (eq.5)
It is a violation of Kirchoff’s Law to admit {a} = 0.30 yet use {ε} = 1 in calculating the Earth’s surface temperature, irrespective of the assumed amount of greenhouse effect.

This reference (and many others like it) show that white paint can have an emissivity of ~ 0.9 for thermal IR, but white paint will have an albedo of ~ 0.9 for visible light. Here is empirical evidence that his interpretation of Kirchhoff’s law is wrong because absorption can and does vary with wavelength.
{a} are weighted averages, where the weightings are completely different. There is no reason to expect the two values to be closely related.

Tim Folkerts
May 10, 2011 8:27 pm

That last sentence should have been:
{e} and {a} are weighted averages, where the weightings are completely different. There is no reason to expect the two values to be closely related.

Martin Lewitt
May 10, 2011 8:28 pm

Dave Springer,
High temperatures are achievable with infrared lasers:
http://en.wikipedia.org/wiki/Shiva_laser
A high enough concentration of photons of any wavelength can cause heating. I think however, there are practical limitations to being able to perform Fresnel lens heating from wall radiation.

Phil.
May 10, 2011 8:33 pm

jae says:
May 10, 2011 at 6:46 pm
Phil: Maybe so, maybe not. You need to EXPLAIN WHY you disagree. With some kind of evidence, links, etc. Sorry, but I don’t know you and cannot take your word on this.

But you accept the unsupported statements in that paper?
Try this for size:
http://hyperphysics.phy-astr.gsu.edu/hbase/phyopt/albedo.html
Includes this “The bond albedo is the total radiation reflected from an object compared to the total incident radiation from the Sun. The bond albedo for the Earth is given as 0.29 by de Pater and Lissauer, ”
The Activist teacher paper asserts: “It is a violation of Kirchoff’s Law to admit {a} = 0.30 yet use {ε} = 1 in calculating the Earth’s surface temperature, irrespective of the assumed amount of greenhouse effect.”
However as shown above the Bond albedo applies for the range of solar wavelengths, i.e. up to ~5 microns. It is therefore inappropriate to apply Kirchoff’s law to determine {ε} for the surface emission range (~5 – 50 microns), the value of 0.7 that Rancort derives is applicable to the UV to 5 microns range, not to the IR.

May 10, 2011 8:36 pm

Joel Shore;
Hope you are still following this thread, could you hit my blog or otherwise provide your email address? I’ve got something I’d like to run by you.

May 10, 2011 11:26 pm

Ira Glickstein, PhD Reur May 9, 2011 at 9:01 pm
You should retract your over-the-top insults to JAE; they give you a negative persona. You should have been able to work-out that JAE made a simple mistake, which BTW he shortly realized and corrected just before your rant. An apology is in order.
I can only conclude that his suggestion that radiative hypotheses are the lesser in importance than other thermodynamics is inconvenient for you to discuss. I’ve followed JAE for several years and find his research and thinking on convection and thermalization etc to be of great interest. BTW Ira, HEAT is a different form of energy to EMR, regardless of wavelength. (the popular restriction of “thermal EMR” to IR is plain nonsense)
JAE:
You may be interested to know that I had an interesting Email exchange with Roy Spencer a while back where I asked in part if in a nominally warming world there would possibly be increased evapo-transpiration. If so, according to Trenberth, given that this is the largest HEAT loss from the surface, there ought be increased surface cooling, or in other words, a negative feedback. Roy eventually withdrew, but to summarise, he admitted that what he collectively called convection was indeed important, but that everyone is too busy working on the radiative stuff. (including himself)
Ho hum.

Bob_FJ
May 10, 2011 11:40 pm

JAE,
Further my post above, where I wrote:
…everyone is too busy working on the radiative stuff…
That might also translate to:
“everyone is too busy competing on the radiative stuff”
(including Roy Spencer)

Bryan
May 11, 2011 12:28 am

Joel Shore
I said ….. ” However from now on I will keep in mind that you should not be taken too seriously. ”
You replied
…….”I would be perfectly happy if you were to just ignore me” …….
If you wish to be taken seriously then avoid distorting words like HEAT which is inexcusable for someone with a Physics degree.
When you do that your give the impression that;
1. Your version of the IPCC position requires this distortion to be convincing.
2. Or perhaps you really don’t know what your talking about.
I am personally much more likely to be persuaded by the likes of Nick Stokes or Rodrigo Caballero who can present the IPCC case without departing from the traditional language of Clausius and Feynman.
Read through Rodrigo Caballero’s Lecture Notes to see what I mean.
http://maths.ucd.ie/met/msc/PhysMet/PhysMetLectNotes.pdf
No wonder an educated person like Professor Gerlach “blew his top” when he found your paper full of elementary mistakes.

R Stevenson
May 11, 2011 2:00 am

Robert Clemenzi says:
May 10, 2011 at 5:39 pm
When I integrate from 2 micron to infinity, assuming a constant pressure, 15c, and 1 km of thickness, CO2 alone absorbs about 73 of the available 390 W/m2. I suspect that there will always be a little more energy absorbed as the thickness is
Robert,
We seem to be in close agreement on my spreadsheet model when I integrate between 0.5 microns and infinity at 15 C etc., CO2 absorbs 25.3 Btu/hft^2 (79.8W/m^2) of the available 124 Btu/hft^2 (391W/m^2) ie 20%.
For water vapour alone, my spreadsheet integration gives 248W/m^2 absorbed of 391W/m^2 available or 63%.
Using charts developed by Hottel I obtain absorptivities giving extinction distances of 3000 to 4000m for CO2 at 380ppm and 2000m when CO2 is doubled. Your extinction distance is less at 1km.

R Stevenson
May 11, 2011 2:25 am

Robert Clemenzi
Further to above on my spreadsheet 97% of the absorbable IR is in 15micron band for CO2.

May 11, 2011 3:07 am

I am impressed with Ira Glickstein’s post, and his clear willingness to share his logic and computations in order to invite criticism, correction, and further development. That speaks to me of scientific integrity, and it is refreshing to encounter it in the climate debate which is often dominated by what might be, at best, called ‘courtroom integrity’ in which antagonists vie with each other to present watertight cases immune to argument and contradiction.
The whole idea of dealing with global mean temperatures and averaged-out energy budgets is by itself crude, and one commenter at least has noted that when an observed global warming of the order of 1C is small compared to the coarseness and sensitivity of such back of an envelope calculations, we need to look elsewhere to resolve disputes.
I presume the answer lies in admitting more of the complexity of real case into the computations: if not the spinning, irregularly surfaced sphere, then at least the huge differential in solar heating ‘twixt the equatorial and the polar regions, the great daily poleward energy transfers which compensate thanks in large part to massive convective systems. I am with the commenter who sees the atmosphere primarily as a coolant – taking heat out of the tropical surface, and helping it escape to the temperate/polar zones and to space. Surely in there somewhere lies the possibility of crucial hypotheses to help clarify our differences of opinion and be capable of observational test/refutation/confirmation. The effects of additional CO2 seem, to me, to be likely to be so small that such hypotheses will be for correspondingly small, probably only regional or local, effects that might yet be measurable. I presume further that we would benefit from increased measurement of radiation environments and temperatures at various heights, coupled with measurements of moisture, cloud, and CO2 levels. A few years of good data there would, it seems to me (a mere outside observer of this science), produce a productive environment for scientific progress.

R Stevenson
May 11, 2011 3:09 am

Dave Springer says:
May 10, 2011 at 8:55 am
CO2 extinction altitude can be seen in the spectrum looking down from 20 kilometers above the arctic ocean. IIRC the spectrum follows a 270K blackbody curve where the atmosphere is IR transparent i.e. it “sees” the temperature of the ocean surface but in the 15um region it drops down to follow a 250K blackbody curve. Going by dry adiabatic lapse rate (arctic air is fairly dry) of 1K per 100 meters the IR sensor is “seeing” the air temperature in the 15um range at a height of 2000 meters. I’m not an optics expert but I believe that altitude represents the optical depth of the atmosphere at 15um or in other words the extinction altitude. In any case there is no way in hell the extinction depth is in tens of meters.
That is the best contribution so far

Dave Springer
May 11, 2011 5:06 am

JAE says:
May 10, 2011 at 8:56 am
“Huh? I hope you are not saying that everything radiates in the IR frequencies. To repeat a comment I made above somewhere:”
Solids, liquids, and dense gases emit continous blackbody spectra characteristic of their temperature. We were talking about atmospheric nitrogen which is a dense gas and its temperature, at least in the lower and middle atmosphere, puts its blackbody emission spectrum in the far infrared.
What part of that don’t you understand?

Joel Shore
May 11, 2011 5:15 am

Smokey says:

When Joel Shore says: “your ideologically-driven point-of-view on AGW,” you can be sure it’s merely Joel’s psychological projection …

I just want to point out for the benefit of the readers what has transpired here. For the last several months, Smokey (often quoting me out of context) has hounded me for saying that I thought it was more likely that the models are correct and the data incorrect in regard to tropical tropospheric amplification.
Now that it has been pointed out to him in my previous comment http://wattsupwiththat.com/2011/05/07/visualizing-the-greenhouse-effect-light-and-heat/#comment-657690 that Richard Lindzen says the same thing…except in even more unambiguous terms (stating flat-out that the data must be wrong), one might expect that a person with decent standards of personal responsibility would either
(1) Apologize to me.
(2) Rebuke Lindzen in equally-harsh words as he has rebuked me.
What has Smokey done instead? He has avoided the issue entirely.

May 11, 2011 5:35 am

Some links to comments on this thread which encouraged or informed my earlier one (3:07am)
(1) ‘Too simple’ models:
“This goes to the heart of the problem. Anthropogenic effects are so tiny as to easily buried in the noise of the real world and in the error margins of all these toy models of the real world.” Dave Springer May 8 8:08am
Anthony Zeeman May 8 10:58am
Roger Sowell May 8 4:02pm
David May 10 12:54am
richard verney May 10 4:03pm
(2) Coolants:
Richard M May 9 4:16pm
Boris Gimbarzevsky May 9 8:38pm
Radiation and/or temperatures at different heights and contexts:
wayne May 7 11:56pm
Bryan May 8 1:13am
Martin Lewitt May 9 3:02am
richard verney May 9 9:53am
Robert Clemenzi May 10 12:25am
Boris Gimbarzevsky May 10 1:51am
(3) Importance of convection:
Cematafried May 8 5:55am
Leanard Weistein May 8 6:36am
Dave in Delaware May 8 7:03am
Nullius in Verba May 8 7:06am and May 8 12:53pm
Alistair May 8 7:56am
richard verney May 9 9:53am
Dave Springer May 9 9:23am
(4) meridional contrasts / transfers:
davidmhoffer May 8 3:05am
Gilles May 8 10:02am

Joel Shore
May 11, 2011 5:36 am

Bryan says:

When you do that your give the impression that;
1. Your version of the IPCC position requires this distortion to be convincing.
2. Or perhaps you really don’t know what your talking about.

In regards to our paper, you (and G&T) seem to be the only ones who are incapable of doing simple word replacements in a few places that have been described to you in detail. Besides which, I have been more careful in these threads to use the word “heat” more precisely; however, it seems that people peddling pseudoscience not only get inordinately hung up on word choices but then also don’t accept the concept of corrections.

No wonder an educated person like Professor Gerlach “blew his top” when he found your paper full of elementary mistakes.

While any actual physicists reading our paper may say we should have been a little more precise in our usage of the word “heat” in a few places, they will consider this to be “going a few miles above the speed limit” in comparison to Gerlich and Tscheuschner’s “serial killing”, i.e., huge scientific blunders in regards to the 2nd Law of Thermodynamics and the greenhouse effect. It is only people peddling pseudoscience who purposely try to misinterpret what other people are saying in order to confuse others.

May 11, 2011 5:38 am

Typo: Need (3) added to give
‘(3) Radiation and/or temperatures ate different heights and contexts:’
and then existing (3) -> (4), and existing (4) -> (5).

Dave Springer
May 11, 2011 5:49 am

John Shade says:
May 11, 2011 at 3:07 am
“I am with the commenter who sees the atmosphere primarily as a coolant – taking heat out of the tropical surface, and helping it escape to the temperate/polar zones and to space.”
I try to make analogies to help people (myself included) understand things. In other commentary on WUWT I compared the poles to the radiator on an automobile. The equator is the engine. When we have a continent over a pole that’s like a blockage in the radiator limiting how well it can dump the heat (south pole today). When we have continents surrounding a pole with restricted openings that also, to a lesser degree, also limits the effectiveness of the radiator.
I think heat transport in the atmosphere is almost all vertical with evaporation, convection, and condensation being the major players there.
As far as heat capacity goes the ocean has roughly 1000 times as much as the atmosphere. So that relationship is akin to a dog and its tail. The tail doesn’t wag the dog of course. However if the dog is sleeping a tug on its tail can wake it up. The climate boffins I think are worried that anthropogenic CO2 is tugging the tail on a sleeping dog and believe it’s always best to let sleeping dogs lie. Or maybe it’s more like a tiger to them. 🙂

Phil.
May 11, 2011 5:59 am

ferd berple says:
May 10, 2011 at 1:43 pm
Here is a thought experiment. If we hold everything else equal and double the amount of N2 in the atmosphere, because N2 does not participate in the back radiation, the surface equilibrium temperature should remain “unchanged” according to the radiative transfer model.
However, comparisons of earth and venus suggest that the temperature of the atmosphere will only remain unchanged at 1 atmosphere pressure. As we now have more atmosphere, the pressure at the surface will be increased as compared to present. Under the gravitational model, the surface temperature would be expected to increase as a result of doubling the N2 in the atmosphere.
Which of these two is correct? Adding N2 will not increase temperature, or adding N2 will increase temperature (holding all else equal)?

Adding N2 will not increase temperature since it will have no impact on the radiative emission to space.
The problem for me is the lapse rate. Doubling the N2 in the atmosphere will raise the height of the atmosphere. If the surface temperature does not increase, then the lapse rate dictates that since the atmosphere extends higher, that further up, past the present altitude it must get colder than it is at present. That seem nonsensical.
Your assumption of a continuation of the lapse rate is flawed, the tropopause is the region where the positive lapse rate of the troposphere changes to the negative lapse rate of the stratosphere (heated from above). I see no reason why addition of N2 would change that very much.

May 11, 2011 6:02 am

Someone hand Joel Shore a hanky.
Blogrule #1: Never demand an apology. It is pitiful and impotent.
Joel’s begging for an apology for his hurt feelings is simply his tactic to avoid answering the points I raised above. Here is the comment I made, which he still avoids answering:

When Joel Shore says: “your ideologically-driven point-of-view on AGW,” you can be sure it’s merely Joel’s psychological projection. I personally have no such ‘ideological’ view. I simply pointed out the fact that there is zero evidence of global harm due to CO2, and that CO2 enhances plant growth; therefore CO2 is harmless and beneficial.
Joel Shore always avoids trying to show evidence of global CO2 damage, because there is none. He is blinded by his own ideology. Alarmists like Shore have the onus of showing that their beliefs in CAGW are supported by empirical, testable evidence. Since they have failed, they project their own faults onto scientific skeptics.

What is untrue about that comment? Truth doesn’t require apologies. I regularly ask Joel Shore and others to produce verifiable evidence of global damage caused by the rise in CO2. But rather than address that point, Joel has repeatedly accused me of being ideological; I am not. He is deliberately avoiding the central question in the whole trumped-up “carbon” debate.
Joel Shore is wrong; I am only ‘biased’ against alarmist scientists feeding at the public trough, and who are trying to scare the public by spending a large part of their workdays wasting our tax money by writing long comments on blogs, instead of doing what they are paid to do.
I am sorry Mr Shore’s boss allows him to misappropriate public funds in this manner. Gavin Scmidt and James Hansen have set a terrible precedent in this regard. There is no doubt that if someone like John Christy began blogging throughout the workday like Joel Shore does, the screams of outrage from the hypocritical alarmists would be deafening.
If it is ‘hounding’ Joel Shore for any putative real world, testable evidence showing that rising CO2 – a tiny trace gas – causes measurable harm to the planet, he needs to stop prevaricating and produce it, or take being ‘hounded’ until he produces evidence, or admits that there is none. Calling others ideologically biased is just Joel Shore’s way of avoiding the question.

Tim Folkerts
May 11, 2011 6:12 am

I agree with much of what Dave Springer says, but I can’t agree with some of his conclusions about optical depth.
Dave Springer says: May 10, 2011 at 8:55 am
> IIRC the spectrum follows a 270K blackbody curve where the atmosphere is
>IR transparent i.e. it “sees” the temperature of the ocean surface but in the
>15um region it drops down to follow a 250K blackbody curve.
From the graph at the top, the temperatures are closer to 265 and 225 K. That gives a difference of 40 K, not 20 K. Not a huge change, but significant.
> Going by dry adiabatic lapse rate (arctic air is fairly dry) of 1K per 100 meters
I would love a little more info here. The key is the relative humidity. Certainly cold arctic air has a very low absolute humidity, but is the relative humidity low?
The observed environmental lapse rate is closer to 0.65 K/100 m (speaking in general — I don’t know details specifically for the arctic), and the temperature difference is closer 40 which would put the altitude closer to 40K / (0.65 K/100m) = 6000 m. So we have a very large uncertainty in the altitude here.
>the IR sensor is “seeing” the air temperature in the 15um range at a height of
>2000 meters. I’m not an optics expert but I believe that altitude represents
>the optical depth of the atmosphere at 15um.
I would not define the optical depth the way you seem to be doing.

Optical depth, or optical thickness is a measure of transparency, and is defined as the negative logarithm of the fraction of radiation (e.g., light) that is not scattered or absorbed on a path. The optical depth is a measure of the proportion of radiation absorbed or scattered along a path through a partially transparent medium…
Wikipedia

In other words, of a region of the atmosphere has an optical depth of 1, then 70% of the incoming light has been blocked. (Or equivalently, 70% of the light we see would come from that region). If most of the light can pass from 2000 m up out of the atmosphere (passing thru well over 1/2 the total mass of the atmosphere) then, then most of the light passing up from the surface could presumable ALSO pass thru well over half the atmosphere. I.e. the optical depth looking up would be well over 2000 m. However as the altitude gets higher, the optical depth gets smaller.
There is, however, another issue here. All we really know is that the photons come from a region where the temperature is about 220 K. First, I put the altitude closer to 6000 m, not 2000 m. But the temperature of 220 K is pretty close to the temperature of the tropopause, which typically extends for several km upward. The photons could come from pretty much anywhere over a range of several km of altitude and still appear to be ~ 220 K. The photons might be traveling a relatively short distance — for example, from space down to just 15 km altitude. So a very tiny part of the atmosphere could stop most of the photons — so a relatively short distance near the surface could stop the photons. We just can’t know very accurately using this method.
(If someone has more details about the temperature and altitude range of the arctic tropopause, I would love to hear it.)
>In any case there is no way in hell the extinction depth is in tens of meters.
After all that, the simper way to determine optical depth near the surface is to look UP. From the graph at the top, the “temperature” of the 15 um IR radiation when looking up is practically the same as the temperature of the surface (certainly no more than 5 K different). The photons must come from no more than 500 m up (using the lapse rate argument), so the optical depth near the surface is well below 500 m.
Whether the optical depth for 15 um photons near the surface is 3 m or 30 m or 300 m, I can’t tell from just this. But there is no way in hell it could be 2000 m.

Phil.
May 11, 2011 6:23 am

Dave Springer says:
May 11, 2011 at 5:06 am
JAE says:
May 10, 2011 at 8:56 am
“Huh? I hope you are not saying that everything radiates in the IR frequencies. To repeat a comment I made above somewhere:”
Solids, liquids, and dense gases emit continous blackbody spectra characteristic of their temperature. We were talking about atmospheric nitrogen which is a dense gas and its temperature, at least in the lower and middle atmosphere, puts its blackbody emission spectrum in the far infrared.

As far as blackbody radiation is concerned N2 is not a dense gas in the earth’s atmosphere and doesn’t emit as a blackbody. If it were a blackbody it would emit in the far IR.

jae
May 11, 2011 6:30 am

Heh. Come on, folks, I am not supporting the teacher’s different methods of calculatig the GHE; I only pointed it out. Don’t shoot the messenger. I have a strong hunch that NONE of these radiative calcs mean anything, anyway, because any of the effects are instantly overwhelmed by other phenomenon, such as convection and water evaporation/condensation.
In that vein, I’m still really interested why there is so much silence surrounding this (and other related expositions):
http://theendofthemystery.blogspot.com/2010/11/venus-no-greenhouse-effect.html

Dave Springer
May 11, 2011 6:56 am

Martin Lewitt says:
May 10, 2011 at 8:28 pm

Dave Springer,
High temperatures are achievable with infrared lasers:
http://en.wikipedia.org/wiki/Shiva_laser
A high enough concentration of photons of any wavelength can cause heating. I think however, there are practical limitations to being able to perform Fresnel lens heating from wall radiation.

I’ll need to think about that some more. Shiva operates in the very near infrared (1um) which correlates to a temperature in the thousands of degrees. Moreover, it focuses 20 beams coming from different directions on its target and achieves heating to millions of degrees through compression (shock waves).
Perhaps a more familiar technology is the microwave oven. Exceedingly cold matter emits blackbody spectra in the microwave region of the spectrum. The cosmic microwave background radiation is 3K for instance. That then raises the question of how a microwave oven can heat up a cup of coffee. It isn’t accomplished by microwave photons per se but rather by phase variation which causes dialectric molecules (primarily water in this case) to constantly re-orient with the magnetic phase. Friction then does the actual heating because microwave radiation in and of itself can’t raise the temperature of anything that isn’t within a couple degrees of absolute zero.

Phil.
May 11, 2011 7:08 am

jae says:
May 11, 2011 at 6:30 am
Heh. Come on, folks, I am not supporting the teacher’s different methods of calculatig the GHE; I only pointed it out. Don’t shoot the messenger.

You asked my reasons for saying that his method was wrong, I gave them, that’s all.

Dave Springer
May 11, 2011 7:12 am

Phil. says:
May 11, 2011 at 6:23 am
“As far as blackbody radiation is concerned N2 is not a dense gas in the earth’s atmosphere and doesn’t emit as a blackbody. If it were a blackbody it would emit in the far IR.”
A gas at 1 bar isn’t dense? In general a gas is considered dense if the molecules are close enough to each other to conduct heat. Nitrogen at 1 bar will certainly conduct heat. Try again.

Vince Causey
May 11, 2011 7:46 am

Joel quoted Richard Lindzen:
“The dominant role of cumulus convection in the tropics requires that temperature approximately follow what is called a moist adiabatic profile. This requires that warming in the tropical upper troposphere be 2-3 times greater than at the surface. Indeed, all models do show this, but the data doesn’t and this means that something is wrong with the data.”
I’m sorry Joel, but you need to get your sarcasm detector seen too. Lindzen’s tongue was so far in his cheek it should have been clearly visible to everyone from at least a mile away.

Joel Shore
May 11, 2011 7:51 am

Smokey says:

Joel’s begging for an apology for his hurt feelings is simply his tactic to avoid answering the points I raised above.

No…It is you who are desperately trying to avoid the fact that you have spent months attacking me (including in this thread) for a statement that Richard Lindzen makes in even more extreme terms than I do. That you are unwilling to address this when called out on it shows a completely lack of ethics and personal accountability on your part.
As for what you ask: I have explained this stuff many times. It is also explained in many reports by reputable scientific organizations. I can’t convince someone who is unconvinceable.

Bryan
May 11, 2011 7:53 am

Joel Shore
I said…… “No wonder an educated person like Professor Gerlach “blew his top” when he found your paper full of elementary mistakes.”……
You said …”While any actual physicists reading our paper may say we should have been a little more precise in our usage of the word “heat” in a few places”…
You have a particularly snide way of phrasing your replies.
Are you here implying that Professor Gerlach is not a physicist?
That a Professor from a German University is an impostor perhaps!
That’s very impertinent from someone who finds coping with reading a big problem.
For someone who is a bit “rusty” in the thermodynamics department yet was happy to add his name to a collection of elementary gaffs.
Its too late now to withdraw you name from the Halpern et al embarrassment.
I could recommend some useful textbooks to bring you back up to speed with Physics.

Tim Folkerts
May 11, 2011 8:33 am

David Springer, you need to slow down a little!
“A gas at 1 bar isn’t dense? In general a gas is considered dense if the molecules are close enough to each other to conduct heat. “
No. A gas is considered “optically dense” if it absorbs most of the photons at that wavelength.

In spectroscopy, the absorbance A (also called optical density)[2][3] is defined as:[4]
A_\lambda = \log_{10}(I_0/I)\,,
where I is the intensity of light at a specified wavelength λ that has passed through a sample …
Wikipedia

N2 does not absorb much IR at any wavelength through earth’s entire atmosphere, so the entire depth of N2 in earth’s atmosphere is not considered “optically dense”. You have already been corrected once on this. You might consider doing a little research before being so sure of your own knowledge.
In another post David discusses microwaves.
“Perhaps a more familiar technology is the microwave oven. Exceedingly cold matter emits blackbody spectra in the microwave region of the spectrum. The cosmic microwave background radiation is 3K for instance. That then raises the question of how a microwave oven can heat up a cup of coffee. It isn’t accomplished by microwave photons per se ….”
Once again, this is not right. It IS true that THERMAL microwaves would come from objects at a few Kelvins and that they could not warm an object above that same temperature. But microwave ovens don’t produce their microwaves by heating the walls of the oven to a few K. They use devices called “klystrons” or “cavity magnetron”. These devises are quite capable of producing very intense beams of microwave photons — several hundred watts of microwaves from a few square inches. It is exactly those microwave photons that heat the food.

May 11, 2011 8:41 am

Let me, as a “thought experiment”, slow the radiation circuits down to a speed where we can see it happening (At least in “our mind’s eye”). And let’s also think of things as single units i.e. radiation = 1 ray, energy =1 unit (Watt) & CO2 = 1 molecule etc. Furthermore lets do what the scientists do on occasions like this; ignore everything other than radiation, i.e. adiabatic warming /cooling, Conduction, convection and advection etcetera. (after all the atmosphere is not needed for radiation.)
Short Wave Radiation (SWR) comes in from space and warms the surface which begins to radiate Long Wave Radiation (LWR) back towards space, say at a constant energy rate of 1 W/m². Having sent this 1 W/m² away the surface must cool down respectively. CO2 intercepts and absorbs this energy and now CO2 warms up respectively and therefore begins to increase it’s radiation, – half of which (estimates may vary) returns to the surface and is now near to the end of it’s first circuit.
Meanwhile a ray of SWR has been absorbed by the surface and the 1 W/m² that LWR took away earlier has been replenished, so now that one part, however small, of LWR returns for absorption it must mean that the surface will warm a little bit extra. (That is if this theory is correct)
However what happens on the dark side of the Earth (or what we call at night)?
Where the Sun does not shine and SWR therefore does not replenish the energy which LWR takes away, – there can be only cooling.

To find out if the “cooling capacity” is great enough to bring the temperature right back to what it was the previous morning (i.e. heating starts again at “The zero line”) or if a part of the extra heating remains in the system and the heating starts again at a slightly higher temperature than yesterday it seems logical to me to bring back the Atmosphere (especially the Troposphere) and all it’s air-movements, pressures and lapse rates etcetera. And we must not forget the “Atmospheric Window”
Do scientists have enough data to anything bar guessing?

Joel Shore
May 11, 2011 8:42 am

Dave Springer says:

A gas at 1 bar isn’t dense? In general a gas is considered dense if the molecules are close enough to each other to conduct heat. Nitrogen at 1 bar will certainly conduct heat. Try again.

We are not talking about conduction but radiative absorption. For N2, radiative absorption does not occur for the isolated molecule and can only occur because of collisional processes between molecules. Here is a paper discussing the measurement of such absorption lines at pressures of 0 to 10 atmospheres: http://www.opticsinfobase.org/view_article.cfm?gotourl=http%3A%2F%2Fwww.opticsinfobase.org%2FDirectPDFAccess%2FBC73981D-EC8E-6BE9-5F66346A49A16C1A_60399.pdf%3Fda%3D1%26id%3D60399%26seq%3D0%26mobile%3Dno&org=Rochester%20Institute%20of%20Technology
Note that even for the strongest absorption line, the measurements rely on obtaining ultra-high purities of nitrogen because any small contamination by CO or CO2 overwhelms the measurement:

At the path length used in this study, trace impurities in the sample presented a serious problem, and we outline in some detail our purification method. Both CO and CO2 have strong bands that fall near to or on top of the collisionally induced absorption band of N2. Concentrations of these molecules at the 10^-9 level cause interfering lines to be observed in the spectrum, and, in fact, commercial Ultra-High Purity nitrogen with a stated purity of 99.9995% contained enough CO and CO2 to render the measurements useless. Intensity measurements on individual impurity
lines showed that the concentrations of CO and CO2 were 0.4 and 1 x 10^-6, respectively.

So, apparently only 1 part per million of CO2 was, along with the CO, enough to render the measurements useless. Imagine what 380 parts per million does! Furthermore, as this graph shows http://www.learner.org/courses/envsci/visual/img_med/electromagnetic_spectrum.jpg , the absorption line of N2 that they are talking about, which is at 4.3 um, would already place it quite far out in the wings of the terrestrial radiation spectrum.

May 11, 2011 8:45 am

Supplement to my previous comment:
Oh yes, – “The Atmospheric window” – last time I looked at how it was once presented by “Climate Scientists” it showed that out of the 390 W/m² of long wave Infra Red (IR) radiation emitted by the surface 40 W/m² was let through that window “scott free”.
Well, according to my rough calculations that is just around 10.25 – (or 10 %) – How can that be correct? – Yes, it may be correct in so far as they can say that; “around 10% of the wavebands emitted by IR radiation are made up of wave-lengths that cannot be absorbed by “Greenhouse Gases” (GHGs), but that cannot possibly mean that 0.04%, in the case of CO2 concentration but certainly less than 10% of the Atmosphere as a total has got what must be a “supernatural” ability to stop LWR. After all, according to scientific measurements the bulk of the Atmosphere, which is transparent to IR radiation, is made up from; Nitrogen (78%), Oxygen (21%) and Argon (0.9%) and the rest = 0.1% is made up from “trace gases” of which some are said to be “GHGs”. – Water Vapour (WV) does not mix evenly in with the other gases and atmospheric WV content varies from location to location but I believe it is estimated to be around 4 to 5%. – Of course the most dense WV concentration must be in the lower half of the Troposphere which may give the impression (by looking at Hydrometers) that is far more WV than a miserly 4-5%. However I doubt very much that the WV concentration near the surface amounts to more than say 40 – 50%. But of course I have got no official data as proof – and I could be wrong.
So, as far as I can see, the case must be that whatever percentage of LWR passes through the “Atmospheric window” must be added to whatever passes through the “Transparent” part of the Atmosphere.

JAE
May 11, 2011 8:50 am

Joel Shore:
You say: “Silence?”
Yeah, silence! Have you even read the article at the link? I don’t see how any of your links relate to it; all they do is repeat again the radiatitive GHE theory. What I want to know is why ALL planetoids with atmospheres, regardless of the gases (and solids) in those atmospheres, have the same temperature at 1000 bar, when distance to the sun is factored in. Are the articles wrong? If not, that simply cannot be due to any GHE. Just curious, man….

Joel Shore
May 11, 2011 8:51 am

Bryan says:

[a bunch of nonsense and then…]
I could recommend some useful textbooks to bring you back up to speed with Physics.

That is very generous, but I think you had better look through the latest edition of such textbooks to make sure they pass through your “political correctness” filter. The last time that you quoted a physics textbook as a source, it turned out that the latest edition of that textbook had a whole page on the greenhouse effect and global warming (see http://wattsupwiththat.com/2011/03/29/visualizing-the-greenhouse-effect-molecules-and-photons/#comment-644823 ). To put it mildly, it did not provide any support whatsoever for your views on the subject!

May 11, 2011 8:58 am

Several people are having trouble understanding the adiabatic lapse rates.
Dry adiabatic lapse rate (DALR) means the change in temperature with a change in pressure (altitude) when the contained water vapor is NOT changing state.
The Moist adiabatic lapse rate (MALR) (aka the Saturated adiabatic lapse rate – SALR) means the same thing except that the water vapor is condensing to form clouds.
The difference has nothing to do with the amount of water in the atmosphere, but only with whether or not the water vapor is changing state.

May 11, 2011 9:04 am

Bryan says:
“You have a particularly snide way of phrasing your replies.”
I would put it: whiny.
Instead of misdirecting the conversation to Prof Lindzen, I’m still waiting for an answer based on the scientific method to the question: where is the putative global damage caused by CO2? In fact, if there was such evidence, it would have been trumpeted 24/7/365 by the CAGW crowd.
Mr Shore, please provide testable, measurable, reproducible and convincing evidence demonstrating verifiable global harm specifically due to the rise in CO2 – you know: per the scientific method. Claiming that “I have explained this stuff many times” is a copout. We need solid evidence, because $trillions more in wasted tax dollars are at risk, based not on empirical evidence, but on GIGO models invented by people who can’t even get the definition of heat right.
Alternatively, Joel Shore can be a man-up and admit that there is no such evidence, and then the whole baseless CAGW conjecture becomes clear to everyone: the purpose of the scare is to provide a free ride on the grant gravy train for these climate charlatans at taxpayer expense.

May 11, 2011 9:13 am

R Stevenson,
I would very much like a link to that spreadsheet. My attempts to locate Hottel’s method keep hitting paywalls.
Using 1.0% water vapor, my program computes 282.68 W/m2 .. close to your value.
For power absorbed, my program incorrectly assumes 1 km at constant temperature and pressure. However, it also plots the distance to absorb x% of the available radiation. As long as those values are under 100 meters, I feel that assuming constant temperature and pressure causes only minor errors.
At any rate, I would like to investigate WHY the programs are giving different values to absorb the radiation.

JAE
May 11, 2011 9:24 am

David Springer must won’t let go of this nonsense:
“Solids, liquids, and dense gases emit continous blackbody spectra characteristic of their temperature. We were talking about atmospheric nitrogen which is a dense gas and its temperature, at least in the lower and middle atmosphere, puts its blackbody emission spectrum in the far infrared.
What part of that don’t you understand?’
I would have ignored this, except for the snide remark.
Come on, David, I gave you a link to a very clear textbook exposition on the interaction of IR with gases, which states unequivocally that N2 is not “IR active,” and it provides specific detailed reasons why this is true (no dipole moment). Density probably has nothing to do with it, unless it is so dense it is a liquid (and that occurs at about 1600 psi at 25 C).
I can see how we can debate something as complicated as the existence or amount of the GHE effect, but we should not be debating such basic, easily demonstrable, and widely accepted facts like the lack of signficant emissions of IR from N2.

JAE
May 11, 2011 9:44 am

Smokey says:
“Instead of misdirecting the conversation to Prof Lindzen, I’m still waiting for an answer based on the scientific method to the question: where is the putative global damage caused by CO2? In fact, if there was such evidence, it would have been trumpeted 24/7/365 by the CAGW crowd.”
Come on, Smokey, you must not be listening: IPCC and the warmistas keep yelling to us that there are “multiple lines of evidence” for CAGW. That’s Orwell-speak for “we don’t have any direct clear evidence, but, considering all things that MIGHT happen…(whatever)…” When dealing with the “progressives,” you gotta learn a new vocabulary. Such phrases as “Kinetic Military Action” (war); “Overseas Contingency Operations” (war on terror); “man-caused disaster” (terrorist act), “climate disruption” (man destroying the world), “we must be civil” (the Conservative bastards must be civil). Ad nauseum.
Problem with the CAGWers is that they can’t seem to locate one single clear line of evidence. An even bigger problem for them is that the “multiple lines of evidence” are actually demonstrating rather clearly that they are wrong, I’m sure to their inner horror (it’s a travesty…). Nothing seems to be working out for them, since the sea level is leveling off, the glaciers are not really melting everywhere, the temperature is not going up for the last 10-15 years, no “hot spot,” as predicted by models, etc., etc.
On the other hand, I guess if you allow yourself to blame cold on warming, storms and any sort of malady on “climate change,” then you can allow yourself to say that, “multiple lines of evidence” show that we are killing ourselves through some sort of unclearly defined mechanism, which we will just call CAGW. LOL.
It all just Brings tears to my eyes.
But they are gonna fix the problem by setting up special forums and getting the liberal MSM to explain their (false) science better, so we morons can understand their brilliance. Or something like that.

Tim Folkerts
May 11, 2011 9:45 am

O H Dahlsveen says: May 11, 2011 at 8:45 am
“but that cannot possibly mean that 0.04%, in the case of CO2 concentration but certainly less than 10% of the Atmosphere as a total has got what must be a “supernatural” ability to stop LWR. ”
hmmmm … lets see try an order of magnitude estimation …
A “typical” IR photon is ~ 10 um, or 2E-20 J
The ground emits ~ 400 W/m^2, which implies around 400/ (2e-22) = 2E22 photons per second being emitted.
Air applies 100,000 N/m^2 of pressure, so a square meter column of contains about 10,000 kg of air. CO2 is ~ 400 ppm by volume (and slight more than 400 ppm by mass), so there are about 10,000 kg x (400/1,000,000) = 4 kg =4000 g of CO2 in each square meter column of atmosphere.
At 44 g/mole, that would be 100 moles of CO2, or ~ 6E25 molecules of CO2 per square meter.
That makes 6E25 / 2E22 = more than 1,000 CO2 molecules for every IR photon passing by. CO2 only absorbs ~ 1/10 of the possible IR wavelengths, so that’s ~ 10,000 CO2 molecules per absorbable photon passing by each second. And the CO2 molecule only “holds” that energy for a faction of a second, so a given molecule could in principle absorb many photons every second.
Each CO2 molecule absorbing 1 out of 10,000 photons passing doesn’t seem “supernatural” to me ….
(I did this pretty quickly, so people are welcome to find flaws. I would not be surprised if my numbers are off by 10x or possibly even 100x, but that still doesn’t cahnge the fact that there are plenty of CO2 molecules around to absorb photons even at an “insignificant 0.04%” concentration.)

May 11, 2011 10:04 am

First I apologise if I am rehashing something that has already been discussed, I have been out in the field and haven’t had a chance to catch up on all the posts in this thread as yet, however this one did.
The equation cited does not, in my opinion, imply backradiation. If it did there would be terms for the emissivity of both objects as well as the surface area of both objects. There aren’t. What the equation does account for is the net difference in temperatures between the two objects or one object and its surroundings.
The net difference in temperature is obviously one of the factors which determines the flow of energy from the hot body to the colder one. If this were water we would call it the hydraulic head, if it were electricity it would be the voltage. Furthermore, if the surroundings or other object did heat the hotter object through “backradiation” wouldn’t the equation not only require terms for the surface area and emissivity of the colder object, but also require integration of the effect of the backradiation? In other words, to consider just the colder object, as it emits energy to the warmer object it cools, but it also warms from the energy input of the warmer object which causes its temperature to rise as a function of the difference between its absortivity and its emissivity. The increased temperature would then cause it to backradiate more energy etc… While at the same time the hotter object would be behaving in a like fashion. So you would need to integrate the energy inputs to each from each as a function of not only emissivity but absorbtivity as the net temperature difference tended towards 0 ( assuming it could in fact tend towards 0)?

Joel Shore
May 11, 2011 10:27 am

Martin Lewitt:

The lack of the temperature profile that you claim is expected from more turnover of the water cycle, can possibly be explained by the models under representing the efficiency with which greenhouse gases cool the upper troposphere, there is already good evidence that the stratosphere is cooler, that is attributed to this mechanism.

I’m not sure what you mean by this. Are you trying to say that you don’t believe that the estimates of the forcing due to increases in CO2 is not around the value that nearly all scientists, including Roy Spencer and Richard Lindzen, accept it to be? And, if so, what evidence do you have to make the assertion? It seems to me that you are taking one rather slender piece of evidence (the Wentz paper, which is interesting but not completely definitive) and then extrapolating it like crazy to suggest all sorts of things that they haven’t suggested…and which don’t really seem in line with the empirical data that we have.

Joel Shore
May 11, 2011 10:31 am

Smokey says:

Please provide testable, measurable, reproducible and convincing evidence demonstrating verifiable global harm specifically due to the rise in CO2 – you know: per the scientific method.

How am I supposed to convince you of something that involves seriously weighing of scientific evidence when I cannot even get you to admit you have made in error when it is right in front of your face for everyone to see? I might as well convince a rock to do the tango! You are so far from a reasonable person able to objectively weigh scientific evidence that it is a fool’s errand to try to convince you of anything.

wayne
May 11, 2011 10:33 am

ferd berple says:
May 10, 2011 at 1:43 pm
Here is a thought experiment. If we hold everything else equal and double the amount of N2 in the atmosphere, because N2 does not participate in the back radiation, the surface equilibrium temperature should remain “unchanged” according to the radiative transfer model.
However, comparisons of earth and venus suggest that the temperature of the atmosphere will only remain unchanged at 1 atmosphere pressure. As we now have more atmosphere, the pressure at the surface will be increased as compared to present. Under the gravitational model, the surface temperature would be expected to increase as a result of doubling the N2 in the atmosphere.
Which of these two is correct? Adding N2 will not increase temperature, or adding N2 will increase temperature (holding all else equal)?

Adding additional N2 to doubling it’s contribution to total mass of the atmosphere will absolutely increase the temperature at the surface, quite a bit. Mainly water vapor and carbon dioxide will continue to be mixed and will absorb, thermalize and re-excite as normal, the lapse rate in a deeper atmosphere will do the rest.
The problem for me is the lapse rate. Doubling the N2 in the atmosphere will raise the height of the atmosphere. If the surface temperature does not increase, then the lapse rate dictates that since the atmosphere extends higher, that further up, past the present altitude it must get colder than it is at present. That seem nonsensical.
Your view of the lapse rate is right on the spot. At the one bar level temperatures will be very close to what we experience today.

Joel Shore
May 11, 2011 10:50 am

jae says:

Yeah, silence! Have you even read the article at the link? I don’t see how any of your links relate to it; all they do is repeat again the radiatitive GHE theory.

No…I also explained why the general claims (by people like Postma) that the lapse rate alone can explain the surface temperature are completely wrong.

What I want to know is why ALL planetoids with atmospheres, regardless of the gases (and solids) in those atmospheres, have the same temperature at 1000 bar, when distance to the sun is factored in. Are the articles wrong? If not, that simply cannot be due to any GHE.

That article just shows this as being true for Venus and Earth (assuming the data shown is accurate), which could very well be a coincidence, especially considering that the factoring in of the distance from the sun without taking the very different albedos into account seems rather unjustified. Can you show me the data for the other planetoids? Then maybe it would be worth thinking about why this might be the case.
However, it would almost certainly have no bearing whatsoever on whether there is the GHE or not, since nobody has successfully told me how one can propose the earth and Venus have the surface temperature they do without invoking the greenhouse effect or abandoning conservation of energy.

Bryan
May 11, 2011 11:50 am

Tim Folkerts
Perhaps you could shed some light on the CO2 photon capture aftermath.
I did a calculation some time ago (and from memory) at atmospheric temperatures about 4% of CO2 molecules have the vibrational mode active.
This means that 96% are available to absorb.
So in the Earth surface up there will be a reasonable flux of 15um photons which can be absorbed to increase the active population.
The relaxation time in this mode is relatively long compared with the chance of losing the energy by collision to probably N2, O2 molecules.
This causes local heating and a local temperature rise.
The fact that the re-emission of a 15um photon requires a CO2 molecule to reach a high and statistically unlikely translational speed means that the following seems more probable;
1. Local temperature rise.
2.15um Emissions are less than absorptions as a energy consequence of 1.
3. Radiative emissions (when they happen) will favour the longer wavelength, lower energy, more probable and readily available H2O bands.
Some people have trouble with 2 as Kirchhoff no longer holds.
This is perhaps a needless concern as;
1. LTE no longer holds – a condition for Kirchhoff
2. Photon energy has been diverted along more probable paths.
Thats my take – any comments?

May 11, 2011 12:12 pm

Joel Shore says:
May 11, 2011 at 10:50 am
“…since nobody has successfully told me how one can propose the earth and Venus have the surface temperature they do without invoking the greenhouse effect or abandoning conservation of energy.”
Pressure, Joel, pressure. There I have successfully told you. Whether you accept is a different story.

May 11, 2011 12:16 pm

JAE, regarding some other comments made here, corrections are in order.
Dr Richard S. Lindzen states: “If one assumes all warming over the past century is due to anthropogenic greenhouse forcing, then the derived sensitivity of the climate to a doubling of CO2 is less than 1C.” [my bold]
None but the most radical alarmists claim that all of the warming over the past century is anthropogenic. Natural variability is always at work. Lindzen also points out:
“There is ample evidence that the Earth’s temperature as measured at the equator has remained within ± one degree centigrade for more than the past billion years. Those temperatures have not changed over the past century.” [source]
As one approaches the poles  the Earth’s temperature begins to vary more and more from one region to another, and from summer to winter, and day to night.  This is normal, and has always been so.  Greenland has very wide temperature swings, while Egypt is extremely uniform from year to year, from decade to decade, and from century to century.
By selectively picking a base year for comparison, anyone can show either warming or cooling. Pick a warm base year such as 1997, and you can show cooling. Pick a cool base year like 2001, and you can show a warming trend. But the  Earth’s overall temperature is extremely constant, and a slight variation of a few tenths of a degree is routine and natural. Temperatures have fluctuated much more over the Holocene, sometimes by tens of degrees. Dr Lindzen points out how ridiculous the current arm-waving is, over the extremely *mild* natural variability of only 0.7°C over the past century and a half:
“Future generations will wonder in bemused amazement that the early 21st century’s developed world went into hysterical panic over a globally averaged temperature increase of a few tenths of a degree, and, on the basis of gross exaggerations of highly uncertain computer projections combined into implausible chains of inference, proceeded to contemplate a roll-back of the industrial age.”
Finally, regarding the question of the climate’s sensitivity to a doubling of CO2, the UN/IPCC preposterously claims it is 3°C or more. That is absurd. If sensitivity were that high, the temperature would closely track rises in CO2. It doesn’t.
More knowledgeable scientists than the IPCC’s political appointees, including its head railroad engineer/bodice-ripper author, and its WWF advisors, provide much lower sensitivity numbers:
Dr Chylek estimates the sensitivity at 1.4°C; less than one-half the IPCC’s lowest number. Dr Schwartz gives the sensitivity as 1.1°C. Prof Lindzen puts the sensitivity at under 1°C. Dr Spencer puts the sensitivity at 0.46°C, based on observations. Drs Idso, fils & pere, put the sensitivity at 0.37°C. Dr Miskolczi puts the climate’s sensitivity to a doubling of CO2 at 0.0°C.
And there is still zero evidence of any global harm as the result of the increase in that beneficial trace gas. Conclusion: CO2, at current and projected concentrations, is both harmless and beneficial. And that answers the key AGW question: Is the rise in CO2 a problem? In fact, the increase in CO2 is, on balance, a net benefit to the biosphere.

Tim Folkerts
May 11, 2011 12:36 pm

Bryan says: May 11, 2011 at 11:50 am
“The fact that the re-emission of a 15um photon requires a CO2 molecule to reach a high and statistically unlikely translational speed …
I agree with things before this line, but I disagree with this. The average KE of a gas molecule is 3/2 kT. At 250 K this works out to ~ 5e-21 J. This is the same approximate energy as photons with a wavelength of 15 um. Thus it seems that the KE due to thermal motion should be able to excite the modes needed to emit the IR photons. (it would take considerably more time to work out actual probabilities of having various specific energy.)
Without thinking about it too deeply, it seems that peak in BB radiation would correlate quite naturally with the peak in kinetic energies of the molecules emitting the radiation. The fact that the name “Boltzmann” is associated with both ideas adds to the supposition that the two are related.

Tim Folkerts
May 11, 2011 12:59 pm

>>Joel Shore says: May 11, 2011 at 10:50 am
>>“…since nobody has successfully told me how one can propose the earth
>>and Venus have the surface temperature they do without invoking the
>>greenhouse effect or abandoning conservation of energy.”
>mkelly says: May 11, 2011 at 12:12 pm
>Pressure, Joel, pressure. There I have successfully told you. Whether you accept is a >different story.
IMHO, pressure (and lapse rate) can indeed explain temperature differences for various layers within the atmosphere.
But, mkelly, can you explain how pressure can determine either the location or the temperature of the “top of atmosphere”? What calculation will predict that a pressure of 1 bar should be ~ 15 C? Or that the temperature at 5 km altitude should be ~ -20 C?
Until you can calculate such a temperature from first principles, you have not successfully told anything — you have simply made a bold, unsupported assertion.

May 11, 2011 1:37 pm

Tim Folkerts says:
May 11, 2011 at 12:59 pm
IMHO, pressure (and lapse rate) can indeed explain temperature differences for various layers within the atmosphere.
Joel asked about the surface. I answered him. I believe your statement above would be valid for the layer that is at or immediately adjacent to the surface.

May 11, 2011 1:55 pm

5) Once we scientifically-oriented Skeptics accept the reality of the Atmospheric “greenhouse effect” we are, IMHO, better positioned to question the much larger issues which are: a) HOW MUCH does CO2 contribute to that effect, b) HOW MUCH does human burning of fossil fuels and land use changes that refuce albedo affect warming, and, perhaps most important, c) Does the resultant enhanced CO2 level and higher mean temperature actually have a net benefit for humankind?
How about using real heat transfer equations to explain this rather than back radiation. There is no input for back radiation in any heat transfer equation so taking as if it does something cannot be shown via standard equations.

Joel Shore
May 11, 2011 2:22 pm

mkelly says:

Pressure, Joel, pressure. There I have successfully told you. Whether you accept is a different story.

Sorry…Saying the world “pressure” does not get you around having to satisfy conservation of energy. As I have explained, those sorts of things explain the slope of the line (how temperature varies with height…or at least a limit on how steeply temperature can fall with height) but it does not determine the constant “b” in the equation “y = m*x + b”.
The only way you can get around the requirement that the steady-state temperature be such that the energy in from the sun equals the energy out from terrestrial radiation is to have some other significant source of energy. For a gaseous planet undergoing gravitational collapse, that source of energy could be the conversion of gravitational potential energy to other forms of energy…but that is not happening here on the earth. Energy flow from the interior of the earth is negligible.

Joel Shore
May 11, 2011 2:25 pm

mkelly says:

How about using real heat transfer equations to explain this rather than back radiation. There is no input for back radiation in any heat transfer equation so taking as if it does something cannot be shown via standard equations.

This is utter and complete hogwash. The term “back radiation” is just a label given to radiation emitted by the atmosphere that happens to travel toward the earth. All actually calculations of the greenhouse effect are done with the standard heat transfer equations. If you don’t like that label, you can get rid of it and it will make not one iota of difference in what you get out of the equations.

gnomish
May 11, 2011 2:45 pm

If the environmental lapse rate is larger than the dry adiabatic lapse rate, it has a superadiabatic lapse rate, the air is absolutely unstable — a parcel of air will gain buoyancy as it rises both below and above the lifting condensation level or convective condensation level.
what happens to the black.body spectrum of water vapor as it condenses, joel and ira and other radiation freaks? say it loud, if you dare.

Tim Folkerts
May 11, 2011 2:52 pm

mkelly says: May 11, 2011 at 1:55 pm
How about using real heat transfer equations to explain this rather than back radiation. There is no input for back radiation in any heat transfer equation so taking as if it does something cannot be shown via standard equations.

1) “back radiation” is radiation
2) radiation is included in standard heat transfer equations
Therefore “back-radiation” is included in standard heat transfer equations
If you want more detail, here are some “standard equations” that specifically deal with thermal radiation from one object of arbitrary size, shape, emissivity, and temperature to a second object of arbitrary size, shape, emissivity, and temperature:
Standard dQ/Dt (scroll about 2/3 of the way down).
“The radiation heat transfer from one surface to another is simply equal to the radiation entering the first surface from the other [added: in other words “back radiation”], minus the radiation leaving the first surface.”
http://en.wikipedia.org/wiki/Thermal_radiation
Standard form factors to go with the above equation.
http://www.engr.uky.edu/rtl/Catalog/tablecon.html
Do you contend these are not standard equations?
IF SO, what would you use instead?

Tim Folkerts
May 11, 2011 3:12 pm

Ira,
I have just a minor quibble with your wording here:
“Thus, any form of energy contains “heat” and “potential energy” and “electrical energy” and so on. As I am sure you know, that water in the elevated lake got there because the Sun heated Surface water”
I would change “contains heat” to “contains thermal energy”. In traditional thermodynamics terminology, “heat” is always energy being transferred, not contained in an object. An object doesn’t “contain heat” any more than it “contains work”. In many ways, it would be better if the language was more parallel. For example:
dU = δQ – δW
would be “the change in internal energy is equal to the ‘heat done to the system’ minus the work done by the system.” It sounds really awkward in English, but that is the traditional sense of the words in thermodynamics.
It seems nitpicky to many, I’m sure. But when talking about laws of thermodynamics in discussions like this, much of the confusion comes from imprecise wording, like using “to heat” to as a synonym for “to warm”. Or using “heat” when one means “U” (as you did above).
I do approve of your later use “the Sun heated Surface water”. There was indeed a transfer of energy from sun to water due to a temperature difference. 🙂
And I think you hit the nail on the head with:
“5) Once we scientifically-oriented Skeptics accept the reality of the Atmospheric “greenhouse effect” we are, IMHO, better positioned to question the much larger issues which are: a) HOW MUCH does CO2 contribute to that effect, b) HOW MUCH does human burning of fossil fuels and land use changes that reduce albedo affect warming, and, perhaps most important, c) Does the resultant enhanced CO2 level and higher mean temperature actually have a net benefit for humankind?”

sky
May 11, 2011 3:57 pm

Soft-science students of climate woul benefit greatly from performing a mass-weighted vertical integration of the standard atmospheric temperature profile. It nearly matches the theoretical expectation for the average graybody temperature of the Earth. This illustrates two vital points that are often overlooked: 1) the LWIR escaping into space comes NOT directly from the surface (except in the “window” wavelengths) but from a diaphanous mass of gases, and 2) there is NO additional thermal energy produced by the “greenhouse effect.”
Air temperature is a nonconservative, intensive variable whose local value depends not only upon the radiative fluxes driven by thermalization of insolation, but upon upon the atmoshperic pressure, in accordance with Boyle’s law. Indeed, if a parcel of air is moved adiabatically from the surfaceto aloft, the temparature drops accordingly. The ~33K increase in temps that we enjoy is available only because we live at the bottom of the atmosphere. It is NOT the product of any additional thermalization, for that would require TOA LWIR planetary emissions of ~390w/m^2 to maintain a steady state. The fundamental flaw of the “radiative greenhouse” paradigm is the illusion that thermalization is a recursive process. In fact, it can occur only once. Like income, it cannot be reused.

wayne
May 11, 2011 4:16 pm

Ira Glickstein, PhD says:
May 11, 2011 at 3:08 pm
PLEASE EXPERTS: Comment on this very interesting link and argument and either confirm my conclusions or modify
them or (Horrors!) accept the new argument as basically correct. advTHANKSance

Since you have never responded to all of the real aspects I have supplied to you to think about, I don’t know why I am responding. Since you said please I guess. Seems you keep asking for an expert, which I will continue to maintain I am not, whether the surface of the Earth is a black body to long wave radiation peaking at 10 µm, the answer is no. It is very close to a gray body if the spectrum is taken merely meters from the surface. The is a paper by some Swiss scientists on the current radiation flux in the Alps that gives a great multi-altitude spectrum of the upwelling LW radiation. Search it out.
Have you read Miskolczi’s papers yet as I suggested you do, by an atmospheric physicist, which explains plainly why this is so?
Also, that is a rather cheap shot when you said if someone did not respond they will accept all you believe as laid out in this thread. That make you look so incredibly shallow.

Bryan
May 11, 2011 4:21 pm

Tim Folkerts said….”I agree with things before this line, but I disagree with this. The average KE of a gas molecule is 3/2 kT. At 250 K this works out to ~ 5e-21 J.”
I agree with this
Tim Folkerts said….” This is the same approximate energy as photons with a wavelength of 15 um.”
I dont agree with this.
Energy of 15um photon is almost three times bigger.
Redo your calculations and work out probabilities from Maxwell Boltzmann Statistics

Bryan
May 11, 2011 4:25 pm

Tim
I left a redundant top line of your post – ignore that part.

Steve
May 11, 2011 4:34 pm

I am trying to figure out why a doubling of atmospheric N2 would necessarily cause a dramatic increase in temperatures at the surface, as I have seen some propose. N2 has little to no insulative properties, right?
By ideal gas laws, PV=nRT. N2 is roughly 75% of the mass of the atmosphere. I know that n is moles, not mass, so let’s assume that we just increase N2 by “a whole helluva lot”, so that n(2) = 1.75n(1), in terms of the entire atmosphere.
So we release a whole helluva lot of N2 gas into the atmosphere and then, much later, sample a cubic meter at sea level. What would we find is different about this volume V(2) as compared to V(1)? Is there any possibility that T(2) will have increased insignificantly compared to T(1)?
Assuming n is the only change on the right side of the equation, sure. I can think of a pretty simple solution. P could increase by a factor of 1.75, and that’s it – equation balanced. We end up with a thicker atmosphere (a factor of 1.75 more moles per cubic meter), but the same surface temperature.
I don’t think that is exactly what would happen, but I don’t see why increasing the mass of an atmosphere guarantees a dramatic increase in surface temperatures. It does guarantee an increase in surface pressures, but that just means a thicker atmosphere, not necessarily a hotter atmosphere.

IAmDigitap
May 11, 2011 5:20 pm

A frigid fluid immersion bath, never was, isn’t, and won’t be tomorrow, a warmer of the object immersed therein.
No matter how many times you try to claim you ‘know a guy who calculated’ it, and showed in a graph,
the colder object,
didn’t heat a warmer one.
Thank you for calculating it but all we have to do, is sit two anvils down with thermocouples mounted in them.
Not once has the cool one, subtracted it’s 75 degrees’ room temperature from itself and added it to the initially warmer one, creating a 150 degree anvil and a 0 degree one.
Now; in case you think there’s some distantly possible way you know this is capable of happening, congratulations: you have made it to fundamental law number two, before you fell out of contention for the title ‘Grasper of Reality.’
Thank you for playing, the attendant will see you to your perpetual motion-mobile.
Your magical freezing and heating anvils look nice, with the little blinking thinglies alleged to be entropy violators.
They do not however show us a cold object subtracted some of it’s own temperature, and added it to another object, already warmer. Oh no they did not, you did not, and if you claim you did, well you’re either a R.A.I.L. or another word using the same letters.
Entropy 1.
You, O.
Your move.
You don’t have one.
That’s called checkmate.
BECAUSE, when CALCULATING ENGINEERING of:
ANY SYSTEM
Y.O.U. WILL
C.H.E.C.K.
YOUR WORK,
by ASKING YOURSELF
if WHAT YOU ARE SAYING,
AGREES
with E.N.T.R.O.P.Y.
If NOT,
YOU WILL N.O.T.
DETECT IT’S EXISTENCE in APPLIED systems, of ANY KIND, natural, or MANNMADE, because E.N.T.R.O.P.Y. WILL NOT BE DISREGARDED as the FAILSAFE for ANY MATHEMATICAL ENDEAVOR.
And kids, PhD or not,
you have tried to claim that ENERGY RESIDING in FINITE INTERSTITIAL VOLUME
has VIOLATED entropy,
and that a VOLUME of INTERSTICES
containing a LOWER concentration of photons,
sent that energy
to a place with it’s interstices already MORE dense with them.
Your claim is that *pOp* went the WEASEL, and magically, energy traveled the wrong way – from a miles deep, frigid, fluid bath,
to the object submerged therein.
Shiny blinkie spreadsheets or not, you did not warm the object submerged in the frigid gas bath,
using that frigid gas bath.
No, you did not, and that’s why you’re reduced to showing up without an instrument on earth that verifies the ***false claim*** you made.
All we had to do was check and see if you’d wind up obeying entropy. And when you made net energy travel from the cooler to the warmer of two objects, y.o.u.
were
d.o.n.e.
D.O.N.E. and OUT to PASTURE
deep, D.E.E.P:
in Area 51/Bigfoot/Psychic Spoon Bending/A.G.W. Country

Joel Shore
May 11, 2011 5:45 pm

sky says:

This illustrates two vital points that are often overlooked: 1) the LWIR escaping into space comes NOT directly from the surface (except in the “window” wavelengths) but from a diaphanous mass of gases,

Yes…because some of the radiation from the surface gets absorbed and then radiation is emitted from the atmosphere…but less because the atmosphere is at a colder temperature.

and 2) there is NO additional thermal energy produced by the “greenhouse effect.”

Nonsense. What you have shown is there is NO additional thermal energy produced by the “greenhouse effect” other than that which is produced by the “greenhouse effect”. What do you think would happen if there were no IR-absorbing elements in the atmosphere? Would any of the radiation from the surface be absorbed?

Indeed, if a parcel of air is moved adiabatically from the surfaceto aloft, the temparature drops accordingly.

All that this does is tell you is the maximum rate that the temperature can drop with altitude, i.e., the lapse rate. It does not by itself tell you what the temperature at the surface is.

The ~33K increase in temps that we enjoy is available only because we live at the bottom of the atmosphere.

Yes…The bottom of an atmosphere that has IR-absorbing elements and hence a greenhouse effect.

It is NOT the product of any additional thermalization, for that would require TOA LWIR planetary emissions of ~390w/m^2 to maintain a steady state. The fundamental flaw of the “radiative greenhouse” paradigm is the illusion that thermalization is a recursive process. In fact, it can occur only once. Like income, it cannot be reused.

Not a clue what you’re trying to say here.

May 11, 2011 5:54 pm

OK, apparently I got the blockquote wrong and thus my previous post doesn’t make sense, so here goes again:
“Tim Folkerts says:
May 10, 2011 at 9:42 am
mkelly,
You needed to look a little farther and understand what you are reading. This is from the SAME source you quoted:
Net Radiation Loss Rate
If an hot object is radiating energy to its cooler surroundings the net radiation heat loss rate can be expressed as
q = ε σ (Th^4 – Tc^4) Ac (3)
http://www.engineeringtoolbox.com/radiation-heat-transfer-d_431.html
Please note there ARE inputs for back radiation (the Tc^4 term).”
First I apologise if I am rehashing something that has already been discussed, I have been out in the field and haven’t had a chance to catch up on all the posts in this thread as yet, however this one did.
The equation cited does not, in my opinion, imply backradiation. If it did there would be terms for the emissivity of both objects as well as the surface area of both objects. There aren’t. What the equation does account for is the net difference in temperatures between the two objects or one object and its surroundings.
The net difference in temperature is obviously one of the factors which determines the flow of energy from the hot body to the colder one. If this were water we would call it the hydraulic head, if it were electricity it would be the voltage. Furthermore, if the surroundings or other object did heat the hotter object through “backradiation” wouldn’t the equation not only require terms for the surface area and emissivity of the colder object, but also require integration of the effect of the backradiation? In other words, to consider just the colder object, as it emits energy to the warmer object it cools, but it also warms from the energy input of the warmer object which causes its temperature to rise as a function of the difference between its absortivity and its emissivity. The increased temperature would then cause it to backradiate more energy etc… While at the same time the hotter object would be behaving in a like fashion. So you would need to integrate the energy inputs to each from each as a function of not only emissivity but absorbtivity as the net temperature difference tended towards 0 ( assuming it could in fact tend towards 0)?
and just recently Tim said:
“Standard dQ/Dt (scroll about 2/3 of the way down).
“The radiation heat transfer from one surface to another is simply equal to the radiation entering the first surface from the other [added: in other words “back radiation”], minus the radiation leaving the first surface.”
http://en.wikipedia.org/wiki/Thermal_radiation
Sorry, but this is just a staight radiative energy transfer, no back radiation involved here at all. The energy transfered to an object is equal to the incoming energy minus the energy the object re-radiates. If back radiation existed the definition would need to be modified to say something like:
‘The radiation heat transfer from one surface to another is simply equal to the radiation entering the first surface from the other, minus the radiation leaving the first surface, plus the percentage of the energy leaving the first surface and entering the second object.’
This would of course pose a problem because the energy transfer would never end and we would have created a perpetual motion machine, not to mention a really hot one 🙂 In other words you would never reach energy and temperature equilibrium between the two objects, which is obviously contrary to reality.
Back radiation from a colder object cannot heat a warmer object, energy only travels from a warmer to a colder object, unless external work is applied.

Steve
May 11, 2011 6:30 pm

Dear Digitap,
Which will cool off faster, and to a lower temperature:
A 275K anvil in a 200K frigid bath…
A 275K anvil in a 150K frigid bath…

gnomish
May 11, 2011 7:11 pm

Whereas the condensation of water vapor liberates heat
and whereas that phase change does not exhibit a temperature change
therefore, black bodies from outer space are not telling much about the planet’s energy flux.
We all know that on a sunny day it’s cooler to be in the shade.
Gardeners know that at night, clouds will reflect heat and keep it warmer than a clear sky.
It seems to go without saying enough, though, that all the while our water vapor clouds are moving the heat from the oceans to a place near you, CO2 sits there with it’s thumb up it’s butt, useless for shade or reflecting heat. It needs to be said and repeated, I guess.
The amount of heat transported by the CO2 in any volume of our atmosphere is less than 1/50,000 that of the water vapor in that same volume. It just doesn’t count IRL.

jae
May 11, 2011 7:15 pm

Ira:
“You criticize my reply to “JAE” (all upper case) but actually I was replying to “jae” (all lower case). I don’t know if both “JAE” and “jae are the same person, but “jae” has earned my reply, at least IMHO.”
Yes they are the same, but I think your reply was a little harsh. But I also don’t care.

jae
May 11, 2011 7:25 pm

Steve:
“Dear Digitap,
Which will cool off faster, and to a lower temperature:
A 275K anvil in a 200K frigid bath…
A 275K anvil in a 150K frigid bath…”
Answer obvious, and pertains only to anvils in frigid baths (or similar concepts), but it is completely irrelevant to the discussion at hand: it has NO relationship to atmospheric physics, due to “other factors,” such as convection and evaporation/condensation of water.
Sorry.
I am SO damn sick of the radiative effects being presented without any mention of all the other variables. It is, to put it as kind as I can muster, STUPID! This planet does NOT operate solely on the basis of some radiation cartoons! Please tell us how these radiative effects interact with convective and evaporative effects, and then you won’t have to deal with so many skeptics.
Here’s the real question: Will an atmosphere of almost pure OCO warm the surface more than an atomsphere of 0.04% OCO?
Evidently not, according to the links I provided above.

Tim Folkerts
May 11, 2011 7:25 pm

Alleyne says:
“Sorry, but this is just a staight radiative energy transfer, no back radiation involved here at all. The energy transfered to an object is equal to the incoming energy minus the energy the object re-radiates. ”
I think we just have different definitions of the words. What you call “the incoming energy” (from radiation from some second object directed at the first object) is what I call “back radiation”. I know the name is poor and can lead to confusion.
Then you say “the definition would need to be modified to say something like:
‘The radiation heat transfer from one surface to another is simply equal to the radiation entering the first surface from the other, minus the radiation leaving the first surface, plus the percentage of the energy leaving the first surface and entering the second object.’ ”
I sort of see your point, but again, I think it is just a question of terminology. The “radiation back toward the first object from the second object” is certainly just a function of the temperature of the second object — as you say, no “percentage of the energy leaving the first surface and entering the second object” is needed in the equations. I agree 100% with you.
However, I would ask what sets the temperature of the second object that is radiating back? Well, it could have many sources of energy, but one of them will be the energy it receives from he first object. That energy will raise the temperature higher than if that second object was not receiving energy from the first object. This elevated temperature will cause the second object to radiate more energy. In particular, the “radiation back toward the first object from the second object” will increase. In otherword, while the “back radiation” is completely determined by the temperature of the second object (along with geometry & emissivities), that temperature is at least in part a function of the “forward radiation” from the first object.
I don’t expect the “divergent series problem you suggest here: “This would of course pose a problem because the energy transfer would never end and we would have created a perpetual motion machine, not to mention a really hot one 🙂 In other words you would never reach energy and temperature equilibrium between the two objects, which is obviously contrary to reality.”
There will indeed be an equilibrium reached because the series will converge. There is a discussion in this thread if you want to see what was said before. Search for “chicken” to find the relevant parts of the discussion. http://wattsupwiththat.com/2011/03/29/visualizing-the-greenhouse-effect-molecules-and-photons/

Tim Folkerts
May 11, 2011 7:38 pm

IAmDigitap says: May 11, 2011 at 5:20 pm
A frigid fluid immersion bath, never was, isn’t, and won’t be tomorrow, a warmer of the object immersed therein.

The earth is “immersed” in an “amazingly cold” fluid of 3 K microwave photons. With no energy source, the earth would cool to 3 K.
The earth IS heated — it absorbs ~ 240 W/m^2 of sunlight. This is enough to raise the surface temperature quite a bit (the exact value depends on what assumptions you make about emissivity & albedo).
Now immerse the earth in a “frigid fluid” of IR photons from the atmosphere. This fluid is indeed cooler than the surface. (For simplicity, we could leave a thin vacuum layer around the earth to prevent conduction & convection so that only radiation was important.)
QUESTION. Will this surface, immersed in the “frigid” fluid of IR photons, be warmer than the surface immersed in the “amazingly cold” 3 K fluid of photons?

Tim Folkerts
May 11, 2011 7:56 pm

Bryan,
I agree that the energies involved in the MB distribution for room temperature gases would put the IR energies out toward the tail.
It would be interesting to explore that issue more, but right now I don’t have time to dig into that. (There would also be messy details introduced my the fact that the energies of the vibrational modes are quantized). My hunch is that there will be fat enough tails for the molecules to emit the IR, but I could be wrong.

ferd berple
May 11, 2011 8:21 pm

It sounds like science doesn’t know what effect doubling the N2 in the earth’s atmosphere will have on surface temepratures, all else being held equal.
As such, there is no way to know how much of the 33K “missing” in the radiative transfer model is due to N2/O2. Without that basic knowledge, the assumption that all of the 33K heating is due to GHG seems an enormous leap of faith. Especially as the main prediction of this model, the “hot spot” cannot be observed except through creative squinting. This in itself would in most science fields constitute falsification of the GHG theory of global warming.
The fact that no hot spot has develped suggests that the effects of GHG are to increase the vertical convection rate, increasing the rate at which heat is returned to space. That gravity is responsible for the 33K in unexplained heating and contrary to the assumptions of the radiative transfer model, increasing the weight of N2O2 in the atmosphere will increase the surface temperature, as more and more molecules are packed into a smaller volume, resulting in a net increase in energy per cubic meter of atmosphere at the surface, which we measure as an increase in temperature.

Steve
May 11, 2011 9:00 pm

ferd berple says: “…increasing the weight of N2O2 in the atmosphere will increase the surface temperature, as more and more molecules are packed into a smaller volume, resulting in a net increase in energy per cubic meter of atmosphere at the surface, which we measure as an increase in temperature.”
No, packing more molecules of a gas into a smaller space does not, in and of itself, raise the temperature of the gas. It raises the pressure. If what you are thinking were true, scuba divers would have one hell of a problem, given that tanks are typically filled to 200 bars. Those metal tanks are not insulated, and with internal temperatures 200 times the ambient air temperature… those would be some seriously hot tanks!
jae says: “Here’s the real question: Will an atmosphere of almost pure OCO warm the surface more than an atomsphere of 0.04% OCO? Evidently not, according to the links I provided above.”
Are you referring to the link regarding “Venus atmospheric pressure explains Venus surface temperature?” That link only demonstrates a misunderstanding of the ideal gas law.

Phil.
May 11, 2011 9:03 pm

jae says:
May 11, 2011 at 6:30 am
In that vein, I’m still really interested why there is so much silence surrounding this (and other related expositions):
http://theendofthemystery.blogspot.com/2010/11/venus-no-greenhouse-effect.html

Presumably because it’s nonsense?
An example, it starts with:
“Venus is closer to the Sun, and gets proportionally more power from it. Earth is 93 million miles from the Sun, on average, while Venus is only 67.25 million. Since the intensity of the Sun’s radiation decreases with distance from it as 1 over r-squared, Venus receives (93/67.25) squared, or 1.91 times the power per unit area that Earth receives, on average.”
It goes on to assert that any point in the Venusian atmosphere at the same pressure as the Earth’s would be at the same temperature with appropriate adjustment for the relative power received from the sun. This is said to be the 4th root of the ratio of the powers (∜1.91), and lo and behold they match!
The trouble is that ~30% the power received at the Earth’s TOA is reflected and ~90% of the power received at the Venusian TOA is reflected, so by his argument Venus receives less power than Earth!

May 11, 2011 9:09 pm

Joel Shore says:
“How am I supposed to convince you of something that involves seriously weighing of scientific evidence when I cannot even get you to admit you have made in error when it is right in front of your face for everyone to see?”
Quit blustering. My comment was: “Please provide testable, measurable, reproducible and convincing evidence demonstrating verifiable global harm specifically due to the rise in CO2 – you know: per the scientific method.”
I’m still waiting.

Phil.
May 11, 2011 9:12 pm

Alleyne says:
May 11, 2011 at 5:54 pm
“Tim Folkerts says:
May 10, 2011 at 9:42 am
mkelly,
You needed to look a little farther and understand what you are reading. This is from the SAME source you quoted:
Net Radiation Loss Rate
If an hot object is radiating energy to its cooler surroundings the net radiation heat loss rate can be expressed as
q = ε σ (Th^4 – Tc^4) Ac (3)
http://www.engineeringtoolbox.com/radiation-heat-transfer-d_431.html
Please note there ARE inputs for back radiation (the Tc^4 term).”
The equation cited does not, in my opinion, imply backradiation. If it did there would be terms for the emissivity of both objects as well as the surface area of both objects. There aren’t.

There certainly are, the energy balance is done at the surface of ‘h’, and the area is the same for both outgoing and incoming radiation, ‘Ac’, since ‘h’ is immersed in ‘c’.
Strictly the equation should be:
q =(ε σ Th^4 – a σ Tc^4) Ac
however following Kirchoff’s Law a=ε
so:
q = ε σ (Th^4 – Tc^4) Ac

wayne
May 11, 2011 9:32 pm

Alleyne, very clearly and properly said. Can I use your words in a pinch?
(I should have taken a writing course)
You know, I searched for the first reference to the words “back radiation” and the only official place I could find it used is in I.P.C.C.’s AR4 WGI Chapter 3 – 3.4.4.2 Surface Radiation. It is not in any pre-1990’s physics books I can find. I sure would love to find out where and when this term and warped view of energy transfer came to be.

Tim Folkerts
May 11, 2011 10:00 pm

ferd berple says: May 11, 2011 at 8:21 pm
“It sounds like science doesn’t know what effect doubling the N2 in the earth’s atmosphere will have on surface temepratures, all else being held equal.”
More specifically, it sounds like us amateurs posting here in our spare time don’t know the answer. I suspect there are calculations that have been done that could answer your question.
The biggest question to me is how would that affect the distribution of CO2 & H2O since those are the key players in the radiation balance. Part of me says that more gas of any sort (ie the N2) would raise the whole atmosphere, including the CO2; part of me says each gas is independent and of the partial pressure of CO2 stays the same, then vertical distribution of CO2 would stay the same. That question is non-trivial and would require a little more thought for me to reach any firm conclusion.
In the first case, I suspect the extra N2 would have a significant impact; in the second case, the N2 would have only a minor impact.

Tim Folkerts
May 11, 2011 10:24 pm

Wayne,
If you don’t like “back radiation”, you could always use “plain old thermal radiation from the molecules in the atmosphere that happens to be directed back down in the general direction of the surface”.
Just like the light that happens to be heading west from a light bulb could be called “west radiation”. Just like microwaves aimed up and to the northwest could be called “(-i + j + k) radiation”. Just like a photographer’s flash that is aimed back toward the camera from the far side is called “back light”.
Adding a designation to the direction of a flow of photons doesn’t create any new ideas or new laws of physics. It doesn’t invalidate any old laws of physics. It simply tells people the direction of the relevant EM radiation.

Bryan
May 12, 2011 12:57 am

Ira Glickstein might care to comment
Tim Folkerts says …..”I agree that the energies involved in the MB distribution for room temperature gases would put the IR energies out toward the tail.”……
Yes its all about probabilities for air at 250K.
1. From temperature considerations 4% CO2 molecules in the active vibrational state.
2. Now add a flux 15um radiation from Earth surface to further enhance the active numbers.
3. The relaxation time of vibrational state is longer than the chance of collision deactivation
4. The collision route will share the vibrational energy between CO2 and O2 molecules leading to local temperature increase.
5. The radiative route takes two paths
a. Collision activation of longer wave IR emissions from H2O is favoured for two reasons
(i) Nearly 40 times more H2O molecules than CO2
(ii) Plenty H2O bands at wavelengths > 15um => much more probable.
6. Still about 4% chance of CO2 getting enough energy to be vibrationally active but even then back to 3 above.
Net result of all this
Photon energy from 15um Earth surface up going to increase in temperature of atmosphere and increasingly more radiation in wavelengths > 15um.
Some might find a Kirchhoff problem here;
But I think this worry is unfounded
1. Photon energy being transformed into translational KE in N2,O2 molecules.
2. Kirchhoff requires LT E to be valid and in this case this is not satisfied.
My clincher evidence comes from the “bite around 15um ” seen in the satellite looking down spectrum.

Bryan
May 12, 2011 1:10 am

Tim Folkerts says
…..”Wayne If you don’t like “back radiation”, you could always use “plain old thermal radiation from the molecules in the atmosphere that happens to be directed back down in the general direction of the surface”…….
However Tim, climate science seems to have saddled itself with labels that are to say the least highly misleading.
Back radiation includes a lot of solar radiation moderated by the clouds; in other words it NOT come BACK from the Earth surface.

R Stevenson
May 12, 2011 2:54 am

Robert Clemenzi writes:
R Stevenson,
I would very much like a link to that spreadsheet. My attempts to locate Hottel’s method keep hitting paywalls.
Using 1.0% water vapor, my program computes 282.68 W/m2 .. close to your value.
For power absorbed, my program incorrectly assumes 1 km at constant temperature and pressure. However, it also plots the distance to absorb x% of the available radiation. As long as those values are under 100 meters, I feel that assuming constant temperature and pressure causes only minor errors.
At any rate, I would like to investigate WHY the programs are giving different values to absorb the radiation.
Robert Clemenzi
In my method I calculate the absorptivity of a hemispherical mass of gas (dry air for CO2 alone) of radius L (L= extinction distance) using graphs developed by Hottel and Mangelsdorf which can be found in Heat Trnsmission by McAdams .

Tim Folkerts
May 12, 2011 5:53 am

Bryan says:
“Back radiation includes a lot of solar radiation moderated by the clouds; in other words it NOT come BACK from the Earth surface.”
I think the preposition you chose is a big part of the misunderstanding. The radiation is back TOWARD the surface, not back FROM the surface.
Beyond that, I’m not sure I agree with your wording that back radiation “includes a lot of solar radiation moderated by the clouds”. I interpret the Trenberth energy balance diagram as follows:
* If the solar radiation is simply scattered/reflected AWAY from the surface, it is part of albedo, which is included in the 77 W/m^2.
* If the solar energy is reflected/scattered TOWARD the ground, it is part of the 168 W/m^2.
* If the solar energy is ABSORBED by the clouds, it is part of the 67 W/m^2.
So the “solar radiation moderated by the clouds” is already all accounted for.
The clouds do indeed provide part of the 324 W/m “back radiation”, with the rest provided by the GHGs. But that 324 W/m^2 of thermal IR comes from the energy OF THE CLOUDS (and air molecules) at a rate determined by the temperature OF THE CLOUDS (and air molecules).
While there is a little more potential for misunderstanding from this term than from many others,with a small effort, anyone who knows some science should readily be able to understand the meanings.
As another example, the phrase “for every action there is an equal and opposite reaction” is misleading to many people. That doesn’t mean Newton’s 3rd Law needs to be replaced. It just means you need to be a little more careful when teaching Newton’s 3rd Law.

Dave Springer
May 12, 2011 6:39 am

Tim Folkerts says:
May 11, 2011 at 10:00 pm

ferd berple says: May 11, 2011 at 8:21 pm
“It sounds like science doesn’t know what effect doubling the N2 in the earth’s atmosphere will have on surface temepratures, all else being held equal.”
More specifically, it sounds like us amateurs posting here in our spare time don’t know the answer. I suspect there are calculations that have been done that could answer your question.

Or less specifically formulating an answer is an exercise in wool gathering.
Let’s start off with one effect. It would increase atmospheric pressure at sea level to 1.7 bar. That would raise the boiling point of water to 250F or so. The three-minute boiled egg would become the two-minute boiled egg. There would be a zillion other things that change.

Dave Springer
May 12, 2011 7:49 am

Ira Glickstein, PhD says:
May 11, 2011 at 3:08 pm

New calcs of radiative GHE shows the GHE is only 8-9 deg. C:
http://activistteacher.blogspot.com/2011/05/radiation-physics-constraints-on-global.html
//////////////////////////////////////
WOW! I just read the linked webpage and it seems fairly convincing. I hope Joel Shore and Dave Springer and others comment and either set me straight or agree that I and virtually everybody else on the Internet who claims the Atmospheric “greenhouse effect” contributes about 33º to the average temperature of the Surface. If the correct number is closer to a third of that amount, established science has got a lot of ‘splainin to do. (I am pretty sure there is an error in the new claims, but I cannot figure it out on my own.)

Tim beat me to it but I wouldn’t use white paint as the example. I’d use the ocean.
The ocean absorbs close to 100% of the visible spectrum that reaches it and the absorption continues to a depth of about 100 meters where there is effectively no more light.
It doesn’t re-emit visible light. It emits in the far infrared and it sucks as an infrared emitter. It also sucks as an infrared absorber. Infrared radiation only penetrates the ocean to a depth of 1 micrometer. Instead of warming the water down to a significant depth it rather serves to increase the evaporation rate and the heat is carried off the surface as latent heat of vaporization. So the daytime radiative heating of the ocean isn’t followed by radiative cooling at night because water is quite opaque to IR. Only a thin 1 micrometer film at the surface can transmit IR. Convection does the heavy lifting. As the surface water cools at night through evaporation it gets denser and sinks while warmer water from below rises to replace it.
The truism that good absorbers are good emitters has a caveat: it only applies to absorption and emission of the same wavelength. The nut of the greenhouse effect is dependent on this. CO2, like water, is largely transparent to visible light but opaque in the infrared spectrum. Energy enters the system as visible light but must escape as infrared because nothing gets hot enough to emit in the visible spectrum.
[Update 3:30PM Florida time. THANKS Dave springer for this info. I fixed the blockquote stuff. – Ira]

Dave Springer
May 12, 2011 8:04 am

Speaking of caveats I owe Phil an explanation on atmospheric nitrogen thermal radiation. It must remain true that nitrogen radiates. ALL matter above absolute zero has thermal radiation which is a consequence of atoms being composed of oppositely charged particle in motion relative to each other. However, nitrogen is a poor absorber of infrared so it is also a poor emitter of infrared. So while it must have a thermal spectrum characteristic of its temperature the intensity of that spectrum is down in the dirt so it contributes nothing by itself, per se, to the infrared glow of the atmosphere. However it can still itself be thermalized kinetically and it can transfer kinetic energy to good IR emitters like water vapor and CO2 so it can’t just be ignored like it isn’t there at all.

May 12, 2011 8:12 am

R Stevenson says:
May 12, 2011 at 2:54 am
My heat transfer book has a rough mean beam length formula of Le=3.6(V/A) where V is volume of gas and A is area.

Bryan
May 12, 2011 8:26 am

Tim Folkerts said ….”The clouds do indeed provide part of the 324 W/m “back radiation”, with the rest provided by the GHGs. But that 324 W/m^2 of thermal IR comes from the energy OF THE CLOUDS (and air molecules) at a rate determined by the temperature OF THE CLOUDS (and air molecules).”….
The natural meaning of Backradiation is radiation that has come “back” from somewhere.
The only place “back” can refer to in this context is the planet surface.
Since it plainly does not all originate from the planet surface the term should be avoided.
Downwelling long wavelength radiation is a much better title.
If we were to accept your definition then from KT97 we have 350W/m2 up and 324 W/m2 back.
The readily available spectra graphs show that this is wrong.

Phil.
May 12, 2011 8:44 am

Bryan says:
May 12, 2011 at 12:57 am
Ira Glickstein might care to comment
Tim Folkerts says …..”I agree that the energies involved in the MB distribution for room temperature gases would put the IR energies out toward the tail.”……
Yes its all about probabilities for air at 250K.
1. From temperature considerations 4% CO2 molecules in the active vibrational state.
2. Now add a flux 15um radiation from Earth surface to further enhance the active numbers.
3. The relaxation time of vibrational state is longer than the chance of collision deactivation
4. The collision route will share the vibrational energy between CO2 and O2 molecules leading to local temperature increase.
5. The radiative route takes two paths
a. Collision activation of longer wave IR emissions from H2O is favoured for two reasons
(i) Nearly 40 times more H2O molecules than CO2

True near the surface, becomes less so as you increase altitude or go near the poles or to deserts. So near the surface the energy absorbed by CO2 is likely to be shuttled through N2/O2 to be emitted at longer wavelength by H2O. It’s like how a Helium/Neon laser works, the He is excited by electric discharge collides with Ne thereby exciting it (3s level) and then Ne emits (3s->2p) the familiar red beam. As seen from the ‘down’ spectra though some CO2 is still emitted.
(ii) Plenty H2O bands at wavelengths > 15um => much more probable.
Again depends where you are in the atmosphere. Also depends on the relative radiation lifetime of the excited states of H2O, if shorter than the CO2 lifetimes or not. From what I recall from shocktube studies, H2O has a slightly shorter lifetime but that was at wavelengths 14 micron and below.

Dave Springer
May 12, 2011 8:51 am

mkelly says:
May 11, 2011 at 12:12 pm
“Pressure, Joel, pressure. There I have successfully told you. Whether you accept is a different story.”
So if I compress air into tank until the compressional heating raises it above the boiling point of water then I can use it to run a steam engine forever, right? Because it will never cool off until I let the pressure escape.

Tim Folkerts
May 12, 2011 8:57 am

Bryan says: May 12, 2011 at 8:26 am
“Downwelling long wavelength radiation is a much better title.”

I think everyone agrees this is a much more descriptive and accurate name. So every time you hear “back radiation” just substitute in your mind “Downwelling long wavelength radiation”. The terminology is secondary, as long as everyone who really cares understands what is meant.
“The readily available spectra graphs show that this is wrong.”
Could you provide a link to specific spectra that you are referring to? What is wrong with them?

Bryan
May 12, 2011 9:37 am

Phil thanks for your input and your qualification of particulars in my post are well founded.

May 12, 2011 9:42 am

Dave Springer says:
May 12, 2011 at 8:51 am
Sir, if you disgree that pressure causes an increase in temperature if volume is held steady that is well odd. PV= nRT discribes what is termed STP (Standard temperture and pressure). At 1 atmosphere the temperature of air is 0 C. On earth that accounts for 18 degrees of the so called 33 degrees of warming by GHG.
Is it hotter in Death Valley or the top of Everest on the same day of the year?
Further, I repeat that none of the heat transfer equations have an input for back radiation. So talking of something that does not exist in formulated heat transfer equations is strange. Please show me an equation that has an input for back radiation and is called back radiation in the formula. If you do I will speak no more of it.

Tim Folkerts
May 12, 2011 10:42 am

mkelly says: May 12, 2011 at 9:42 am
“At 1 atmosphere the temperature of air is 0 C.”
No, at 1 Atm the temperature of 1 MOLE OF GAS occupying 22.4 LITERS is 0 C. Go around the global at sea level and you will be right around 1 Atm the whole time, but I will guarantee the temperature will not be 0 C simply because the pressure is 1 Atm.
“STP” is simply an ARBITRARY definition for uniform comparisons. It is in no way related to what temperature the air “should” be on earth or Jupiter or Pluto. Choosing a standard is only done to allow — well — standard measurements that can be compared easily.
For example, in the US you buy natural gas by the cubic foot. But, of course, you would get a different number of moles in each cu ft depending on the temperature and pressure. The standard is 60 °F and 14.73 psi. This in no way says that natural gas “should” be 60 °F. It simply says that if you buy the natural gas at high pressure, you get charged more for the actual cu ft of gas because it would have more ‘standard cu ft’.

Joel Shore
May 12, 2011 10:42 am

mkelly says:

Sir, if you disgree that pressure causes an increase in temperature if volume is held steady that is well odd.

Nobody is disagreeing with that. However, the volume is not “held constant” in the atmosphere.
You’ve had this all explained to you a million times: Considerations of the adiabatic lapse rate tell us the maximum rate at which temperature can decrease with height. However,
(1) It does not tell us that the temperature must decrease with height at this rate, and in fact it doesn’t once you get beyond the troposphere. The actual temperature distribution depends on how the atmosphere is heated, with the adiabatic lapse rate just providing a stability limit…i.e., an upper bound on the allowed lapse rate. Since the bottom of the atmosphere (the troposphere) is heated strongly from below, it turns out that the actual lapse rate in this part of the atmosphere is usually close to the (appropriate dry or moist) adiabatic lapse rate.
(2) Even once you know the lapse rate, that doesn’t determine the temperature at the surface alone. You must have an additional criterion, the temperature at some level in the troposphere, in order to determine the surface temperature. In particular, this criterion turns out to be that the average temperature at the effective radiating level in the atmosphere has to be equal to the “blackbody temperature” so that the earth system emits back out into space the same amount of energy it absorbs from the sun. This effective radiating level depends on atmospheric composition and, in particular, on those constituents that absorb terrestrial radiation.

Further, I repeat that none of the heat transfer equations have an input for back radiation.

If you repeat it a thousand times, it will be no less false than it was the first time. We have explained this to you. What you call it is totally irrelevant. If you don’t like the term, don’t use it. The greenhouse effect does not depend on calling the radiation that is downwelling from the atmosphere “back-radiation”. You can call it “Magical mystery radiation from the planet Zircon” and as long as you use the same equations, you will get the same result: namely, the greenhouse effect.

Joel Shore
May 12, 2011 10:45 am

[Perhaps this is obvious, but since nothing seems to obvious to state explicitly here, when I say “you use the same equations”, I mean the standard equations used in all radiative transfer calculations, i.e., those used by scientists and engineers in practical calculations every day (as David M Hoffer has pointed out).]

May 12, 2011 11:29 am

Joel Shore says:
May 9, 2011 at 2:09 pm
“Any text on thermodynamics that treats it from a statistical physics perspective ought to do fine. In the context of radiative heat transfer specifically, any text that talks about the exchange of radiation between two objects or any object and its surroundings ought to give you the basic radiative transfer equations that are used in computing the greenhouse effect.”
Thank you Joel, but I’m afraid, not being as widely read as yourself, that I can’t find a textbook that supports your theory. Could you perhaps cite the document in question, and even better, quote the relevant passage(s)?  I am familiar with blackbody and greybody (to their surroundings) radiation equations and I confess that I find no evidence of backradiation in the equations.  Perhaps you could enlighten me?
Joel Shore says:
May 9, 2011 at 2:09 pm
“Yes…So, heat (which is a macroscopic concept by its definition) always flows from hotter to colder (in the absence of work). However, note that the interpretation that the radiative energy flows in both directions, while having an abundance of empirical support, is not necessary to show there is a greenhouse effect. All that is really necessary to say is that the heat flow between two objects depends on the temperature of both objects and not just on the temperature of the hotter object.
Of course the energy flow between the two objects depends on the difference in temperature between the two objects.  However that does not imply that energy flows from the cooler object to the warmer object.  That merely accounts for the difference in energy states.  The greater the difference in energy states, the more energy will flow from the higher state to the lower state.  The potential energy of an object 5 meters above the ground is greater than that of an object 1 meter above the ground.  The energy transfer from the higher object to the ground (via transformation to kinetic energy) will be greater than the energy transfer from the lower object to the ground.  However the energy transfer does not require that the ground move towards either object.  The energy transfer (current) in an electric circuit is unidirectional and proportional to the difference in the energy states between the source of the energy and its destination.  Energy is energy, it doesn’t change nature or characteristics, whether it is potential, kinetic or radiative.
I don’t dispute that radiative energy can flow in both directions, I don’t think anyone does.  Any object above 0K will radiate some energy, just because you put a hotter one next to a colder one doesn’t mean the colder one stops radiating – but neither does it mean that the colder one will warm the hotter one.  
The SB law of the radiation energy of a black body
q = σ*T^4*A 
states that the radiation energy of an object is proportional to the fourth power of its absolute temperature.
The grey body equation for the radiation loss rate of an object to its surroundings 
q = ε*σ*(Th^4 – Tc^4) Ah
must correct for the temperature of the surroundings in order to correctly reflect the absolute temperature of the object, hence the term (Th^4-Tc^4).  This however in no way implies that the surroundings (which are radiating energy, not being at 0K) transfer energy to the object.
Furthermore the equation has a term for the emissivity of the object and no term for the emissivity of the surroundings or the absorptiveness of the object, both of which would be required if the surroundings were transferring energy to the object.
If you put a mirror a few feet away from an infrared source, say a heat lamp, the mirror radiates energy as visible white light and LWIR, both radiative energy, but that won’t increase the temperature of the heat lamp any.
Joel Shore says:
May 9, 2011 at 2:09 pm
“I have no clue what you are trying to say in this last sentence. The term “heat pump” is sometimes used as a category that includes air conditioners and refrigerators. Other times, it is used to represent the subcategory of the “heat pump” in the above context that are actually used to heat a house rather than cool it. However, regardless of how the term is used, heat pumps used for heating and air conditioners operate by the same basic principle. Both use work to “pump” heat energy from cold to hot, i.e., in the opposite direction from which heat spontaneously “flows”.”
I was using the term “heat pump” in the colloquial sense, meaning of a machine which heats a house by extracting heat from the surroundings and moving it into the house.  The reason I contrasted the two types of ‘heat pump’, heat pumps and air conditioners, was to illustrate the large amount of work (energy) required to circumvent the Second Law – ie to cool an object further by moving energy from it to a warmer one or warmer surroundings.
I don’t necessarily disagree at this point with the concept that GHGs trap heat, I disagree with the idea contention that they heat the earth through backradiation.  They can’t, they don’t.
But just for the sake of argument, tell me in your theory what the temperatures of the earth and atmosphere are, the amount of energy radiated from the earth to the atmosphere and backradiated from the atmosphere to the earth.  Please state all units in Kelvin if you don’t mind.  I’ll run the calcs and see if the numbers prove me wrong.

May 12, 2011 12:00 pm

Joel Shore says:
May 12, 2011 at 10:42 am
What you call it is totally irrelevant.
Sorry I totally disagree. What a thing is named is important. What the equations call temperaturee you seem to want to call back radiation. Temperture is temperature and radiation is radiation. They maybe intertwined but they are not interchangeable.
And since you did not supply an equation with a back radiation input I must declare I am correct.
Alleyne says:
May 12, 2011 at 11:29 am
q = ε*σ*(Th^4 – Tc^4) Ah
The above from an astute person shows clearly that if two objects of the same temperature are radiating at each other there is no (none, nada, zippo) energy exchange. No back radiation heating the other up as you claim back radiation heats the earth.

R Stevenson
May 12, 2011 12:03 pm

MKelly says:
My heat transfer book has a rough mean beam length formula of Le=3.6(V/A) where V is volume of gas and A is area.
The original charts were for a hemispherical gas mass of radius L. Constants by which the characteristic dimensions of simple shapes are multiplied use 3.4(V/A) to obtain an equivalent mean hemispherical beam length L.

May 12, 2011 12:18 pm

Ira, thank you very much for your reply to my comment on your article “IraGlickstein, PhD says May 11, 2011 at 2:23 pm: “——-“
I have not had a chance to reply until now. – Well, “until now” is not strictly correct as I finished an explanatory reply as to why you “get a disconnect”. However before I submitted my posting I read through some of the later comments and found that Tim Folkerts says it much more elegantly in his comment on May 11, 2011 at 3:12 pm, than I did in mine so I scrubbed my entry and recommend his instead.

Tim Folkerts
May 12, 2011 12:32 pm

Mkelly,
The above from an astute person shows clearly that if two objects of the same temperature are radiating at each other there is no (none, nada, zippo) energy exchange HEAT (ie no NET energy exchange). You even say specifically that they are radiating at each other; since radiation = traveling EM energy, you just admitted that energy is being exchanged in the very sentence where you say there is no energy exchange.
As you just said, what you call things DOES matter — ESPECIALLY if you call them the wrong thing (ie saying that “q” is “energy” rather than “heat”)! There IS energy exchange when the temperature is the same — but the energy going each way is the same. There is no HEAT from one to the other.
“What the equations call temperature you seem to want to call back radiation. ”
This is just silly! Joel has always been way too careful to mistake temperature (measured in K) with radiation (in W in this case). Where does he do such an odd thing? Or is this another case where being careful to use the right words was not important?

Steve
May 12, 2011 12:37 pm

mkelly says: “Sir, if you disgree that pressure causes an increase in temperature if volume is held steady that is well odd. PV= nRT discribes what is termed STP (Standard temperture and pressure).”
How are you applying pressure to your fixed volume? If I increase the pressure inside a scuba cylinder by pumping in more air, I get a proportional increase of n, not T, for my fixed volume.
Are you trying to imagine some other way to apply pressure to a fixed volume without compressing it (not lowering V) AND without adding more gas (not increasing n)? That would indeed be an odd theoretical device – the non-compressing presser that heats a fixed volume. If I was trying to increase the pressure of a fixed volume, I would work the other way around – heat it to induce a temperature increase, leading to a pressure increase.
In the real world, I do not think you would have “pressure causes an increase in temperature if volume (and moles!) is held steady”, I think you would have “temperature causes an increase in pressure if volume (and moles) is held steady.”

R Stevenson
May 12, 2011 12:38 pm

Robert Clemenzi says:
‘Using 1.0% water vapor, my program computes 282.68 W/m2 .. close to your value.
For power absorbed, my program incorrectly assumes 1 km at constant temperature and pressure. However, it also plots the distance to absorb x% of the available radiation. As long as those values are under 100 meters, I feel that assuming constant temperature and pressure causes only minor errors.
At any rate, I would like to investigate WHY the programs are giving different values to absorb the radiation.’
Robert,
For water vapour I use a partial pressure of 0.231 atm. I calculate the emissivity/absorptivity of water vapour again using Hottel’s emissivity v absolute T graphs.

Tim Folkerts
May 12, 2011 1:20 pm

This equation has shown up several times:
q = ε*σ*(Th^4 – Tc^4) Ah
Every equation applies in some specific set of circumstances (for example, d= vt only applies when v is constant). Looking at this equation, I would say it applies to radiation between two large, flat surfaces with area Ah with the same emissivity close to each other. It should work for a convex polyhedron of area Ah completely by the second object. It might well work for some other circumstances. (People are welcome to suggest other circumstances, or to suggest why I might be wrong, but that is not the point.)
The given equation most certainly does NOT work for two arbitrary objects at arbitrary distances with arbitrary orientations with different emissivities. Expecting this simplified equation to work in general is a losing proposition.
If you want an equation that applies to more general circumstances, you will need a more general equation. So the conclusion (furthermore the equation has a term for the emissivity of the object and no term for the emissivity of the surroundings or the absorptiveness of the object, both of which would be required if the surroundings were transferring energy to the object.) is more a statement about the limitation of the equation, not a statement about whether objects with different emissivities can transfer energy just one direction or both directions.
PS As I pointed out once before, a more general equation is in Wikipedia http://en.wikipedia.org/wiki/Thermal_radiation . This equation DOES include the areas of the two objects, the emissivities of the two objects, and the geometry of the two objects. This removes the previous objection.

JAE
May 12, 2011 1:30 pm

Phil informs:
“It goes on to assert that any point in the Venusian atmosphere at the same pressure as the Earth’s would be at the same temperature with appropriate adjustment for the relative power received from the sun. This is said to be the 4th root of the ratio of the powers (∜1.91), and lo and behold they match!
The trouble is that ~30% the power received at the Earth’s TOA is reflected and ~90% of the power received at the Venusian TOA is reflected, so by his argument Venus receives less power than Earth!”
Can you please share with us the source of your 90% reflectance number?

May 12, 2011 1:42 pm

Wayne,
Sorry I missed this reply of yours.  Thank you for the kind words, you may of course use my words anytime you wish, for what they are worth 🙂
I too don’t remember the concept of “back radiation” occuring before AR4.  As with many things the IPCC claims, I find the actions they attribute to it unscientific and a misjudgement of cause and effect, as with the effect of clouds.  But I don’t want to open that can of worms at this time…

Joel Shore
May 12, 2011 1:42 pm

Alleyne says:

The grey body equation for the radiation loss rate of an object to its surroundings
q = ε*σ*(Th^4 – Tc^4) Ah
must correct for the temperature of the surroundings in order to correctly reflect the absolute temperature of the object, hence the term (Th^4-Tc^4). This however in no way implies that the surroundings (which are radiating energy, not being at 0K) transfer energy to the object.

Well, yes, the 1st term is the transfer of radiant energy from the hotter object to the colder surroundings and the 2nd term is the transfer of energy from the colder surroundings to the hotter object. The heat q is the net radiative transfer and is clearly always positive when Th > Tc, meaning heat flows from the hotter object to the cooler object as the 2nd Law requires. [It is not necessary that you interpret the equation as two countervailing flows of energy in order to get the greenhouse effect, but since the actual radiation in each direction can be measured, it is worth noting that there really is a transfer of radiative energy back and forth.]
Now, how does this lead to a greenhouse effect? What you have when Th is the temperature of the earth and Tc is the temperature of the atmosphere is a heat flow away from the earth that, while always positive, has a magnitude that depends on both Th and Tc. In radiative balance, this heat flow away from the earth will balance the heat flow to the earth from the sun. Now imagine that you start at Tc = 0 and then ramp it up. In order to maintain radiative balance, you have to ramp up Th. That is the greenhouse effect.
[As Tim points out, the equation below has some simplifications in it already in regards to having a graybody object and blackbody surroundings and the surrounding completely surround the object…but for what we are using it to demonstrate, these simplifications arefine.]

JAE
May 12, 2011 1:56 pm

The -18 C “average temperature of the Earth” is the temperature where more radiation is leaving than is entering–about 5 km in the sky. As measured accurately by satellite. Because of the lapse rate (environmental lapse rate = -6.49 K/km), that means that the surface of the Earth has an average temperature that is about 33 C higher, or about 15C. We know that the lapse rate is not caused by a GHE, since it is simply a function of gravity and thermal capacity. I is caused by the relationship explained by the ideal gas law pV=RT. Therefore, it looks to me like there is no need to “explain” anything with a GHE.
Of course, other planets with an atmosphere should show the same relationships, and it appears that they do. They all show the same type of increase in temperature with a decrease in altitude, such that the “black body temperature” of the planetoid is at a level that coincides with about 100 mb pressure (Earth is somewhat different, probably due to the presence of water). Doesn’t seem to matter what the gaseous constituents are. See the figure towards the end of this article:
http://www.tech-know.eu/uploads/Greenhouse_Effect_on_the_Moon.pdf
All right, now, tigers. Tell me why all this is wrong.

Joel Shore
May 12, 2011 2:03 pm

Alleyne says:

I was using the term “heat pump” in the colloquial sense, meaning of a machine which heats a house by extracting heat from the surroundings and moving it into the house. The reason I contrasted the two types of ‘heat pump’, heat pumps and air conditioners, was to illustrate the large amount of work (energy) required to circumvent the Second Law – ie to cool an object further by moving energy from it to a warmer one or warmer surroundings.

Both a heat pump and an air conditioner use work to move heat from colder to hotter, so I don’t really see the distinction. The only substantive qualitative difference is that for the heat pump, the “hot reservoir” is your house and the “cold reservoir” is the outside and for the air conditioner, the “cold reservoir” is your house and the “hot reservoir” is the outside.

I don’t necessarily disagree at this point with the concept that GHGs trap heat, I disagree with the idea contention that they heat the earth through backradiation. They can’t, they don’t.

At some point, this comes down to terminology. It is most correct to say that because of back-radiation (in combination with the radiation from the sun), the earth is warmer than it would be if all the radiation from its surface escaped into space.

But just for the sake of argument, tell me in your theory what the temperatures of the earth and atmosphere are, the amount of energy radiated from the earth to the atmosphere and backradiated from the atmosphere to the earth. Please state all units in Kelvin if you don’t mind. I’ll run the calcs and see if the numbers prove me wrong.

This is an ill-defined question. The average flows of energy radiated and back-radiated have been measured to fairly good accuracy and are summarized in the diagram of Trenberth and Kiehl (e.g., http://www.nar.ucar.edu/2008/ESSL/catalog/cgd/images/trenberth9.jpg ). However, the atmosphere is not just at one temperature (nor is it a blackbody emitter). To compute what happens quantitatively, one must solve the equations for radiative transfer absorption-line by absorption-line through the atmosphere. This is quite an undertaking (although I think there are codes available on the web).
To get a qualitative picture, you can use simpler blackbody (or graybody) shell models like the one we discuss in Section 2.3 of our comment on G&T: http://scienceblogs.com/stoat/upload/2010/05/halpern_etal_2010.pdf

Joel Shore
May 12, 2011 2:11 pm

mkelly says:

q = ε*σ*(Th^4 – Tc^4) Ah
The above from an astute person shows clearly that if two objects of the same temperature are radiating at each other there is no (none, nada, zippo) energy exchange. No back radiation heating the other up as you claim back radiation heats the earth.

Perhaps the most fundamental thing to understand about that equation is that if Tc was zero then there would be more heat going from the hotter object to the colder object than when Tc is not zero. That, along with the fact that the surface temperature of the earth is determined by balancing the heat it receives from the sun with the heat it radiates away tells you there is a greenhouse effect. In other words: the temperature Th necessary to radiate away the 240 W/m^2 of radiant energy that the earth receives from the sun depends on the temperature Tc.
It is sort of amazing how such a simple concept seems to illusive for some people!

Tim Folkerts
May 12, 2011 2:14 pm

JAE asks:
“Can you please share with us the source of your 90% reflectance number?”
Look for “bond albedo” of Venus, for example at
http://nssdc.gsfc.nasa.gov/planetary/factsheet/venusfact.html
http://en.wikipedia.org/wiki/Bond_albedo
Google is your friend 🙂

Joel Shore
May 12, 2011 2:15 pm

JAE says:

The -18 C “average temperature of the Earth” is the temperature where more radiation is leaving than is entering–about 5 km in the sky. As measured accurately by satellite. Because of the lapse rate (environmental lapse rate = -6.49 K/km), that means that the surface of the Earth has an average temperature that is about 33 C higher, or about 15C. We know that the lapse rate is not caused by a GHE, since it is simply a function of gravity and thermal capacity. I is caused by the relationship explained by the ideal gas law pV=RT. Therefore, it looks to me like there is no need to “explain” anything with a GHE.

Are you reading anything we write here?!?! There is indeed something to explain: Why, from space, does the earth look like it has the temperature that is actually 5 km up in the sky? It certainly wouldn’t if all the radiant energy from the surface escaped to space. Then it would look like it had the temperature of the earth’s surface!
For heaven’s sake, how many times do we have to say this?

Joel Shore
May 12, 2011 2:24 pm

JAE says:

http://www.tech-know.eu/uploads/Greenhouse_Effect_on_the_Moon.pdf
All right, now, tigers. Tell me why all this is wrong.

That paper has been debunked elsewhere. Basically, Hertzberg is clueless about how to calculate what the surface temperature has to be in the absence of the greenhouse effect and relate it to measured temperatures. What the blackbody temperature that you compute from the solar absorption actually constrains is the average of T^4 on the surface. It does not constrain the instantaneous temperature at a certain location, nor does it constrain the average temperature. (For a planet like earth where the temperature variations are not that large on an absolute scale, the difference you get from worrying about the average of T^4 vs the average of T is not that significant; for the moon, it is more significant).

Tim Folkerts
May 12, 2011 2:45 pm

JAE says:
May 12, 2011 at 1:56 pm
The -18 C “average temperature of the Earth” is the temperature where more radiation is leaving than is entering–about 5 km in the sky. … Therefore, it looks to me like there is no need to “explain” anything with a GHE.”
But the radiation that is leaving is coming in part from the GHGs, and also from clouds and also from the ground. The net radiation leaving is from all three added together. If there were no GHGs, they would not block the “warm” IR from the surface and they would not emit “cool” IR from the upper atmosphere. This would create a greater energy flow to space. To balance, the surface would have to cool off (or the cloud cover could change, or perhaps a few other things). You need to know about the GHE to understand this.
Or put another way, why 5 km up? What physics leads to this? If the atmosphere was transparent to IR, all the IR would come from the surface, and the surface would be -18C. With a little GHG, the effective altitude might be 1 km — then the surface would -18 C + 6.5 C. With a lot, it might be 10 km and the surface would be (-18 C + 6.5C * 10) = 47.
You NEED the GHGs & knowledge of the GHE to know the effective height. (Or equivalently, if you measure the effective height as 5 km , that tells you something about the actual GHE).
The GHE sets the “effective altitude” where IR is emitted. The lapse rate sets how much warmer the surface is compared to this “effective altitude”. Both are needed.

JAE
May 12, 2011 3:09 pm

Sorry, Joel, but no entiendo. I’m reluctant to just accepting your words about “debunking,” “clueless,” etc. This talk reminds me too much of RC, which I can no longer bear to read. Sorry. Less armwaving, more specific info., and a link or two would be appreciated.
But let’s go step-by-step. Earth first! What is wrong about my statements regarding this planet? Is it not true that the equilibrium radiation in/out is consistent with a temperature of about 255K, which is the average temperature at about 5 km elevation?

Tim Folkerts
May 12, 2011 3:43 pm

“That paper has been debunked elsewhere. ”
I think we need a “Snopes for Science”. Instead of looking into urban legends, it could look into scientific claims made on the internet. Then people could go there for info, rather than rehashing a topic every time it comes up in a discussion like this. 🙂
[Reply: WUWT is the ‘Snopes for Science’.☺ ~dbs, mod.]

May 12, 2011 3:44 pm

Phil. said on Visualizing the “Greenhouse Effect” – Light and Heat
May 11, 2011 at 9:12 pm
“The equation cited does not, in my opinion, imply backradiation. If it did there would be terms for the emissivity of both objects as well as the surface area of both objects. There aren’t.
There certainly are, the energy balance is done at the surface of ‘h’, and the area is the same for both outgoing and incoming radiation, ‘Ac’, since ‘h’ is immersed in ‘c’.
Strictly the equation should be:
q =(ε σ Th^4 – a σ Tc^4) Ac
however following Kirchoff’s Law a=ε
so:
q = ε σ (Th^4 – Tc^4) Ac”
Sorry Phil, you are correct, I should have been more accurate.  I should have said: the equation does not imply or support the claim that back radiation transfers energy to the hotter object.  All it says is that the energy transfer is equal to the difference between the radiative energy transfer per unit of time of the hotter object less the radiative energy transfer per unit of time of the cooler object.
As I understand it Kirchoff’s Law states αλ = ελ, but only at thermal equilibrium and does not require that the absorbtivity and emissivity of the surroundings be the same as that of the emitting object, otherwise what you are suggesting is that everything has the same absortivity and emissivity and αλ = ελ = a constant.

Dave Springer
May 12, 2011 4:05 pm

Tim Folkerts says:
May 12, 2011 at 2:45 pm
“Or put another way, why 5 km up? What physics leads to this?”
The same physics that stops sunlight from penetrating the ocean more than 300 feet deep. It’s called optical depth.

JAE
May 12, 2011 4:06 pm

Tim:
“With a little GHG, the effective altitude might be 1 km — then the surface would -18 C + 6.5 C. With a lot, it might be 10 km and the surface would be (-18 C + 6.5C * 10) = 47.”
Well, the evidence does not appear to be supporting that concept. We have more GHGs now, but no “hot spot,” like your theory suggests.
And is the level higher over Samoa (high levels of GHGs) than over the Sahara (very low levels of GHGs)?
Was the average temperature 47 when OCO levels were at 2,000 ppm?

Dave Springer
May 12, 2011 4:16 pm

Joel Shore says:
May 12, 2011 at 2:15 pm
“Why, from space, does the earth look like it has the temperature that is actually 5 km up in the sky? It certainly wouldn’t if all the radiant energy from the surface escaped to space. Then it would look like it had the temperature of the earth’s surface!”
Actually there’s a fair size IR “window” where it does look the temperature at the surface. More or less water vapor can open/close other windows.
Back radiation however is experimentally proven by pointing an IR spectroscope straight up in the air at night. If there were no back radiation it would only “see” the 3K cosmic background radiation. Instead it sees lots of energy at much higher temperatures up to and including the temperature of the dirt or water beneath it. The atmosphere glows (in all directions including downward!) in the infrared primarily around the emission frequencies of water vapor and carbon dioxide.

Dave Springer
May 12, 2011 4:34 pm

FYI: an air conditioner is a heat pump. If you put one in your window to cool the house in the summer you can turn it around the other way to heat the house in the winter. More sophisticated models reverse themselves through plumbing so you don’t have to physically turn them around.

Dave Springer
May 12, 2011 4:40 pm

mkelly says:
May 12, 2011 at 12:00 pm

Sorry I totally disagree. What a thing is named is important.

A rose by any other name smells as sweet. ~Wm. Shakespeare
Your lack of education evidently isn’t confined to science and extends into classic literature as well.

Dave Springer
May 12, 2011 4:49 pm

The atmosphere gets colder as you move away from the surface because the surface is the source of the heat***. It’s works the same way as wearing layers of clothing in the winter. The layers are warmest nearest your body and get coldest nearest the outside air.
***This doesn’t apply once you reach the upper atmosphere where high energy radiation (which doesn’t penetrate the atmosphere very far) heats it up to thousands of degrees, ionizing it, and even causing weird stuff like the northern and southern lights. The international space station is orbiting about halfway through the upper atmosphere. The reason is doesn’t burn up is because the atmosphere is so tenuous at that height that it can’t transfer enough heat to the solid shell of the space station to have any effect on it. Nonetheless it is quite hot.

Dave Springer
May 12, 2011 4:51 pm

The sun heats the ocean. The ocean heats the atmosphere.
Write that down.

Dave Springer
May 12, 2011 5:03 pm

The concentration of CO2 being very small compared to other gases is meaningless.
It only takes a similar small percentage of pigment to turn white paint into dark paint. That tiny difference in pigment makes a huge difference in how hot the hood of your car gets on a clear day. CO2 is like a pigment – a little bit goes a long ways.

sky
May 12, 2011 5:25 pm

Joel Shore says:
May 11, 2011 at 5:45 pm
It’s a small wonder that if you don’t understand the difference between PRODUCING energy, instead of merely spatially CONCENTRATING/STORING it, you’ll remain clueless about what I’m saying regarding thermal energy fluxes through the system.

Joel Shore
May 12, 2011 6:26 pm

JAE says:

But let’s go step-by-step. Earth first! What is wrong about my statements regarding this planet? Is it not true that the equilibrium radiation in/out is consistent with a temperature of about 255K, which is the average temperature at about 5 km elevation?

Which part of the 25 times that Tim and I have explained this to you are you not understanding?!?

Well, the evidence does not appear to be supporting that concept. We have more GHGs now, but no “hot spot,” like your theory suggests.

You are just spouting meaningless nonsense talking-points that you pick up from garbage you read on the internet! The hotspot has to do with the moist adiabatic lapse rate; it says nothing to do about the mechanism causing the warming.
Look, if you want to remain ignorant, then please just tell us and we’ll stop wasting our time (at least I will). In fact, I encourage you to go wherever you can and advertise that you are an AGW skeptic and believe the nonsense that you are defending here. You will just be confirming many intelligent people’s suspicions that AGW skeptics are a bunch of flat-earthers.
I have better things to do with my time than waste it trying to teach others who don’t want to learn. David M Hoffer has already pretty much given up in hopeless disgust … and the poor fellow has more incentive than me to persist in that he has strong desire for the AGW skeptics not to look ignorant to the larger world. I actually feel quite a bit of pity for him and Ira and Roy Spencer who actually want to remain credible.

jae
May 12, 2011 6:47 pm

David Springer:
“Back radiation however is experimentally proven by pointing an IR spectroscope straight up in the air at night. If there were no back radiation it would only “see” the 3K cosmic background radiation. Instead it sees lots of energy at much higher temperatures up to and including the temperature of the dirt or water beneath it. The atmosphere glows (in all directions including downward!) in the infrared primarily around the emission frequencies of water vapor and carbon dioxide.”
??. I think I know what you mean, but you sure didn’t say it. It looks up and “sees backradiation.” OK, sure. But then you say “Instead it sees lots of energy at much higher temperatures up to and including the temperature of the dirt or water beneath it.” I guess you mean it sees radiation that is consistent with the temperature of the ….dirt or water. That part is nonsense, unless there is an inversion (which is why I think the Kiehl-Trenberth cartoon is also nonsense). The lapse rate dictates that the radiation “from the sky” will indicate a colder temperature than the dirt or water.

Tim Folkerts
May 12, 2011 6:55 pm

Dave Springer says:
May 12, 2011 at 4:05 pm
>>Tim Folkerts says:
>>“Or put another way, why 5 km up? What physics leads to this?”
>Dave Springer says:
>The same physics that stops sunlight from penetrating the ocean more than
>300 feet deep. It’s called optical depth.
EXACTLY! That optical depth is a result of the GHGs absorbing/emitting IR. This is exactly why GHGs and the GHE MUST be included!

Joel Shore
May 12, 2011 7:11 pm

Dave Springer says:

Actually there’s a fair size IR “window” where it does look the temperature at the surface. More or less water vapor can open/close other windows.

Yeah…I meant in a total-integrated sense. But…you make a good point in noting that the earth doesn’t really look like some sort of blackbody object emitting at a certain temperature, corresponding to a certain level of the atmosphere. Rather, the emission varies strongly with wavelength due to the optical thickness of the atmosphere at these various wavelengths. The 5km “average (or effective) radiating level” is just that…an average that sweeps under the rug a lot of interesting spectral dependence.
And, by the way, I should have also mentioned you in that list in my last post that included Ira, David M Hoffer, and Roy Spencer.

jae
May 12, 2011 7:59 pm

“EXACTLY! That optical depth is a result of the GHGs absorbing/emitting IR. This is exactly why GHGs and the GHE MUST be included!”
Eureka, eh?
Possible logical error alert, I say.
You are making the usual EXTREMELY big jump in saying that because there is absorption/emission by GHGs that there is a radiative GHE. That link HAS NOT BEEN ESTABLISHED EMPIRICALLY, JUST ASSUMED. You still don’t seem to get the FACT that there are all kinds of other things going on in the atmosphere. Convection might (and probably does) wipe out any GHE effects, just like with the “real greenhouse” effects.
Science is NEVER settled until there is empirical evidence. And the empirical evidence, so fare, is against the link, as I pointed out many times above. Nice theory, though.

richard verney
May 12, 2011 8:39 pm

Ira
Your articles always generate much interest and comment, and this one is no exception.
I am surprised that so many people hold such bullish views given the sparcity of empirical data and the complexity of the system and lack of understanding how it all fits together and works in practice. I remain unsure about many issues but I am reasonably confident of the following:
1. That it is inappropriate to treat the Earth as if it were a blackbody (a rotating spherical object with oceans that act as heat sinks, swirling and variably heated atmosphere with variable latent heat characteristics, imprecisely known and changing emissivity etc is nothing like a blackbody). This fundamental misconception inevitably leads to a misconception as to what should be the ‘theoretical notional’ temperature of the Earth as if it had no atmosphere. Wrong model wrong answer.
2. We have imprecise data with respect to albedo (Trenberth in his paper has albedo measurements varying between 27.1 and 35.8% and others have it is high as 40% and the albedo is constantly changing) and imprecise data with respect to the amount of solar energy absorbed by the atmosphere and imprecise data as to the amount of solar energy reflected by the atmosphere. These errors mean that we cannot accurately calculate the ‘theoretical notional’ temperature of the Earth as if it were a blackbody. These (potential) errors are not insignificant and afre such that they could lead to perhaps a 10 to 20K error in the assessment of the ‘theoretical notional’ temperature of the Earth (as if it were a blackbody).
3. The water cycle alone, makes it inappropriate to assess ‘theoretical notional’ atmospheric temperature on the basis that the Earth is a blackbody.
4. We do not have an accurate figure for the average temperature of the Earth. Indeed, this could be out by many degrees (say somewhere between 1 to 3K).
5. That irrespective of the presence of GHGs, commonsense suggests that the atmosphere at or near the surface would be warmer than the ‘theoretical notional’ temperature of the Earth (assessed as if it were a blackbody). According to Trenberth some 78 W/m^2 of the incoming solar energy is absorbed by the atmosphere. Since the warmists case is that CHGs are transparent to incoming solar radiation, CHGs cannot account for this absorption. The absorption of this energy must have some heating effect on the atmosphere since energy is fungible and this incoming energy must have gone somewhere also not forgetting the latent heat content. The rotation of the Earth and the movement of the atmosphere (winds, convection etc) must inevitably lead to heating of the atmosphere (after all heat is a by product of work and considerable work is involved in moving the atmosphere). Most importantly, the oceans (which are heated by the sun during the day) are constantly heating the atmosphere day and night.
6. If GHGs were responsible for all the warming of the atmosphere there would be some correlation between the concentration of GHGs and temperature both laterally and vertically but there does not appear to be such correlation.
7. Adiabatic lapse/compression far better explains the temperature profiles of the atmosphere (although this is not a complete explanation in itself). You have likened it to a tyre and suggested that the temperature is radiated away. However, that analogy is incomplete. If you pump up a tyre and then drive on it (steering, braking, going over bumps etc), the side wall flexes slightly only slightly increasing the pressure as the tyre goes in and out of shape and this small movement is sufficient to maintain the temperature as long as the car is being driven (you see this in motorsport). The circulation of the atmosphere (hot air rising and cooling etc) is the equivalent and helps sustain the temperature of the atmosphere. The atmosphere is in constant flux and the work done inevitably generates heat.
8. That Trenberth’s energy budget is wrong. It defies commonsence that DWLIR exceeds incoming solar energy given that the DWLIR can have been created only by the incoming solar energy (and energy radiated/released by the core which latter energy is said to be miniscule such that this can be ignored). The use of ‘average’ figures disguises and distorts what is truly going on. Just imagine for a moment what would be the position if the Earth’s speed of rotation was different. This would have little effect on the figures used in the Trenberth diagram but conditions on Earth would be radically different.
9. That if DWLIR of the magnitude as indicated by Trenberth really existed, we would be able to extract energy from this and/or there would be considerable research into exploiting this natural green resource. After all, this DWLIR would have potential exceeding that of solar and wind.
10. At most, the effect of GHGs is merely to delay heat loss being radiated. However, it is likely that at night (when there is no incoming solar energy) or at other times when atmospheric conditions are such that there is a temperature inversion, any LWIR that has been delayed has sufficient opportunity to radiate to space there by meaning that no excess heat is ‘trapped’ (ie., the heat in the atmosphere does not build up). Ira, see the article posted by E.M.Smith on Temperature Inversion titled Grostbite Falls January 23 2011 which gives a practical example of a ‘safety valve’ which serves to release any ‘excess’ temperature build up.
I doubt that many people challenge the assertion that GHGs can absorb LWIR and re-emit some of this as potentially DLWIR (but bear in mind thermalisation which no doubt plays an important part). I doubt that many people would challenge the assertion that the warmer the atmosphere, the slower the heat loss from the surface of the Earth. The debate is not how gases behave in isolation in laboratory conditions but rather how the atmospheric system behaves as a whole with all its complexity and nuances, and it is because of this that many are skeptical as to whether in reality there is any significant ‘greenhouse’ effect.
The upshot of the above is that I do not dismiss out of hand the contention that GHGs may contribute something to the warming of the atmosphere but I am very skeptical that the Earth is 33K warmer because of the presence of GHGs in the atmosphere. I consider that such a contention is far from proven both on figures and on principles involved.

Steve
May 12, 2011 10:13 pm

richard verney says: “According to Trenberth some 78 W/m^2 of the incoming solar energy is absorbed by the atmosphere. Since the warmists case is that CHGs are transparent to incoming solar radiation, CHGs cannot account for this absorption. ”
The GHGs are transparent to incoming visible light, not all solar radiation. I think you have read an incorrect account of the “warmists” view.

Steve
May 13, 2011 12:01 am

jae says: “You are making the usual EXTREMELY big jump in saying that because there is absorption/emission by GHGs that there is a radiative GHE.”
“You still don’t seem to get the FACT that there are all kinds of other things going on in the atmosphere. Convection might (and probably does) wipe out any GHE…”
How is convection going to “wipe out” a molecule’s ability to absorb/emit certain wavelengths of radiation? Are you saying that once energy is absorbed, a volume of gas is more likely to rise and expend all of that energy as work (adiabatic cooling), instead of emitting that energy as IR radiation? So the only GHE is an increased occurrence of rising warm air?

Bryan
May 13, 2011 12:35 am

Those advocates of the IPCC “greenhouse theory” should get real.
If the radiative effects of H2O and CO2 are supposed to “heat” the atmosphere to 33K higher than “it would otherwise be” then they need to show significant radiative heating in a volume of the atmosphere.
This they cannot do.
Instead they distort equations and say look “there’s the proof”.
If we went to the installer of our home central heating system with a complaint that it wasn’t working would we be satisfied with a couple of equations as an excuse.
Call the bluff of IPCC “greenhouse theory” advocates!
Ask for proof of radiative heating in a volume of the atmosphere.
They will be stumped!
The fact that the temperature profile of the troposphere can be derived from thermodynamics without reference to radiation sums it up.
A good place for the IPCC advocates to start would be the radiative heating of a space the size of a real greenhouse.
Lets get some real figures for say 30 cubic metres of air at STP on a KT average day.

Bryan
May 13, 2011 4:18 am

Tim Folkerts asks
“The readily available spectra graphs show that this is wrong.”
Could you provide a link to specific spectra that you are referring to? What is wrong with them?
Tim, I have a spreadsheet of the Planck function similar to Ira’s
So I can call up a blackbody spectrum for any temperature.
This gives a surface up value.
I then compared the atmosphere spectrum profile from Petty(2004) as given in page 127
Physical Meteorology on line is Rodrigo Caballero’s Lecture Notes
http://maths.ucd.ie/met/msc/PhysMet/PhysMetLectNotes.pdf
With the surface continuous spectrum.
My calculation is that all the radiation shown in this chart is reduced by 25% compared to the surface up radiation.
So the maximum possible back radiation would be 75% of the surface radiation!
K&T on the other hand have the reduction of only 8%.
Thus they claim a figure of 92% of radiation is surface bound.
I used the older K&T diagram .
390 up (subtract 40 from ‘window’ as everyone agrees this radiation doesn’t come back).
This leaves 350W/m2 up and 324 W/m2 “backradiation”.
(350-324)/350 = 7.4%
Caveat’s;
1. A fairy rough and ready calculation
2. The Petty spectrum might not be representative.
However even with these reservations a reasonable conclusion can be drawn.

Joel Shore
May 13, 2011 4:41 am

jae says:

Convection might (and probably does) wipe out any GHE effects, just like with the “real greenhouse” effects.

Convection does not occur between the earth and space…and hence cannot get you around having to satisfy conservation of energy. Besides which, the convective-radiative models include convection. You are just spouting more nonsense.

Dave Springer
May 13, 2011 4:52 am

Bryan says:
May 13, 2011 at 12:35 am

If the radiative effects of H2O and CO2 are supposed to “heat” the atmosphere to 33K higher than “it would otherwise be” then they need to show significant radiative heating in a volume of the atmosphere.
This they cannot do.

There are some things that are easily provable yet there seems to be no dearth of skeptic who don’t accept what is proven.
Point an infrared thermometer up at the clear sky at night. It shows a temperature much higher than the empty cosmos. That’s because the atmosphere glows in the far infrared. Point an infrared spectrometer up at the clear sky at night. It breaks the glow down into component frequencies and those are primarily the absorption/emission bands of water vapor and CO2. In the stronger absorption bands of water vapor when the absolute humidity is high the spectroscope will show temperatures in those bands closely approaching the ground temperature. The instrument in that case is seeing the temperature of the water vapor very close the ground.
Emergency blankets commonly known as “space blankets” are made of exceedingly thin highly reflective mylar. A blanket of this material big enough to cover your body fits folded into a package the size of a teabag. It is made of a material with a mirror finish because that material reflects infrared radiation from your body back at your body and thus keeps you warmer than the same blanket made of non-reflective material.
These things are indisputable proof of so-called “back radiation” and the FACT that back radiation slows the loss of heat from the body which it wraps whether that’s a human body on a cold night or the body of the planet.
The person denying this just looks dumber than a fifth grader and makes it impossible to move on to the bits of the global warming story that aren’t well established facts like whether clouds have a net warming effect due to them being an effective insulator at night or a net cooling effect due them shading the ground underneath during the day. Or exactly how much daytime heating of the ocean gets lifted away from the surface by evaporation and convection. Or exactly how much heating of the equatorial ocean gets transported to the poles by ocean currents. Or exactly how much the albedo of the planet changes from year to year due to variation in type and amount of cloud cover and wind driven waves which break up the ocean surface and make it less able to absorb sunlight while at the same time increasing the evaporation rate which combine to make it much cooler than it would be if the winds were calm.
There are a million details which are poorly characterized that are highly relevant to the anthropogenic global warming narrative. The basic physics of greenhouse gases are simply not one of those things that are not well-enough understood and if you don’t understand how greenhouse gases work you can’t possibly move on to any reasonable debate about other phenomena which can and do (IMO) largely negate the effects of increasing greenhouse gases and leave us in a situation where the modest increase in carbon dioxide has vast beneficial effect by warming the planet at high latitudes where warming is welcome, not warming it at low latitudes where it is already warm enough, increasing the growth rate of green plants, and decreasing the water needs of green plants at the same time.

richard verney
May 13, 2011 5:26 am

Steve says:
May 12, 2011 at 10:13 pm
richard verney says: “According to Trenberth some 78 W/m^2 of the incoming solar energy is absorbed by the atmosphere. Since the warmists case is that CHGs are transparent to incoming solar radiation, CHGs cannot account for this absorption. ”
The GHGs are transparent to incoming visible light, not all solar radiation. I think you have read an incorrect account of the “warmists” view.
///////////////////////////////////////////
Steve. Thanks your comment. I stand corrected on that point ( I was posting at about 4:30 am and was not thinking sufficiently clearly). I do need to check whether all of the 78 W/m^2 is said to be absorbed in the absorption wavelengths of GHGs or whether some of this is component is absorbed in the absorption wavelenghts of other gases and/or by atmospheric aerosols.

Dave Springer
May 13, 2011 5:43 am

Someone asked for actual spectrographs.
Looking up from the ground:
http://www.sundogpublishing.com/AtmosRad/Excerpts/AtmosRad212.pdf
Looking down from above:
http://www.sundogpublishing.com/AtmosRad/Excerpts/AtmosRad125.pdf

Tim Folkerts
May 13, 2011 5:52 am

richard verney says: May 12, 2011 at 8:39 pm
8. That Trenberth’s energy budget is wrong. It defies commonsence …”
Richard, you make some good points, but this is a pretty bold statement: science is wrong because it defies YOUR common sense!
There are many ideas in science that defy the common sense of many people. That just means people need to look at it more carefully and THEN decide what is correct. I think the diagram is correct (at least within reasonable uncertainties). Many other commenters also think it is correct. Many scientist in many fields think it is correct.
So … what specific numbers in the diagram do you think are wrong? How would you change them and still satisfy conservation of energy and still agree with observed temperatures?

Dave Springer
May 13, 2011 6:05 am

richard verney says:
May 12, 2011 at 8:39 pm
“The upshot of the above is that I do not dismiss out of hand the contention that GHGs may contribute something to the warming of the atmosphere but I am very skeptical that the Earth is 33K warmer because of the presence of GHGs in the atmosphere. I consider that such a contention is far from proven both on figures and on principles involved.”
33K appears to be accurate +-10% going by average measured temperature of the surface of the moon and the surface of the earth after taking into account difference in albedo between the two where the inaccuracy is almost soley due to lack of accuracy in determining the average albedo of the earth.
However, I question whether the GHE is soley in the atmosphere. The ocean has the same properties of being transparent to visible light and opaque to infrared light that greenhouse gases exhibit. The sun warms the ocean to a depth of about 100 meters during the day but it cannot radiate infrared from a depth greater than 1 micrometer. So the energy absorbed from the sun at depth must find its way to the surface by some means other than radiative which of course leaves only convection and conduction. The question is whether convective and conductive cooling are more or less efficient than radiative heating and under what conditions. I suspect the net effect under most circumstances is that a substantial amount of greenhouse warming is contributed by the ocean alone. Of course if the ocean surface is frozen then all bets are off and any GHG warming in that case comes from atmospheric fluids not oceanic fluids. CO2 I believe is most important in helping to raise average temperature from below freezing to above freezing then water in liquid and gaseous phase becomes the major player. CO2 is the “kindling” which ignites the water cycle.

Tim Folkerts
May 13, 2011 6:33 am

Bryan says:
“My calculation is that all the radiation shown in this chart is reduced by 25% compared to the surface up radiation.
So the maximum possible back radiation would be 75% of the surface radiation!”
The chart in question is for “Whole-atmosphere clear-sky absorptivity”. For a cloudy sky, the clouds would absorb close to 100% of the radiation. The numbers I have seen put cloud cover at ~ 0.7 (therefore clear sky @ ~ 0.3). The back radiation would then be limited to approximately.
(100% * 0.7) + (75% * 0.3) = 95% of the upward radiation.
This limit assumes the radiation comes back from the same temperature as the surface, which is clearly incorrect. The fact that the clouds and atmosphere are on average cooler than the surface will decrease this 95% ratio significantly. If the surface emits at an average of 270 K and the atmosphere & clouds emit at an average of 260 K, the downward radiation would be reduced by (260^4 / 270^4) = 86%. So the downward radiation would be 0.86 * 0.95 = 81% of the upward radiation.
The observed ratio of downward to upward is 324/390 = 83%. This seems within reasonable agreement with the rough estimates.
PS. I disagree with your thinking when you say “subtract 40 from ‘window’ as everyone agrees this radiation doesn’t come back”.
The EM radiation from the surface is determined by the surface temperature and surface emissivity. Whether that energy is absorbed or reflected or transmitted by the atmosphere doesn’t change how much IR the ground emits. The ground emits ~ 390 W/m^2 of thermal IR at its current average temperature, so that is the number that should be used.

May 13, 2011 6:43 am

Dave Springer says:
May 12, 2011 at 4:40 pm
mkelly says:
May 12, 2011 at 12:00 pm
Sorry I totally disagree. What a thing is named is important.
A rose by any other name smells as sweet. ~Wm. Shakespeare
Your lack of education evidently isn’t confined to science and extends into classic literature as well.
Mr. Springer, you cannot call temperture back radiation. In science words have meanings. Velocity is not distance although it is intertwined with distance. I subscribe to the Mark Twain version. see below
“If we call a dogs tail a leg how many legs does a dog have? Four just because you call a tail a leg does not make it one.” Mark Twain.

Tim Folkerts
May 13, 2011 6:47 am

richard verney says: May 12, 2011 at 8:39 pm
“9. That if DWLIR of the magnitude as indicated by Trenberth really existed, we would be able to extract energy from this and/or there would be considerable research into exploiting this natural green resource. After all, this DWLIR would have potential exceeding that of solar and wind.”
Once again, your common sense is incorrect. The diffuse IR radiation from the atmosphere cannot be concentrated in the way you suggest. No matter how you concentrate the energy, this IR by itself cannot heat something above the temperature of the material emitting the light.
In the same way, you cannot heat something above 5700 K no matter how you arrange lenses & mirrors to focus sunlight. If you had a furnace at 5700 K, you could not use sunlight to warm it any further (although you could reduce the amount of other energy required to heat the oven).

Bryan
May 13, 2011 8:06 am

Dave Springer you will need to read the posts before you launch into a rant.
I said …… show significant radiative heating in a volume of the atmosphere.
This they cannot do.
Did you notice the word SIGNIFICANT ?
Ive made it larger so that you can read it.
You then go on to give a whole pile of irrelevant tripe.
Of course CO2 and H2O radiate in the Infra Red but does this amount to a 33K increase in the atmosphere of the planet
You go on “These things are indisputable proof of so-called “back radiation” ”
Who are you arguing with here?
If you had read any of my previous posts you would know that I have always maintained that colder surfaces can radiate to warmer surfaces but they cannot HEAT them.
I will send a second post with experimental evidence that the radiative heating of a volume of air the size of a greenhouse is so small that it is almost negligable

May 13, 2011 8:17 am

Roy W. Spencer, Ph. D. says in his own blog on May 11, 2011 at 9:15 AM as a response to comments to his posting “UAH Temperature Update for April, 2011: +0.12 deg. C”:
“EVERYONE:
It turns out the stratosphere is responsible for the difference between the Discover ch 5 data and our official LT data. There was an abrupt switch from a westerly QBO to easterly last month, with strong lower stratospheric cooling in the tropics, which then influenced ch. 5.
ALSO:
You are making too much out of month-to-month temperature variations. We have computed before that these large changes can be caused by less than 1% fluctuations in the convective heat transport from ocean to atmosphere.
Stop thinking in terms of only radiation causing temperature changes.”
============================
I hope Dr. Spencer does not mind me sharing his comment with you here on WUWT but I thought the line; “Stop thinking in terms of only radiation causing temperature changes.” is a significant one when it comes from him. I have known for a long time that Dr. Spencer has always considered factors other than radiation. But I have also had the impression; “That not a lot of people know that”
I am also one who happen to think too many people – by far, are making too much of radiation. – The so-called “Back radiation” is also “just” radiation and cannot possibly just radiate back to whence it came.

Bryan
May 13, 2011 8:35 am

Tim Folkerts
Why is it that IPCC advocates have a tendency to “bend” the numbers?
We were talking about the KT diagram
You say……. If the surface emits at an average of 270 K
Tim the actual surface temperature for this diagram is +15C = 288K
You say …… atmosphere & clouds emit at an average of 260 K
A more realistic figure particularly if you are stressing the clouds would be 240K.
Remember also that this post was in answer to your initial contention that all downwelling LW radiation is really back radiation.
Hence my calculation ignored the window radiation which nobody thinks “comes back” or do they?
I don’t think that that is a tenable position

Bryan
May 13, 2011 8:53 am

Dave Springer as promised two experiments that show the radiative heating of the atmosphere is very small.
The first experiment was the classic experiment by R W Wood.
He was was probably the best experimental physicist that America ever produced.
Wood nailed two points in this experiment.
1. Greenhouses(glasshouses) work by stopping convection.
2. The radiative effects of CO2 are very weak at atmospheric temperatures.
G&T did an experiment to confirm the conclusions of Wood and also give an account of his experiment.
“Falsification Of the atmospheric CO2 greenhouse effects within the frame Of Physics” by Gerhard Gerlich and Ralf D. Tscheuschner; International Journal of Modern Physics B, Vol. 23, No. 3 (2009) pages 275-364.
http://arxiv.org/PS_cache/arxiv/pdf/0707/0707.1161v4.pdf
The second experiment is an interesting paper especially as it comes from a source with no “spin” on the AGW debate.
The way I read the paper is it gives massive support for the conclusions of the famous Woods experiment.
Basically the project was to find if it made any sense to add Infra Red absorbers to polyethylene plastic for use in agricultural plastic greenhouses.
Polyethylene is IR transparent like the Rocksalt used in Woods Experiment.
The addition of IR absorbers to the plastic made it equivalent to “glass”
The results of the study show that( Page2 )
…”IR blocking films may occasionally raise night temperatures” (by less than 1.5C) “the trend does not seem to be consistent over time”
http://www.hort.cornell.edu/hightunnel/about/research/general/penn_state_plastic_study.pdf
Now if you think that the “greenhouse theory” has any evidence that the radiative effects of CO2 and H2O can significantly heat a volume of air at STP please let us know.

May 13, 2011 9:16 am

Joel Shore says:
“Convection does not occur between the earth and space.”
As a matter of fact, convection does occur between the earth and space. Warmer air both rises and radiates. That area is called the atmosphere, and convection provides a mechanism for dumping heat into space.

JAE
May 13, 2011 9:26 am

Tim Folkerts:
“So … what specific numbers in the diagram do you think are wrong? How would you change them and still satisfy conservation of energy and still agree with observed temperatures?”
It doesn’t make sense to me because it suggests that the only way the surface temperature can average 288 K (and therefore emit 390 wm-2) is through backradiation from a colder sky. It is hard to believe that so many scientists think that is correct.
But then, that seems to be the only way that the magic 33 C increase from “black body earth” can really be explained by radiatiation. How funny!

JAE
May 13, 2011 9:32 am

Smokey:
““Convection does not occur between the earth and space.”
As a matter of fact, convection does occur between the earth and space. Warmer air both rises and radiates. That area is called the atmosphere, and convection provides a mechanism for dumping heat into space.”
I think you are actually saying the same thing as Joel is: the only way that energy can actually move from the atmosphere to outer space is through radiation.
But then Joel said in reply to my:
“Convection might (and probably does) wipe out any GHE effects, just like with the “real greenhouse” effects. ”
that
“Convection does not occur between the earth and space…and hence cannot get you around having to satisfy conservation of energy. Besides which, the convective-radiative models include convection. You are just spouting more nonsense.”
Which is a total (willful?) misunderstanding of my comment.
Yeah, the “convective-radiative” models include convection, but my position is that they don’t treat it properly.

Tim Folkerts
May 13, 2011 9:39 am

Bryan says: May 13, 2011 at 8:35 am
>>You say……. If the surface emits at an average of 270 K
OK, that number is low, but that is not the main point. I will grant that 285 K is a better number
>>You say …… atmosphere & clouds emit at an average of 260 K
>A more realistic figure particularly if you are stressing the clouds would be 240K.
I don’t agree. A large faction of clouds are within 2000 m of the surface. Based on lapse rates, these must be no more than 20 K cooler than the surface. Many would be within 10 K of the surface temperature. Very few clouds would have their bases high enough to be 285K – 240K = 45 K colder than the surface that you suggest as more realistic.
So we can be more accurate. I am not a meteorologist, but …
* the surface average would be ~ 285K (using pretty much anyone’s numbers)
* the observed environmental lapse rate is 6.5 K/km, which we will use a a rough estimate.
* 0 m and 3000 m will be 6.5 K – 20 K cooler than the surface.
* many common types of clouds (stratus, stratocumulus, cumulus, cumulonimbus, fog) have their base between 0 – 3000 m above the ground.
* therefore, the bases of many clouds are 0-20 K cooler than the surface.
* therefore estimating the temperature drop between ground and clouds as 10K is not too far off.
I’d be happy to see either more accurate estimates or experimental data for the typical temperatures of cloud bases. There are definitely clouds above 2000 m (6500 ft) and a more accurate analysis would need to include a better average of the cloud heights.
Furthermore, estimates I have seen for CO2 & H2O vapor suggest most of the absorption (and hence emission) comes in the first few 100 m where the temperature is within a few K of the surface temperature. So for the 75% you were looking at, we can use about the same temperature for the back radiation as for the forward radiation for CO2 & H2O. (And the satellite data at the very beginning of this post confirms that)
Whether the the numbers are (270 K surface and 260 K clouds & GHG) or (285 K surface and 275 K clouds & GHG), the basic result is the same.
Bryan then says: “Why is it that IPCC advocates have a tendency to “bend” the numbers?”
1) I don’t consider myself an “IPCC advocate” — just a scientist interested in the subject of climate change and interested in seeing the basic science presented correctly.
2) I admit I presented one inaccurate number (270 K vs 285) — it must have been too late when I wrote that. However, that doesn’t actually make a fundamental difference in the final results because it is primarily the temperature DIFFERENCE that matters. I think the estimate of 10 K temperature difference is closer than your estimate of 45 K.
3) You missed the effect of clouds entirely! That would seem to be much more egregious than “bending” one or two numbers.

May 13, 2011 9:44 am

Joel Shore says:
May 12, 2011 at 2:11 pm
mkelly says:
q = ε*σ*(Th^4 – Tc^4) Ah
“Perhaps the most fundamental thing to understand about that equation is that if Tc was zero then there would be more heat going from the hotter object to the colder object than when Tc is not zero.
It is sort of amazing how such a simple concept seems to illusive for some people!”
So you agree with me that there is no back radiation only temperature gradient. Two things must exist for heat transfer to take place 1. path 2. temperature gradient. Temperature is not back radiation. A simple concept Joel.
However the must fundamental thing to note is that if both objects have the same temperature there is zero back radiation as you call it. The back radiation from the atmosphere cannot heat the ground higher it takes a temperature change in the atmosphere.
Please now specify the temperature you would like to use for the atmosphere and we can proceed.
By the way that should be “too illusive” not “to illusive”.

Tim Folkerts
May 13, 2011 10:15 am

Bryan says: “Hence my calculation ignored the window radiation which nobody thinks “comes back” or do they?”
That suggest a fundamental misconception. (Or else I have some fundamental misconceptions! 🙂 ) Here is what I think …
* Nobody thinks (or at least nobody should think) that any of the 390 W/m^2 radiation “comes back”.
* The earth emits radiation. Once the energy leaves, it is gone; it is no longer the earth’s energy. All that matters is the surface’s emissivity and the surface’s temperature. Whether the atmosphere lets 40 W/m^2 thru or 390 W/m^s will not matter (at least not until the surface starts to cool off).
* The atmosphere emits its own energy. Wherever that energy may have come from (from the sun, from the water cycle, from the surface’s radiation, from convection), it is the atmosphere’s energy. All that matters is the temperature and emissivity (complicated by the fact that emissivity is a function of wavelength and that IR can travel considerable distances, so there is not a clear “surface” for calculations).
* Some of the energy from the atmosphere heads left; some head right; some heads up; some heads down. Since the radiation heading down is heading “back toward us” we call it “backradiation” (not because any photons are heading back where they came from).
The surface currently emits ~ 390 W/m^2 in a generally upward direction.
The atmosphere current emits ~ 324 W/m^2 in a generally downward direction.
There would only be radiation “coming back” if the atmosphere acted as a reflector, which is not the case (to any appreciable extent).

richard verney
May 13, 2011 10:27 am

Tim
Regarding your post at 5:32 am regarding Trenberth’s energy diagram. I am not saying that his diagram is wrong, merely that I am fairly confident that when more is known and understood about the system (this may be 20, 30, 50 years hence – hopefully not that long), it will be found to be wrong.
As an engineer, you often have a ‘feel’ for what will or will not work. Hopefully, one’s ‘feel’ for something is right although I accept that inevitably there will be cases when it is not right (or not fully right).
I know that this is unscientific, but to me, the Trenberth energy budget does not have the right ‘feel’. The fact that there is famously ‘missing heat’ may indicate that his energy budget is wrong, but I am not going to carp on about that.

richard verney
May 13, 2011 10:41 am

Tim
Perhaps, I should have added to my last comment that I accept that I am not in a position to put forward alternative figures (partly because I consider that there is a lack of accurate empirical data on a number of aspects that affect the budget) but in general, I consider that the use of average figures is not simply an over simplification of the system but more significantly it disguises and distorts what is in practice going on thereby leading to a failure to properly understand how the system works as a whole. My gut tells me that it underplays the effect of solar and overplays the effect of DWLIR.
The real driver of the Earth’s climate is the oceans and the solar energy being poured into these which is then distributed around the globe by currents (both ocean currents and air currents). Until this is properly understood (which will involve a proper understanding of clounds), we will never properly understand the Earth’s climate.

Tim Folkerts
May 13, 2011 10:42 am

>>> Richard says “8. That Trenberth’s energy budget is wrong. It defies commonsence ”
>> Tim replies: “There are many ideas in science that defy the common sense”
>>”So … what specific numbers in the diagram do you think are wrong?”
> JAE replies: “It doesn’t make sense to me ”
I obviously didn’t my point about common sense, since more people are claiming their common sense is being violated, as if violations of common sense are remarkable in science.
Once again, science does not always make sense, especially if you don’t really dig into it.
Let me try one more time on radiation…. Using the way over-simplified equation as a starting point:
q = ε*σ*(Th^4 – Tc^4) Ah
We could call the term related to Th^4 (390 W/m^2) the “up-radiation” or “away-from-us radiation” from the surface and we could call the term related to Tc^4 ( 324 W/m) the “down-radiation” or “back-toward-us radiation” from the atmosphere.
Or we could call the net result (390 W.m^2 – 324 W/m^2 = 66 W/m^2) the net radiation. I really don’t care.
The bottom line is that the surface has a net lost of thermal IR radiation of 66 W/m^2.
The surface has a loss of energy via convection and evaporation as well.
All these together add to the other energy input (primarily the sun’s visible light) and the energy of the surface is pretty well balanced. Any slight change in these numbers will lead to global warming/cooling.)
(And yes, there are other inputs/outputs of energy from the surface. There is geothermal energy flowing out at a rate I have seen estimated as 0.1 W/m^2 — pretty insignificant it would seem. The much bigger factor would be ocean currents eg El Nino/La Nina. Over the course of years or decades, changes in the oceans currents could have a major impact on surface temperatures because of the huge heat capacity of the oceans.)

Bryan
May 13, 2011 10:53 am

Tim Folkerts .
I’m afraid you did not get it quite right even on the second attempt.
>>You say……. If the surface emits at an average of 270 K
OK, that number is low, but that is not the main point. I will grant that 285 K is a better number
I didn’t mention 285K, the actual number is 288K and corresponds to the average surface temperature of 15C
This is from my post above…………….
“Tim the actual surface temperature for this diagram is +15C = 288K”
I don’t think it is worth pursuing this rather trivial point as I have better things to do.
However I think that you should review your opinion that all downwelling Long Wavelength radiation is backradiation.
Why not instead of “bending” the numbers to suit an IPCC position try bending them the other way and take an average.

Bryan
May 13, 2011 11:02 am

Tim Folkerts
Your tying yourself up in knots here;
You say ……… “Some of the energy from the atmosphere heads left; some head right; some heads up; some heads down. Since the radiation heading down is heading “back toward us” we call it “backradiation” (not because any photons are heading back where they came from). “………
So by this definition, the Solar Radiation is also “backradiation”!!!

richard verney
May 13, 2011 11:29 am

Dave
Regarding your post Dave Springer says:May 13, 2011 at 6:05 am.
My understaning is that theoretical calculations based upon black body calculations for Venus, Mars, Titan and the Moon do not give the correct observed surface temperatures. This may suggest that fundamentally, planets cannot be equated with black bodies. I believe this to be particularly so in the case of a water planet such as the Earth with its variously heated and swirling atmosphere.
Much of the input data is not known with sufficient certainty (eg. the extend of cloudiness which impacts upon how much solar energy is received by the ground, the amount of solar energy absorbed by the atmosphere and the amount of solar energy simply reflected by the ground which impacts upon the amount of solar energy received by the ground, the albedo, the emissivity etc). These uncertainties (some of which may tend ro cancel each other out but which could easily be cummulative) could lead to substantial errors. May be by as much as 10 to 20K. I think your +/- 10% is far to low unless this is applied to the 255K figure.
I also consider that the average temperature of the Earth is not known with sufficient certainty. We are all fanilkier with the arguments as to whether this uncertainty plays a significant role when assessing trends, but this potential uncertainty certainly does have a significant role when seeking to assess absolute (as opposed to a trend) temperature. We have all seen various comments wheresome has set out the IR temperatures around there property (garden, drive, paths, flower beds) which have shown 5 or so different temperatures in a 1000 sqm plot. These differences are everywhere not just in down. If you go for a country walk you can often feel differences as you walk in fields, up hills, in hollows, by hedgerows, near dykes, lakes canal pathways etc.
There have been so many station drop outs I can’t recall the number of measuring stations used, but I would say that if we were to have any prospect of getting a reasonable handle on average globabl temperatures this would have to be increased a billion fold (or by even more than that). It would not surprise me if we were 1 to 3K out on the true and accurate global average temperature which wis within 10% of the assessed 33K figure.
Much of what you say regarding oceans, I agree with or at least in part agree with. I also agree, that once above freezing point, water would become the major player. However, I am far from convinced that even if there was no CO2 (leaving aside issues about life on Earth), all the oceans would be frozen. It may be that oceans around the Equator would not freeze such that there would always be some water vapour.
I am not convinced that there is sufficient evidence of a snowball Earth (although there may well have been). I recall one post on how Earth came out of a snowball, which explanation may be a possibility but then again there may be other explanations (eg., oceanic volcanos splitting in the ice and thereby releasing some water vapour, soot deposits changing albedo, even meteor collision – who knows given the lack of evidence).

Joel Shore
May 13, 2011 11:33 am

Smokey says:

As a matter of fact, convection does occur between the earth and space. Warmer air both rises and radiates. That area is called the atmosphere, and convection provides a mechanism for dumping heat into space.

Convection provides a mechanism for moving heat around in the atmosphere. The only way that any significant amount of heat gets transferred out of the earth system (which means, including the atmosphere) is by radiation.
As I discussed in response to Martin Lewitt, some people seem to believe they can vaguely invoke the water cycle to transport heat back out into space. However, because heat can only get out into space via radiation, this proposal has testable consequences: The atmosphere must warm up enough somewhere in order to increase the amount of heat that it emits to space.
JAE says:

Yeah, the “convective-radiative” models include convection, but my position is that they don’t treat it properly.

And, your evidence of this is, what exactly?
Besides which, like I said, we don’t derive the ~33 K temp difference between the earth’s surface and its effective blackbody radiating temperature by modeling. We get it empirical observation and applying conservation of energy.
mkelly says:

Mr. Springer, you cannot call temperture back radiation. In science words have meanings. Velocity is not distance although it is intertwined with distance. I subscribe to the Mark Twain version. see below

Nobody is calling temperature “back radiation”. What is being called “back radiation” is radiation that the atmosphere emits (back toward the earth) by virtue of having a nonzero temperature.
Bryan says:

If you had read any of my previous posts you would know that I have always maintained that colder surfaces can radiate to warmer surfaces but they cannot HEAT them.

Once again, you make a statement that can be interpreted as correct but completely irrelevant to discussions of the atmospheric greenhouse effect. Or, alternately, it can be interpreted as incorrect. You seem to thrive on this ambiguity, just as G&T did, in order to peddle pseudo-scientific nonsense.

Tim Folkerts
May 13, 2011 11:38 am

Bryan says: May 13, 2011 at 11:02 am
“So by this definition, the Solar Radiation is also “backradiation”!!!”

Sure. Why not? This is simply semantics, not physics.
The sun provides “solar back radiation” or “solar downward radiation” or “solar toward-the-surface short-wavelength EM radiation” to the surface (~ 168 W/m^2). There is no “solar upward radiation” so there is really no need for the directional adjective, but we could use it if we want.
The surface provides “surface forward radiation” or “surface upward radiation” or “surface away-from-the-surface long-wavelength EM radiation” to the atmosphere (350 W/m). It also provides “surface forward radiation” of 40 W/m^2 to outer space.
The atmosphere provides “atmospheric back radiation” or “atmospheric downward radiation” or “atmospheric toward-the-surface long-wavelength EM radiation” to the surface (324 W/m^2).
Personally I’d use a simpler notation like q=radiant energy flow, S = sun, A = atmosphere; G = ground; O = outer space.
Then we would have:
q(S–>G) = 168 W/m^2
q(G–>A) = 350 W/m^2

Bryan also says:
“However I think that you should review your opinion that all downwelling Long Wavelength radiation is backradiation.”

Don’t worry about the labels — worry about the physics.
* All q(x –>S) radiation adds energy to the surface
* All q(S –>x) radiation removes energy from the surface
It is as simple as that.
If you want to call some or all of the q(x–>S) “back radiation” or “downwelling radiation” then feel free.

Tim Folkerts
May 13, 2011 11:52 am

OOPS! I MIXED UP THE ABBREVIATIONS
I said:
Don’t worry about the labels — worry about the physics.
* All q(x –>S) radiation adds energy to the surface
* All q(S –>x) radiation removes energy from the surface
It is as simple as that.

“S” was already taken for “solar”, so “G” was used for ground (which also would include the oceans). That should have read:
Don’t worry about the labels — worry about the physics.
* All q(x –>G) radiation adds energy to the ground (ie adds energy to the surface)
* All q(G –>x) radiation removes energy from the ground
It is as simple as that.

Joel Shore
May 13, 2011 12:20 pm

richard verney says:

1. That it is inappropriate to treat the Earth as if it were a blackbody (a rotating spherical object with oceans that act as heat sinks, swirling and variably heated atmosphere with variable latent heat characteristics, imprecisely known and changing emissivity etc is nothing like a blackbody). This fundamental misconception inevitably leads to a misconception as to what should be the ‘theoretical notional’ temperature of the Earth as if it had no atmosphere. Wrong model wrong answer.

I don’t understand this description. If you are talking about the earth + atmosphere, it is not treated as a blackbody. There is merely a number given that tells us what its temperature would be if it were emitting as a blackbody. If you are talking about the earth alone, then determining the relation between its temperature and what it radiates using the blackbody formula is a very good approximation because in the IR range of interest, the materials making up the earth’s surface do indeed have emissivities very close to 1.

2. We have imprecise data with respect to albedo (Trenberth in his paper has albedo measurements varying between 27.1 and 35.8% and others have it is high as 40% and the albedo is constantly changing) and imprecise data with respect to the amount of solar energy absorbed by the atmosphere and imprecise data as to the amount of solar energy reflected by the atmosphere. These errors mean that we cannot accurately calculate the ‘theoretical notional’ temperature of the Earth as if it were a blackbody. These (potential) errors are not insignificant and afre such that they could lead to perhaps a 10 to 20K error in the assessment of the ‘theoretical notional’ temperature of the Earth (as if it were a blackbody).

You have exaggerated the error ranges in the albedo by looking at the separate estimates of land and ocean albedo. For the global albedo, the table that you have quoted from shows four estimates in the range of 31 to 31.3% and two outlier estimates of 28.1% and 33.8%. [Paper here: http://content.imamu.edu.sa/Scholars/it/net/trenbert.pdf ]
At any rate, a better estimate of the “theoretical notional” temperature of the Earth is using the measured emission OLR (outgoing longwave radiation). The table lists four measurements in the range of 233.3 to 237.4 W/m^2 and then two outliers at 245 W/m^2 and 253.9 W/m^2. Even if we consider the most extreme high outlier of 253.9 W/m^2, that only gives an effective blackbody temperature of 258.7 K, which compared to 255 K is far less than a 10 to 20 K error.

3. The water cycle alone, makes it inappropriate to assess ‘theoretical notional’ atmospheric temperature on the basis that the Earth is a blackbody.

The water cycle does not get you around having to obey conservation of energy, which tells us that the earth has a current surface temperature that is roughly 33 K higher than what it would support if its atmosphere were transparent to the radiation that its surface emits (but it otherwise had the same albedo, etc.).

4. We do not have an accurate figure for the average temperature of the Earth. Indeed, this could be out by many degrees (say somewhere between 1 to 3K).

I doubt the higher figure, but this is still way smaller than 33 K.

5. That irrespective of the presence of GHGs, commonsense suggests that the atmosphere at or near the surface would be warmer than the ‘theoretical notional’ temperature of the Earth (assessed as if it were a blackbody). According to Trenberth some 78 W/m^2 of the incoming solar energy is absorbed by the atmosphere. Since the warmists case is that CHGs are transparent to incoming solar radiation, CHGs cannot account for this absorption. The absorption of this energy must have some heating effect on the atmosphere since energy is fungible and this incoming energy must have gone somewhere also not forgetting the latent heat content. The rotation of the Earth and the movement of the atmosphere (winds, convection etc) must inevitably lead to heating of the atmosphere (after all heat is a by product of work and considerable work is involved in moving the atmosphere). Most importantly, the oceans (which are heated by the sun during the day) are constantly heating the atmosphere day and night.

I still don’t see how all of these words get you around having to satisfy conservation of energy.

6. If GHGs were responsible for all the warming of the atmosphere there would be some correlation between the concentration of GHGs and temperature both laterally and vertically but there does not appear to be such correlation.

I love how the same sort of people who want to tell us about the importance of convection and such in order to get around limits that convection cannot get you around then turn around and make statements like this, ignoring convection and advection when it suits their purpose…and when it is vitally important not to do so. You have presented absolutely no data to support either your contention that there is no correlation nor showing what correlation would be expected once one fully includes other factors.

7. Adiabatic lapse/compression far better explains the temperature profiles of the atmosphere (although this is not a complete explanation in itself). You have likened it to a tyre and suggested that the temperature is radiated away. However, that analogy is incomplete. If you pump up a tyre and then drive on it (steering, braking, going over bumps etc), the side wall flexes slightly only slightly increasing the pressure as the tyre goes in and out of shape and this small movement is sufficient to maintain the temperature as long as the car is being driven (you see this in motorsport). The circulation of the atmosphere (hot air rising and cooling etc) is the equivalent and helps sustain the temperature of the atmosphere. The atmosphere is in constant flux and the work done inevitably generates heat.

Again, these arguments are not going to get you around conservation of energy considerations.

8. That Trenberth’s energy budget is wrong. It defies commonsence that DWLIR exceeds incoming solar energy given that the DWLIR can have been created only by the incoming solar energy (and energy radiated/released by the core which latter energy is said to be miniscule such that this can be ignored).

As Tim has pointed out, the universe is governed by the laws of physics, not your “commonsence”. If the laws of physics contradict your common sense, I’ll go with the laws of physics. The only reason your argument even seems plausible is that the numbers for DWLIR only exceed the solar energy by a little bit on earth; on Venus, they exceed it by such a huge amount that it would be immediately obvious that your common sense is not even close to correct.

The use of ‘average’ figures disguises and distorts what is truly going on. Just imagine for a moment what would be the position if the Earth’s speed of rotation was different. This would have little effect on the figures used in the Trenberth diagram but conditions on Earth would be radically different.

It doesn’t distort what is going on at all. Yes, looking at averages throws out a lot of information, but that just means that the Trenberth budget has its uses and its limitations, like anything does.

9. That if DWLIR of the magnitude as indicated by Trenberth really existed, we would be able to extract energy from this and/or there would be considerable research into exploiting this natural green resource. After all, this DWLIR would have potential exceeding that of solar and wind.

This has already been commented upon by others (and myself) upthread.

10. At most, the effect of GHGs is merely to delay heat loss being radiated. However, it is likely that at night (when there is no incoming solar energy) or at other times when atmospheric conditions are such that there is a temperature inversion, any LWIR that has been delayed has sufficient opportunity to radiate to space there by meaning that no excess heat is ‘trapped’ (ie., the heat in the atmosphere does not build up).

This is just pernicious nonsense that can only fly with someone who has never actually done a simple radiative balance calculation. It is not just a “delay”; the greenhouse effect reduces the rate of energy loss out into space (for a fixed surface temperature), requiring a higher average surface temperature to restore radiative balance.

May 13, 2011 12:25 pm

Joel Shore says:
“Convection does not occur between the earth and space.”
Unequivocal statement. And unequivocally wrong. Convection does in fact occur between the earth and space, despite the convoluted attempt to explain that mis-statement of fact.
I only mention this because of Joel Shore’s regularly labeling my rejection of CAGW as ‘ideological,’ which is simply projection on his part. I base that rejection on the fact that there is no evidence of global damage from CO2, despite a ≈40% increase. Only the GIGO models predict global harm from that essential trace gas, while real world observations show that CO2 is harmless.

Bryan
May 13, 2011 12:30 pm

Tim Folkerts
Yes I suppose you could operate like that.
Just as a physics lecturer might decide to do all his lectures standing on his hands upside down.
You would however be the only person on the planet who calls Solar Radiation under the title of “backradiation”.
Others might think you a bit odd,….. but don’t let that put you off.

Tim Folkerts
May 13, 2011 1:08 pm

Bryan,
You are missing my point.
The names do not ultimately matter, only the physics. If people want to use unusual nomenclature, that does not change the physics. There is “backradiation” because people have given that label to “downward directed thermal IR radiation from the atmosphere toward the surface”. It is what I labeled q(A–>G).
There is also q(S–>G). We can call it “sunlight”. Or we could call it “solar backradiation” if we really wanted to emphasize that is heads the same direction as the IR backradiation from the atmosphere. But as long as we both understand the concept, the name doesn’t matter.
Two fundamental physics questions related to climate science are:
1) Does q(A–>G) exist?
2) Is the value around q(A–>G) = 324 W/m^2?
I think the answer to both is “YES!” It seems you think (1) is correct but you question (2).
Some people seem to think that either there is no such thing as q(A–>G), or that it could possibly be larger than q(S–>G). I’d love to hear specific reasoning to support these positions.
THAT should be the starting point for a discussion, not ” ‘backradiation’ is a poor name”!

May 13, 2011 1:14 pm

Joel Shore says:
May 13, 2011 at 11:33 am
“Nobody is calling temperature “back radiation”.
Joel says: “At some point, this comes down to terminology. It is most correct to say that because of back-radiation (in combination with the radiation from the sun), the earth is warmer than it would be if all the radiation from its surface escaped into space.”
Ira says: “The only rational explanation is the back-radiation from the Atmosphere to the Surface.”
You say nobody is calling temperature back radiation. Well it seems as you and Ira are at the very least. You use the word warmer which indicates a temperature change. If Ira wants to say “The only rational explanation is a temperature change in the atmosphere that is causing the surface to warm.” then say that and not use back radiation.

Tim Folkerts
May 13, 2011 1:17 pm

Joel said “Convection does not occur between the earth and space.”
and Smokey questioned this.
Smokey, how about this?
“Energy transfer via convection does not occur between the earth as a whole and space.”
That seems to be very clearly what Joel meant.
I suppose you could warp the sentence to mean:
“Convection does occur within the atmosphere, and the atmosphere is between the earth’s surface and outer space. Thus, convection occurs in the region ‘between the earth and space’ “.
Frankly, I can’t see anyone actually thinking that is what Joel meant!
Or do you have some other meaning for “convection occurring between earth and space” ?

May 13, 2011 1:40 pm

Tim Folkerts,
Thanks for your input. Note that I also commented: “Warmer air both rises and radiates.” But there’s not much wiggle room to parse here:
“Convection does not occur between the earth and space.”

Jim D
May 13, 2011 1:50 pm

mkelly, back radiation can increase without a temperature increase, for example by just adding CO2. It is very important to separate the temperature effect from atmospheric constituent effect to understand back radiation.

May 13, 2011 2:03 pm

Ira and Joel Shore, thanks for your comments.
Joel, I think that you make the mistake to assume that nothing will be reflected nor absorbed in a GHG free atmosphere. Reflection will be there as long as there are clouds, regardsless of water in vapour phase. (I am assuming here that you do not regard water in liquid or solid state as a GHG.) Particles will also absorb (and to a lesser degree reflect). Whatever the mechanism one has to note that 23% of incoming solar light is absorbed today in the atmosphere – not due to GHGs by definition. And that somewhere between 23% and 38% of outgoing thermal radiation is reflected.
So, if we seek a minimum temperature that earth holds without IR absorbing gases we need to count back radiation of absorbed sunlight, 39 W/sqm, 161 W/sqm absorbed sunlight + 38% reflection of outgoing radiation we get 322 W/sqm or 0 deg celsius at surface. This analysis leaves a lot out (e.g. no spatial analysis of temperature distribution, nor time) but yields us a mere 15°C to be explained by GHGs. It matters as CO2 is at least 12 doublings over the saturation levels of the base band. 15/12 yields a maximum of 1,25 °C per CO2 doubbling. This assumes that no water vapour, methane etc would be present in a CO2 free atmosphere, which is obviously not true and we can safely say that a doubbling of CO2 levels would give less than a degree temperature increase. (All heat flows according to Trenberth, 2009)
Joel and Ira, please comment.
And now, for a beer. Happy weekend to you all.

JAE
May 13, 2011 2:33 pm

Tim Folkerts says:
May 13, 2011 at 10:42 am
Well, I’ll be —! Thank you, Tim! By juxtaposing that equation with the K&T diagram, I finally got it through my thick head just what the diagram is trying to show! It now actually makes sense to me (although I still don’t know if I agree with it).

May 13, 2011 3:35 pm

Joel,
I confess that the Trenberth diagram you directed me to leaves me with more questions than answers.
According to the diagram:
– the Earth only absorbs 161 W/m^2 from the sun yet emits 493 W/m^2
– the Atmosphere only absorbs 78 W/m^2 from the sun yet emits 502 W/m^2
Also his numbers don’t add up, though they are out only by a few W/m^2 which perhaps is not important…
 
Unfortunately this doesn’t make sense to me, if the extra 333 W/m^2 the earth needs to radiate 493 W/m^2 comes from the atmosphere, then where did the atmosphere get the energy from?  It only gets 78 W/m^2 from the sun after all…
Also, I have reviewed a couple of books on statistical thermodynamics, and nowhere do I find any evidence that a cold object can heat/transfer energy to a warmer object without the application of external work.  As a matter of fact they state that this cannot spntaneously occur.  This is not surprising, statistical thermodynamics still have to repsect the first & second laws.
I quote from an excellent book, which you might enjoy:
“In all cases the expressions for the changes in entropy correspond exactly to those deduced from Clausius’s definition, and we can be confident that the classical entropy and the statistical entropy are the same.”
Peter Atkins, Four Laws That Drive the Universe; Oxford University Press 2007
So if, as you claim, the energy from the atmosphere heats the earth, where is the external work that allows this to occur?  Unless of course the atmosphere is warmer than the surface of the earth – but then the earth wouldn’t spontaneously heat the atmosphere with the 356 W/m^2 it somehow emits from an input of only 161 W/m^2.
Another point, you Ira and others have stated that it is the NET difference of energy transfered which must go from hot to cold in order to respect the second law.
However the second law states that entropy must stay the same or increase, it can’t decrease as a result of any spontaneous event.  So even if a cold object could transfer energy to a hotter object spontaneously, using the numbers in Trenberth’s diagram the net result is a decrease in entropy inasmuch as the 333 W/m^2 emitted from the colder atmosphere represents a greater decrease in entropy than the 356 W/m^2 is an increase in entropy.
So, to me at least, this diagram of Trenberth’s seems very problematic.
While I have enjoyed reading the many threads on adiabatic heating etc… I really can’t get by what appears to me to be a very fundamental problem with this theory of Greenhouse Gas Warming, namely that a cold object can heat a warmer one.

May 13, 2011 5:12 pm

A question that interests me is one that has come up a few times lately, namely:
“So … what specific numbers in the diagram do you think are wrong?”
Below is my version of it. – I know some of you guys have tried to “shoot me down” on this one before, but here I am, once again, as you never did convinced me I was wrong.:
As early as 1859, Gustav Kirchhoff proposed that “At thermal equilibrium, the emissivity of a body (or surface) equals its absorptivity” and as far as I can understand, nobody objected and his proposition was accepted as part of “Kirchhoff’s Law”, and, to me, it seems logical and should be unavoidable as it is based on “energy conservation”.
– But even so in 1997 some scientists, Kiehl, Trenberth & al (K&T), published (an IPCC “peer-reviewed”) “Energy Flow Chart” (EFC) which showed equilibrium, in as much as energy in = energy out.
– The surface is, in the EFC, shown to absorb 168 W/m² of incoming Solar radiation, but does not even attempt to conserve any energy as it gets rid of (24+78)= 102 W/m² via thermals and “evapo-transpiration” and then in stead of being contented with radiating away the remaining 66 W/m², it sends out a whopping 390 W/m²
– Yes I have posted questions about this conundrum as to where the surface finds the 324 W/m² that forms the basis for this “Radiation Circuit” (RC) between atmosphere and surface. But none of the answers given have so far convinced me.
– Kirchhoff was kicked into touch and no scientist grumbled.
-Well, I doubt very much that the surface knew when it emitted the 324 extra W/m² of energy from it’s “energy store” that the atmosphere was going to send it straight back and thus conserve that energy on it’s behalf.
Q: “So … what specific numbers in the diagram do you think are wrong?”
A: “ Below is how I would have changed some of the numbers”
To my way of reasoning, if the basics of the EFC is correct, then it would be reasonable to accept that the result of the 168 W/m² absorbed by the surface would be that (24+78) = 102 W/m² are leaving in the form of thermals and “evapo-transpiration” and then the remaining energy 66 W/m², after having done it’s job, would leave via radiation.
– And then, – and only then – as what is lost from the surface is absorbed by the atmosphere (which, by the way has absorbed 67 W/m² directly from the Sun,) would I enter the GHGs and with them “The Atmospheric Window” which gives a free passage to 40 W/m². – In which case there are (66 – 40) = 22 W/m² of energy left to bounce back and forth between surface and GHGs creating a “delay” before those 22 W/m² also left for space, then yes I too may have accepted a bit of warming down here in the Troposphere, but only as a result of the delay in the 22 W/m² leaving the system.
But if the surface is in the habit of radiating away 222 W/m² more than it receives then we must assume that the Moon’s surface will do the same and that the 107 ° C that is reported on as “measured data” on it’s sunny side must be a myth as the Moon gets nothing back from GHGs

Tim Folkerts
May 13, 2011 5:27 pm

Alleyne muses: May 13, 2011 at 3:35 pm
“Unfortunately this doesn’t make sense to me, if the extra 333 W/m^2 the earth needs to radiate 493 W/m^2 comes from the atmosphere, then where did the atmosphere get the energy from? It only gets 78 W/m^2 from the sun after all…”

Here is a different way to think about it that might help. I haven’t seen it explained quite like this before, but it suddenly makes a lot of sense to me to use this approach.
The total thermal energy of a 1 m^2 column of the atmosphere could be roughly estimated as:
U = mcT = 10,000 kg * 1000 J/kg*K * 300 K = 30,000,000 J
There are a variety of little problems with this estimation, but it is clear that a 1 m^2 column has millions of joules of thermal energy. Suddenly the idea of it losing a few hundred of those joules doesn’t seem so noteworthy.
Or think of it this way. At some point in time, an average 1 m^2 column of air will have some amount of thermal energy U (~ 30,000,000 J). During the next second, that column of air will
* give 324 J to the surface via thermal EM radiation
* give 195 J to outer space via thermal EM radiation
* get 350 J from the surface via thermal EM radiation
* get 78 J from the surface via evaporation/condensation
* get 24 J from the surface via convection
* get 67 J from the sun via sunlight
30,000,000 J – 324 J – 195 J+ 350 J + 78 J + 24 J + 67 J = 30,000,00o J
The atmosphere has managed to hold steady, giving up only a TINY fraction of its total energy and absorbing only a TINY fraction of its total energy, with no net gain or loss. (Of course, the number will change from day to night and from winter to summer, but these are “typical” numbers.)
Or just for dramatic effect I could write it as ….
30______ MJ
– 0.000324 MJ
– 0.000195 MJ
+ 0.000350 MJ
+ 0.000078 MJ
+ 0.000024 MJ
+ 0.000067 MJ
———————
= 30 _____ MJ
The atmosphere losing 0.001% of its energy over the course of a second does not sound so dramatic anymore!

May 13, 2011 5:30 pm

Allyne,
Peter Atkins’ handy little book is a really excellent tutorial for anyone interested in understanding the zeroth through third Laws and entropy.
Also, I have bit of a problem with any explanation that goes beyond a simple time delay due to GHGs. There is certainly no empirical evidence that they generate heat. If they do, I want to see testable, replicable and convincing real world evidence. All we have now are models.
And is Trenberth implying that if a warmer object were surrounded by hundreds of slightly cooler objects, all radiating toward the slightly warmer object, that the combined radiation from the slightly cooler objects would warm the single slightly warmer object? Entropy doesn’t allow that, unless work is performed between. What is doing that work?

richard verney
May 13, 2011 5:30 pm

The observations made by Alleyne at 13, 2011 at 3:35 pm appear particularly pertinent. Trenberth uses the problematic statement “Absorbed by Surface” with respect to the back radiated 333 W/m^2.
Tim Folkerts’ comments at May 13, 2011 at 10:42 am in which he suggests that the surface is (in effect) radiating 66 W/m^2 is a very different take on the diagram and is not simply interpreting the diagram but re writing what Trenberth is saying.
Tim may have been forced into this revision since he has commented several times that radiation can not heat an object to a temperature greater than the temperature of the radiating source. In the case of the the source of the 333 W/m^2 this is back radiated from a height in the atmosphere which is at a temperature lower than the ground surface and therefore this radiation cannot be “Absorbed by the Surface”. Thus Tim’s revised take significantly contradicts the assertion put forward by Trenberth.
As soon as you look at net energy flows, you are looking at the extent to which GHGs merely delay energy departing the Earth’s surface. In which case one needs to properly consider what happens at night (when there is no solar input) and what happens when there are temperature inversions (which must be taking place somewhere on this planet on a reasonably regular basis) which act as a safety valve venting any excess temperature built up (see the article Frostbite Falls by E M Smith on WUWT in which he givces an illustration where local temperatures fell to -43C, ie., 230K which is significantly below the calculated BB temp of the Earth without GHGs)

Tim Folkerts
May 13, 2011 6:11 pm

Alleyne again muses:
“However the second law states that entropy must stay the same or increase, it can’t decrease as a result of any spontaneous event. So even if a cold object could transfer energy to a hotter object spontaneously, using the numbers in Trenberth’s diagram the net result is a decrease in entropy inasmuch as the 333 W/m^2 emitted from the colder atmosphere represents a greater decrease in entropy than the 356 W/m^2 is an increase in entropy.”

I don’t see the problem. Entropy is calculated as
dS = dQ/T
For the sake of argument, lets call the ground 285 K and the atmosphere 250 K (but the values don’t matter as long as the number for the atmosphere is lower than the number for the surface). Let’s look at 1 m^2. And I use the numbers from the original energy balance diagram.
For convection between surface & atmosphere.
The entropy of the ground decreases each second by
dS = – 24 J/ 285 K = – 0.084 J/K
The entropy of the atmosphere increases by
dS = +24 / 250 K = + 0.096 J/K
NET CHANGE from the process is +0.096 J/K – 0.084 J/K
= 0.012 J/K
This is positive, as required.
For Evaporation/Condensation between surface & atmosphere.
The entropy of the ground decreases each second by
dS = – 78 J/ 285 K = – 0.274 J/K
The entropy of the atmosphere increases by
dS = +78 / 250 K = + 0.312 J/K
NET CHANGE from the process is +0.312 J/K – 0.274 J/K
= + 0.038 J/K
This is positive, as required.
For radiation between surface & atmosphere.
Looking specifically at the exchange between surface and atmosphere, the radiation rates are 350 W/m^2 up from surface to atmosphere and 324 W/m^2 down from atmosphere to surface, for a net 26 W/m^2 upward.
For the surface,
dS = – 26 J/ 285 K = – 0.091 J/K
The entropy of the atmosphere increases by
dS = +26 / 250 K = + 0.104 J/K
NET CHANGE from the process is +0.312 J/K – 0.274 J/K
= + 0.0.013 J/K
This is positive, as required.
(You could also use the 350 J & 324 J numbers directly, but that will give the same results).
I’ve actually given numbers to show entropy always increases as required. What specific example of a process with a net entropy decrease were you thinking of?

Joel Shore
May 13, 2011 6:20 pm

vindsavfuktare says:

Joel, I think that you make the mistake to assume that nothing will be reflected nor absorbed in a GHG free atmosphere. Reflection will be there as long as there are clouds, regardsless of water in vapour phase.

First of all, let me be clear that what I have talked about is an atmosphere that does not absorb terrestrial IR radiation. That is, the total greenhouse effect does include a contribution due to clouds. Note that the clouds do not significantly reflect at these mid- and far-IR wavelengths. They only absorb the radiation or (if not optically-thick) transmit it. So, the way they act is in the same way that the gaseous contributors to the greenhouse effect act (albeit with a broader absorption spectrum).
The rough number for the amount that clouds contribute to the natural greenhouse effect is 25% as discussed here http://pubs.giss.nasa.gov/docs/2010/2010_Schmidt_etal_1.pdf . (These numbers are necessarily rough in that the effects are not completely additive, so you get one number if you start with the current atmospheric composition and take clouds out and another number if you start with an atmosphere with no GHGs and put clouds in.)

This analysis leaves a lot out (e.g. no spatial analysis of temperature distribution, nor time) but yields us a mere 15°C to be explained by GHGs.

I lost you…particularly in the part about how you treat incoming sunlight being absorbed. That doesn’t make sense to me. The point is that the earth’s surface emits as a blackbody at a temperature of ~288K and the earth system as a whole emits an amount of radiation that corresponds to what the earth system absorbs from the sun in shortwave radiation, and corresponds to a blackbody temperature of 255 K. The difference in these temperatures is the greenhouse effect. Of this, about a quarter of it is attributable to clouds, as I noted above.

It matters as CO2 is at least 12 doublings over the saturation levels of the base band. 15/12 yields a maximum of 1,25 °C per CO2 doubbling. This assumes that no water vapour, methane etc would be present in a CO2 free atmosphere, which is obviously not true and we can safely say that a doubbling of CO2 levels would give less than a degree temperature increase.

Woe…There’s a lot of assumptions there (in addition, as I noted to attributing too small an amount to GHGs). First off, an atmosphere without CO2 is indeed expected to lead to the water vapor condensing out, so you do lose its effect. Furthermore, the earth’s surface will become much more ice- and snow-covered leading to a larger albedo and hence a further temperature decrease. So, no, I don’t buy your claim about how much the temperatures will drop when you take CO2 out. See the discussion here: http://www.sciencemag.org/content/330/6002/356.abstract (Technically, that paper looks at removing all of the non-condensable GHGs, so it is an interesting question what the models predict if you just remove CO2 and leave in the others. For removing all of the non-condensable GHGs, they get a temperature drop of about 35 K.)
Second, I am not sure where you get your number for 12 doublings over saturation. It may be right…but I’d at least like to see the reference. And, there are additional questions as to whether the forcing and climate sensitivity would really be constant over that many doublings.

Tim Folkerts
May 13, 2011 6:42 pm

It’s like a hydra ! ! ! :-0
Smokey says:
“There is certainly no empirical evidence that they generate heat. “

Quite true. GHG’s absorb IR energy. They emit IR energy. They are part of heat exchange. But they generate no energy.
Smokey says:
“And is Trenberth implying that if a warmer object were surrounded by hundreds of slightly cooler objects, all radiating toward the slightly warmer object, that the combined radiation from the slightly cooler objects would warm the single slightly warmer object? “

In a word — “NO!”
(If you need more explanation, each cool object, not matter what the size, shape position, or emissivity, will receive more energy from the warm object than it gives to the warm object — energy is conserved and entropy increases.)
richard verney says:
“Tim Folkerts’ comments at May 13, 2011 at 10:42 am in which he suggests that the surface is (in effect) radiating 66 W/m^2 is a very different take on the diagram and is not simply interpreting the diagram but re writing what Trenberth is saying. “

No, I suspect that Trenberth would interpret it just the way I do.
richard verney says:
“Tim may have been forced into this revision since he has commented several times that radiation can not heat an object to a temperature greater than the temperature of the radiating source. In the case of the the source of the 333 W/m^2 this is back radiated from a height in the atmosphere which is at a temperature lower than the ground surface and therefore this radiation cannot be “Absorbed by the Surface”. Thus Tim’s revised take significantly contradicts the assertion put forward by Trenberth.”

No, there is no revision. I have tried to say things several ways, but I always profess that there is “upward thermal IR from the surface” (~ 396 W/m^2, most of which gets absorbed by the atmosphere, but some passes directly thru the atmosphere and out into space) and a “downward thermal IR from the atmosphere” (~ 333 W/m^2, nearly all of which gets absorbed by the surface).
The entropy calculations in my previous post show that there is no problem with entropy for such a process of energy exchange via IR radiation.

Joel Shore
May 13, 2011 6:52 pm

Alleyne says:

Also, I have reviewed a couple of books on statistical thermodynamics, and nowhere do I find any evidence that a cold object can heat/transfer energy to a warmer object without the application of external work. As a matter of fact they state that this cannot spntaneously occur. This is not surprising, statistical thermodynamics still have to repsect the first & second laws.

Well, that’s good because nobody is arguing that point. We all know what the 2nd Law says. And, all models of the greenhouse effect, be they simple shell models or full convective-radiative models have the heat going from the warmer earth to the colder atmosphere.
However, the point I am making is that this does not mean that colder objects don’t radiate toward warmer objects. It only means that the colder object will always absorb more radiant energy from the warmer object than warmer object absorbs from the colder. That is what the real 2nd Law says.
Believing that this means that a cold object won’t radiate toward a warm object (or that the warm object won’t absorb any of this radiation) is believing in what I call “The Magical 2nd Law”, which is not a law of physics and, in fact, is not in accordance with the statistical physics principles on which the 2nd Law is ultimately based. If you don’t believe in “back radiation” or somehow imagine that the final steady-state temperature of the earth doesn’t depend on how much “back radiation” it receives from the atmosphere, then you are a believer in the Magical 2nd Law, not the real 2nd Law.
Furthermore, the equations for radiative transfer that are used in all models of the greenhouse effect will, if correctly applied (e.g., with objects obey Kirchkoff’s Law), will automatically satisfy the 2nd Law.

So if, as you claim, the energy from the atmosphere heats the earth, where is the external work that allows this to occur?

Look, you are simply confusing terminology here. If you mean by “the atmosphere heats the earth” that the net energy flow (i.e., heat) goes from the atmosphere to the earth, then no, that is not what I am claiming; that violates the 2nd Law.
If you mean by “the atmosphere heats the earth” that the atmosphere causes the earth to be at a higher steady-state temperature than if all of the radiation that the earth emitted went back out into space, then yes, that is what I am claiming; however, it doesn’t violate the 2nd Law because the heat still goes from the earth to the atmosphere. It is just that (for a given earth surface temperature) the heat flow away from the earth is less than it would be if the (IR-absorbing) atmosphere were not present. As a result, the earth’s surface temperature must increase until it reaches a point where the earth system (earth + atmosphere) is radiating back into space as much energy as it receives from the sun.
Read this 20 times until you understand it so that you stop attacking “strawman” arguments and address what we are actually saying.

However the second law states that entropy must stay the same or increase, it can’t decrease as a result of any spontaneous event.

But…you can’t define your spontaneous event as being just one part of the process. That won’t happen spontaneously. The point of the 2nd Law is that you can’t have the radiation from the colder object toward the warmer occurring without having the warmer object radiating toward the colder object too. Again, you seem to believe in the 2nd Law as some form of magic, rather than understanding the modern basis of it as following from statistical physics.

Joel Shore
May 13, 2011 7:01 pm

Alleyne: I see that Tim has interpreted your misunderstanding on the entropy considerations at the end of your post differently than I did. But, hopefully, one or the other or both of our posts will address this issue or issues that are bothering you. Note that for heat transfer between two bodies, the statement that entropy must increase and the statement that heat flows from the hotter body to the colder body are equivalent because of the fact that dS = dQ/T.

Dave Springer
May 13, 2011 7:03 pm

Bryan says:
May 13, 2011 at 8:53 am
The RW Wood real greenhsoue experiment (glass vs. rock salt) is bogus. The glass panes heat up when they absorb IR and lose it right away through conduction. The G&T experiment repeats the mistake. A kilometer’s thick layer of greenhouse gases doesn’t conduct very well at all, unlike a thin pane of glass or polyethelene.
G&T need to repeat the experiment with double-paned polyethelene with vacuum separating the two panes to imitate the poor conduction through kilometers of atmosphere. Woods needed to do the same thing with double-paned rock salt.
The better (much more expensive greenhouse panels) are constructed in such a way that they don’t conduct heat very well. IIRC correctly they use some sort of closed cell expansion which makes them sort of like transparent styrofoam. They aren’t clear like glass or solid plastic and don’t transmit visible light quite as well but the increase in R-factor is quite a lot and it’s still far less expensive than double-pane vacuum panels.

jae
May 13, 2011 7:07 pm

Tim: I fear that you are just confusing folks (of course, it is not intentional?):
“Or think of it this way. At some point in time, an average 1 m^2 column of air will have some amount of thermal energy U (~ 30,000,000 J). During the next second, that column of air will
* give 324 J to the surface via thermal EM radiation
* give 195 J to outer space via thermal EM radiation
* get 350 J from the surface via thermal EM radiation
* get 78 J from the surface via evaporation/condensation
* get 24 J from the surface via convection
* get 67 J from the sun via sunlight
30,000,000 J – 324 J – 195 J+ 350 J + 78 J + 24 J + 67 J = 30,000,00o J”
WHY DO YOU DO THIS?
You are simply “reading” the K&T diagram, using joules, instead of watts. Doesn’t help most folks, so don’t know why you do this? Probably confuses many. Is this your goal?
Sorry, K&T is still just a nice theory, without empirical evidence. So, still no real science, so far. Just an old cartoon that attempts to explain the GHE, but which has NO empirical support that I have seen (as I am sure you know, there can be NO science without empirical evidence. And the CAGW folks have exactly NONE that I know of, to date).
But some questions about the details:
Only 24 J for convection? And only 78 joules for evaporation? (which you get back in the upper atmosphere, but that’s another subject). Come on, man! These are probably the major places where the BS is hiding.
\
A “photon-warmed” OCO molecule instantly rises and imparts energy with other molecules in the air, which then also rise. I think that climate scientists vastly underestimate the convection. Otherwise, we should be seeing a change in the lapse rate as OCO concentrations increase. Don’t think we see this.
The “wet areas” on Earth NEVER see higher temperatures, because of the effects of water. See Willis Eschenbach’s articles on this. So the “average 78 joules for evaporation” meme is complete junk science.
IF your analysis (theory) is correct, THEN we should be seeing SOME measurable effects of the additional OCO. We are NOT seeing ANY effects of which I’m aware. In fact, we are seeing things going the “wrong” way, despite NASA’s disgusting corrupt attempts at revising the data history. Therefore, we must have some reservations about the adequacy of the construct.
Amen, have a good day.

Joel Shore
May 13, 2011 7:14 pm

richard verney says:

As soon as you look at net energy flows, you are looking at the extent to which GHGs merely delay energy departing the Earth’s surface.

I told you that this was pernicious nonsense the last time you made the statement and it doesn’t get to be less pernicious upon repetition. That is like saying that a beaver dam can’t create a pond because all it does is merely delay the flow of water.
The point is that the temperature of the earth’s surface is determined by what is needed to achieve a balance between the power it receives from the sun and the power the earth system radiates back out into space. When you reduce the flow of heat back out into space for a given surface temperature, the surface temperature must increase until the flow of heat back out into space once again equals the amount of heat being received from the sun.
This is not all that complicated to understand if one actually wants to understand science rather than to try to make the science fit one’s ideological preconceptions.

Phil.
May 13, 2011 8:10 pm

Alleyne says:
May 12, 2011 at 3:44 pm
Phil. said on Visualizing the “Greenhouse Effect” – Light and Heat
May 11, 2011 at 9:12 pm
“Strictly the equation should be:
q =(ε σ Th^4 – a σ Tc^4) Ac
however following Kirchoff’s Law a=ε
so:
q = ε σ (Th^4 – Tc^4) Ac”
As I understand it Kirchoff’s Law states αλ = ελ, but only at thermal equilibrium and does not require that the absorbtivity and emissivity of the surroundings be the same as that of the emitting object, otherwise what you are suggesting is that everything has the same absortivity and emissivity and αλ = ελ = a constant.

‘ε’ refers to the emissivity of the hot object, ‘a’ refers to the absorptivity of the hot object, it is nowhere implied that “the absorbtivity and emissivity of the surroundings be the same as that of the emitting object”

Joel Shore
May 13, 2011 8:27 pm

jae says:

Sorry, K&T is still just a nice theory, without empirical evidence. So, still no real science, so far. Just an old cartoon that attempts to explain the GHE, but which has NO empirical support that I have seen (as I am sure you know, there can be NO science without empirical evidence. And the CAGW folks have exactly NONE that I know of, to date).

You just make this stuff up, don’t you? The K&T diagram is based on empirical data, from satellite measurements mainly.

Only 24 J for convection? And only 78 joules for evaporation? (which you get back in the upper atmosphere, but that’s another subject). Come on, man! These are probably the major places where the BS is hiding.

You do realize that once you have a reasonable estimate for the amount of precipitation that falls over the earth then it is trivial to compute how much heat is transported by the evaporation / condensation mechanism? I almost had my students do it as an exercise in an introductory physics course.

I think that climate scientists vastly underestimate the convection. Otherwise, we should be seeing a change in the lapse rate as OCO concentrations increase. Don’t think we see this.

Again…You are just making stuff up. The models do predict a change in lapse rate…They predict that, overall, the lapse rate will DECREASE (mainly because of the decrease in lapse rate in the tropics…You know, the so-called “hot spot” that in a post above you claimed is missing). That’s a negative feedback in all the climate models.
If the models are underestimating convection, that would mean that the lapse rate should be decreasing even more…i.e., that the “hot spot” should be larger than the models predict. Your view of the view of the science is not even internally consistent.

Tim Folkerts
May 13, 2011 8:29 pm

jae says: May 13, 2011 at 7:07 pm
“Doesn’t help most folks, so don’t know why you do this? Probably confuses many. Is this your goal?”

There is always a possibility for being too simplistic for some and too complex for others. This assumed people know about watts and joules and heat capacity. If they don’t they can skip the analysis (but then they should also skip trying to draw scientific conclusions about the topic as well).
The point is that there is LOTS of energy around. Being able to gain or lose a few hundred joules from the atmosphere is not that amazing. In any given second, we do not need too supply all the 330 J per square meter for “backradiation” from the 168 J coming from the sun.
“Sorry, K&T is still just a nice theory, without empirical evidence. “
What sort of empirical evidence do you want? Every factor on the energy balance can be measured to fair accuracy, can be modeled to fair accuracy and can be calculated from first principles to fair accuracy.
I will admit that the exact values are not something that can be calculated/presented in a forum like this. I will admit that estimating global averages can be a challenge. I would not be surprised if some of the number are off a little. I would not be surprised if some of the numbers change significantly from year to year (for example, cloud cover & albedo). And small difference can make large differences in the overall climate.
But the general scheme seems pretty solid (keeping in mind that the energy balance diagram is a major simplification).
“Only 24 J for convection? And only 78 joules for evaporation? (which you get back in the upper atmosphere, but that’s another subject). Come on, man! “
Yes, I agree there could be significant uncertainty in the numbers. If you truly think there are major problems, then write them up and submit them to a scientific journal. Or submit them here.
The 78 W/m^2 for evaporation can be estimated pretty well from global precipitation estimates. I did it once — I challenge you to try it once.
If you just “feel” there is a problem, then I really have nothing I can say to you.
“A “photon-warmed” OCO molecule instantly rises…”
Why do you say this? Rising hot gas is a result of the bulk expansion of the gas making it less dense. A single hot molecule would know rise AFAIK.
Your question about changing lapse rate would be interesting to explore further. I don’t know if it has changed or how it should change with more CO2.
The “wet areas” on Earth NEVER see higher temperatures, because of the effects of water. See Willis Eschenbach’s articles on this. So the “average 78 joules for evaporation” meme is complete junk science.
I am completely confused by what you think is junk science here. Perhaps you could explain?
IF your analysis (theory) is correct, THEN we should be seeing SOME measurable effects of the additional OCO. We are NOT seeing ANY effects of which I’m aware.
Even most skeptical scientists seem to agree there is SOME sensitivity of temperature to CO2 and that at least SOME of the increase over the last century should be attributed to CO2. I am not in a position to know which estimates are most accurate, or how much of the warming would have occurred without CO2 increases.
See
http://wattsupwiththat.com/2011/04/26/the-climate-sensitivity-and-the-surface-temperature-record-question-answers-from-major-players/
for example for more info.

Joel Shore
May 13, 2011 8:54 pm

Alleyne: At this point, both Tim and I have patiently explained to you how the greenhouse effect does not violate the 2nd Law.
I’ll also point out that both of the major physics textbooks that we use in our introductory physics courses (one for the calculus-based course and one for the algebra-based course), popular texts used by hundreds…if not thousands…of colleges and universities to teach physics, contain discussions of the greenhouse effect and global warming. Don’t you think there would be an uproar in the physics community if the textbooks were teaching about things that violate the 2nd Law?
You have absolutely no credible shred of an argument left to believe that the greenhouse effect violates the 2nd Law. It is ludicrous to continue to make this argument…and, if you can’t understand why it doesn’t from our explanations, then you need to help us understand what your confusions are.

Phil.
May 13, 2011 8:55 pm

richard verney says:
May 12, 2011 at 8:39 pm
8. That Trenberth’s energy budget is wrong. It defies commonsence that DWLIR exceeds incoming solar energy given that the DWLIR can have been created only by the incoming solar energy (and energy radiated/released by the core which latter energy is said to be miniscule such that this can be ignored).

Well you know what they say about ‘commonsense’, it’s neither common nor sensible!
Try this:
You have a container into which water is flowing at 100l/min and flowing out through an overflow pipe, clearly outflow equals inflow at steady state. Now recycle 60% of that outflow back to the input, what’s the new outflow?
Answer: 250, so the recycle flow is greater than inflow, does this defy commonsense?

May 13, 2011 9:02 pm

Tim Folkerts says:
“Even most skeptical scientists seem to agree there is SOME sensitivity of temperature to CO2 and that at least SOME of the increase over the last century should be attributed to CO2. I am not in a position to know which estimates are most accurate, or how much of the warming would have occurred without CO2 increases.”
Thank you for that. It is also my own view. CO2 has caused some warming. But it is insignificant because it is too insignificant to measure. The null hypothesis of natural climate variability has never been falsified, therefore we must assume that what we are observing is mostly natural climate cycles.

Joel Shore
May 13, 2011 9:41 pm

Smokey says:

The null hypothesis of natural climate variability has never been falsified

That is because such a hypothesis is unfalsifiably vague. You guys often complain (without justification) that AGW is not falsifiable when in fact there are lots of tests that it has passed and lots more tests that it will continue to have to pass in the future. However, your null hypothesis really is unfalsifiable. If the global temperature shot up by 10 K in one year, you could claim it was just a natural climate cycle caused by some unspecified mechanism.

wayne
May 13, 2011 9:46 pm

jae,
I’m going to hand you the key if have the mind to see it and recognize it, I’ll call it wayne’s “Natural Geometric Greenhouse Effect”. It only strictly applies to atmospheres with thick enough GHG’s to make it opaque at relative short distances.
Take Tim’s numbers from KT97’s diagram, just the (78 + 24) + 67 to get 169, but one of those three numbers are off by one for the only possible number is 170 Wm-2, will explain the +1 later. But we see 235 being ejected at the TOA, we are missing 65 Wm-2.
Here’s the key to this entire discussion on GHE. Take the 65 * 6 (the six sides of a cube), that equals 390 Wm-2 or the maximum possible power that the surface can ever create since it is at 288 K. But radiation from the bottom of the one meter cube cannot pass net energy to the surface from whence it came, an object cannot be affected by it’s own radiation. So forget the bottom of the six sides of that cube, no net energy can pass backwards downward.
Now concentrate on the four sides of that cube. Any radiation horizontally is totally cancelled by the homogeneous nature of our atmosphere, for ever plus there is an equal minus nearby (see Miskolczi’s papers for a better worded description). So forget energy moving through the four sides of that imaginary cube.
This is very close to the same logic of three degrees of freedom described in the derivation of the ideal gas laws and how we came to know how pressure of a gas with those 3 degrees of freedom manifests itself as pressure.
So, you are left with a maximum of 65 Wm-2 that can move and it is upward through the top of the cube.
Playback: 170 + 65 = 235 Wm-2 seen at the TOA.
Also: 390 * 5/6 = 325 Wm-2, what KT97 is colloquial calling “back radiation”. (a misnomer and once again off by one)
This one sixth’s of energy that can use the upward degree of freedom is an absolute maximum, more flux can never occur upward. The (1 – 1/6) or 5/6 is the lower limit of what is colloquial call “back radiation” must always be equal or more than this limit.
That’s it, it is all simple geometry and I will wait no longer for someone to figure it out. I have tried to hint all of the necessary connections in the past month’s but have yet to find anyone who can seem to think out of the box. It is now recorded.
See? The 169 Wm-2 above violates that principle, it must be 170 and TFK09 has it corrected. Miskolczi passes with flying colors. Their numbers though hard to extract are very close to each other. No surprise, all of the base numbers come from measurements.
It seems any atmosphere, even if the energy transfer might be slow and by conduction and convection, follows this same principle. So even a pure nitrogen atmosphere would have it’s “greenhouse effect”.
Also curious is this seems to place a lower limit on the optical thickness of any planetary atmosphere and it is:
–ln(1/6) = 1.791759469…. .
There you go fellows in science, disprove that, I can’t seem to do it.
I know I don’t write word pretty, please excuse.
Can you and anyone else who can fathom this help me to get others to see the greenhouse IR light?

savethesharks
May 13, 2011 10:36 pm

Joel Shore says:
May 13, 2011 at 9:41 pm
That is because such a hypothesis is unfalsifiably vague. You guys often complain (without justification) that AGW is not falsifiable when in fact there are lots of tests that it has passed and lots more tests that it will continue to have to pass in the future. However, your null hypothesis really is unfalsifiable. If the global temperature shot up by 10 K in one year, you could claim it was just a natural climate cycle caused by some unspecified mechanism.
============================
That is an absolutely stupid and asinine analysis.
You are arguing from the negative…or, at worse, from the absurd….so what you are saying is flawed to the core.
Nobody…absolutely nobody…is claiming the absurdities that you claim (10K in one year)…and that absurdness speaks for itself.
Chris
Norfolk, VA, USA

savethesharks
May 13, 2011 10:47 pm

Joel Shore says:
May 13, 2011 at 8:54 pm
You have absolutely no credible shred of an argument left to believe that the greenhouse effect violates the 2nd Law. It is ludicrous to continue to make this argument…and, if you can’t understand why it doesn’t from our explanations, then you need to help us understand what your confusions are.
========================
Who is “OUR” in “our explanations”….and the “US” in “help us understand”?
Or you more than one person, Joel?
Or are you writing for a groupthink cluster? An amalgam? A super-individual?
Or are you using that tired and old scientific nomenclature of “we”?
Who is “we” and “us”?
Define it.
Chris
Norfolk, VA, USA

wayne
May 13, 2011 11:06 pm

Errata, to correct paragraph above:
It seems any atmosphere, even if the energy transfer might be slow and by conduction and convection, follows this same principle. So even a pure nitrogen atmosphere would have it’s “greenhouse effect”.
Sorry, convection is one of the other two energy transfers that is singled out in KT97 diagram and is added in separately (24 Wm-2).

Bryan
May 14, 2011 12:00 am

Dave Springer Says “G&T need to repeat the experiment with double-paned polyethylene”…….
You obviously did not read the second experiment which backs up the findings of RW Wood
Basically the project was to find if it made any sense to add Infra Red absorbers to polyethylene plastic for use in agricultural plastic greenhouses.
Polyethylene is IR transparent like the Rocksalt used in Woods Experiment.
The addition of IR absorbers to the plastic made it equivalent to “glass”
The results of the study show that( Page2 )
…”IR blocking films may occasionally raise night temperatures” (by less than 1.5C) “the trend does not seem to be consistent over time”
http://www.hort.cornell.edu/hightunnel/about/research/general/penn_state_plastic_study.pdf
You have also failed to come up with any evidence that radiative heating can produce significant effects in a volume the size of a large greenhouse
Now if you think that the “greenhouse theory” has any evidence that the radiative effects of CO2 and H2O can significantly heat a volume of air at STP please let us know.

Bryan
May 14, 2011 12:14 am

Dave Springer says
The RW Wood real greenhsoue experiment (glass vs. rock salt) is bogus. The glass panes heat up when they absorb IR and lose it right away through conduction. The G&T experiment repeats the mistake. A kilometer’s thick layer of greenhouse gases doesn’t conduct very well at all, unlike a thin pane of glass or polyethelene.
Dave try this on the next sunny day.
Go to your car, the dashboard might be so hot that it is too hot to touch.
Touch the windows and you will find them quite cool.
So much for IR absorbers causing a large temperature rise.

Bryan
May 14, 2011 12:20 am

For sceptics that think the IPCC “greenhouse effect” is bogus please note that none of the defenders has risen to the challenge of my yesterday post!!!!
Those advocates of the IPCC “greenhouse theory” should get real.
If the radiative effects of H2O and CO2 are supposed to “heat” the atmosphere to 33K higher than “it would otherwise be” then they need to show significant radiative heating in a volume of the atmosphere.
This they cannot do.
Instead they distort equations and say look “there’s the proof”.
If we went to the installer of our home central heating system with a complaint that it wasn’t working would we be satisfied with a couple of equations as an excuse.
Call the bluff of IPCC “greenhouse theory” advocates!
Ask for proof of radiative heating in a volume of the atmosphere.
They will be stumped!
The fact that the temperature profile of the troposphere can be derived from thermodynamics without reference to radiation sums it up.
A good place for the IPCC advocates to start would be the radiative heating of a space the size of a real greenhouse.
Lets get some real figures for say 30 cubic metres of air at STP on a KT average day.

Jim D
May 14, 2011 2:25 am

Bryan, greenhouse gases don’t radiatively heat the atmosphere. They cause the surface to warm by shielding it from cooling it would otherwise have. The surface then causes the atmosphere to warm through convection. This is not a difficult concept.
As far as CO2 having any effect, up to about 50 W/m2 out of that 324 W/m2 back radiation is from the presence of CO2 molecules. This is measurable via the IR spectrum.

Bryan
May 14, 2011 3:09 am

Jim D says “Bryan, greenhouse gases don’t radiatively heat the atmosphere. ”
I say …..That good we agree on that point.
This means my earlier post on R W Wood and the polytunnel experiments are vindicated
Jim D says “They cause the surface to warm by shielding it from cooling it would otherwise have.”
I say….In other words like the radiative component of insulation.
The Sun heats the surface should come next!
Jim D says “The surface then causes the atmosphere to warm through convection.”
I say …..Once a surface temperature is obtained the air next to the surface increases temperature by conduction and the laws of thermodynamics determine the temperature profile via the lapse rate.
The “greenhouse theory” seems to be missing or is it simply the radiative effect of CO2 and H2O in the IR.
I think there is not a lot of difference between our views.

Jim D
May 14, 2011 3:41 am

Bryan, it is an insulating or shielding effect reducing cooling. The surface with more CO2 and H2O above it is warmer than otherwise. Convection distributes this extra warmth to the atmosphere. If you agree with this mechanism, you are agreeing with the basic ideas needed for AGW.

Bryan
May 14, 2011 4:26 am

Jim D
There are many versions of the “greenhouse theory”.
A great number of them involve significant direct heating of the atmosphere.
My post was directed at the lack of evidence for such a view.
A much more sophisticated version of the “greenhouse theory” is mainly concerned with the radiation to space at the TOA.
The lapse rate going back from TOA to Earth surface gives the surface temperature.
This viewpoint is proposed by Leonard Weinstein and Nullius in Verba and seems quite plausable

May 14, 2011 5:06 am

Jim D says:
May 14, 2011 at 2:25 am
Bryan, greenhouse gases don’t radiatively heat the atmosphere. They cause the surface to warm by shielding it from cooling it would otherwise have. The surface then causes the atmosphere to warm through convection. This is not a difficult concept.
As far as CO2 having any effect, up to about 50 W/m2 out of that 324 W/m2 back radiation is from the presence of CO2 molecules. This is measurable via the IR spectrum.
All gases “shield” the surface from warming not just GHG’s. With 99 % N2 and O2 what part do you assign to them? Again with the back radiation. Show the temperture of the 324 W/m^2 using standard SB equation and the CO2 portion you attribute.

Jim D
May 14, 2011 5:15 am

The TOA is the other part of the theory. CO2 reduces outgoing radiation at the top, but as explained above, the warming required to compensate it occurs via the surface warming and resulting convection through the troposphere. (Other things happen in the stratosphere). If anyone is saying adding CO2 directly warms the air, they would be wrong. I don’t think you would find any proper science sites or articles saying that.

Tim Folkerts
May 14, 2011 5:16 am

Wayne says,
“Here’s the key to this entire discussion on GHE. Take the 65 * 6 (the six sides of a cube), that equals 390 Wm-2 or the maximum possible power that the surface can ever create since it is at 288 K. But radiation from the bottom of the one meter cube …”
A black body @ 288 K emits 390 W/m^2 from each square meter. The phrase “one meter cube” implies a cube 1m x 1m x 1m. That cube has an area of 6 m^2. It can emit 390 W/m^2 from EACH side, not a total of 390 from all sides.
Assuming the above interpretation of your set-up is correct, either you need to
1) not divide by 6 and just use 390 W/m^2 for each side.
2) think if your cube as having an area of 1/6 m^2 on each side (length = 0.41 m, volume = 0.068 m^3).
I haven’t gone any farther through your explanation, because this needs to be addressed before any of the further analysis would make sense.

May 14, 2011 5:39 am

Joel Shore says:
“If the global temperature shot up by 10 K in one year, you could claim it was just a natural climate cycle caused by some unspecified mechanism.”
That is exactly right, unless you can specify the mechanism using the scientific method. The temperature has risen more than that in the past, well prior to the industrial revolution. Explain that mechanism. You crave fame, don’t you? Here’s your chance.
No wonder Trenberth wants to replace the climate null hypothesis with his own cherry-picked version. The real null hypothesis falsifies his alternative CAGW hypothesis, and he knows it. So like all climate charlatans, he wants to turn the scientific method on its head, and replace it with mumbo-jumbo.

Joel Shore
May 14, 2011 5:45 am

Bryan says:

There are many versions of the “greenhouse theory”.
A great number of them involve significant direct heating of the atmosphere.
My post was directed at the lack of evidence for such a view.

There are not different versions of the theory. There are just different degrees of sophistication in discussing what goes on in the atmosphere. At a low level of sophistication, just radiative effects are considered. In such models, there is naturally a lapse rate due to the fact that the atmosphere is heated mainly from below (because most of the sunlight makes it down to the earth’s surface or pretty close to the surface).
At a higher level of sophistication, convective effects…which lead to the actual lapse rate observed in the troposphere are considered. This is because the role of convection is basically to set a stability limit on the lapse rate that is given by the appropriate adiabatic lapse rate: Lapse rates higher than the adiabatic lapse rate lead to convection, which lowers the lapse rate back down to the adiabatic lapse rate.

A much more sophisticated version of the “greenhouse theory” is mainly concerned with the radiation to space at the TOA.
The lapse rate going back from TOA to Earth surface gives the surface temperature.

Since you seem to be very concerned about using terminology correctly, I will point out that you are not using terminology very precisely here. Here is a more correct way to say things: When one considers convection, the best quantity to consider is the radiative balance at the top of the atmosphere. This is because you know that at the only heat flows between the atmosphere and space are radiative whereas within the atmosphere, especially the troposphere, there is a combination of different heat flow processes.
However, it is not really correct to say that the surface temperature is determined by the lapse rate from the top of the atmosphere. It is determined by the lapse rate from the effective radiating level, which is the level in the atmosphere from which, on average, the emitted radiation successfully escapes to space. By radiative balance, this level will have a temperature of ~255 K.

Joel Shore
May 14, 2011 5:55 am

Smokey says:

That is exactly right, unless you can specify the mechanism using the scientific method. The temperature has risen more than that in the past, well prior to the industrial revolution. Explain that mechanism. You crave fame, don’t you? Here’s your chance.

I thought falsifiability was supposed to be a good thing. I guess it is only a good thing for the AGW hypothesis? For the null hypothesis, apparently you are completely comfortable with having it be essentially impossible to falsify.
And, the mechanisms by which past climate change has occurred in the past are in fact in large part understood. Are they understood perfectly? No…but neither are all the steps in the evolution of humans understood perfectly.
You are essentially admitting what any intelligent person has known for a long time, which is that it is impossible to convince you in regards to AGW. Your belief in AGW not being important is simply a belief that follows from your political ideology since it would be very inconvenient for that ideology if it were important. You pretend that it is about science but really your scientific views are slave to your ideological views, which is of course why your views of the science differ so violently from the views of most scientists and all of the scientific societies. The existence of such societies and the willingness of governments to respect their advice on scientific matters has taken us from the Dark Ages to the age where science guides policy at least to some degree rather than ideology being the only policy guide.

Joel Shore
May 14, 2011 6:16 am

savethesharks says:

You are arguing from the negative…or, at worse, from the absurd….so what you are saying is flawed to the core.

And yet, Smokey has admitted that, yes, my “absurb” statement is correct in his mind.

Who is “we” and “us”?
Define it.

The people here who are trying to explain the actual science regarding the greenhouse effect. In addition to myself, they include Ira, Tim Folkerts, Jim D, Phil., Dave Springer, and David M Hoffer.

Bryan
May 14, 2011 6:24 am

Jim D said ……. “Bryan, it is an insulating or shielding effect reducing cooling. The surface with more CO2 and H2O above it is warmer than otherwise. Convection distributes this extra warmth to the atmosphere. If you agree with this mechanism, you are agreeing with the basic ideas needed for AGW.”……
There is an active debate about whether CO2 always tends to restrict the flow of heat or whether in some instances it could actually speed up the process.
The temperature record does not show a relentless rise with rising CO2 and there seems to be many other factors affecting global temperatures.

May 14, 2011 6:50 am

Joel Shore mentions “ideology” four more times in his post above. As Ronald Reagan said to Jimmy Carter, “There you go again.” Joel is fixated on his wacky world view that science is based on politics. In his grant-driven world, it probably is. Name-calling is all Joel Shore has left in his bag of tricks, because the planet isn’t cooperating with his failed predictions.
Ideology has nothing to do with the facts – which is why Joel Shore falls back on his endlessly repeated ‘ideology’ canard. The plain fact is that the current *mild* warming cycle over the past century and a half has only amounted to about 0.7°C. Even if all of that warming was attributable to AGW, it would not justify taking any action, particularly when the world’s biggest CO2 emitter won’t play the game. And of course, no credible authority claims that all of the rise is due to AGW. In fact, there is no evidence that any of that rise is due to AGW.
Natural variability is a perfectly sufficient explanation of what we observe. There is no evidence that the very mild rise in temperature is anything other than the planet emerging from the LIA. CO2 is harmless and beneficial; falsify that, if you can.
Joel Shore is always trying to re-frame the skeptical position, which in general is that there may be some harmless, beneficial and minor warming due to AGW, but that there is no evidence whatever supporting his belief in catastrophic AGW. And without wild-eyed claims of catastrophe, the public loses interest.
So without any evidence of CAGW, Joel Shore resorts to labeling skeptics’ questions – and skeptics’ demands that the scientific method must be followed – as “ideology.” That is pure psychological projection; it is Joel Shore who falls back on the failed alarmist CAGW ideology, which does not have a shred of evidence supporting it. Wake me when there is evidence of global damage due to CO2. Until then, it’s all impotent arm-waving.

Jim D
May 14, 2011 7:06 am

mkelly, as I mentioned, CO2 contributes up to 50 W/m2 of the 324. O2 and N2 contribute approximately zero, O3 and other trace gases contribute some more, clouds contribute too, but mostly it is H2O vapor.

Dave Springer
May 14, 2011 7:08 am

Jim D says:
May 14, 2011 at 5:15 am
“The TOA is the other part of the theory. CO2 reduces outgoing radiation at the top, but as explained above, the warming required to compensate it occurs via the surface warming and resulting convection through the troposphere. (Other things happen in the stratosphere). If anyone is saying adding CO2 directly warms the air, they would be wrong. I don’t think you would find any proper science sites or articles saying that.”
Technically true in the same sense that putting on an extra layer of clothes doesn’t directly warm the clothes. CO2 works as an insulator. It slows down the rate of cooling of the water beneath it but the practical effect is that the air is warmer than it would have been otherwise so it’s kind of splitting hairs to say it doesn’t directly warm the air.

richard verney
May 14, 2011 7:09 am

Joel regarding your post at Joel Shore says:May 13, 2011 at 7:14 pm
I am not one for criticising spelling or sloppy languague since I recognisse that people posting often post late at night, type quickly and inevitably do not give the same care and attention to posting that they would to their day job. Likewise, I do not comment on emotive language since this merely distracts and adds nothing to the debate. In fact, save for having once commented to the effect that commentators should not be so discourteous to authors who go to the trouble of writing and posting articles on this site, this is the first time I have commented on language.
When a person starts inferring that another person has ideological beliefs that are clouding the judgment of that other, this is usually an indicator that the person making the inference is themselves clouded by their own ideology.
It would appear that you yourself are the one with ideological beliefs on this issue as is apparent from your describing my comment as pernicious. My comment may be incorrect. It may even be nonsence, but it can only be perceived as pernicious if it offends your own ideology. My comment is not ruinous, hurtful, evil, wicked, one that causes insidious harm etc and could only be viewed as such by someone whose own objectivity has been distorted by ideological beliefs such that they view comments not fitting in with their perception of events as hurtful and wicked.
I can assure you that I have no ideological views when it comes to the science behind the ‘theory’ of CAGW, My only concern (goal) is the search for the truth and to gain a better understanding of how the world in which we live in works.
I would suggest that you stick to the science.

Jim D
May 14, 2011 7:13 am

Bryan, well aerosols and cloud albedo may be having an effect too, but the effects of CO2 aren’t really being debated because they are understood very well. At the top of the atmosphere it accelerates atmospheric heat loss to space, and at the bottom it inhibits cooling from the ground to space. This happens by enhancing IR radiation both up and down, just due to its presence, not any extra warmth.

May 14, 2011 7:22 am

Joel Shore says:
“…Smokey has admitted that, yes, my “absurb” statement is correct in his mind.”
Not sure what that refers to, but since Joel Shore has raised the issue, I note that he previoulsy admitted to some confusion regarding the definition of heat. For someone who presumes himself to be the authority explaining the ‘actual science’ to the ignorant hoi polloi here, it’s amusing that Joel Shore was confounded regarding such a basic definition. Feet of clay, eh?
Joel Shore should also learn the definition of evidence. Because there is no evidence of AGW, is there?

ferd berple
May 14, 2011 7:37 am

How can CO2 over come the lapse rate? It can’t except by heating the surface, leading to increased convection, leading to a warming of the atmosphere, leading to an expansion of the atmosphere, similar to the expansion of the oceans. You need a taller column of air to maintain the lapse rate if you warm the surface.
Is this what is observed?
Earth’s Shrinking Atmosphere Baffles Scientists
http://news.discovery.com/earth/earth-atmosphere-shrinking.html

Jim D
May 14, 2011 7:40 am

I want to modify my 7:13am statement. CO2 does enhance downward radiation at the surface, but its effect at the top is to reduce upward radiation, because it radiates to space from higher generally colder layers, being more efficient as an emitter. There is a part of the spectrum that emits from the stratosphere, where higher is warmer, and there CO2 enhances the outgoing radiation (eventually causing a cooling effect).

Joel Shore
May 14, 2011 7:42 am

richard verney says:

It would appear that you yourself are the one with ideological beliefs on this issue as is apperent from your describing my comment as pernicious.

Perhaps “pernicious” is not the best choice of words, but the point is that you view that GHGs merely cause a slight delay is just not the correct scientific way of looking at things. It encourages an incorrect view of the science and, for those of us who are scientists and like to see science correctly understood, that’s a bad thing.
Smokey says:

That is pure psychological projection; it is Joel Shore who falls back on the failed alarmist CAGW ideology, which does not have a shred of evidence supporting it. Wake me when there is evidence of global damage due to CO2. Until then, it’s all impotent arm-waving….

What you fail to understand again and again is that you are not the judge and jury in regards to scientific evidence and the scientific method. You are certainly entitled to your opinion but why should anybody respect your opinion over the opinion of…say…real scientists?
That your statements are basically identical to the sorts of statements made by those who argue against evolution is not a coincidence. The arguments made by those who attack science are almost always very similar.
You have illustrated on this site again and again that you are unable to evaluate the scientific evidence. Right here in this thread, we have an example of the extremely low standards that you set for yourself and your refusal to engage in substantive discussions: You have been shown that a statement of mine that you have attacked for months is actually weaker than a similar statement made by Richard Lindzen, one of the few respectable atmospheric scientists who agrees with you on AGW. However, just you refuse to address this. You have shown by your actions here and in other threads that you are not someone whose views should be taken at all seriously and who people should waste their time trying to discuss science with. A discussion necessarily involves good faith and high ethical standards on both sides.

Joel Shore
May 14, 2011 7:45 am

ferd berple says:

How can CO2 over come the lapse rate? It can’t except by heating the surface, leading to increased convection, leading to a warming of the atmosphere, leading to an expansion of the atmosphere, similar to the expansion of the oceans. You need a taller column of air to maintain the lapse rate if you warm the surface.

For the 120th time (okay, maybe it’s only the 19th), the temperature at the earth’s surface is determined by the lapse rate and the level in the atmosphere at which the temperature is constrained, which is the effective radiating level, i.e., the average level from which radiation can successfully escape to space. Adding CO2 increases that level; hence the level in the atmosphere at which the temperature must be 255 K is now higher, meaning that the temperature at the surface is now higher.

Dave Springer
May 14, 2011 7:47 am

Bryan says:
May 14, 2011 at 12:20 am
“If the radiative effects of H2O and CO2 are supposed to “heat” the atmosphere to 33K higher than “it would otherwise be” then they need to show significant radiative heating in a volume of the atmosphere.”
That’s a straw man, Bryan. The sun heats the ocean. The ocean heats the air. Greenhouse gases slow down the rate at which daytime warming can escape from the ocean at night.
And by the way the 33C (or 33K if you prefer) number is completely bogus because the average temperature of the ocean is a mere 4C which is a true reflection of the average surface temperature averaged over a complete glacial cycle which is long enough for the deep water to equilibrate with the surface water. So for the past 3 million years the average temperature of the earth hasn’t been the warmish 15C of the past 10,000 years of interglacial period but rather the brutal cold 3C of the glacial periods which last ten times as long as the interglacials.
Positive feedback from snow & ice, which raises albedo and accelerates the cooling to produce even more snow & ice, is what drives the big climate shifts of the past several million years. The only “tipping point” the planet is on is tipping back into a full blown glacial period that will cover everything north of Washington, D.C. with a mile-thick sheet of ice. It would be great if anthropogenic CO2 could end the ice age but I sincerely doubt whether we can pump it up enough to warm that huge bucket of nearly freezing water called the global ocean. It’s too late now. Maybe if we’d been doing it beginning 5000 years ago when interglacial period was only half over but now it’s near the end and anything we can do is too little, too late. It doesn’t even appear to be enough to raise the temperature of the shallow surface layer by more than a fraction of a degree to say nothing of imparting any significant warmth to the other 90% of the volume of the global ocean below the thermocline (400+ meters deep).
So for the past few million years at least the greenhouse effect has only been good for an average of about 23C not 33C.
We need all the atmospheric CO2 we can get for a number of very good reasons and there’s no real reason at all why we’d want less of it. There are good reasons to slow down the rate of fossil fuel consumption but those all have to do with conservation of a finite resource and not a damn thing to do with global warming because global warming and more CO2 is a hugely positive thing for the primary producers in food chain – green plants.

May 14, 2011 8:35 am

Joel Shore says:
“A discussion necessarily involves good faith and high ethical standards…”
Let’s discuss that. I’d like to know how it fits in with “high ethical standards” that Joel Shore can post long comments here and on other blogs so often throughout the workday? My apologies in advance if Joel is self-employed, or retired – or if posting on assorted blogs is in his official job description. But if Joel Shore is receiving any part of his pay from the public, which I suspect he is, then his claims of “high ethical standards” are risible.
And of course we hear the sounds of crickets chirping whenever I ask for evidence of global damage from CO2. Instead, we get misdirection about “ideology”, and comparing someone with a different point of view to “those who argue against evolution”. That sounds pretty desperate, my friend. I’d be more interested in seeing evidence of global harm from CO2. But since there is no such evidence, is labeling people anti-evolution, and calling skeptics ideologues your fallback position?

Bryan
May 14, 2011 8:35 am

To all
I was trying to catch up with previous posts when I came to a familiar refrain from Joel Shore
…….”Your belief in AGW not being important is simply a belief that follows from your political ideology since it would be very inconvenient for that ideology if it were important. You pretend that it is about science but really your scientific views are slave to your ideological views, which is of course why your views of the science differ so violently from the views of most scientists and all of the scientific societies. The existence of such societies and the willingness of governments to respect their advice on scientific matters has taken us from the Dark Ages to the age where science guides policy at least to some degree rather than ideology being the only policy guide.”….
When Joel loses the argument on science he attacks the political and religious opinions of other posters.
I for one am getting heartily sick of Joel’s guessing game as to the motives of other posters.
Also some posters are ordinary members of the public who have a concern about the way the IPCC operates. Often they make insightful comments missed by the scientists among us. I deplore any language used to demean or ridicule non specialist posters.

Bryan
May 14, 2011 8:57 am

Dave Springer says…..”So for the past few million years at least the greenhouse effect has only been good for an average of about 23C not 33C.”
Never mind about the last million years do you agree with my post today about the experiment carried out by R W Wood and the polytunnel study.
Jim D seems to agree and I hate to go on about it, but it is rather important.
Is there a significant direct radiative heating of the atmosphere by CO2 and H2O?
It seems to me that the only significant role that the greenhouse gases perform is to perhaps insulate the Earth Surface and to radiate to space at their effective radiating level.

richard verney
May 14, 2011 9:04 am

Tim and Joel
Energy is the ability to do work and in this regard not all energy is equal.
Tim thinks that you can simply deduct the 333 W/m^2 back radiation from the 396 W/m^2 surface radiation thereby yielding a net emission from the surface of 66 W/m^2. However, question marks arise as to whether this photonic energy is equal and can accordingly simply be deducted as suggested since the 396 W/m^2 has the ability to do work capable of heating an object to about 288K whereas the back radiated 333 W/m^2 is only capable of heating an object to about 240K. Since they do not have equal ability to do work, is it correct that one can simply deduct one from the other?
As mentioned by me in an earlier post, I am surprised that any one has particularly bullish views since there is a lack of empirical data and evidence showing how things work. So what empirical data and evidence exists showing (i) that the 333 W/m^2 of back radiation is actually absorbed by the surface; and (ii) and if so absorbed, what happens to the photonic energy so absorbed? I would like to see the empirical data and evidence. Please provide any such evidence that you may have.
In making this request, I am not suggesting that objects above absolute zero do not radiate inaccordance with their temperature, but it does not follow from that that the radiation so admitted is absorbed by something else.
It may be that radiation eminating from a cooler object can come up to the ‘door’ of a warmer object but cannot pass the threshold and is therefore not absorbed by the warmer object. It may be that the photons being radiated from the warmer object effectively block the path of those eminating from the cooler object, possibly even extinguishing the cooler photons as the warmer photons make there way towards the cooler object (I understand that you do not dispute that net flow is always from warmer to cooler). I am not saying that this is what happens but rather what may happen.
Thus in summary, lets see what evidence you have suggesting that cooler photons are actually absorbed by a warmer object. I will then consider that evidence and see whether I need to revise my views.
If there is no such evidence then it is about time that some physical experiments are conducted to get some real empirical data and evidence. There is enough money being poured into this area of science to be able to fund some practical and/or field experiments looking at some of the contenious issues raised, or is is it beyond the ingenuity of man to mimic and examine these contentious issues by way of practical experiment (not computer modelling)?

Joel Shore
May 14, 2011 9:40 am

Smokey says:

Let’s discuss that. I’d like to know how it fits in with “high ethical standards” that Joel Shore can post long comments here and on other blogs so often throughout the workday? My apologies in advance if Joel is self-employed, or retired – or if posting on assorted blogs is in his official job description. But if Joel Shore is receiving any part of his pay from the public, which I suspect he is, then his claims of “high ethical standards” are risible.

My current job is that I work for a private university. My job basically entails just teaching and they are not worried about exactly what my hours are but whether or not I do a satisfactory job. In fact, I often do lots of work on evenings and weekend. In the fall quarter, when I had a particularly challenging schedule, I was working close to 70 hours per week (which is why I was posting a lot less back then) in addition to the considerable amount of prep time I had put in over the summer.
I think you would do best to focus on your own ethical standards. I have personally never had the pleasure of meeting anybody who had such high standards for others and such abysmally low standards for himself. You have actually made this into a warped philosophy whereby AGW skeptics are not expected to have any accountability for their falsehoods (whether they be incorrect statements or deceptions) whereas all of the responsibility and accountability falls onto those on the other side. Frankly, I would be way too embarrassed to ever suggest the sort of double standard that you suggest. I am surprised that your fellow skeptics here don’t express more embarrassment at such nonsense.

I’d be more interested in seeing evidence of global harm from CO2.

Been there…Done that. And, it’s like talking to a brick wall. Besides which, you are capable of reading: These issues are all well-summarized in the IPCC reports, reports from the National Research Council (e.g., http://wattsupwiththat.com/2011/05/12/weve-heard-this-before/ ) and many other places.
This current thread is about the greenhouse effect.

jae
May 14, 2011 9:53 am

wayne says:
May 13, 2011 at 9:46 pm
Most fascinating and intriguing! So Ferenc is still in the game, and my SUV may not be guilty of anything, after all? How come we don’t hear from Ferenc anymore?
This certainly fits my observations that the temperature at various places on the Planet don’t seem to have anything to do with the quantity of GHGs in the atmosphere above those places, even when altitude and latitude are the same.
However, I have to eat crow on some of K&T’s data (but that’s OK, since I eat it regularly).
What say ye to this CAGWers? Don’t be shy.

Joel Shore
May 14, 2011 9:55 am

richard verney:

In making this request, I am not suggesting that objects above absolute zero do not radiate inaccordance with their temperature, but it does not follow from that that the radiation so admitted is absorbed by something else.

The earth is almost a perfect blackbody emitter in the mid & far-IR and since Kirchkoff’s Law imply that the emissivity and absorptivity must be equal at each wavelength, the means that essentially all of the radiation that is incident on these objects is absorbed.

It may be that radiation eminating from a cooler object can come up to the ‘door’ of a warmer object but cannot pass the threshold and is therefore not absorbed by the warmer object. It may be that the photons being radiated from the warmer object effectively block the path of those eminating from the cooler object, possibly even extinguishing the cooler photons as the warmer photons make there way towards the cooler object (I understand that you do not dispute that net flow is always from warmer to cooler). I am not saying that this is what happens but rather what may happen.

It may be that the moon is made out of green cheese. It may be that the sky is pink with purple polka dots when we are not looking at it and only turns blue when we look. It is not up to us to disprove every nonsense idea that you come up with. How radiative transfer works is well-understood and if you think it works differently, then the onus is on you to show that your new theories are in better agreement with empirical data than the accepted theories. If you want to believe nonsense badly enough, then there is little that we can do to stop you. But, don’t fool yourself into thinking you are somehow being scientific.

There is enough money being poured into this area of science to be able to fund some practical and/or field experiments looking at some of the contenious issues raised, or is is it beyond the ingenuity of man to mimic and examine these contentious issues by way of practical experiment (not computer modelling)?

These are not contentious issues any more than it is contentious whether or not the earth is more than 6000 years old. The whole field of remote sensing is based upon the current understood radiative transfer equations, as are calculations done by scientists and engineers worldwide every day.
Why should scientists waste their time responding to people who are just completely ignorant of the science? You should appreciate the fact that some of us do invest the time and energy to try to help you overcome your own ignorance despite the fact that we have to fight against your apparent desire to remain ignorant every step of the way.

May 14, 2011 10:07 am

Joel Shore,
The second comment under the link you posted says it all. There is still no evidence showing global damage due to CO2. None. If that link is the best you can do, your CO2=CAGW conjecture fails. “Been there, done that” is nothing but bluster.
Ball’s back in your court now: produce empirical, testable evidence, per the scientific method, showing global harm due to CO2, or admit that CO2 is harmless.

jae
May 14, 2011 10:45 am

“Ball’s back in your court now: produce empirical, testable evidence, per the scientific method, showing global harm due to CO2, or admit that CO2 is harmless.”
If any of the religious-zealot scaremongers could do that, the facts would be plainly clear and stand on their own, and the “experts” wouldn’t be developing “Information Task Forces” and other propaganda gimmicks to convince the majority of people (yes, the majority) that the theory is correct, that “climate change” is killing us all, that we can do something about it, but, alas the majority of us are just too dumb to understand it.
Anyone still citing the IPCC is desperate, IMHO. What really sickens me is that the politics has entered the administrations of the scientific societies, because of all the grant money that would disappear if the truth about the AGW scam (and there is now absolutely no doubt about it being a sick scam) becomes well-enough known (that point seems near; hence all the yelling). I quit the American Chemical Society because of this crap; but it keeps sending me applications; I keep telling them again why I quit; but I get no response, just another application. Sorta sounds like the government, doesn’t it?
Don’t worry, I think the “environmentalists” shot themselves in both feet this time.

Bryan
May 14, 2011 11:18 am

Joel Shore says
…”Why should scientists waste their time responding to people who are just completely ignorant of the science? You should appreciate the fact that some of us do invest the time and energy to try to help you overcome your own ignorance despite the fact that we have to fight against your apparent desire to remain ignorant every step of the way.”…
Perhaps Joel because it gives you the chance to insult people!

sky
May 14, 2011 2:08 pm

richard verney says:
May 14, 2011 at 9:04 am
Backradiation is best understood as a component of an EXCHANGE of thermal energy; it’s NOT a PRODUCER of it. LWIR already IS thermal energy. The net energy FLUX resulting from that exchange is INVIOLABLY from warmer to cooler object, i.e., from surface to atmosphere on climatic time-scales. The rate of flux is solely a function of the temperature difference. Field experiments show that the net flux is very small in comparison to the transport of LATENT heat by moist convection. Even on a dry, cold planet with a CO2 atmosphere, such as Mars, convection remains the dominant factor in heat transport aloft. This is a consequence of the fact that radiative equilibrium in a gravity-bound atmosphere results in a top-heavy hydrostatic instability, as rigorously shown from first principles by the eminent Swiss astrophysicist/meteorologist Emden nearly a century ago. The parallel path of latent heat transport spares us from the violent hemispheric dust storms that sporadically envelop Mars, but not from tropical cyclones. It is only high aloft that radiative transport becomes dominant.
In it’s myopic preoccupation with CO2, climate “science” largely ignores the foregoing facts. It stands entirely apart from all other branches in attempting to do thermodynamics on the basis of a simplistic radiative algebra that does not even recognize the significance of the density of the atmosphere in setting the surface temperature. And it it forgets that the Stefan-Boltzman equation applies only to a black body in equilibrium WITHOUT an atmosphere in into which thermal energy can be convected.

richard verney
May 14, 2011 2:57 pm

Joel
I see that you have run out of science. Your response to my enquiry is disappointing to put it mildly.
I readily ackowledge that I am skeptical of the the entire AGW theory (for many reasons some of which are outside the ambit of this article). I consider that any genuine scientist would be (after all that is the scientific approach to any theory). In holding this view, I am quite prepared to acknowledge that the GHG ‘theory’ is superficially attractive. I understand the superficial attractiveness behind the proposition that the atmoshpere contains some gases that are largely transparent to incoming solar radiation and therefore the majority of this solar radiation finds its way through the atmosphere to the surface whereupon it heats the surface and this heat is, inter alia, radiated from the surface at a different wavelength at which wavelength the atmosphere (or some gases within the atmosphere) is not transparent such that some of this radiated enerrgy is ‘trapped’ thereby effectively warming the planet. Such a proposition is not difficult to understand and has a certain superficial attractiveness. I do not find the CAGW argument to be superficially attractive given that we are here some 4 1/2 billion years after the planet was formed during which time there have been significant climatic changes (far exceeeding those presently happening) and in particular times when CO2 has been about 1000 times higher without catastrophic effect. This suggest that it is extremely unlikely that there are tipping points such that I find the CAGW argument to be superficially unattractive.
Of course, whether an argument is superficially attractive or unattractive does not mean that it is right or wrong merely that it has a simplistic attraction or not.
There are many factors that go to the heart of the global warming theory. One of these is whether radiation emitted by a colder object is actually absorbed by a warmer object (and thereby increasing the radiation that the warmer object thereby emits).
The relevant equations have (several times) been set out above but they do not answer the question whether radiation only flows from hot to cold, or whether it is a two way street but with net flow from hot to cold. In particular, they do not answer the question whether a hotter object actually absorbs radiation emitted by a colder object and if so what effect this has on the hotter object and what becomes of the radiation so absorbed by the hotter object.
If radiation emitted from a colder object is actually absorbed by the hotter obeject then some fingerprint (signal) of this should be discernible. It seems to me that some physical expirementation should be carried out with a view to finding this fingerprint (signal) to try and ascertain whether it exists or not.
Hopefully some grown up scientists will not simply assert that “It may be that the moon is made out of green cheese. It may be that the sky is pink with purple polka dots when we are not looking at it and only turns blue when we look.” but will instead conduct some real science so as to try and answer these unknown and unanswered scientific questions.
Rhetorically you ask: “Why should scientists waste their time responding to people who are just completely ignorant of the science?” I would say that if their response is along the lines of your post (Joel Shore says:May 14, 2011 at 9:55 am) not only should you not wast your time but you should not waste my time engaged in reading such a childish rant.
Lets see the science. Your latest repsonse does not do your cause any good still less does it take the dabate forward.

May 14, 2011 3:08 pm

Joel Shore says:
May 13, 2011 at 12:20 pm
“the greenhouse effect reduces the rate of energy loss out into space (for a fixed surface temperature), requiring a higher average surface temperature to restore radiative balance.”
So where does the energy to get this higher surface temperature come from?

richard verney
May 14, 2011 3:31 pm

Brian
I fully endorse the sentiments expressed in your comment “Bryan says:
May 14, 2011 at 8:35 am”
Not only are the posters (or vast majority of them) on this site entirely genuine, they are the people who are expected to make the (immense) sacrifices and dig (deeply) into thier pockets should political governace force through a policy of (attempted) mitigation to the (percieved) threat of manmade global warming such that it is necessary that the scientific community carries these people with them on the journey to the sacrificial altar.
The recent comments by Joel show why the public is tiring of this policy agenda and why they are no longer prepared to take this dogmatic mantra at face value.

wayne
May 14, 2011 3:57 pm

jae says:
May 14, 2011 at 9:53 am
Most fascinating and intriguing! So Ferenc is still in the game, and my SUV may not be guilty of anything, after all? How come we don’t hear from Ferenc anymore?
This certainly fits my observations that the temperature at various places on the Planet don’t seem to have anything to do with the quantity of GHGs in the atmosphere above those places, even when altitude and latitude are the same.

Isn’t that? Came to me one night while slipping between dreams, then it hit me. Got up, multiplied 390 by 5/6 and said, of course, the rest just started to fall in place. Now that finally makes some scientific sense to me. It took me close to seventeen months of sifting through the physics and it ends up being mainly simple geometry. Love that science, but it makes me feel a bit dense at the same time, why so long for such a simple principle. How could I overlook it for so long? Well, people like Tim, Joel, and I.P.C.C. is why, filling minds, like mine, with warped science logic in areas that really don’t matter.
Really, thanks for the reply! I wasn’t sure if anyone would really read this comment to you, and really “get” that approach, I though I thought that you would bee able if I could have you stop enough to grasp it. I’m like you, CO2, especially at these concentrations makes no scientific sense, even as hard as AGW based science tries, and I have yet for anyone as such to show one instance that it is the cause. Trouble is, what to do with such a thought out of the blue. Feel like writing a scientific paper but have little chance of being published. I’m in the middle of extending that to the other atmospheres, Venus, Titan, Mars, to see if it properly holds in all aspects and it does seem to hold so far. With this you can even extract the tau (optical thickness) of one bar pure CO2 from Venus’s atmospheric profile, super neat. As soon as I triple check the numbers I will post them here.
See where I am heading? If you can extract from Venus the influence of CO2 across 92 levels of one bar deep shells, logarithmic of course, then that same figure can be diluted to tell pretty close to how much CO2 influences our atmosphere as a whole with diluted CO2. My numbers so far show very small indeed, nearly, but not zero. The remaining influence must come from water vapor and I am working on that and all we are really speaking of is less that 6 Wm-2 even if our atmosphere were totally comprised of GHGs themselves, as Venus. That is what seems to be coming out of this, but I do want this to follow proper physics and radiative transfer keeping out of areas which do not really even apply if you can approach it correct. I have never thought of something so drastically new in science even after forty years of constant following and learning. I keep saying someone surely has already thought of this but I find it nowhere. Have you?
Seemed you might be able to help for if you can follow the areas where you have been arguing with Joel, you know enough to see this. So I chose you. Willing to follow on as I get deeper? (didn’t want to write 50 paragraphs if no one was even listening or cared)

richard verney
May 14, 2011 3:59 pm

Sky
Whilst I stand to be corrected, (save that I have some reservation with exchange) my perception of matters is much along the lines of your post at May 14, 2011 at 2:08 pm

wayne
May 14, 2011 4:27 pm

Ira…
If you cannot state clear objections, I believe you must accept that about 320 Watts/m^2 is being ouput from the Atmosphere and being absorbed by the Surface. QED. Any objection to that?

Absolutely object unless you also say that any radiation accepted by the surface (left T in SB) from the atmosphere itself (right T in SB) originally came from the surface and has absolutely ZERO effect on any temperature, surface or atmosphere. SB handles it perfectly just from the two T’s. You seem to keep taking the right temperature and saying it’s possible power is special, it matters even though the temperatures do not change at all, sorry Ira, right there you are wrong. Don’t feel alone, seems David, Joel, and Tim, maybe Phil., seem to agree with you.
The two temperatures plus an optional emissivity properly handles the energy power at that interface just fine. Net energy never flows from cool to warm. You can imagine gazillion photons going back and forth with no change in either temperature, fine, I can imagine that too, it may even be real, by quantum amplitudes and probabilities, it may really not, but none of that really matters in this entire discussion in these five posts. If you continue to say it matters then I will continue to say I object, only the temperatures matter in a macro environment and our climate system is a macro system.

May 14, 2011 5:03 pm

“Joel Shore said on Visualizing the “Greenhouse Effect” – Light and Heat
May 13, 2011 at 6:52 pm
However, the point I am making is that this does not mean that colder objects don’t radiate toward warmer objects. It only means that the colder object will always absorb more radiant energy from the warmer object than warmer object absorbs from the colder. That is what the real 2nd Law says.”
Alleyne replies:
Perhaps it would be useful if I set out the definitions of the terms I am using, and how I understand them
Temperature is the proxy for the energy state of an object.
The energy state of an object is defined by the Boltzmann distribution of its molecules.
Heat is the manifestation of the transfer of energy from an object at a higher energy state to one at a lower energy state.
Spontaneous means not needing to be driven by work of some kind (Water flows downhill spontaneously)
Heat/energy always travels from a higher temperature/energy state to a lower one.
Heat/Energy can not be transfered from a colder/lower energy state to a warmer/higher energy state without the application of external work/energy (Clausius’s Law) (Water won’t flow uphill without being pumped)
This is where I am having the difficulty with what you and Tim are saying.  I have no problem with the energy traveling from the surface (warmer) to the atmosphere (colder).  What I am having a hard time accepting is the energy being transferred from the atmosphere (colder) to the surface (warmer).
Based on your theory, we could build and engine with the surface as a hot source and the atmosphere as a cold sink, and run the engine to produce some work.  In the process we discard some of heat/energy into the cold sink, as is required by Clausius’s Statement and the 2nd law. Now the clever part of the design is that we direct all but 1/10,000th of a percent of the energy radiated from the cold sink (atmosphere) back to the hot source (surface), thereby returning all but 1/10,000th of a percent of the energy we discarded into the cold sink.  This respects the second law, according to you, because the NET entropy increases.
It also very significantly increases the efficieny of our engine.
However it doesn’t appear to respect Carnott’s Law.
I agree that a colder object can radiate towards a warmer object.  I agree that a colder object will warm up due to the radiated energy from a warmer object.
However, if a warm object absorbs radiant energy from a colder object, this implies that the lower energy state is somehow able to raise the higher energy state to an even higher energy state.  This is the part I am having difficulty with…
“Joel Shore said on Visualizing the “Greenhouse Effect” – Light and Heat
May 13, 2011 at 6:52 pm
“It is just that (for a given earth surface temperature) the heat flow away from the earth is less than it would be if the (IR-absorbing) atmosphere were not present. As a result, the earth’s surface temperature must increase until it reaches a point where the earth system (earth + atmosphere) is radiating back into space as much energy as it receives from the sun.”
Alleyne replies:
If on the other hand you mean that the GHGs act like insulation which slows the heat loss from the surface, I would have to think about that, but can’t at this point disagree that it is possible.  However the backradiation “heating” the surface is not necessary in this scenario.
“Joel Shore said on Visualizing the “Greenhouse Effect” – Light and Heat
May 13, 2011 at 6:52 pm
Read this 20 times until you understand it so that you stop attacking “strawman” arguments and address what we are actually saying.”
Alleyne replies:
I am not attacking anything Joel, I am merely asking questions and making statements which, hopefully, will elicit informative responses so that I may understand your theory.
“Joel Shore said on Visualizing the “Greenhouse Effect” – Light and Heat
May 13, 2011 at 6:52 pm
But…you can’t define your spontaneous event as being just one part of the process. That won’t happen spontaneously. The point of the 2nd Law is that you can’t have the radiation from the colder object toward the warmer occurring without having the warmer object radiating toward the colder object too. Again, you seem to believe in the 2nd Law as some form of magic, rather than understanding the modern basis of it as following from statistical physics.”
Alleyne replies:
I was using spontaneous in the thermodynamic sense, as defined above, and because energy flows from hot to cold, the flow of energy from the surface to the atmosphere is spontaneous, whereas the energy flow from the atmosphere (ability of the atmosphere to heat the surface) is not spontaneous in the thermodynamic sense
“Joel Shore said on Visualizing the “Greenhouse Effect” – Light and Heat
May 13, 2011 at 8:54 pm
Alleyne: At this point, both Tim and I have patiently explained to you how the greenhouse effect does not violate the 2nd Law.
I’ll also point out that both of the major physics textbooks that we use in our introductory physics courses (one for the calculus-based course and one for the algebra-based course), popular texts used by hundreds…if not thousands…of colleges and universities to teach physics, contain discussions of the greenhouse effect and global warming. Don’t you think there would be an uproar in the physics community if the textbooks were teaching about things that violate the 2nd Law?
You have absolutely no credible shred of an argument left to believe that the greenhouse effect violates the 2nd Law. It is ludicrous to continue to make this argument…and, if you can’t understand why it doesn’t from our explanations, then you need to help us understand what your confusions are.”
Alleyne replies:
Thank you Joel, I am trying to help you understand my confusion, as you put it.  As to the textbooks, there has been a lot of foolish stuff published in textbooks and the scientific concensus has been wrong more often than it has been correct – however that is not really germane to this discussion.
“Tim Folkerts said on Visualizing the “Greenhouse Effect” – Light and Heat
May 13, 2011 at 6:11 pm
For radiation between surface & atmosphere.
Looking specifically at the exchange between surface and atmosphere, the radiation rates are 350 W/m^2 up from surface to atmosphere and 324 W/m^2 down from atmosphere to surface, for a net 26 W/m^2 upward.
For the surface,
dS = – 26 J/ 285 K = – 0.091 J/K
The entropy of the atmosphere increases by
dS = +26 / 250 K = + 0.104 J/K
NET CHANGE from the process is +0.312 J/K – 0.274 J/K
= + 0.0.013 J/K
This is positive, as required.
(You could also use the 350 J & 324 J numbers directly, but that will give the same results).
I’ve actually given numbers to show entropy always increases as required. What specific example of a process with a net entropy decrease were you thinking of?”
Alleyne replies:
Thanks Tim for the numbers.  Perhaps it is simplistic of me, or a further misunderstanding on my part, however I thought the formula for the change in Entropy was
dS= dCold – dHot
In other words the change in entropy was equal to the change of entropy in the cold sink less the change in entropy of the warm sink, therefore, using 356W/m^2 and 288K for the surface and 333W/m^2 & 253K for the atmosphere I got:
For 356J flux from the surface to the atmosphere:
decrease in entropy for the surface:      -356J/288K = -1.236J/K
increase in entropy for the atmosphere:    356J/253K = 1.407J/K
dS for the surface to Atmosphere: 1.407J/K – (-1.236J/K) = 2.643J/K
For 333J flux from the atmosphere to the surface:
increase in entropy for the surface:      333J/288K = 1.156J/K
decrease in entropy for the atmosphere: -333J/253K = -1.316J/K
dS for the Atmosphere to the surface: -1.316J/K – 1.156J/K = -2.472J/K
Net dS = -2.472J/K – 2.643J/K = -5.115J/K
Which is a reduction in entropy and contrary to the 2nd law.  I believe you have to calculate the effect of the energy flux on entropy in each direction and then sum them.  If you were correct that it is the net which counts, then it could be calculated either way and arrive at your answer, but it doesn’t seem to work out that way.

richard verney
May 14, 2011 5:34 pm

Ira
I am glad to see that you have come back with a summary. As previously mentioned, I find your articles amongst the most interesting posted on this site and the fact that this article has generated nearly 600 comments is testament to that. I thank you for the time spent compiling your articles.
As a general observation, a 10% error in some of the figures would more than produce the feared global warming of more than 2 degC per century. It is therefore important to know the correct figures within a better margin of error than 10% if we are to ascertain whether there may or may not be any significant warming.
Claim 1: I have considerable problems with this. If I was to go out into my garden on a summers day and record temperatures, I probably would find 100 or may be even a 1000 different temperatures. Many of these would vary by only relatively small amounts but some would vary by upwards of 40 degC. I live in semi rural conditions about 1 Km from the sea in the low foothills of the mountains. The mountains behind me begin in earnest about a further 1 km inland and extent about a further 20 or so km inland. In the early Spring, it may be 15 deg C by the coast but only 3 deg C in the mountains. The idea that temperatures for my area should be set by a thermometer at an airport some 60km away reading say 18 degC is fatuous and misconceived. Look at what Hansen does with large swathes of the Artic with only one or so station readings. To get a proper handle on the average global temperature we would need trillions of measurements. Obviously satellite measurements go some way to addressing this but even these are too course.
Further whilst the average surface sea temperature may be about 15 deg C, it is an over simplification to consider that over 70% of the earth’s surface is at the average sea surface temperature. One has to bear in mind the average temperature of the sea which is only about 4 deg C not its surface temperature (of about 15 degC) since the oceans are a heat sink with far more stored latent heat capacity than either the atmosphere or the the ground itself. The oceans are long term temperature drivers.
In summary, I think that there is good reason to be cautious with respect to the accuracy of the 288K figure.
Claim 2. If NASA TOA measurments ascertain incoming radiation at 1366 Watts/m^2, I would accept this.
Claim 3. Some caution is required. Actual measurements do not accord with actual measurements for Venus, Mars Titan or even the moon. The moon being devoid of an atmosphere ought to be the simplest to model but even this does not accord. See http://www.tech-know.eu/uploads/Greenhouse_Effect_on_the_Moon.pdf
Claims 4 and 5 . Potential for much error. The area, height (leading to volume) and compostion of clouds constantly varies 24 hours a day. No 2 days are the same. Nor are the location of clouds in the same place on any 2 days. It is difficult to know with reasonable precision what percentage of solar energy finds it way past the cloud shield so as to be able to warm the surface. Further the albedo of the surface is variable (not only from place to place but also from season to season) and therefore difficult to assess. Further there is an interaction between variation in cloud pattern and ground albedo, ie., it makes a difference whether the cloud window is open over surface areas which are more or less reflective. If clear skies are prevalent over areas that are more reflective of solar radiation one inevitable gets a different absorption. There is potential for considerable variation year on year with 4 and 5 and potential for much error in the assessment of these factors.
Claim 6. It is not a black body curve. See my comment under 3 above. Caution required with respect to interpretation.
Claim 7. This may be so, however, the issue is to what extent is it reasonable to consider the earth as if it were a black body when in fact it is, at best, a gray body.
Claim 8. This may be under estimated. Very difficult to perform accurate world wide measurements.
Claim 9. There is a deficiency in available empirical data to support the figures cited. (d) and (f) are interlinked and much more field studies are required to validate those figures and their order of magnitude.
There is no empirical data showing that about 320 Watts/m^2 is being ouput from the Atmosphere and being absorbed by the Surface, nor what the effect of that radiation has on the surface if so absorbed. It is difficult to envisage how 320 Watts/m^2 at a temperature of about 240K to 250K could be absorbed by the surface which is at about 288K. In making that latter statement, I am not excluding that the general temperature of the atmoshere (whatever that may be) does not slow down the heat loss from the surface or affect the height at which energy is radiated into space.

Joel Shore
May 14, 2011 5:55 pm

sky says:

In it’s myopic preoccupation with CO2, climate “science” largely ignores the foregoing facts. It stands entirely apart from all other branches in attempting to do thermodynamics on the basis of a simplistic radiative algebra that does not even recognize the significance of the density of the atmosphere in setting the surface temperature. And it it forgets that the Stefan-Boltzman equation applies only to a black body in equilibrium WITHOUT an atmosphere in into which thermal energy can be convected.

This is, quite frankly, nonsense. Simple models of the greenhouse effect to illustrate it make certain approximations. However, more quantitative models do not ignore convective effect nor do they use the S-B Equation. It continually amazes me that people who have clearly never even read a textbook on the subject feel that they are experts on it nonetheless. The Dunning Kruger effect ( http://en.wikipedia.org/wiki/Dunning%E2%80%93Kruger_effect ) is really in full force here.
richard verney says:

If radiation emitted from a colder object is actually absorbed by the hotter obeject then some fingerprint (signal) of this should be discernible. It seems to me that some physical expirementation should be carried out with a view to finding this fingerprint (signal) to try and ascertain whether it exists or not.

Do you think such experimentation to understand the basic laws of radiative physics has never been done?!?! If you want to understand the laws of radiative physics, I suggest that you go read about them. These are laws that are used everyday by physicists and engineers; they are the laws that govern the field of remote sensing; they are even the basic principles that explain our human metabolism. (Example: Why does a human feel comfortable naked in a room at a temperature of about ~25 C when, given our skin temperature, we emit radiation at a rate of several hundred Watts, much greater than our metabolic production of heat of ~100 W. The answer is because the net radiative flow is only about 100 W because the objects around us emit radiation that we absorb.)
Why do you think that people need to do additional experimental testing just because you aren’t yet convinced that they work?!? The people who actually use them sure as heck are convinced. If you want to learn more about the historical discovery of these laws and some of the original experiments done, be my guest…but don’t expect people to perform experiments for your benefit to show things that have been verified literally millions of times!

The recent comments by Joel show why the public is tiring of this policy agenda and why they are no longer prepared to take this dogmatic mantra at face value.

Even people on your side of the larger AGW debate have expressed admiration for my patience in dealing with you guys. Everyone else wonders why I even spend the time doing this.

Joel Shore
May 14, 2011 5:58 pm

Hans says:

So where does the energy to get this higher surface temperature come from?

It comes ultimately from the sun. The surface temperature of the earth is determined by the balance between what it receives from the sun and what it emits back out into space.

richard verney
May 14, 2011 6:14 pm

With respect to my latest post. I need to correct. This should read:
Claim 3. Some caution is required. Theoretical black body calculations do not accord with actual measurements for Venus, Mars Titan or even the moon. The moon being devoid of an atmosphere ought to be the simplest to model but even this does not accord. See http://www.tech-know.eu/uploads/Greenhouse_Effect_on_the_Moon.pdf
Alleyne
Many good points raised in your post of May 14, 2011 at 5:03 pm
“I agree that a colder object can radiate towards a warmer object. I agree that a colder object will warm up due to the radiated energy from a warmer object.
However, if a warm object absorbs radiant energy from a colder object, this implies that the lower energy state is somehow able to raise the higher energy state to an even higher energy state. This is the part I am having difficulty with…”
This is what I have been trying to say to Joel and Tim. I don’t see how the warmer body can actually absorb this radiation.
I don’t know what happens to the warmer body if it absorbs this radiation. It is difficult to envisage how the absorption of cold energy increases the energy state since surely the warm body will have to expend energy in warming up the colder photons towards the energy state of its own warm photons . This will then reduce the capability of the warmer object to emit photons, ie., the photons that it now emits will be slightly cooler than the photons that it was emitting prior to the absorption of the cold photons. Problem with that is that it is that it is akin to saying heat can flow from hot to cold and this is why I dont see that there can be absorption.

Joel Shore
May 14, 2011 6:55 pm

Alleyne says:

Heat/energy always travels from a higher temperature/energy state to a lower one.
Heat/Energy can not be transfered from a colder/lower energy state to a warmer/higher energy state without the application of external work/energy (Clausius’s Law) (Water won’t flow uphill without being pumped)

No…The Second Law talks about heat, which is a macroscopic concept representing the net flow of energy.

This is where I am having the difficulty with what you and Tim are saying. I have no problem with the energy traveling from the surface (warmer) to the atmosphere (colder). What I am having a hard time accepting is the energy being transferred from the atmosphere (colder) to the surface (warmer).

Why? How does the Second Law work in your picture of the world? Is there a little “Maxwell’s Demon” that stops a photon and asks where it came from and where is it’s going and says, “I am sorry, sir, but you can’t travel in this direction”? Do objects refuse to radiate if toward any hotter objects that they see or do the hotter objects just refuse to accept photons from objects that are colder?
Why do you believe your conception of the Second Law is correct and the conception of physicists, who actually work intimately with it and understand its derivation from statistical physics principles, is wrong?

Now the clever part of the design is that we direct all but 1/10,000th of a percent of the energy radiated from the cold sink (atmosphere) back to the hot source (surface), thereby returning all but 1/10,000th of a percent of the energy we discarded into the cold sink.

Yeah…Well, that’s pretty clever but also impossible to implement. How do you direct the energy radiated from the cold sink back to the hot source without having the hot source radiate energy to the cold sink? That’s the whole point: Such a scenario is impossible. The Laws of Radiative Transfer, coupled with Kirchkoff’s Laws regarding emissivity and absorptivity guarantee that this can’t happen.
I think I see where you might be getting confused: You seem to think that I am just saying as long as some overarching process has flow from hot to cold, we can have different separable sub-processes that have flows in the other direction.
I am not saying that: What I am saying is that the spontaneous process of radiative heat transfer between two objects consists of two sub-processes: radiation from the hot object passing to the cold object and radiation from the cold object passing to the hot object. However, the sub-processes necessarily occur together; you can’t separately have one without the other. Try as you might, you will not be able to produce (without the input of work) the energy flow from the cold to the hot without necessarily having the larger energy flow in the other direction.
Of course, there are lots of clever ways that people like to imagine how they can get around this, which is why so many people seem convinced that they have invented a concept for a perpetual motion machine. If the Laws of Thermodynamics were not so subtle, it would be obvious that one couldn’t violate them. It is their very subtlety (particularly the 2nd Law) that makes it so tempting for people to believe that they can find a way around it if they are just a little bit more clever.

If on the other hand you mean that the GHGs act like insulation which slows the heat loss from the surface, I would have to think about that, but can’t at this point disagree that it is possible. However the backradiation “heating” the surface is not necessary in this scenario.

Yes…That insulation effect is essentially what is happening. So, if you want to describe it without reference to radiation passing from the atmosphere to the surface then I won’t object. (The fact is that the mechanism by which this insulation effect occurs is by radiation occurring in both directions, as can easily be verified by empirical observation. But if you just want to think about just the net heat flow because it violates your principles to imagine heat as the net result of energy flowing in both directions, then I won’t complain. Mathematically, the greenhouse effect comes out of the equations without labeling the terms as forward and back radiation. So, my advice to people who are hung up about the concept of “back radiation” to just forget about it and simply tell themselves that the heat flow away from the earth’s surface depends not only on the temperature of the earth’s surface but also the temperature of the atmosphere, which is what the radiative transfer equations basically express.)

As to the textbooks, there has been a lot of foolish stuff published in textbooks and the scientific concensus has been wrong more often than it has been correct – however that is not really germane to this discussion.

Why would you jump to the conclusion that the physics textbooks are wrong on a very fundamental and basic matter of physics and that the scientific consensus is wrong in using equations that have been tested by scientists and engineers and used to build all sorts of technology rather than acknowledge that your own knowledge may be what is limited here?

Alleyne replies:
Thanks Tim for the numbers. Perhaps it is simplistic of me, or a further misunderstanding on my part, however I thought the formula for the change in Entropy was
dS= dCold – dHot

Yes…You are misunderstanding. You have put in a negative sign so that you end up subtracting numbers that you should be adding. You correctly calculate changes in entropy and then, instead of adding them together to get the total change, you subtract one from the other.

jae
May 14, 2011 7:35 pm

Wayne:
Keep up your work.
I have always had a very strong hunch that there is something wrong with the conventional AGW/GHG-forcing view. For several reasons. First, because this planet has been around for a very long time and has sustained extreme variations in GHGs, without any kind of ghg-caused collapse–or even a hickup (meteors cause big hickups, but not OCO). Second, there has been absolutely no correlation between the increases of any GHGs and temperature, over a period of a couple of millenium or more. OR even in the past 15 years, LOL. Au contraire, many scientists are now even fearing a little ice age, again! Three, the temperatures are NOT higher in areas with very high levels of GHGs (e.g., Atlanta, GA) than in areas with very low levels of GHGs that are at the same latitude and altitude. This indicates that Planet Earth has a very good “thermostat” (WATER). See the posts by Willis Eschenbach.
Bottom line: there is ABSOLUTELY no empirical evidence for a GHE or CAGW. All three of the above reasons should be critical to a scientist: without empirical verification, the theory is junk. So far, all we have is junk science, wrt “climate change,” “global warming,” and even “warming due to addition of GHGs.”

Joel Shore
May 14, 2011 7:48 pm

richard verney:

Claim 3. Some caution is required. Theoretical black body calculations do not accord with actual measurements for Venus, Mars Titan or even the moon. The moon being devoid of an atmosphere ought to be the simplest to model but even this does not accord. See http://www.tech-know.eu/uploads/Greenhouse_Effect_on_the_Moon.pdf

You consider yourself a skeptic, I assume, and yet you accept any piece of garbage that you find on the internet? Hertzberg, Schreuder, and Siddons are well-known for being purveyors of pseudo-scientific nonsense. Why would you possibly believe their nonsense when you won’t accept real science that has been tested thoroughly? It seems like your skepticism is very selective.
Here is a detailed debunking of their paper: http://scienceofdoom.com/2010/06/03/lunar-madness-and-physics-basics/

Tim Folkerts
May 14, 2011 8:32 pm

richard verney wonders:
“I don’t know what happens to the warmer body if it absorbs this radiation. It is difficult to envisage how the absorption of cold energy increases the energy state since surely the warm body will have to expend energy in warming up the colder photons towards the energy state of its own warm photons . “

Maybe it would help to think on the microscopic level. Think about one atom and one photon at a time.
A photon — any photon — has energy. Energy is not “cold” or “warm”. Photons are not “cold” or “warm”.
When that photon hits an atom, that energy can be absorbed. The photon is destroyed, giving all its energy to the atom. It doesn’t matter how fast the atom is already moving (ie how “hot” the atom is). If the atom absorbs that photon, the atom will have more energy than before. If the photon is low energy, the atom will only gain a little energy (ie get a little hotter). If the photon is high energy, the atom will gain more energy (ie get a lot hotter).
The atom now has EXTRA energy to share with other atoms around it. It does not need the atoms around it provide energy to “warm up” the photon. Instead, it can share the energy the photon had with all he neighbors around, making all of them a little warmer.
(This is different than adding a cold atom to the other atoms. When a cold atom runs into warm atoms, the cold atom is not destroyed. The cold atom had low energy and will tend to gain energy, sapping the other atoms of energy and cooling then a bit.)

jae
May 14, 2011 8:41 pm

Ira:
“Any objection to that?”
Suggestion: by repeating this phrase over and over again, you appear to me (and probably many others) as an elitist, smart-assed, all-knowing, sacrosanct, etc. hack. Drop that attitude, please, since it turns some of your readers off.
Signed, JAE, PhD

wayne
May 14, 2011 9:23 pm

Ira, I do understand that you do not trust what is going on but you also seem unwilling to let go, you know, the same old tack I have seen over the last month’s, some 600 papers, 3000 posts, quarter million comments later. It is not you but IPCC I blame. I understand you build your view on the latest peer-reviewed climate science and I don’t always. If you are comfortable with it, stay right where you are.
I will just point out the one fact of contention on the IR aspect of Trenberth’s chart, if I take an special averaging full spectrum radiometer to read Trenberth averaged “world” and point it at the ground, it says 390, I point it upward toward the cold atmosphere, all lines considered, it says 324, so what is going up from where I am standing, 390-324 or 66 Wm-2.
Why do you have trouble stopping there and quit trying to read into that 324 any more meaning, there is no further meaning to those numbers, read Trenberth’s papers, it’s data or derivatives of. It is just the screwy way the graphic was drawn that is the problem and you seem too to have bought into that whole logic. You seem to see the 324 Wm-2 coming from way up in the atmosphere, and, I see it located no more that a few meters off of the surface, just above my head except for the further reaching window frequencies.
Why does the ‘up’ reading not also say 390? Because it reads the frequency lines with various transparencies that allow that 66 W to escape from ground level upward (window) every second of every averaged day of every averaged year in Trenberth’s simple view of the world (not a cut to Trenberth here, the simplification must occur at this point with limited data and accuracies involved). If the atmosphere were totally opaque to all lines it would be like me, right now in my room, with everything 15 ºC and that is what SB would be showing, all temperatures locally are equal, net energy flow is zero. But, this never happens, even on Venus, there is always leaking upward and it has exactly the dependence of the total averaged optical thickness across all frequencies that determines that leakage that determines the temperatures of the layers above (gauging the natural lapse with H2O adjustments applied of course).
If you don’t like my words I can’t seem to help you if you will not get to the point where you see MY view clearly, I have seen yours for over a year now. I am no longer going to accept climate science’s consensus viewpoint. For whatever physicist I have in me I am now going simplify, simplify, simply down to some established principles in science that can not be questioned without dumping major laws and principles found true over the years. If I fail, well, so be it. I think I have said all of this before but there it is again. I just wish you could see my side, not agree but understand it..
Wave mechanics expert argued the same thing against quantum mechanics with a totally different way to look that matched the same data. I don’t ever want to conform, that is not what I do. The minute the consensus of the science world matches the data and I see why and that they are right, I am off to some other area of science that will challenge me. So far the consensus has been missing the data miserably so this is fertile ground to me. Think out of the box, their box is not working. Stop trying to get me to agree.
I wish I could converse with you over a beer or coffee, could give you so many different viewpoints, all proper in science (well, I do make mistakes), that tend to help me to get a real “feel” of the limits of the system and parameters. Have you taken the extremes yet? For instance, what would happen if the atmosphere if it were so opaque that only one watt per square meter of SW could reach the surface. What would Trenberth’s budget look like then? Heh? It’s calculable with optical thickness as you gauge (the units are mean free path). What would happen? You can plot various possibilities against the new bond albedo and get some rather surprising results. Try it, don’t ask for mine, I did it on a hand calculator. Such tests end up giving you a close approximation to what such changes would do to our atmosphere and your end up with a better “feel”.
To me that is science as I know it and I have been non-stop since college, I simply enjoy science, heavy on physics, I like to have a “feel” when available, as accurate as possible, so I can tell without much trouble sorting through important aspects and those that you can simply ignore or are irrelevant to the question. Why ignore? We only know albedo shy of two digits. I’ve seen in papers 29.x, 30,x. 31.x. Which is right, eh? And in reality there are really only four fairly accurate data points averages that really matter to begin with. I’ll chose to take a new tack on the problem every time. Just ignore me.
If you have a further reply, please don’t pick on a sentence out its entire context. I have not the time to make every single sentence stand on it’s own. That is the way I write when not making a firm point.
I’ve probably said way too much, made too many shortcomings in describing my thoughts, it happens.

wayne
May 14, 2011 10:33 pm

jae, well you’ve got all of that right!
Thanks for giving that a look. I’ll stay on it, wouldn’t be able to get it out of my mind if I wanted to, not until I can find it is flawed. But it seems pretty sound so far and now I have to see if numerically reproduces the numbers correct for the other atmospheres if I can find enough available data to do so.
One other thing, did you notice that the relation I gave should holds true at any point on the globe, any season, any surface temperature just as Miskolczi’s relations hold for any time and place? It is a global relationship. Also 2 x 1/6 or 1/3 is the greenhouse effect constant mentioned in his papers. That also is curious. Thought that might have slipped by you.

wayne
May 15, 2011 12:24 am

Ira, I want to commend you for having these posts, I have gathered many things from them all, no doubt. Some are things that are “out there” to avoid like the plague, others some great thoughts and most of those are from reading between the lines of what others are saying. All of it is great (except maybe the petty bickering, but even there I find value).
You wrote nine points so here are some of mine that I find completely true:
#1) First Law of Thermodynamics
#2) Second Law of Thermodynamics
#3) A solid surface, solid or liquid, can never get it’s temperature raised by energy that originated from said surface and included in it’s temperature at a macro level, by conduction or radiation. (see #1 & #2)
#4) At a macro level once energy is within the atmosphere that portion of energy is committed to be ejected to space for the average atmosphere’s temperature is always lower with increased altitude. (See #2, #3)
#5) On the average the atmosphere is homogeneous and any component of energy transfer (components of a vector) horizontally is totally cancelable and can be reasonably considered zero at the local macro level.
#6 ) … and so on.
See, my thoughts are on proper physics without getting into whatever current climate science has morphed it into. If you really want to make a mark, if you truly care, help, stay in pure physics and I will step into the background, I have no ego to support, no visions of fame, I just want proper physics applied to all of this mess but insist on it.
Maybe you should have a series, with Anthony’s blessing of course, on one tiny question at a time, not trying to wrap the entire question of CO2 warming into one huge post. Always too messy, nothing ever seems to come from it.
For instance, one might be #3, it’s a good question for everyone to get their minds around. On the macro level I stand by my claim above. At the micro level that question goes off of the cliff. There are principles addressing that of whether entropy can be decreased momentarily or not at all. Can gray, a mixture of black and white particles, have a probability of actually turn into black and white again. That’s deep water but everyone needs to really know what happens at the macro scale of our climate system (and of course it is NO, period, according to today’s best science). Maybe delve into the exact reason why. One focused question at a time.
Maybe we could have some real science here answering real questions one by one. I am tiring of the arguing. This isn’t that hard. Climate science WANTS is to be EXTREMELY hard and complex and most know exactly why. Approach it with simplicity as all real physics is. If your 0.2 ºC is correct it will float to the top. I think it is less than that, it could be more, and will try to show and explain as I get the numbers, it will be using a breakdown of Venus’s atmosphere of pure CO2 to show why our atmosphere is acting the way it is but the number have to make sense and be correct.

Myrrh
May 15, 2011 4:57 am

Ira said /#comment-660443
May 14, 2011 at 7:45 pm
Also, Richard, in a previous posting you seemed to imply that radiation could come from a cooler body towards a warmer one and then, at the very doorstep, be rejected somehow because the radiation was somehow marked and detected as coming from a cooler object. How in the world of physics and engineering would this work? If you were very rich and I poor and you gave me $1000 and, out of gratitude, my wife baked you a $2 cake and took it over to your house, would it be rejected at your doorstep?
An analogy which has no bearing on the example being explained is worthless in science. To confuse the standard laws of science by creating a childrens’ story by personifying hotter and colder objects who have the ability to choose, is a fun project, but not science.
It is not the energy from the colder being rejected by the warmer, it is the colder taking from the warmer by imposing its nature on the warmer. The colder comes up to the doorstep, or they meet somewhere in the street, and shake hands; the warmer immediately loses heat to the colder who begins to warm up because that is the nature of their relationship.
That’s what the 2nd Law says because the process is not one of choice. For choice, one has to bring in another energy, work, to change the process. Therefore, Heat always flows from the hotter to the colder.
The claim that it is a “net” heat exchange from hotter to colder makes no sense in light of this. Just because ‘radiation can be emitted in all directions’, doesn’t mean it always is. The 2nd Law limits this possibility, or rather, articulates the limit.
Therefore, the ‘statistical net exchange of heat is from hotter to colder by including heat from colder to hotter’ is gobbledegook in science, because it violates the 2nd Law in its base premise.
The only net exchange going on is within the bounds of the 2nd Law; for example, when pouring hot water into a cup already half filled with cold water the first amount of hotter will have its heat taken by the colder so reducing its own and it then becomes the colder to the hot water coming after it, even while it is still hotter than the colder first in the cup still taking its heat – and so the net exchange in this to equilibrium.
If you claim that the 2nd Law breaks down at the level of photons, then you must prove it.
It would be worthy of a Nobel Prize for Physics for you to show that the tried and tested and well understood in Science 2nd Law breaks down at this level.
There are three of you here claiming it..

Dave Springer
May 15, 2011 5:39 am

richard verney says:
May 14, 2011 at 6:14 pm
“This is what I have been trying to say to Joel and Tim. I don’t see how the warmer body can actually absorb this radiation.”
What other disposition is possible for the radiation moving from the cooler object to the warmer object? They must either be transmitted straight through, absorbed, or reflected. They can’t just disappear as that would violate conservation of energy. The books have to balance. Every quantum of energy must be accounted for.
The simple fact of the matter is that the warmer object throws off more photons than it receives from the colder object.
Imagine a wall with you on one side and me on the other. We are throwing tennis balls over the wall to each other. For the sake of argument lets say I can’t pick up and throw any faster than you so for every ball I throw over you throw one back at the same time. We are in tennis equilibrium. So now I get someone else to help me and for every ball you throw we throw two balls back. We’re the hotter team now and there is no tennis equilibrium any longer. But just because we can throw faster doesn’t mean you’ve stopped throwing. There a net transfer of tennis balls from the hotter to the colder team.
Radiative exchange between two objects of same or different temperatures works the same way. Both objects are absorbing energy from the other. If they are both the same temperatures the exchange is tit for tat and they are said to be in thermal equlibrium. If one is warmer than the other then there is a net transfer from warmer to colder: EnergyNet = EnergyOut – EnergyIn. For the colder object the net is a positive number and for the warmer object it’s negative. The transfer continues forever. It doesn’t ever stop. It simply reaches a point where EnergyOut = EnergyIn for both objects.
Why is that so difficult to accept? It is not at all counter-intuitive and is in fact exactly how radiative exchange works. It isn’t rocket science at this level. The rocket science starts when we start adding more objects with complex properties and transmission media with complex properties. But simple radiative exchange between two ideal bodies in a vacuum is very basic and very comprehensible.

Dave Springer
May 15, 2011 5:57 am

Shore
“The Dunning Kruger effect ( http://en.wikipedia.org/wiki/Dunning%E2%80%93Kruger_effect ) is really in full force here.”
Arthur C. Clarke’s three laws of prediction are worth mentioning in this context:
1.When a distinguished but elderly scientist states that something is possible, he is almost certainly right. When he states that something is impossible, he is very probably wrong.
2.The only way of discovering the limits of the possible is to venture a little way past them into the impossible.
3.Any sufficiently advanced technology is indistinguishable from magic.

Tim Folkerts
May 15, 2011 6:39 am

Wayne postulates:
“#4) At a macro level once energy is within the atmosphere that portion of energy is committed to be ejected to space for the average atmosphere’s temperature is always lower with increased altitude. (See #2, #3)”

Let me see if I can convince you (and others) that at a micro level, energy can travel either way.
1) Suppose you have a convection current moving upward @ 1 m/s. It might be tempting to say air molecules are “committed” to move upward because the net speed is 1 m/s. But the random thermal motion of air molecules is about 400-500 m/s (depending on temperature and just which type of molecule we are talking about).
The speed of the molecules could be stated as 1 +/- 500 m/s. A given molecule could be moving upward at 203 m/s one moment and a moment later it might be heading downward at -337 m/s. If you followed a given molecule, it would take a long time before you could determine that there was a breeze.
Or put another way, if you look at the bottom of a tiny cube (say 10^-8 m on a side), nearly as many molecules will be passing downward as upward. in some second, it is possible that more molecules actually passed down than up even when the air is blowing upward!
2) Suppose a wire is hooked to a battery and we look at a cross-section of that wire. Electrons move:
a) only from + to –
b) only from – to +
c) about evenly in both directions.
If you answer (a), you are thinking of conventional current, not movement of electrons.
If you answer (b), you have had a bit more training in circuits.
If you answer (c), you are correct.
Conduction electrons have a mean speed around 10^6 m/s (google “Fermi energy” if you want to know more).. A typical drift velocity of electrons in a wire is 0.001 m/s. This is an even more extreme case compared to (1). The conduction electrons are moving at 0.001 m/s +/- 1,000,000 m/s!
3) Consider two identical blocks of metal in contact, one slightly warmer than the other. The average energy of an atom in each block will be 1/2 kT per degree of freedom. For the sake of argument, suppose the average energy in one is E and the average energy of the other is 1.1 E (ie it is 1.1 times hotter, eg one might be 300 K and the other might be 330 K).
Energy is transferred by conduction when an atom of one block hits an atom of the other block. There are plenty of atoms in the cold block with an energy > 2E. There are plenty of atoms in the warm block with an energy < 0.5 E. If an above average atom from the cold block hits a below average atom from the warm block, the warm block will gain energy!
Of course, if you average enough collisions, the net flow of energy is from the warm to the cold block, but individual collisions can transfer energy either way. (This might be labeled "back-conduction" and "forward-conduction", but I can think of any cases practical where "back-conduction" is important as a separate concept from net conduction.)
And the conclusion, of course, is …
Suppose an air molecule with above average energy emits a photon with above average energy to a surface atom with below average energy ….
So we can state
“#4b) At a
micro level once energy is within the atmosphere that portion of energy is will go almost randomly in any direction. Nearly 1/2 will go downward, accounting for the ~330 W/m^2. “

Tim Folkerts
May 15, 2011 6:52 am

Myrrh expounds:
“It would be worthy of a Nobel Prize for Physics for you to show that the tried and tested and well understood in Science 2nd Law breaks down at this level.”

With all due respect, until you have taken a graduate level course in thermodynamics and statistical mechanics, you are not even CLOSE to being able to decide what is worthy of a Nobel Prize related to the 2nd Law.
I suspect that 2 or three of us here HAVE taken graduate level statistical mechanics. I suspect several others have at least taken upper level undergraduate level thermodynamics classes.
If you REALLY want to discuss statistical mechanics, then I would suggest working through the free course from MIT http://ocw.mit.edu/courses/physics/8-333-statistical-mechanics-i-statistical-mechanics-of-particles-fall-2007/syllabus/ In particular, work through the ideas of microcanonical, canonical and grand canonical ensembles and how they relate to entropy and the 2nd Law.

May 15, 2011 7:00 am

Ira Glickstein, PhD states his 9 claims in a posting on May 14, 2011 at 3:49 pm:
I’ll “focus” mainly on the last one here:
“——————- Claim 9: Accounting.
(a) 390 Watts/m^2 = Radiative energy input from the Surface to the Atmosphere (claim #7).
(b) 70 Watts/m^2 = Radiative energy input from the Sun to the Atmosphere (claim #5).
(c) 100 Watts/m^2 = Net convective and water cycle input from the Surface to the Atmosphere (Claim #9).
(d) 560 Watts/m^2 = Total input to the Atmosphere from all sources.
(e) 240 Watts/m^2 = Output of the Atmosphere towards Space (claim #4).
(f) 320 Watts/m^2 = Atmospheric emission towards the Surface (simple arithmetic: 560 – 240 = 320).
Evidence: Simple arithmetic. Close enough for a “sanity check”. Any objection to that?”
This, his last “Claim” is the “Main Claim” that keeps me from readily accepting the rest of the claims.
My objection to this one is that “Kirchhoff’s Law of thermal radiation states”, as far as I can recall, and as Wikipedia writes is that:
“At thermal equilibrium, the emissivity of a body (or surface) equals its absorptivity.”
Our nearest neighbor in space which must receive a very close to similar radiation in Watts per m² to that which the Earth does, would be a very cold ball indeed if it’s surface was to receive 240 W/m² and at the same time emit 390 W/m². – As I said in my comment on May 13, 2011 at 5:12 pm: “I doubt very much that the surface knew when it emitted the 324 extra W/m² of energy from it’s “energy store” that the atmosphere was going to send it straight back and thus conserve that energy on it’s behalf.
Ira, you say it is “Simple arithmetic” – therefore ok, – so my arithmetic may be simpler than the simplest of all arithmetic(s), but it is still good enough to understand that there is not necessarily the arithmetic that is wrong.
If surface radiation has been measured and does indeed average 390 W/m² then either the stated “input wattage” (your 170 Watts/m^2) is incorrect or the “Earth System”, Globe + Atmosphere, must be looked at as one complete unit where internal heat exchange uses or looses 390 – 170 = 220 W/m², or – Kirchhoff was wrong but nobody, as far as I know, advocates that.
And then lastly in Claim 9 (f) you say:”320 Watts/m^2 = Atmospheric emission towards the Surface (simple arithmetic: 560 – 240 = 320).”
My questions here are: How is it possible for the Atmosphere to radiate 240 W/m² towards space and 320 W/m² towards the surface? – And, what is the “selection method” used by the Atmosphere to enable it to emit more in one direction than it does in the other?
And lastly Ira, think of this one; if you was to substitute 390 W/m² & al with, say 3.9
°C & pro rata, – then, how much would it add to the average surface temperature of 15 °C if 3.9 °C was removed from the surface and just 3.24 °C was returned?

Dave Springer
May 15, 2011 7:04 am

Joel Shore says:
May 14, 2011 at 6:55 pm
It’s tough not getting frustrated, isn’t it? 🙂
I hope you know that many skeptics are very well informed and have no misconceptions about the most basic physics of greenhouse gases. I have tried many times in many ways to explain that greenhouse gases act as insulation allowing visible light from the sun to pass straight through unimpeded to heat the ocean but do not allow infrared light from the ocean to pass through unimpeded on its way out the door to space. The property of being transparent to visible light and opaque to infrared doesn’t seem complex or difficult.
I’d be interested in your thoughts on liquid water acting in the same way. Water is fairly transparent to visible light allowing sunlight to penetrate to a depth of about 100 meters before it is completely absorbed. On the other hand water is opaque to infrared light. Infrared light is absorbed in the first micrometer of water.
So the sun can heat the ocean to a depth of 100 meters but the energy absorbed at that depth can’t escape as infrared because water is opaque to it. Liquid water is thus a GHF (greenhouse fluid). Actually gases are fluid so there’s essentially no difference between a GHG and GHF except for density and viscosity.
So how does the visible light absorbed by the ocean deeper than 1 micrometer escape? If radiative escape is blocked then that only leaves convection and conduction to get the deeper water to the surface where it can escape through radiation, conduction to the atmosphere, and evaporation.
My big question is about whether conduction and convection are as efficient at getting energy to the surface as sunlight is at adding energy at depth.
I don’t believe that it is as efficient and there is in fact a greenhouse effect the comes solely from the fact that the earth is mostly covered by water.
I’ll look into it further.

Myrrh
May 15, 2011 7:19 am

Tim, quit giving me homework. Give me an direct answer, and when I question it, give me the answer in the specific form I’ve asked for it, instead of prevaricating; as you’ve done yet again. I have yet to see any basic science from any of you.
For example, take Ira’s opening here: “..that all energy is fungible” – what on earth is that supposed to mean in context of his post?
So my cup of coffee can turn into a crystal glass bubbling with champagne? Of course it can, in process, and in accordance with the nature of the properties and the limits of their changes at each and every step.
Prove, say, at the photon level that a photon from a colder transfers/add/converts to heat the warmer water added to it in a cup when they are brought into contact.
A statistical analysis made with complete disregard for the actual possibilities of a process taking place is without logical parameters.

Bryan
May 15, 2011 7:21 am

richard verney
Start of with an object radiating because it contains an energy source.
Bring another passive object at a lower temperature near to the first object.
Switch off the power source to the first object
The passive objects temperature increases and the hotter will decrease.
The passive object will be returning some of the energy that was supplied to it by the first object.
Its obvious that the first objects temperature will not increase by absorbing energy it initially supplied to the passive object.
Both objects will exchange energy ( more from the hotter one).
Eventually both objects will be at the same temperature and exchange equal quantities of radiation.
If the power supply was left on the hotter ones temperature will increase in temperature because the passive one acts as an insulator and the radiation it returns can be thought of as the radiative component of insulation.
Radiation in this example is a two way process
Energy exchange is a two way process
Spontaneous Heat transfer is a one way process always from hotter to colder never the reverse.
Heat is always energy but energy is not always heat.
The transfer of heat is the province of the second law of thermodynamics.
Its easy to get the quantities muddled .
Even Physicists if their not careful, get confused.
Joel Shore and Arthur Smith and four other “experts” managed to compose a paper getting very confused about Heat.
Now that is inexcusable.
Particularly when I explained to Joe l (here on WUWT) two months before the papers publication exactly the same points I am making here.

Joel Shore
May 15, 2011 7:29 am

Myrrh says:

An analogy which has no bearing on the example being explained is worthless in science.

Exactly…Which is why when people make analogies, they should (like Ira, myself, Tim, Dave) actually understand the underlying physics and equations that actually describe the process. We do. You don’t. Therefore, any analogies that you make are based on nonsense, not physics.

Joel Shore
May 15, 2011 7:34 am

wayne says:

See, my thoughts are on proper physics without getting into whatever current climate science has morphed it into. If you really want to make a mark, if you truly care, help, stay in pure physics and I will step into the background, I have no ego to support, no visions of fame, I just want proper physics applied to all of this mess but insist on it.

Is there any actual physics textbook that teaches your version of “proper physics”? I ask that because I have seen lots of physics textbooks but they all seem to teach improper physics…So, I am wondering where one can find this proper physics written down. Certainly, there must be one reputable source that you can cite?

Dave Springer
May 15, 2011 7:40 am

Can someone help me out here?
Here is a graph of liquid water absorption across a large spectrum from ultraviolet to microwave.
As on can see water absorbs strongly at all frequencies other than visible light. This is why light from the sun can penetrate the ocean to a depth of about 100 meters.
This is exactly the properties of a greenhouse gas that makes them a greenhouse gas – strong absorber of infrared light and weak absorber of visible light.
The ocean therefore must be a greenhouse fluid.
So far I can find no discussion of water as a greenhouse fluid anywhere. Surely someone other than me has noticed that water has the same properties as greenhouse gases and therefore must have a greenhouse effect.
Maybe I’m not googling for the right words/phrases. No matter how I try when combining water and greenhouse effect in the same search you’re inundated (pun intended) by hits about greenhouse gases. The search for water as a greenhouse fluid is buried in the noise of atmospheric greenhouse effects.
Can someone help me out here? I’m crowd sourcing for an answer.

Dave Springer
May 15, 2011 7:41 am

Forgot the add link to radiative absorption spectrum of liquid water:
http://en.wikipedia.org/wiki/File:Water_absorption_spectrum.png

Joel Shore
May 15, 2011 8:05 am

Myrrh says:

The claim that it is a “net” heat exchange from hotter to colder makes no sense in light of this. Just because ‘radiation can be emitted in all directions’, doesn’t mean it always is. The 2nd Law limits this possibility, or rather, articulates the limit.
Therefore, the ‘statistical net exchange of heat is from hotter to colder by including heat from colder to hotter’ is gobbledegook in science, because it violates the 2nd Law in its base premise.

First of all, an issue of terminology that has sometimes tripped me up too (much to Bryan’s enjoyment): “Heat” is the name given to the net energy flow, so it is not technically correct to talk about “heat from colder to hotter”. Rather, it is most correct to say that the heat, i.e., net flow of energy, is always from hotter to colder but that there are energy exchanges going both ways, with the energy flow from hotter to colder always being larger than that from colder to hotter.
At any rate, I have a question for you Myrrh: In your view, when exactly did these incorrect concepts take over physics? I have the textbook here that I used as an undergraduate. It is “Introduction to Statistical Mechanics and Thermodynamics” by Keith Stowe, copyright 1984. (We actually used the book when it was essentially new…in 1985, plus or minus a year.) Here is some of what it says about the Second Law:

Notice that the second law is based on probabilities, whereas the first law seems to reflect inviolable fact. The second law does not work for small systems, whereas the first law does. Even for large, macroscopic systems, there is some infinitesimal probability that the second law could be violated. However, the chance that any living human will ever see it violated is extreme small. If we had some instrument capable of measuring any system variable with an accuracy of parts per billion, and if this machine repeated these measurements 10^6 times per second, then we would have to wait roughly 10^(360,000,000,0000,0000) times longer than the age of the universe to see even the slightest violation even once, for typical macroscopic systems.
For example, there is some small probability that the air of the room in which you are presently sitting will all rush over to one corner , leaving you to suffocate (Figure 8.5). There is some small probability that water will flow uphill, or begin to boil as ice cubes are added. There is some small probability that your blood could start transporting carbon dioxide to your cells and oxygen away, rather than vice versa. But all these things are extremely unlikely. Even if we had extraordinary accurate machines that could detect even the inception of such anomalous behavior lasting only a microsecond, we would have to wait longer than 10^(10^14) times the age of the universe to record even one such case.
Clearly, we can base our studies on a law whose chance of violation is so minuscule and we can rest assured that we will never witness a violation. In fact, every moment of our existence we bet our very own lives on these odds, and needless to say, we always win.

Joel Shore
May 15, 2011 8:13 am

O H Dahlsveen says:

My questions here are: How is it possible for the Atmosphere to radiate 240 W/m² towards space and 320 W/m² towards the surface? – And, what is the “selection method” used by the Atmosphere to enable it to emit more in one direction than it does in the other?

I am actually pretty annoyed that you bring this up again given how much time and energy we (both Tim and I) expended explaining it to you in the last thread:
http://wattsupwiththat.com/2011/03/10/visualizing-the-greenhouse-effect-emission-spectra/#comment-621266
http://wattsupwiththat.com/2011/03/10/visualizing-the-greenhouse-effect-emission-spectra/#comment-621292
http://wattsupwiththat.com/2011/03/10/visualizing-the-greenhouse-effect-emission-spectra/#comment-621474
http://wattsupwiththat.com/2011/03/10/visualizing-the-greenhouse-effect-emission-spectra/#comment-621511
Why should we waste our time explaining these things to you if you will just continue to recycle your questions again and again?

Joel Shore
May 15, 2011 8:44 am

Dave Springer says:

It’s tough not getting frustrated, isn’t it? 🙂

Indeed!

I hope you know that many skeptics are very well informed and have no misconceptions about the most basic physics of greenhouse gases.

Yeah…And, I am glad that many of you have become more vociferous in trying to get the others to understand this basic physics, although it seems to be a losing battle in many cases.

My big question is about whether conduction and convection are as efficient at getting energy to the surface as sunlight is at adding energy at depth.

Well, conduction may not do that much, but I think convection will. I.e., like in the atmosphere, convection will probably try to move things back to the stability limit, which means a much smaller lapse rate in a liquid medium than in a more-compressible gaseous medium.

I don’t believe that it is as efficient and there is in fact a greenhouse effect the comes solely from the fact that the earth is mostly covered by water.

An interesting idea. However, my guess (admittedly without much thought) is that this will be very small exactly because convection in a liquid will place a much lower limit on lapse rates than convection in a gaseous medium. Also, although light can make it down to large depths, I don’t think the heating is as concentrated at the bottom of the ocean as it is at the bottom of the atmosphere (although this may depend on the ocean depth…e.g., how much light gets all the way down to the ocean floor).

philincalifornia
May 15, 2011 9:02 am

Dave Springer says:
May 15, 2011 at 7:40 am
Interesting concept. I tried Googling “water absorption spectrum pH” as I suspected there may be pH-related changes in the absorptivity of water (there are). I agree, surely there must be some serious literature on this ??
I thought that the way this page started, it might touch upon your question, but it doesn’t really, although it’s an interesting resource:
http://www.btinternet.com/~martin.chaplin/vibrat.html#uv

gnomish
May 15, 2011 9:53 am

A scientist said cogently, as his hand came from his pants-
“I understand an elephant is fetching mighty grants!”
A cohort, then, who felt the wind emerging from his ass,
declared “I see the elephant is very like a gas!”
“Then they began to quarrel, shouting, ‘Yes it is!’ ‘No, it is not!’ ‘An elephant is not that!’ ‘Yes, it’s like that!’ and so on, till they came to blows over the matter.
“Just so are these preachers and scholars holding various views blind and unseeing…. In their ignorance they are by nature quarrelsome, wrangling, and disputatious, each maintaining reality is thus and thus.”
Weather used to be ‘the safe topic’ before it became blood sport for sophomores with no other outlet for their raging hormones.
More circus, less bread! More taxes to pay for this! The alto section of the choir can always use more eunuchs, too.

Dave Springer
May 15, 2011 9:57 am

philincalifornia says:
May 15, 2011 at 9:02 am

Dave Springer says:
May 15, 2011 at 7:40 am
Interesting concept. I tried Googling “water absorption spectrum pH” as I suspected there may be pH-related changes in the absorptivity of water (there are). I agree, surely there must be some serious literature on this ??
I thought that the way this page started, it might touch upon your question, but it doesn’t really, although it’s an interesting resource:
http://www.btinternet.com/~martin.chaplin/vibrat.html#uv

Yeah, I found that page. It’s about as close as I got but after all the nice info it focuses on water vapor in atmosphere. Nothing I could find anywhere about the same effect from liquid water. Presumably the effect is much greater in liquid water since the first 30 feet of it is equal the entire weight of the atmosphere above it and the first foot of it equal to the weight of all the water vapor above it.
I still can’t believe the seeming utter dearth of information pertaining to liquid water as a greenhouse fluid given it obviously has the required properties of transparency to visible light and opacity to infrared.

May 15, 2011 9:59 am

O H Dahlsveen says:
May 15, 2011 at 7:00 am
My questions here are: How is it possible for the Atmosphere to radiate 240 W/m² towards space and 320 W/m² towards the surface? – And, what is the “selection method” used by the Atmosphere to enable it to emit more in one direction than it does in the other?
This is rather easy – at some frequencies, the atmosphere is opaque. As a result, the warmer lower atmosphere emits more energy towards the surface than the cold upper atmosphere emits towards space.

May 15, 2011 10:01 am

Come on guys, this stuff is really easy. Several people keep arguing that raising the average temperature from -18C to 15C violates the second law. However, they are ignoring the fact that the actual temperature fluctuates from high values to low values. The correct analysis is for recognize that, without an atmosphere, the daily surface temperature would raise to about 120C (249F). When the Sun goes down, the surface wants to cool to about 45K. (Notice, this is not the 3K background everyone keeps talking about. The difference is due to the zodiacal light that shines 24/7.) What the atmosphere does is to cool the surface so that the daily maximum value is only 57C (135F). As the temperature drops at night, the atmosphere returns some of the energy it stored during the day so that the minimum temperature is only slightly below the maximum. When analyzed like this, it is obvious that the Second Law is not violated.
Get it? Instead of cooling to -18C, the surface cools to 15C. It isn’t being heated by the atmosphere. It simply does not cool as much as it would without an atmosphere.
At the poles, it works a little different. During the long night, the atmosphere is always warmer than the surface. (This has been measured with thermometers tied to balloons.) As a result, the only source of winter heat is the radiation from the atmosphere.

Myrrh
May 15, 2011 10:19 am

Joel Shore says:
Re “An analogy which has no bearing on the example being explained is worthless in science.”
Exactly.. Which is why when people make analogies, they should (like Ira, myself, Tim and Dave) actually understand they underlying physics and equations that actually describe the process. We do. You don’t. Therefore, any analogies that you make are based on nonsense, not physics.
Well, you can keep claiming you and your gang understand, but I’ve yet to see real evidence of it. I don’t have to have a science PhD to think logically..
If all you can reply to my questions is that you are so much more educated in this science so it follows that whatever you say it correct, then that is illogical, and against the principles of Science; that is religion claiming its priesthood knows all the answers. I thought this was a Science blog.
Take for example the oft touted analogy of “insulating” and “insulating blanket”, which anyone with half a brain and without science background can look up to check what that means as an analogy – that it’s a blanket among more than 99.95% non-blanket. And which, coupled with the claim that this “insulating blanket” is well mixed in that 99.95%, that doesn’t even count as a stitch.
You’ve shown immediately and obviously that you don’t have the skills you’ve just claimed, because your gang’s analogy is demonstrably nonsense.
If you can’t defend your science claims with anything better than that, it would be really dumb of me to take your claims about your science superiority seriously.
So I don’t.

Dave Springer
May 15, 2011 10:31 am

This gets much closer:
JOURNAL OF GEOPHYSICAL RESEARCH, VOL. 108, NO. C5, 3146, doi:10.1029/2002JC001584, 2003
Seasonal mixed layer heat budget of the tropical Atlantic Ocean

Net surface heat flux is a combination of latent and
sensible heat loss, shortwave radiation absorption, and net
longwave emission. Sensible heat loss is insignificant (<10
W m2) due to small air-sea temperature differences, while
net emission of longwave radiation is a relatively constant
50 W m2 [da Silva et al., 1994].

This is fatal for the GHG hypothesis. It is experimentally found that the ocean absorbs 200 w/m2 of visible light and only loses 50 w/m2 via longwave emission.
As I suspected the energy is primarily carried off in latent heat of vaporization.
In a nutshell: greenhouse gases only get a chance to act on 25% of the energy being emitted by the ocean and CO2 with its narrow absorption bands only gets to act on a fraction of that 25%.
Moreover the paper above discusses how solar energy absorbed by the ocean in the humid summer remains trapped until dry winds in the winter remove through evaporation.
I friggin’ knew I had to be right. The goddamn ocean is responsible for most of the greenhouse effect. Atmospheric gases play only a small role.

Dave Springer
May 15, 2011 10:41 am

There’s an elephant in the room responsible for most of the 33K greenhouse warming on this planet. That elephant’s name is “the global ocean”.
If water vapor and CO2 had no IR absorptivity we’d still have most of the greenhouse warming because it’s liquid water that generates most of it!
Now the question is why no one talks about it. If a physics piker like notices that water is transparent to visible light and opaque to infrared, thus fulfilling the requirement to act as a greenhouse agent, the surely others must have noticed as well. I don’t get it. This is too obvious but try as I might I can find no flaw in the reasoning.

Joel Shore
May 15, 2011 11:00 am

Dave Springer:

If water vapor and CO2 had no IR absorptivity we’d still have most of the greenhouse warming because it’s liquid water that generates most of it!

No…In addition to the objections that I raised above, how is the liquid water in the oceans going to cause the temperature at the earth’s surface to be higher than required by simple energy balance considerations? The substances that do this have to be above the earth’s surface.

Joel Shore
May 15, 2011 11:08 am

Myrrh says:

If you can’t defend your science claims with anything better than that, it would be really dumb of me to take your claims about your science superiority seriously.
So I don’t.

Myrrh: There is not a scientist on this planet who can dispel the scientific delusions that you live under. As David M Hoffer has pointed out, you are utterly and completely impervious to science and reason.
We can’t fight ignorance when someone is dead-set on remaining ignorant. I am sorry but it is just not possible.

philincalifornia
May 15, 2011 11:18 am

Dave Springer says:
May 15, 2011 at 9:57 am
Nothing I could find anywhere about the same effect from liquid water. Presumably the effect is much greater in liquid water since the first 30 feet of it is equal the entire weight of the atmosphere above it and the first foot of it equal to the weight of all the water vapor above it.
———————————–
Maybe it’s just a nomenclature thing. Since no one doubts the fact that the oceans absorb and retain energy from the sun, your visible light observation has got lost in the noise.
Given the parameters you mention above, I can’t help thinking that you might be on to something important here, i.e. with respect to changes due to dissolved CO2 and the attendant, albeit small pH changes. However, since there does not appear to be much (Googlable) literature on this, it’s difficult to start trying to assess the magnitude of, or even in which direction this potential feedback could be.
Apologies for extrapolating from your initial observation, but you piqued my curiosity.

philincalifornia
May 15, 2011 11:26 am

PS Just saw your latest posts. I guess I was looking for the baby elephant !!!!

Tim Folkerts
May 15, 2011 1:13 pm

David Springer,
I seems the paper you reference deals with tropical oceans. It is not surprising that they absorb more then they emit, since warm ocean currents carry large amounts of energy from the tropics toward the poles. That might affect your conclusions a bit.
I’m not convinced overall that your analysis is correct, but I don’t have time right now to dig deeper into it. At first glance, it would seem that the most your “ocean greenhouse effect” could do would be to warm lower layers of the ocean warmer than they would be without such an effect.

Joel Shore
May 15, 2011 1:23 pm

Dave Springer says:

It is experimentally found that the ocean absorbs 200 w/m2 of visible light and only loses 50 w/m2 via longwave emission.

By the way, note that it says NET emission of longwave radiation. Compare this to the Trenberth and Kiehl estimate for the whole globe that you have 396 W/m^2 going up from the surface and 333 W/m^2 coming down from the atmosphere, for a net of 63 W/m^2 upward. So, the two numbers (one a global estimate and one just an estimate over the oceans) are in reasonably good agreement considering that they don’t measure exactly the same thing.
Can you be a little clearer on what part is “fatal for the GHG hypothesis” and why?

Myrrh
May 15, 2011 2:30 pm

Joel Shore says:
May 15, 2011 at 8:05 am
First of all, an issue of terminology that has sometimes tripped me up too ..:”Heat” is the name given to the net energy flow, so it is not technically correct to talk about “heat from colder to hotter”.
Nope. Heat is energy on the move from one location to another on a molecular level as a result of temperature difference. It always flows spontaneously from the hotter to the colder.
Rather, it is most correct to say that the heat, i.e. net flow of energy, is always from hotter to colder but that there are energy exchanges going both ways, with the energy flow from hotter to colder always being larger than that from colder to hotter.
You can say that is correct, but as it doesn’t make sense in the physical world where heat always flows spontaneously from hotter to colder I have no reason to take your saying so seriously.
At any rate, I have question for you Myrrh: In your view, when exactly did these incorrect concepts take over physics? I have the textbook here that I used as an undergraduate. It is “Introduction to Statistical Mechanics and Thermodynamics” by Keith Stowe, copyright 1984.
I don’t know. But obviously before that book was published..
Here is some of what it says about the Second Law:

Notice that the second law is based on probabilities, whereas the first law seems to reflect inviolable fact. The second law does not work for small systems, whereas the first law does.


Prove it. Claiming it doesn’t, when it is quite clear in Science that the law refers to heat energy in transit on a molecular level, which can be by conduction, convection or radiation, is contradicting the law; the onus is on you to prove your change of it.

For example, there is some small probability that the air of the room in which you are presently sitting will all rush over to one corner, leaving you to suffocate.. There is some small probability that water will flow uphill, or begin to boil as ice cubes are added. ..


Yeah, and pigs might fly.
Probability which excludes taking into consideration the physical nature of the properties and their real interactions, is Alice through the looking glass. Heat does not flow spontaneously from colder to hotter, it violates reality. Science is the study of reality, not fantasy. Water does not flow uphill spontaneously. It takes Work to alter that reality. Work is also energy in transit, as Heat is.
Clearly, we can base our studies on a law whose chance of violation is so miniscule and we can rest assured that we will never witness a violation. In fact, every moment of our existence we bet our very own lives on these odds, and needless to say, we always win.
Irrelevant.
Prove that heat flows from a colder to a hotter spontaneously or you are violating the 2nd Law which says it doesn’t.
Write your own laws for your fantasy world if you must, but quit mangling those articulating our natural reality, and certainly quit claiming that your rewording isn’t novel because as you’ve given the quote it clearly denies the 2nd Law, and as it is novel, it requires to be proved.
So, perhaps it’s you who should find out how come you were taught this violation of the Law where you’ve not only extrapolated probability by including the oh so miniscule impossible, water does not flow uphill spontaneously, but have turned over all your reasoning to saying the impossible is our actual reality.

jae
May 15, 2011 7:10 pm

It seems clear to me that the “jury is still out” regarding even the most basic, simple part of the CAGW junkscience: the GHE theory. I keep changing my mind about what is relevant and what is not, and I still don’t have a definite position about what is factual, after years of reading this stuff. I find that fascinating. I’m not a physicist, but I have a PhD in carbohydrate chemistry; so I think I can easily understand the basic science, here. Whatever your position, you gotta admit that it is strange that we have so little (in fact, none) empirical evidence on something that appears to be so basic and important. Everyone think s(he)’s right, but nobody can really demonstrate it to the satisfaction of everyone. Weird.
The real marvel that separates planet earth from all others (that we know of (and probably all we will ever know of)) is HOH. Most of the mystery of all that is being discussed here can probably be attributed to this “weird” substance that behaves so differently from nearly all known substances.
I doubt that this means much for anyone here, but I find it fascinating, none the less.

R Stevenson
May 16, 2011 4:21 am

Wayne in answer to your question re my equation for absorption of LWIR – below is part of spreadsheet (out of alignment slightly) showing band energy absorbed after 10, 1000, 2000m etc as emissivity and absorptivity progressively change.
Emissivity and absorptivity
Gray body
L metres 10 1000 2000 3600
Nominal gas temp deg C ‘C4’ 20 20 20 20
Emissive power entire spectrum 119.8 119.8 119.8 119.8 Btu/hft^2
Absorption by CO2 of land IR L m 4.9 20.1 23.6 24.4 Btu/hft^2
Total absorbable energy in bands 24.4 24.4 24.4 24.4 Btu/hft^2
Energy absorbed by gas 4.9 20.1 23.6 24.4 Btu/hft^2
Energy remaining after L m 19.5 4.2 0.8 0.0 Btu/hft^2
Typical equation for energy absorbed by gas is:
(1+0.9)/2*o.193*0.1713*10^-8*(20*1.8+492)^4

R Stevenson
May 16, 2011 4:49 am

Wayne, this a similar spreadsheet for water vapour showing a much shorter distance 120m to absorb 64% of the land LWIR
Emissivity and absorptivity
Gray body
L metres 10 100 120 120
Nominal gas temp deg C 20 20 20 20
Emissive power entire spectrum 119.8 119.8 119.8 119.8 Btu/hft^2
Absorption by H2O of land IR L m 39.9 75.9 76.3 76.3 Btu/hft^2
Total absorbable energy in bands 76.4 76.4 76.4 76.4 Btu/hft^2
Energy absorbed by gas 39.9 75.9 76.3 76.3 Btu/hft^2
Energy remaining after L m 36.5 0.4 0.0 0.0 Btu/hft^2
In this case H2O absorptivity ranges from 0.300 to 0.573

Joel Shore
May 16, 2011 5:36 am

jae says:

It seems clear to me that the “jury is still out” regarding even the most basic, simple part of the CAGW junkscience: the GHE theory.

As a physicist, it seems clear to me that the “jury is still out” on this to about the same degree as the jury is still out in regards to whether the earth is 6000 years old or 4.6 billion years old. There is absolutely no serious debate in regards to these things in the scientific community; the debate only exists among those who are unwilling to accept scientific evidence that goes against what they desperately want to believe.
I think that what our experience with the whole issue of human origins demonstrates is that whether or not science is doubted by a significant portion of the population, including some trained scientists, depends not on the strength or weakness of the science but instead on whether the science challenges strongly-held religious or ideological beliefs. If it does, then there are people who will argue the science incessantly as a proxy; but their real objections have nothing to do with science.

wayne
May 16, 2011 5:53 am

R Stevenson, thank you very much. This is in a bit of mixed units. No problem at all. However on the final equation I have to guess a bit, I recognize the 0.1713*10^-8*(20*1.8+492)^4 as being the SB in English, Rankine for the temperature, but could you identify the values in the term (1+0.9)/2*o.193 as to what those values are? I don’t immediately recognize what the 0.9 and 0.193 are, or the relationship, but overall that seems to give the total absorbable energy in all bands.

wayne
May 16, 2011 6:39 am

jae, contraire! It matters hugely to some here. I am just realizing that as I extend that new relationship to Venus. The surface is dry, not wet, so when you go to integrate the flux on Venus from the BOA to the TOA you notice the huge difference between Earth and Venus. On Earth you have 168 reaching the surface (KT97 numbers, I think everyone here has them memorized, so without words) of which 78 is immediately subtracted to get the upward flux without the evapo-transpiration influence of water on Earth. Venus has no such subtraction term from the BOA flux upward, it does have dry thermals upward, and to a greater amount partially also because of the lack of water on Venus since the temperature is kept elevated, but still small compared to what evaporation of water would do if it were present. If Venus still had water the temperature would be hugely less at the surface, even less than 370 K, not 740 K. That’s my take so far.

May 16, 2011 7:34 am

Tim Folkerts says:
May 12, 2011 at 12:32 pm
Mkelly,
The above from an astute person shows clearly that if two objects of the same temperature are radiating at each other there is no (none, nada, zippo) energy exchange HEAT (ie no NET energy exchange). You even say specifically that they are radiating at each other; since radiation = traveling EM energy, you just admitted that energy is being exchanged in the very sentence where you say there is no energy exchange.
q is zero so no energy is exchanged.

Joel Shore
May 16, 2011 7:53 am

mkelly says:

q is zero so no energy is exchanged.

This is wrong…but no matter.
At least, we both agree there would be no heat flow from the earth to the atmosphere if the two are at the same temperature. I think we also both agree that there is heat flow between the sun and the earth. Given such a state of affairs, i.e., the earth absorbing heat from the sun and no heat leaving it (in the hypothetical case of an atmosphere at the same temperature as the earth), what would happen: Would the earth remain at the same temperature or would its temperature increase?

May 16, 2011 8:05 am

Joel Shore said on Visualizing the “Greenhouse Effect” – Light and Heat
May 14, 2011 at 6:55 pm
Why? How does the Second Law work in your picture of the world? Is there a little “Maxwell’s Demon” that stops a photon and asks where it came from and where is it’s going and says, “I am sorry, sir, but you can’t travel in this direction”? Do objects refuse to radiate if toward any hotter objects that they see or do the hotter objects just refuse to accept photons from objects that are colder?
Alleyne replies:
Nope, no little demons.  I rather considered the energy radiated from a colder object to act very much like a neutrino, but to pass through matter without affecting it, unlike a neutrino which only very rarely affects it.
Joel Shore said on Visualizing the “Greenhouse Effect” – Light and Heat
May 14, 2011 at 6:55 pm
Why do you believe your conception of the Second Law is correct and the conception of physicists, who actually work intimately with it and understand its derivation from statistical physics principles, is wrong?
Alleyne replies:
The books I have read were written by physicists and chemists who were well versed in, and worked with, these principles.  Admittedly most of what have learned was a long time ago and didn’t include statistical thermodynamics.  As I have said before, I am merely asking questions and making arguments which I hope will help elucidate your argument and from which I may learn something. I don’t know who you are, or what your qualifications are.  The books I have read and the education I have had all taught me that energy flows from regions of high energy to regions of low energy and that heat doesn’t flow from cold to hot, and heat being the transfer of energy ergo energy doesn’t flow from cold to hot.  You are telling me something different.  You recently chastised someone for believing what Hertzberg and others of his ilk said on the internet, surely you don’t expect me to accept everything I am told over the internet without question?  Or is it just because you are saying it that makes it OK 🙂  Not trying to be offensive, just suggesting that you not take these questions so personally.  Not everyone on this site is a physicist or a climate scientist.  There are people like myself that have an interest in the subject and wish to learn more.  If you are indeed a physicist who actually works intimately with and understands these things – wonderful, I may then learn something…  And I can appreciate your frustration, if you are one, however I consider the tone of your comments uncalled for at times.  There are undoubtedly people who come to this site with as much intelligence, expertise and learning in their field of endeavour as you have in yours.  The sarcasm, while you may consider it merited at times, does nothing to elevate the level of the discussion.  If I am misinterpreting your comments, easily done over the internet where tone of voice and affect are not evident, I apologize.
Joel Shore said on Visualizing the “Greenhouse Effect” – Light and Heat
May 14, 2011 at 6:55 pm
Yeah…Well, that’s pretty clever but also impossible to implement. How do you direct the energy radiated from the cold sink back to the hot source without having the hot source radiate energy to the cold sink? That’s the whole point: Such a scenario is impossible. The Laws of Radiative Transfer, coupled with Kirchkoff’s Laws regarding emissivity and absorptivity guarantee that this can’t happen.
Alleyne replies:
Well that is substantially a thought experiment from classical thermodynamics, modified to return the energy radiatively to the hot source instead of in the unspecified fashion of the original.  Would Kirchoff apply in this case?  As we drew off energy and performed work with it the system would no longer be in radiative equilibrium and I thought Kirchoff only applied in that case, or is this another misconception on my part?  A lot of the thought experiments in classical thermodynamics are impossible to implement, perfect gases, perfect insulators etc… however the conclusions reached through them are valid and the basis for your statistical thermodynamics.  I am not trying to invent a perpetual motion machine, that is part of what I have been objecting to with respect to energy flowing from a cold source to a hot source.
Joel Shore said on Visualizing the “Greenhouse Effect” – Light and Heat
May 14, 2011 at 6:55 pm
I think I see where you might be getting confused: You seem to think that I am just saying as long as some overarching process has flow from hot to cold, we can have different separable sub-processes that have flows in the other direction.
I am not saying that…
Alleyne replies:
I understand what you are saying now, thank you.  I need to think on it and do some reading on statistical thermodynamics.  Please don’t throw up your hands in disgust 🙂 but I just wonder, if it is a question of photons (as you say) travelling from a cold object to a hot object, and these photons are, on average, lower energy than the average energy of the matter they are travelling toward, wouldn’t it require, in order to transfer all the energy emitted from the cold source to the hot source, that they impinge upon only the lower energy molecules in the hot object?  Seems statistically unlikely.  I am not arguing against what you say, just trying to understand the mechanism…  I have always known an object cools more slowly and reaches a higher equilibrium temperature in a warm surroundings than a cold one, but had considered that an effect of gradient, not as you say a result of energy input from the colder surroundings.  
Joel Shore said on Visualizing the “Greenhouse Effect” – Light and Heat
May 14, 2011 at 6:55 pm
Why would you jump to the conclusion that the physics textbooks are wrong on a very fundamental and basic matter of physics and that the scientific consensus is wrong in using equations that have been tested by scientists and engineers and used to build all sorts of technology rather than acknowledge that your own knowledge may be what is limited here?
Alleyne replies:
Because there have been some very bad textbooks and textbooks have been wrong before on basic and fundamental matters.  However I really don’t wish to get OT and discuss that here, so let us agree that intelligent, questioning, skepticism is scientific.  I do, and have, acknowledged that my knowledge is limited in this area, I never claimed otherwise.  I’m not sure I would use the term ‘scientific concensus’ though.  Scientific concensus has a bad track record, apart from which I always understood that science wasn’t concensus based but founded on empirical data and repeatable experiments.  However this too is OT and more a question of ideology than science, so let us leave it for another time.
Joel Shore said on Visualizing the “Greenhouse Effect” – Light and Heat
May 14, 2011 at 6:55 pm
Yes…You are misunderstanding. You have put in a negative sign so that you end up subtracting numbers that you should be adding. You correctly calculate changes in entropy and then, instead of adding them together to get the total change, you subtract one from the other.
Alleyne replies:
Oooops, my apologies, you are entirely correct, I was apparently too tired to think straight after a long day in the field.  For some reason, though I did say you have to sum them, I subtracted them – my bad.
Thanks for taking the time to answer my questions.

wayne
May 16, 2011 8:10 am

Joel Shore says:
May 8, 2011 at 12:06 pm
You [Martin Mason] clearly do not know the basis for the Second Law, which is statistical physics. You seem to believe in some sort of magical version of the Second Law. The actual Second Law is based on the fact that although energy transfers occur in both directions, by simple statistics it becomes essentially astronomically improbable that the net flow of energy, what we call “heat”[,] will be from the hotter body to the colder body for any macroscopic bodies.
———
I don’t think it improbable that the net flow of energy will be from the hotter body to the colder body. I think Martin doubts it improbable too.
I am going to take the moment to claim a “Get Out of the Warmist’s Jail Free” card for myself and Martin. Even those who claim statistical thermodynamic and physics superiority make such asinine statements themselves. I didn’t even have to twist your words. Joel, being intelligent I realize you surely did not mean that (but give us the cards anyway) ☺ and lash yourself to that high-horse next time.

R Stevenson
May 16, 2011 8:42 am

Joel Shore said:
Given such a state of affairs, i.e., the earth absorbing heat from the sun and no heat leaving it (in the hypothetical case of an atmosphere at the same temperature as the earth)**, what would happen: Would the earth remain at the same temperature or would its temperature increase?
**Heat would still leave the atmosphere for outer space, I would have thought even though there is hypothetically no interchange between atmosphere and earth.

Bryan
May 16, 2011 8:44 am

Joel Shore
jae says:
It seems clear to me that the “jury is still out” regarding even the most basic, simple part of the CAGW junkscience: the GHE theory.
Joel Shore says
There is absolutely no serious debate in regards to these things in the scientific community.
I say
The science community have NEVER spent any time discussing the “greenhouse theory”.
A tiny clique called the “team” numbering no more than a dozen people in climate science developed this threadbare “theory”.
From this remote and uninteresting branch of science what started out initially as an innocent “spicing up” their papers to get funding has ballooned into a monster.
The average person in the world will see their living standards reduced to pay for the dislocation of the world economy.
Do you think that say Martin Rees Astronomer Royal ever sat down to seriously study the pros and cons of the greenhouse theory?
Or any other notable scientist who was not directly linked to the IPCC bandwagon?
The position of most scientists not in the field would be to assume good faith in those who were.
The climategate tapes indicated that there was something rotten in this branch of science.
It was a wake up call for all in a position to have a critical look at the area.
The hallmarks of the “team” set them apart from the rest of the real science community.
1. Withholding data.
2. Interfering with peer review process
3. Lobbying the media to exclude critical opposition
4. Crude cut and paste graphs (Mann)
Back to the present situation
Joel you know perfectly well that there is no substantial direct heating of the atmosphere by greenhouse gases.
Your simple radiating shells model has gone the way of the plum pudding model of the atom.
Interesting, but of historical interest only.
Some versions of the greenhouse theory are so modest that its a wonder anyone bothered to write them down.
You have witnessed in the last few years several fundamental problems being acknowledged by the IPCC greenhouse theory proponents.
What of the future?
I suspect that even more inconsistencies will be acknowledged .
I would like to see a review of the IPCC conclusions carried out by a dozen or so scientists and statisticians in good standing.
This group should be well qualified but not presently working in the area of climate science.
Effectively it would be a quality audit of the practices of climate science in the previous 20 years or so.

Dave Springer
May 16, 2011 8:50 am

Shore
re; how is this fatal to GHG hypothesis
It wouldn’t matter if downwelling IR was 100% of upwelling IR from the ocean as all the additional downwelling IR does is proportionally increase the evaporation rate in the first micrometer of ocean surface.
Greenhouse warming isn’t thottled by IR absortive gases in the atmosphere over the ocean. It’s throttled by humidity. Over land is a different story but given 71% of the earth’s surface is water that is the lion’s share of the effect. Greenhouse effect is throttled by humidity which determines the rate at which shortwave radiation absorbed by water can escape. Conversely if downwelling IR decreases the evaporation rate decreases and the heat exchange at the ocean/atmosphere boundary remains the same. This is a natural consequence of water being transparent to shortwave and opaque to longwave. Water cannot be heated by longwave IR. All longwave IR does is vaporize liquid water at the surface.
I believe I happened to stumble onto the same hypothesis put forward by Dr. Ferenc Miskolci “The Saturated Greenhouse Effect”
http://www.scribd.com/doc/25071132/The-Saturated-Greenhouse-Effect-Theory-of-Ferenc-Miskolczi
The saturated greenhouse effect is supported by 50 years of TIGR data i.e. radiosonde soundings of the troposphere which reveal that as CO2 level rises water vapor content of the atmosphere falls and thus the total GHG content of the atmosphere remains constant. Miskolczi goes beyond my analysis by looking at what happens to the water vapor after it leaves the ocean surface. It is of course carried upward by convection where it condenses (releasing latent heat far above the surface) and falls back as rain. In a nutshell Miskolczi discovered through analysis of radiosonde sounding data that increased CO2 causes the speed of the water cycle to increase propotionately with the net result being a slightly dryer atmosphere and no change in total greenhouse warming.
This is same claim that I made i.e. that on a water world the ocean is responsible for greenhouse warming and amount of downwelling radiation from GHGs is inconsequential due to the consequences of liquid water being a greenhouse fluid and unlike the atmosphere there is, for all practical purposes, an infinite reservior of greenhouse fluid in the global ocean. In other words if non-condensing atmospheric greenhouse gas content changes the ocean reacts by changing condensing GHG in the opposite direction maintaining a constant maximal greenhouse effect.

JAE
May 16, 2011 8:52 am

Joel:
“There is absolutely no serious debate in regards to these things in the scientific community; the debate only exists among those who are unwilling to accept scientific evidence that goes against what they desperately want to believe.”
More armwaving. Your definition of “serious debate” differs from mine. More “consensus science” crap? No wonder you are so certain about the GHE, too.

R Stevenson
May 16, 2011 9:06 am

Wayne asked
‘could you identify the values in the term (1+0.9)/2*0.193 as to what those values are? I don’t immediately recognize what the 0.9 and 0.193 are, or the relationship, but overall that seems to give the total absorbable energy in all bands.’
0.9 is land emissivity and could be ignored (assume BB) . 0.193 is CO2 absorptivity
at PcL of 4.13406 where Pc is the partial pressure of CO2 (0.00035at) and L is the mean beam length 11812ft (or hemispherical gas mass rad L =3600m) corrected for
temperature PcLTs/Tg.

Dave Springer
May 16, 2011 9:17 am

Shore (con’t)
The red flag went up in my mind when the first paper I cited “Tropical Atlantic Mixed Layer Heat Budget” showed that 25% of insolation energy is removed by LWIR, 5% by conduction, and 70% by latent heat of vaporiation. From other reading, also cited in this thread, only 23% of surface heat loss is assigned to latent heat. If that number is in reality much larger then it drastically changes the surface temperature response because latent heat by definition doesn’t register on a thermometer. It is invisibly carried upward by convection until the vapor condenses adiabatically. So if the lion’s share of insolation energy is removed by latent heat we won’t see any temperature rise near the surface. It’ll happen far above the surface at the cloud layer where the GHGs below the cloud layer now serve to insulate the surface from downwelling IR from the cloud.
You see, you can’t have your cake and eat it too. If GHGs act as insulation between surface and space due to LWIR opacity they must also act as insulation from space to surface for any downwelling LWIR. So if latent heat transport carries a lot of energy aloft, drilling straight through the densest layer of GHG’s like it wasn’t there then when the latent heat becomes sensible again the dense layer of GHGs below it must impede its return to the surface in effect making the easiest radiative path for LWIR emittance by clouds straight up to space.
It was the magnitude of the measured latent heat loss from the ocean in that Atlantic heat budget which inspired me to say it’s fatal to the anthropogenic GHG warming hypothesis because latent heat carries energy away from the surface without sensible warming of near surface air.

May 16, 2011 9:20 am

Joel Shore says:
May 16, 2011 at 7:53 am
mkelly says:
q is zero so no energy is exchanged.
This is wrong…but no matter.
Sorry Joel, let change q to the more oft used q/A or W/m^2. If one side of the equation is zero then the other is too. So no energy exchange. Watt is joule per second and a joule is a unit of energy I will let my statement stand.
Mr. Springer, although you have disparaged me, I am in general in agreement with you concerning water vapor and the oceans having a large influence on the atmosphere that far exceeds CO2.

Dave Springer
May 16, 2011 9:41 am

@ Joel Shore (con’t)
Another way describing the latent heat effect is that increasing CO2 will not change the dry adiabatic lapse rate but it does change the saturated adiabatic lapse rate. Increasing water vapor content decreases the dry adiabatic lapse rate what should be observed with increasing CO2 is a lower lapse rate from surface to cloud layer and increased lapse rate from cloud layer upward. So the net effect is not surface warming but cloud layer warming. I believe this is the notorious “tropical tropospheric hotspot” that GCMs predicted should be found. Where they fail I believe is not at this level but rather in inaccurate modeling of cloud formation and effects on radiation balance i.e. they presume that more clouds cause additional average forcing at the surface and insufficiently account for loss of surface forcing due to high albedo of the cloud tops.
Two presumptions in GCMs that I believe are erroneous are constant average absolute humidity and constant average albedo. Actual instrument measurements of humidity (TIGR) and albedo (Earthshine) indicate they are not the constants they are presumed to be.

wayne
May 16, 2011 10:08 am

R Stevenson, seems I’ve got it all, thanks again.

wayne
May 16, 2011 10:26 am

Dave Springer, now you are speaking real sense in the last few comments. I’m just nodding as I read. If you have time read back on some of my comments and those revised 24hr T&K spreadsheets near the top and you should find they fit to what you have been saying just fine. Miskolczi’s figures as a 24hr spread are quite a bit off of T&K’s but the general view is the same, his shows more atmosphere absorption of SW, less of window LW. (if you find mistakes, let me know, I am human too)

Dave Springer
May 16, 2011 10:35 am

@ Joel Shore
At any rate no matter who is right or wrong regarding AGW the cautionary principle compels looking at what practical ramifications come with AGW.
As far as I can determine the net practical effect of AGW is all reward and little risk. Obervation and theory both support an AGW effect over landmasses at times when temperatures are the coldest and the air is the dryest. In other words we should expect AGW to be milder winters in the high latitudes. If you bother to ask people who live in high nortern latitudes whether they’d welcome milder winters the answer is overwhelmingly yes. Few people enjoy harsh winters and the plants and animals that struggle to survive in sub-freezing temperatures, if they could talk, would certainly vote for milder winters as well. Adding substantially to the benefits of milder northern winters (extended growing seasons, cattle farms and apple orchards in Greenland, and things of that nature) is the effect of increased CO2 on primary production in the food chain (green plants). Higher levels of CO2 almost universally accelerate plant growth rate and at the same time reduce water use per unit of growth. These are fantastically good things and indeed we are already enjoying the fruits (pun intended) of going from 280ppm to 380ppm CO2 and the benefits will continue to accrue at least up through 2000ppm. In the best possible scenario it would end the ice age and allow the earth to return to it’s historically predominant state of being green from pole to pole. That’s a good thing unless for some strange reason you prefer barren rocks and ice to forests and grasslands. I sure prefer a living world to a dead one!
So what’s not to like about AGW? Rising sea level is about it. The thing of it is that sea level will rise very slowly giving us and the rest of the living world many centuries in which to slowy adapt. There is no evidence at all that sea level will increase faster than the current ~3mm/year or about a foot per century. In fact there’s no evidence at all the largest store by far of ice (Antarctica) is reducing at all. If anything it’s getting colder there and continental AGW is confined to the northern hemisphere.
Contrast these great net benefits of continually rising CO2 with what it would cost to stabilize it at current level. The industry that makes if possible for a rising human population to enjoy rising standard of living is driven in large measure by fossil fuel consumption. Anything that makes fossil energy more expensive and/or less available will result in a fall of living standards and the poor will take the brunt of it because they lack the excess disposable income to absorb higher energy costs.
The only possible conclusion for me is that anyone armed with the objective facts surrounding AGW who supports draconian measures to reduce fossil carbon emissions has an agenda driven by misanthropy. The rest who support are simply don’t have the facts at hand to reach a reasoned decision. I’m not trying to influence how people decide but am rather just trying to get the facts on the table trusting that most people will make the right decision once they know the real story instead of the hysterical misrepresentations peddled by miscreants like Al Gore, James Hansen, Michael Mann, what’s his face Pachauri (head of the UN IPCC), and the rest of the usual suspects.

Dave Springer
May 16, 2011 10:47 am

@ Ira
Thanks for the offer of publishing an article I might want to write regarding an oceanic greenhouse but I’m already approved for author level on WUWT and can do it myself. It’s not unfamiliarity with WordPress publishing either as I was the top level adminstrator on a high traffic wordpress blog for several years. During that time I composed and published hundreds of articles.
The bottom line is I get more satisfaction being on an equal footing with regular subscribers. Authors get advantages that make for an unlevel playing field and I’d prefer mine to be level.
That said you are of course welcome to investigate and publish something yourself if you think it’s warranted and I will, as usual, be a prolific commenter in the thread as greenhouse physics fascinates me.

Dave Springer
May 16, 2011 11:12 am

jae says:
May 15, 2011 at 7:10 pm
“It seems clear to me that the “jury is still out” regarding even the most basic, simple part of the CAGW junkscience: the GHE theory.”
Take your blinders off. The “jury” has been “in” regarding the longwave absorptive and emissive properties of greenhouse gases for over 150 years since physicist John Tyndal proved it experimentally. CO2 level sensors commonly employed in commercial building ventilation systems use exactly the same physics that Tyndal provied in his experiments. Infrared thermometers which today are inexpensive thermometers allow the amateur experimenter with the most limited budget to confirm that the sky glows in the infrared. More expensive infrared spectrometers outside the amateur budget looking upward from the ground and downward from satellites also confirm what Tyndal found only with far better ease, accuracy, and precision.
You’re in denial – either mentally or emotionally unable to accept physics facts derived by both theory, experiment, and widespread employment in practical applications.
A common expression from my time in the USMC comes to mind: What the f**k is your major malfunction?

May 16, 2011 11:27 am

R Stevenson says:
May 16, 2011 at 9:06 am
Mr. Stevenson glad to see someone else talking emisivity of CO2. However, if memory serves me when H2O and CO2 are both available my heat transfer book had
(Pw/(Pw+Pc)-1) as the combination emissivity. Pw is partial pressue of water vapor and Pc partial pressure of CO2. And at low temperatures less than 500 R the emissivity of CO2 is very low. Granted this is from memory and the Hottel charts are a touch vague in my mind.

Joel Shore
May 16, 2011 11:42 am

mkelly says:

Sorry Joel, let change q to the more oft used q/A or W/m^2. If one side of the equation is zero then the other is too. So no energy exchange. Watt is joule per second and a joule is a unit of energy I will let my statement stand.

As I pointed out, it doesn’t matter what you say: If you take your logic to its obvious conclusion, it shows the greenhouse effect because the earth receiving energy from the sun must emit heat. You have just admitted that this won’t happen as long as the atmosphere is the same temperature as the surface. So, the surface will increase its temperature…In fact, it has to increase its temperature until the difference in temperature between the atmosphere and the surface is enough to support heat flow away from the earth’s surface that is equal to what it receives from the sun.
RStevenson says:

**Heat would still leave the atmosphere for outer space, I would have thought even though there is hypothetically no interchange between atmosphere and earth.

You are correct that you will eventually have to solve for what the final temperature of the atmosphere will be. However, the point is simply this: As long as you have an IR-absorbing atmosphere that is at a nonzero temperature, the earth’s surface will have to be at a warmer temperature (in order to radiate away the energy that it receives from the sun) than it would be if the atmosphere did not absorb any of the IR radiation that the earth emits.
There is no reason why we ever have to talk about “back-radiation”. As long as you emit that the heat flow between two bodies at different temperatures depends on the temperature of the colder body as well as the hotter body, then the game is over (and you’ve lost). It’s as simple as that.

wayne
May 16, 2011 11:56 am

Ira Glickstein, PhD says:
May 15, 2011 at 11:28 am
I agree that some of the downwellng radiation comes from very low reaches of the Atmosphere, but why does that make a difference in this discussion? Does it matter to the argument that hundreds of Watts/m^2 pass from the Atmosphere to the Surface due to the ill-named “greenhouse effect”?

Ira, you still don’t see what I am saying. You agree that SOME of the downward pointing radiation comes from the low reaches of the atmosphere but I am saying ALL of the downward radiation measurable at the surface is from the low reaches (just 10’s of meters) EXCEPT (minus) that which can and does go upward to space and that is called the “window” frequencies upward IR radiation.
Re-read my statements above, I am not saying some, I am saying all. This is so because the atmosphere is totally opaque to these frequencies outside any particular frequency that you could call a “window” frequency. The window frequencies are a class of frequencies, not necessary a contiguous band, some lines may be anywhere and all are not total but partial by their exact measured radiance. It is the sum of all of these partial window frequencies together that make up the upward “window IR radiation”. That is what Dr. Miskolczi went to great lengths to accurately measure, not only once, but at different seasons and different latitudes at many points on the globe. Now do you see?

Joel Shore
May 16, 2011 11:59 am

Dave Springer says:

The saturated greenhouse effect is supported by 50 years of TIGR data i.e. radiosonde soundings of the troposphere which reveal that as CO2 level rises water vapor content of the atmosphere falls and thus the total GHG content of the atmosphere remains constant.

…And, contradicted by the satellite data that show the opposite. And, it is understood that the trends in the radiosonde data are artifacts due to instrumentation that was never designed to look at trends of this sort.

The red flag went up in my mind when the first paper I cited “Tropical Atlantic Mixed Layer Heat Budget” showed that 25% of insolation energy is removed by LWIR, 5% by conduction, and 70% by latent heat of vaporiation.

I have noted to you ( http://wattsupwiththat.com/2011/05/07/visualizing-the-greenhouse-effect-light-and-heat/#comment-660881 ) that, as near as I can tell, your whole notion here is based on a simple misreading of what they said…They talked about “net LWIR” and you somehow misinterpreted as just the emitted part.

I believe this is the notorious “tropical tropospheric hotspot” that GCMs predicted should be found.

So, you seem to be predicting either a much bigger “hot spot” than what the models predict or some sort of cloud-albedo feedback that you have provided no real evidence for.

Two presumptions in GCMs that I believe are erroneous are constant average absolute humidity and constant average albedo. Actual instrument measurements of humidity (TIGR) and albedo (Earthshine) indicate they are not the constants they are presumed to be.

Climate models make neither of those assumptions. To the extent that either of these end up being constant in the models, they are predictions that come out of the physics in the models, not assumptions.

At any rate no matter who is right or wrong regarding AGW the cautionary principle compels looking at what practical ramifications come with AGW.

Your conclusions about the impacts essentially fly in the face of the actual science on impacts (or, at best, look at only a small part of the impacts). It is too far afield to get into impacts in this thread, but to believe that you understand the impacts better than the scientists studying them is not realistic in my view.

Joel Shore
May 16, 2011 12:08 pm

Dave Springer says:

From other reading, also cited in this thread, only 23% of surface heat loss is assigned to latent heat. If that number is in reality much larger …

The only way it could be much larger is if precipitation was way-underestimated because you can get the amount of energy carried away from the surface as latent heat just by knowing the amount of water evaporated, which is equal to the amount precipitated, as long as you average over a reasonable period of time. Maybe you can try to make some argument at the margin (i.e., for a change) but you certainly can’t say that latent heat transport is current much larger.

Editor
May 16, 2011 12:23 pm

richard verney says:
May 12, 2011 at 8:39 pm

“The upshot of the above is that I do not dismiss out of hand the contention that GHGs may contribute something to the warming of the atmosphere but I am very skeptical that the Earth is 33K warmer because of the presence of GHGs in the atmosphere. I consider that such a contention is far from proven both on figures and on principles involved.”

It depends on your assumptions regarding clouds and albedo. Clouds or no clouds? Surface albedo or blackbody? Here’s the calcs, assuming a solar “constant” of say 342 W/m2 global 24/7 average.
Actual average temperature: ~ 14°C
No atmosphere blackbody temp: 6°C
Less – cloud albedo: -10°C
Less – surface albedo: – 18°C
The final figure is the one usually quoted, and gives the ~ 33K warming you cite above. The net effect of adding our existing atmosphere to a blackbody no-atmosphere earth is much smaller, about 8°C.
Dave Springer says:
May 13, 2011 at 6:05 am

… I question whether the GHE is soley in the atmosphere. The ocean has the same properties of being transparent to visible light and opaque to infrared light that greenhouse gases exhibit. The sun warms the ocean to a depth of about 100 meters during the day but it cannot radiate infrared from a depth greater than 1 micrometer. So the energy absorbed from the sun at depth must find its way to the surface by some means other than radiative which of course leaves only convection and conduction. The question is whether convective and conductive cooling are more or less efficient than radiative heating and under what conditions. I suspect the net effect under most circumstances is that a substantial amount of greenhouse warming is contributed by the ocean alone.

I begin my mental investigation of this claim by saying “warming compared to what”? My first comparison would be to compare a given volume (say ten thousand cubic metres, 100m x 100m x 100m) of ocean with the same volume of land. We’ll assume that they have the same albedo.
In both cases, they have the same surface area for radiation of the energy. And both absorb and radiate infrared (IR) in the same manner.
But the ocean can absorb energy much more efficiently because the light penetrates the upper layer.
On the other hand, the ocean can also lose energy much more efficiently because every night it overturns, bringing warmth to the surface. In addition, it loses more energy from evaporation than does the land.
So the question becomes, in a constant day-night alteration of temperature, what would be the final temperature of the two (ocean and land). If an “oceanic greenhouse effect” exists, the ocean should be warmer.
NEGLECTING EVAPORATION and ASSUMING EQUAL CONDUCTION/CONVECTION LOSSES, I’d say the final temperature of the two, ocean and land, would be the same. I reason thusly. They are both absorbing the same amount of energy. They both have the same identical ways to lose energy (radiation and conduction/convection). Therefore, they’d have to end up at the same temperature.
And that means that including evaporation, I’d say the ocean would end up cooler than the land.
So I’d say no oceanic greenhouse effect … but I must confess I haven’t thought too deeply about the question.
w.

R Stevenson
May 16, 2011 12:24 pm

mkelly says
‘(Pw/(Pw+Pc)-1) as the combination emissivity. Pw is partial pressue of water vapor and Pc partial pressure of CO2. And at low temperatures less than 500 R the emissivity of CO2 is very low. Granted this is from memory and the Hottel charts are a touch vague in my mind.’
Emissivity of CO2 depends on PcL as well as tempertature. At 500 R it can be as high as 0.2 for a PcL of 5.0 ft.atm. When CO2 and water vapour are present together a correction is made to reduce the sum of their emissities using a plot of correction factor v Pw/(Pc+Pw) for particular temperature/PcL +PwL values. In the case of atmospheric calculations with water vapour so dominant I’m not sure how much effect a correction would have.

wayne
May 16, 2011 12:34 pm

Ira, I said ALL and I have told myself over and over to be real careful when saying “all”, “total”, “100%” when talking physics. Please change my word “all” above to “so close to all that the small difference remaining doesn’t matter at the current precision we are speaking of (2-3 digits)”. See how many words it takes to say it properly? I tend to assume someone else is going to know “all” in that particular case of radiation is not absolutely 100%, totally every single photon with no exceptions, for in radiation that so rarely happens when speaking of gases. Sorry.

wayne
May 16, 2011 1:12 pm

I’d have to agree with Willis though there are those absolute words again “same”, I’d just say “very close to the same” and I’m automatically assume Willis meant the same.
Willis, thanks for just reminding everyone here that you cannot have lopsided thoughts touching physics, there is an other side of the equation you have to worry about an account for except in the rarest of cases. If there is a plus effect, look for the minus effect on the other side and account for that too.
I even think this same thought applies to CO2. With increased CO2 there will be more absorption of energy within the atmosphere in the CO2 bands in SW frequencies. Anyone following what I have been saying, that any energy that is “in the atmosphere” is destined to be ejected to space, that net energy cannot find itself EVER warming the surface because lower in the atmosphere is always warmer and ‘that’ net energy (heat) cannot travel downward from cooler to warmer. So, that additional SW absorbed due to the CO2 will cancel some, if not all, or even more, of the absorption upward in LW IR. You never hear about that being accounted for either.

wayne
May 16, 2011 1:19 pm

We could use a new thread… i..t..’..s g..e..t..t..i..n..g s..l..o..w..!..!

May 16, 2011 1:38 pm

Joel Shore says:
May 16, 2011 at 11:42 am
mkelly says:
Sorry Joel, let change q to the more oft used q/A or W/m^2. If one side of the equation is zero then the other is too. So no energy exchange. Watt is joule per second and a joule is a unit of energy I will let my statement stand.
“As I pointed out, it doesn’t matter what you say: If you take your logic to its obvious conclusion, it shows the greenhouse effect because the earth receiving energy from the sun must emit heat. You have just admitted that this won’t happen as long as the atmosphere is the same temperature as the surface. So, the surface will increase its temperature…In fact, it has to increase its temperature until the difference in temperature between the atmosphere and the surface is enough to support heat flow away from the earth’s surface that is equal to what it receives from the surface.”
I agree. Never said anything different. But it is temperature gradient that matters period. Not back radiation as you and Ira say. I have said several times the two requirements for heat transfer are path and temperature gradient.
All gases absorb heat from the surface so we must account for them and 99% give no/little IR radiation.
I also said now lets get to the temperature of the atmosphere you want to use and the emissivity etc to further the discussion.
Thanks for admitting I was correct since you used “temperature” 4 times and not once back radiation.

Joel Shore
May 16, 2011 1:39 pm

Dave Springer says:

If that number is in reality much larger then it drastically changes the surface temperature response because latent heat by definition doesn’t register on a thermometer. It is invisibly carried upward by convection until the vapor condenses adiabatically. So if the lion’s share of insolation energy is removed by latent heat we won’t see any temperature rise near the surface. It’ll happen far above the surface at the cloud layer where the GHGs below the cloud layer now serve to insulate the surface from downwelling IR from the cloud.
You see, you can’t have your cake and eat it too. If GHGs act as insulation between surface and space due to LWIR opacity they must also act as insulation from space to surface for any downwelling LWIR. So if latent heat transport carries a lot of energy aloft, drilling straight through the densest layer of GHG’s like it wasn’t there then when the latent heat becomes sensible again the dense layer of GHGs below it must impede its return to the surface in effect making the easiest radiative path for LWIR emittance by clouds straight up to space.

Besides the fact that we don’t seem to be seeing a much greater-than-predicted temperature rise in the upper troposphere that this sort of picture would seem to need, I think there is a fundamental error that you (and Martin Lewitt in some previous comments) are making here: This error is basically summarized by considering a simple calculation done by L.D. Danny Harvey’s in “Global Warming: The Hard Science”. What he shows is that a change in the radiative balance between the surface and the atmosphere even by a larger amount, such as 10 W/m^2 would result in only a very small surface temperature change while a change in the greenhouse effect (i.e., the radiative balance between the earth and space) by 10 W/m^2 results in a much larger surface temperature change (almost 2 orders of magnitude larger if I recall correctly). The same would presumably be true if instead of a change in radiative balance, you had a change in some other heat transfer process like the water cycle.
The reason is simply this: Because of convection, the lapse rate in the troposphere is primarily determined by the stability limit that it can’t be larger than the (appropriate) adiabatic lapse rate. So, if you “try” to change the temperature structure of the atmosphere by changing the heat flows between the surface and atmosphere, the atmosphere just responds by altering the convection to cancel out most of this change and you end up with basically the same temperature structure you started with (modulo the issues involving the moist adiabatic lapse rate…i.e., the fact that the adiabatic lapse rate changes some with heating, which genuinely does cause a change in the temperature structure and leads to the lapse rate feedback, a negative feedback already included in all of the climate models).
However, if you actually alter the energy balance between the earth and space, as increasing the greenhouse effect does, then the fact that the temperature structure must remain the same leads to a significant increase in surface temperatures.
So, in summary:
Altering the heat transfer between the earth system and space => Significant effect on surface temperatures.
Altering the heat transfer between the earth’ surface and atmosphere => Only a small effect on surface temperatures.
I think this is alluded to, albeit rather cryptically, in the Wentz et al. paper that Martin Lewitt cited above ( http://www.sciencemag.org/content/317/5835/233.full ), and helps to explain why people like Martin have put an interpretation on the paper that you don’t find in the Wentz paper itself or in other papers that talk about how the hydrological cycle changes under global warming. In particular, what Wentz et al. say say is:

For example, variations in modeling cloud radiative forcing at the surface can have a relatively large effect on the precipitation response (4), whereas the temperature response is more driven by how clouds affect the radiation at the top of the troposphere.

Joel Shore
May 16, 2011 1:49 pm

mkelly says:

I agree. Never said anything different. But it is temperature gradient that matters period. Not back radiation as you and Ira say. I have said several times the two requirements for heat transfer are path and temperature gradient.

So, what are we arguing about then? You admit there is a greenhouse effect?

Thanks for admitting I was correct since you used “temperature” 4 times and not once back radiation.

There can be different ways of looking at the same thing. It is not necessarily a matter of one being correct and the other incorrect. The heat flow (i.e., net energy transfer) that occurs is determined by the two objects being at different temperatures.
For radiation, the way that this comes about is that both objects emit radiation, but the hotter one emits more than the colder one. However, if this picture confuses you, then you can just stick to the heat flow equation and don’t have to worry about how it is coming about on a more microscopic level.

wayne
May 16, 2011 4:09 pm

Joel Shore says:
May 16, 2011 at 1:39 pm
Besides the fact that we don’t seem to be seeing a much greater-than-predicted temperature rise in the upper troposphere that this sort of picture would seem to need, I think there is a fundamental error that you (and Martin Lewitt in some previous comments) are making here: …
———–
And here comes all of your bulk AGW science and wrong assumptions IMHO bought in by crappy AGW books. I don’t buy any of it Joel Shore, not without the science, I read your words. You would have to prove it to me one tiny factor at a time and if you are able to make each one bulletproof I would then apologize, if it is true I will usually recognize it as true, or tell you why not, or possibly put on the “just don’t know” stack… not enough data to say either way.
This type of bulk comment is why people are getting skeptical by the droves, the more of this they hear “authority… this author says this, that author says that” the more their gut says no, it makes no sense (observation). I don’t want to hear of some author, don’t care about his name, or his claimed qualifications, I want to hear the science and physics behind his conjectures, each one a tiny step at a time.
You just brought in the concept of trapped heat again, BS and I will show you why if you will let me, there is a very tiny component there but it is small. You just brought in the concept of warped atmospheric profiles by CO2 again, BS and I will show you why if you will let me, it can’t be warped by a GHG that would affect us, it can’t be re-based.
I just felt this was coming. The science Joel, not some book, not some claims somebody made, dig out the science points they are building their claims on and present the science point by point, most here know science well enough to handle it. You know the ones that can’t so let them be.

May 16, 2011 4:13 pm

Dave Springer says on May 15, 2011 at 7:40 am :
“Can someone help me out here? ——- The ocean therefore must be a greenhouse fluid. ———– Can someone help me out here? I’m crowd sourcing for an answer.”
Yes Dave, I can be said to be one in a crowd and I can say I have always thought the “oceans” – that’s plenty of water – may be likened to the little heater that keeps the greenhouse warm at night.
But don’t take to much notice of me as I also believe that Advection i.e. the kind of horizontal air movements that follow isobaric surfaces and therefore are predominantly horizontal) have got more of a Green House Effect (GHE) than does a radiation circuit, of say 324 W/m² originally removed from the surface, and then returned via Green House Gases (GHGs) – which, by the way, show no sign of having warmed at all (no hot spot) But even so, when somehow the same 324 W/m² are delivered back to the surface for absorption it is supposed to be getting warmer.
(Tip – Two bank accounts for frequent money transfers may be enriching)
I hope I have been of some small help to you in your greenhouse fluid research.
PS. Just realised I may be very wrong as in the AGW world there is never night, not as we know it. – Average Solar Irradiation (SI) = 240 W/m², but an average no SI = 0 W/m² or night which would make SI irrelevant when heating by gas happens doesn’t get a look in.

Joel Shore
May 16, 2011 5:57 pm

Ira Glickstein says:

As for temperature increase, when I moved from New York to Florida to gain around 15ºC. How many doublings of CO2 would be equivalent to that temperature increase? Not that I think the world, on average, needs as much as 15ºC, but it it totally ridiculous that the Alarmists (and Warmists) are aghast at the supposed 0.8ºC rise since 1880 they think they’ve measured

The part we are aghast about is not the amount of warming that we have had so far but the amount that we are committing ourselves to in the future.
And, another way to look at temperature changes is this: During the last ice age, when there was like a mile of ice on top of where I currently am, the global temperature was only about 5-7 C colder than it is today. Yes, temperatures of just a few C matter.
wayne says:

This type of bulk comment is why people are getting skeptical by the droves, the more of this they hear “authority… this author says this, that author says that” the more their gut says no, it makes no sense (observation). I don’t want to hear of some author, don’t care about his name, or his claimed qualifications, I want to hear the science and physics behind his conjectures, each one a tiny step at a time.

I explained the basic physics behind it. If you want to see all the gory details, then buy the book and read it. Do you expect to be spoon-fed everything that you want, especially after repeatingly demonstrating that it does no good anyway? Really…we should just be leaving you guys to believe any nonsense that you want to believe!

Dave Springer
May 16, 2011 6:42 pm


Well you may be modestly well informed in the physics department but you suck in biology. The biological effects of CO2 are very well characterized. More CO2 is virtually all upside and no downside. You buy into the hysterics promulgated by Al Gore & company to believe otherwise. That must describe you.

Dave Springer
May 16, 2011 6:52 pm

@Ira
The Hockey Team is doing a fine job consigning themselves to ignomy. The signature of inevitable defeat was when the memo went out saying “global warming” is out and “climate change” is in. They realized at that point the jig was up about global warming. It wasn’t happening at an alarming rate and the pitiful scientific narrative fell apart.
The next shoe fell when “climate change” morphed into “climate disruption” because every fool knows the climate changes and most of them who didn’t sleep through elementary school science classes learned there were glaciers covering the earth 10,000 years ago and these glaciers are overdue for a comeback.
Climate disruption might actually be an accurate term and they should stick with it and hope the next few decades provide some kind of empirical validity to it so at least they won’t go down in the history books as the largest collection of scientific jackasses since Ptolemy’s merry band of epicyclic cipherers were still breathing.

jae
May 16, 2011 7:13 pm

Mr. Springer gets emotional:
“You’re in denial – either mentally or emotionally unable to accept physics facts derived by both theory, experiment, and widespread employment in practical applications.”
Wellll, I think the Title and significant quote from WUWT today helps put you in your deserved place: http://wattsupwiththat.com/2011/05/16/people-underestimate-the-power-of-models-observational-evidence-is-not-very-useful/#more-39837
When you can offer something besides models (calculations which include only a couple of perhaps dozens of variables), then I will bow to your religion.
BTW, the f-word makes you look silly.

Dave Springer
May 16, 2011 7:19 pm


You seem to have lost the plot in the physics of downwelling longwave radiation upon the ocean surface. You can not warm water with LWIR. It is physically impossible. All that happens is the evaporation rate increases. Again following the pesky basic physics involved the water vapor will be at the same temperature as the liquid water which was evaporated. The LWIR energy is in latent heat of vaporization. Once again the physics become inconvenient as water vapor, being lighter than air and thus must necessarily rise carrying all your beloved downwelling LWIR energy back into the sky from whence it came depositing it far from surface when adiatic cooling causes it to condense. Adding insult to injury now at this point (the sadist in me I guess) the ballyhooed insulating effect of greenhouse gases now serves to insulate the surface against the LWIR eminating from the newly condensed vapors and makes the path of least resistance radiative transfer out to the cold dark cosmic void.
Thanks for playing. You were a great sport right up until realized that LWIR cannot insulate a body of water.
If you care to shift your argument to land surfaces we can continue to discuss that because LWIR can certainly insulate rocks. But now that 70% of your potentially insulated surface is gone the GHG effect over land isn’t going to be very consequential. But it’s still good science so I encourage you to continue arguing from your diminished position on dry land.

jae
May 16, 2011 7:22 pm

Addendum to my comment about Mr. Springer goes emotional:
I reread your comment and noticed this factoid of doubtful validity: …”derived by experiment…???
Pray tell where is said experimental evidence?? That is what I crave to get my blinders off!!
The experimental results, so far, show absolutely no OCO-caused warming. Only increases in OCO AFTER warming.
MAYBE, as Misckolski and Wayne seem to think, there is a “ceiling” in the amount of GHE, I can acquiesce. However, the addition of more GHGs to the present system does not appear to be doing squat.

Dave Springer
May 16, 2011 7:52 pm


You say TIGR data is bad because the radiosondes were never designed to measure trends like “that” and satellite data has proven them to be innaccurate.
Well Joel, they’re at least as accurate as the thermometer based surface temperature record.
Fortunately we don’t need accuracy to find trends. All we need for that is repeatability (consistency).
But if you want to play the bad instrument game I allow you to throw out humidity sounding data I’ll have to insist that you throw out the surface station temperature record and only use data available from satellites for that too. I’m a fair guy but won’t tolerate double standards. What’s good for the goose is good for then gander.
Oh sorry, gooses and ganders are biology and we already established biology isn’t your forte. Mibad.

Dave Springer
May 16, 2011 8:06 pm

JAE
Nowhere did I say anything about any net warming caused by back radiation from greenhouse gases. All I claimed was that GHGs do indeed absorb radiation and redirect a portion of the absorbed energy back towards the source. This is the basis upon which most electronic CO2 sensors used in commercial ventilation systems work. This so-called back radiation has been physically measured many times by infrared spectrometers looking up from the surface and down from above. It is even verifiable by anyone who beg, borrow, or steal a $50 infrared thermometer
http://www.amazon.com/gp/product/B0017L9Q9C/ref=pd_lpo_k2_dp_sr_2?pf_rd_p=486539851&pf_rd_s=lpo-top-stripe-1&pf_rd_t=201&pf_rd_i=B000MX5Y9C&pf_rd_m=ATVPDKIKX0DER&pf_rd_r=0A2YZ0RNVN98DZFNB1EG
and verify for themselves that when pointed upward through clear sky it still reads a temperature far closer to the 200-something K of the ground you’re standing on than the 3K temperature of empty space.
If you can’t accept all these things as proof that back radiation is real then you’re a cause and if this was my blog I’d throw your ass out so to reduce the dimwitted clutter that pollutes the more informed converstations.

wayne
May 16, 2011 8:31 pm

@ Ira
“OK, Wayne, let me agree with you that ALL the individual downwelling photons from H2O, CO2, and all other so-called “greenhouse gases” to the Surface comes from “just 10’s of meters”. As a first approximation I am willing to believe that is actually true for some altitude that is a relatively tiny percentage of the total height of the Atmosphere. So let us draw “Wayne’s line”at that level.”
——
Wayne’s line huh? Sorry, no, you keep changing my words every time, not all GHGs, rare GHG’s as CFCs etc have a mean free path that is long (I’m just guessing due to their very tiny concentration), maybe over a kilometer, maybe even more.
But wait…
You keep seeming intent on making me foolish. I did miss including that the ‘ALL’ is what is measured by a radiometer at the surface upward, but you know, I have said that before above. And yes, photons if you insist to stay on the quantum level mix between layers, but I have said that before. There’s “Wayne’s line”, it’s called a division between chosen layers. And as always, there is never net energy passed downward, but I have said that before. I give. You will never understand what I have said, or will say, and I have said one whole lot of comments above. It is time to stop answering your belittling questions. That’s to me is a shame.

wayne
May 16, 2011 8:32 pm

Ira, crx:
There’s no “Wayne’s line”, it’s called a division between chosen layers.

Dave Springer
May 16, 2011 9:11 pm

Shore
I just figured out why my comment about oceanic radiative transfers hit a raw nerve with you.
You were a co-author in a critical comment on this paper:
http://www.tech-know.eu/uploads/Falsification_of_the_Atmospheric_CO2_Greenhouse_Effects.pdf

Section 3.7 discusses the fallacy of radiative balance, from which the following pertinent points are taken:
– For instance, “average” temperatures are calculated for an Earth without an atmosphere and for an Earth with an atmosphere. Amusingly, there seem to exist no calculations for an Earth without oceans opposed to calculations for an Earth with oceans.

I guess I’m not the first to find the dearth of it passing strange. I had no idea that I was echoing comments made by Gerlich et al. It is quite the extraordinary omission.
Seriously I had no idea when I asked your thoughts about it that it was pressing a button on a sore subject. Of course I’d have asked way sooner if I’d known…

wayne
May 16, 2011 10:46 pm

jae says:
May 16, 2011 at 7:22 pm
MAYBE, as Misckolski and Wayne seem to think, there is a “ceiling” in the amount of GHE, I can acquiesce. However, the addition of more GHGs to the present system does not appear to be doing squat.
——
jae, can I clarify? I’m sure that you can’t put Miskolczi’s view with mine, his is a limit on the optical thickness of LWIR through the entire vertical atmosphere.
My concept given above uses the spatial degrees of freedom to calculate the maximum upward flux at a given surface temperature, from the surface upward. I don’t want to tarnish his with mine, if I am off base, could be.
And, my view does not limit the GHE per se, it sets an upper limit on the possible cooling power per a surface temperature and that doesn’t limit the GHE, think of Venus, it insures it’s existence if there are GHG’s present in sufficient enough concentration to have a reasonably short mean free path. But GHE is such a misnomer to me, IMHO it is roughly the local radiative resonance that keeps the air about any point in the atmosphere in radiative and LTE stasis (equipartition across the available degrees of freedom) and therefore maintains maximum warmth and it is performed mainly by GHGs radiation and to a smaller degree, conduction. I though you seemed a bit off my concept.

May 17, 2011 5:07 am

Tim Folkerts says on May 15, 2011 at 6:39 am:
“2) Suppose a wire is hooked to a battery and we look at a cross-section of that wire. Electrons move:
a) only from + to –
b) only from – to +
c) about evenly in both directions. “
And then he goes on to give the answer i.e. :
“If you answer (a), you are thinking of conventional current, not movement of electrons.
If you answer (b), you have had a bit more training in circuits.
If you answer (c), you are correct.”
Dear Tim the batteries that most people operate with produce Direct Current (DC) and you will find that: “Electricity flows in two ways, either in alternating current (AC) or in direct current (DC). The word electricity comes from the fact that current is nothing more than electrons moving along a conductor, like a wire, that has been harnessed for energy. The difference between AC and DC has to do with the direction in which the electrons flow.
In DC, the electrons flow steadily in a single direction, or “forward.” In AC, electrons keep switching directions, sometimes going “forwards” and then going “backwards.”

Joel Shore
May 17, 2011 5:13 am

Dave Springer says:

Well you may be modestly well informed in the physics department but you suck in biology. The biological effects of CO2 are very well characterized.

Then it ought to be no problem for you to find quotes from reputable scientific organizations representing biologists that say that the consequences of increased CO2 (including climate change) are likely to be more positive than negative and that we should put as much CO2 into the air as we can?

Thanks for playing. You were a great sport right up until realized that LWIR cannot insulate a body of water.

I already explained to you at least some of what is wrong with your picture. For all of your pouncing on jae (which is deserved), your views are really only marginally less nutty than his.

But if you want to play the bad instrument game I allow you to throw out humidity sounding data I’ll have to insist that you throw out the surface station temperature record and only use data available from satellites for that too.

Each data set has its own issues and must be analyzed on its own merits. At any rate, the satellite record is large enough to show most of the temperature rise in the latter half of the 20th century into the 21st.

I guess I’m not the first to find the dearth of it passing strange. I had no idea that I was echoing comments made by Gerlich et al. It is quite the extraordinary omission.

If I were you, I would not be proud of this fact! Even you will admit that much of their paper is nonsense, in fact, nonsense to such a degree that agreeing with them on something else ought to in-and-of-itself give you pause!

Dave Springer
May 17, 2011 5:25 am

Joel Shore says:
May 16, 2011 at 5:57 pm

And, another way to look at temperature changes is this: During the last ice age, when there was like a mile of ice on top of where I currently am, the global temperature was only about 5-7 C colder than it is today. Yes, temperatures of just a few C matter.

Unlikely it was anywhere near that little in your neck of the woods. Global warming or cooling isn’t evenly distributed across the globe spatially or temporally. It’s greater over land than over water. It’s greater in higher latitudes than lower. It’s greater in winter than summer.
But hopefully we’re “committing ourselves” to enough to stop those glaciers from returning. The Holocene interglacial period is long in tooth. You knew that already, right?
Did you also know that the indisputable testimony of the geologic column reveals that even when CO2 was 20 times higher than today that global average temperature was only about 7C warmer than today and this is the normal, most stable climate configuration persisting for a hundred million years or more at stretch and furthermore that this is most biologically productive climate configuration with green plants growing from pole to pole?
I have to ask because you haven’t demonstrated much depth of knowledge in geology or biology in the past or present. This is pretty common knowledge for anyone with a wide grounding in science. Hell I knew all that when I was in grade school. It’s encyclopedic and I had the science sections of the World Book Encyclopedia about memorized by the fifth grade. This is all stuff that was well known in the early 1960’s when I was studying it. Maybe studying isn’t a strong enough word. I was inhaling it.

Dave Springer
May 17, 2011 5:44 am

@Ira
Keep in mind that as LWIR migrates upwards there are fewer and fewer GHG molecules left blocking the view to space and more and more GHG molecules below blocking the view back towards the ground.
The insulating effect of GHGs works in both directions so the higher the emission altitude the easier it is to go upwards and the more difficult it is to go downward. That’s why there’s an extinction altitude for 15um radiation a few kilometers off the ground. At that altitude it become impossible for any emissions to make it back to the surface and they rather all head rapidly out to space from there.
Wayne is quite right. At ground level the prepoderance of emissions hitting your upward looking IR spectrometer originated almost close enough to reach up and touch the source.

Dave Springer
May 17, 2011 5:56 am

@Ira
I’d also remind you that when it comes to GHG molecules first molecules in the emission path do most of the work. Tyndall discovered that 150 years ago. I made an analogy to illustrate likening it to the number of people picking apples in an orchard. For a while every additional picker results in a linear increase in harvest speed but after a while there are so few apples left and/or so many pickers they start competing for the same apples and productivity falls off rapidly. So the first molecules in the upward emission path get the most absorption and re-emission done and as you get higher and higher they start competing for the same apples photons and productivity declines exponentially. This also what makes Wayne right about the source of downward emissions – they come from about the height of a tall apple tree or lower.

May 17, 2011 6:13 am

Robert Clemenzi says on May 15, 2011 at 9:59 am:
“O H Dahlsveen says:
May 15, 2011 at 7:00 am
My questions here are: How is it possible for the Atmosphere to radiate 240 W/m² towards space and 320 W/m² towards the surface? – And, what is the “selection method” used by the Atmosphere to enable it to emit more in one direction than it does in the other?
This is rather easy – at some frequencies, the atmosphere is opaque. As a result, the warmer lower atmosphere emits more energy towards the surface than the cold upper atmosphere emits towards space.”
Incorrect Robert, it is not that easy. Radiation is emitted from it’s source equally in all directions. If you do as Joel Shore, Tim Folkerts, Ira Glickstein, Willis Eschenbach and others do and divide the atmosphere up into horizontal layers (all and each one made up, – in our case –not just GHGs but of a mixture of opaque GHGs and other transparent gases) it will mean that once the energy (say 350 W/m²) that relentlessly enters in to the bottom layer from the surface below has had enough energy absorbed for that layer to reach temperature-equilibrium it must emit radiation equally in all directions. And so on – and on, upwards from layer to layer as well as downwards.
If, as according to the plan, 324 W/m² are radiated downwards then 324 W/m² have got to be radiated upwards towards the next layer (hence the origin of the IPCC models’ “Hotspot”.
Do not make the mistaken assumption that the Atmosphere can be likened to a solid piece of, say steel which can easily be warmed more on one side than the other. Such a piece of material will radiate more from the warm side or place than it does from the cooler one.
The Atmosphere progressively gets thinner, or “will contain less atoms and molecules” as altitude increases. That goes for GHGs as much as it does for other gases, which means that radiation will gain more free access to space above in close step with the decrease in opaque gas molecules.
To summarize; “As height is gained each square meter of air will contain less “energy per molecule” but will proportionately let more radiation through from below. – In the end or at the Top of the Atmosphere (TOA), say the Exosphere, each square meter will radiate as LWIR; if the plan is correct, all the incoming/absorbed Solar Irradiation (SI) = 240 W/m² + all the 324 W/m² which the Earth had stored in Oceans, Landmasses and Atmosphere just to keep warm during times when there is no SI which averages near enough 6 months every year. – Due to K& T’s plan we shall soon have a “Snowball Earth””

Dave Springer
May 17, 2011 6:24 am

@Ira
The too many pickers problem is also part of why you can’t slow down the rate of cooling of a body of water with LWIR. The water molecules are so denesly packed in liquid water that the first micrometer absorbs it all. Brownian motion allows the molecules to mix downward another couple micrometers at most but the warmer water is lower density so it wants to fight its way back up to the surface. The end result is there’s virtually no heating beyond the first few micrometers and the molecules near the surface just keep picking up more and more energy as latent heat until they have enough energy to vaporize and then they leave the surface and quickly convect upwards because water vapor is lighter than air. At no point does the average temperature of the vapor exceed the average temperature of the water from which it sprang so the air near the surface never gets any warmer either. The latent heat only becomes sensible again when adiabatic cooling moves it below the dewpoint. This effectively carries off every scrap of downwelling energy up to very near the extinction altitude for 15um and well beyond the extinction altitude of water absorption bands.
We can argue about GHG effect over land which can be substantial but it ain’t a happening thing over water – it’s physically impossible. Liquid water is the uber greenhouse agent to beat all greenhouse agents. Shortwave radiation penetrates easily to a depth of 100 meters while longwave radiation can’t escape except in a billion times thinner layer at the surface and once it escapes it can’t get back in by absorption and reemission by GHGs.
This is why Gerlich et al mockingly said it was amusing that climate boffins calculate the temperature of the earth with and without an atmosphere but don’t calculate it with and without an ocean. The reason is that without an ocean most of the GHG warming disappears. It isn’t the atmosphere doing much of the heavy lifting in that 33C greenhouse warming it’s the ocean doing it.
This giant scientific fraud just keeps getting more and fraudulent the closer you scrutinize it. The biolgical impacts are mostly all bogus, the physics are mostly all bogus, and at this point I’m stuck trying to figure out what small part of the narrative ISN’T bogus. There are grains of truth here and there but they’re not easy to find.

May 17, 2011 6:40 am

Dave Springer,
Thanks for your extremely good series of posts above. Really excellent. The oceans matter, why is that fact being disregarded?

Dave Springer
May 17, 2011 6:44 am

@Ira
Now that I think about it it shouldn’t be very difficult to remove the ocean from the more sophisticated ocean/atmosphere coupled global circulation models.
I know exactly what will happen. Temperature would plummet all over the globe to below freezing, any GHG warming from water vapor disappears because it all freezes out, albedo shoots up to 90% because where there was once an ocean with almost zero albedo there is snow everywhere with a 90% albedo. It would be COLDER than the moon.
Is anyone here proficient enough with GCMs to diddle with the inputs and subtract the ocean entirely? I predict that when you run the model forward from there the earth will be a frigid snowball everywhere within the first two years.

Dave Springer
May 17, 2011 6:59 am

Smokey says:
May 17, 2011 at 6:40 am
Dave Springer,
“Thanks for your extremely good series of posts above. Really excellent. The oceans matter, why is that fact being disregarded?”
I’m pretty sure the reason is because if you strip the ocean out of the coupled models the earth would be completely covered in snow in a matter of weeks and would stay the way for millions of years while CO2 built up in the atmosphere from volcanic discharges until it was as thick as the Venusion atmosphere and then it would be a runaway greenhouse same as Venus with the final stable state hot enough to melt lead on the surface.
But hey, I’m just guessing and I could be wrong. We can find out by stripping the ocean the best GCMs, hit the execute button, and see what happens.
I doubt the GCMs model CO2 emissions by volcanoes so the current models would just turn the earth into a snowball and stay that way forever.

Steve
May 17, 2011 7:42 am

Dave Springer says: “…the molecules near the surface just keep picking up more and more energy as latent heat until they have enough energy to vaporize and then they leave the surface and quickly convect upwards because water vapor is lighter than air. At no point does the average temperature of the vapor exceed the average temperature of the water from which it sprang so the air near the surface never gets any warmer either. The latent heat only becomes sensible again when adiabatic cooling moves it below the dewpoint. We can argue about GHG effect over land which can be substantial but it ain’t a happening thing over water – it’s physically impossible.”
At sea level, the energy content of the evaporated water molecules will be quite high, but the temperature of the air will not be because most of the air is N2 (temperature is an average), which isn’t heated by IR radiation from the water vapor molecules. If most of the air were changed to CO2, the temperature of this air would increase. That is the GHE.
The GHE does not require that the temperature of the air exceeds the temperature of the ocean surface below it. That would require a huge percentage of the air be water vapor (steam) – we aren’t expecting to boil the oceans.
Water vapor molecules do continue to absorb IR radiation (i.e. “heat up”) after they leave the surface of the ocean. Otherwise the cloud layer wouldn’t be there, we’d just have a global fog layer.

RJ
May 17, 2011 7:45 am

Dave Springer
“This so-called back radiation has been physically measured many times by infrared spectrometers looking up from the surface and down from above. It is even verifiable by anyone who beg, borrow, or steal a $50 infrared thermometer”
What does the comment below mean. From
http://www.slayingtheskydragon.com/en/blog/102-climate-follies-encore
Lord: “Back radiation can be simply demonstrated by pointing a simple infrared detector at the underside of a cloud. Try it.”
Chorus: “My IR detector only cost $60! Simple! Agreed! Agreed!”
Slayer: “Clouds do not absorb and re-radiate heat back to Earth. Clouds add THERMAL MASS which takes longer to heat and cool. Warmists ‘support’ this false hypothesis with IR thermometer readings, but the IR readings of a hot Barbie is the same from any distance; ENERGY is not. Your $60 REMOTE thermometer is not measuring the radiant energy you are receiving, it is measuring the resonance of the Barbie.”

Joel Shore
May 17, 2011 7:58 am

O H Dahlsveen says:

In DC, the electrons flow steadily in a single direction, or “forward.” In AC, electrons keep switching directions, sometimes going “forwards” and then going “backwards.”

No…Tim is absolutely right. The “drift velocity” responsible for the DC current is many orders of magnitude smaller than the thermal velocities of the electrons. The DC current is produced by only a very slight bias in the electron velocities.

Joel Shore
May 17, 2011 8:06 am

RJ says:

What does the comment below mean. From
http://www.slayingtheskydragon.com/en/blog/102-climate-follies-encore

It means that the “Slayers” are talking nonsense, as usual.

May 17, 2011 8:08 am

Joel Shore says:
May 16, 2011 at 1:49 pm
mkelly says:
I agree. Never said anything different. But it is temperature gradient that matters period. Not back radiation as you and Ira say. I have said several times the two requirements for heat transfer are path and temperature gradient.
“So, what are we arguing about then?” Correct use of terms so everyone can gain knowledge and not “interpret”. Back radiation is not temperaure and no formula you can show someone to demonstrate heat transfer via back radiaion. Not arguing I consider it discussion although insults and slights seem to venture out from some people.
“You admit there is a greenhouse effect?” The atmosphere retards excessive heating and cooling. Moon vs. Earth CO2 does not not heat the ground.
No GHE via CO2. As I have stated before I am more in tune with the hot water bottle world theory.

Joel Shore
May 17, 2011 8:21 am

Dave Springer says:

I’d also remind you that when it comes to GHG molecules first molecules in the emission path do most of the work. Tyndall discovered that 150 years ago. I made an analogy to illustrate likening it to the number of people picking apples in an orchard. For a while every additional picker results in a linear increase in harvest speed but after a while there are so few apples left and/or so many pickers they start competing for the same apples and productivity falls off rapidly.

Unfortunately, your understanding of the greenhouse effect hasn’t progressed beyond what was understood around 1950. The modern understanding is that you have to consider both radiation and convection. You are way too focused on the surface energy budget, which is more difficult to calculate than the budget at the top-of-the-atmosphere. The surface temperature basically ends up being determined by the top-of-the-atmosphere energy budget plus the lapse rate that is basically constrained come through convection, evaporation / condensation, etc. It does not come about by a singular focus on the radiative budget at the surface.

Dave Springer
May 17, 2011 9:07 am

Steve says:
May 17, 2011 at 7:42 am
That’s not quite right about steam. If you can see it, it isn’t steam it’s condensed water droplets. I’m not implying you said that I just wanted to clarify for others.
Where you wandered off the reservation seemed to be implying that water vapor molecules must carry the energy of 212F sensible heat plus heat of vaporization which is about a thousand times the sensible heat. Water is really weird. The physics of boiling and the physics of evaporation at two seperate and distinct phenomena. The latent heat of evaporation is much lower than latent heat of vaporization (boiling) as is the sensible heat. It’s still much higher than sensible heat in any case. Evaporation ONLY takes place at the surface while boiling takes place everywhere. I believe it boils down to (pun intented) statistics and at any one time there’s just a statistical chance a water molecule will rob enough energy from its neighbors to peel off from the surface as vapor. Even ice evaporates without melting going straight from solid to gas with no change in sensible temperature. I wasn’t aware that evaporation and boiling were separate things until maybe two years ago when I got interested in heat engines and started reading a lot more about the physics of water. Ambient temperature of the water just increases the odds of any one molecule peeling off the surface as vapor. Once the ambient temperature of the water reaches the boiling point determined by the pressure all hell breaks lose and they violently turn to vapor anywhere and everywhere even condensing back into liquid momentarily and then back to vapor. There are also some conditions where it can remain liquid far past the boiling point as the tipping point becomes critically close throughout the volume. Most of us have seen superheating too which is strange. This happens often when heating water in a microwave where we leave it in too long and as soon we remove the cup and set it down it explodes. The shock wave of the glass hitting the countertop triggers the superheated water into turning to vaporizing. It can also be supercooled under some conditions. It’s probably the most exotic substance in the universe. It’s also a unversal solvent, among other things as well a possibly unique property (pure water only) of the solid phase being less dense than the liquid phase. Fresh water lakes would freeze from the bottom if not for the latter property and would freeze solid rapidly because there’s no layer of surface ice insulating the unfrozen water from the below freezing air above.
I happened upon some of the more interesting properties of latent heat, pressure, and boiling temperature farting around in my workshop trying to get a steam engine work with the water at room temperature. I reduced the air pressure around the working fluid enough to lower the boiling point down to something not much above room temperatures. As it turns out this lowers the latent heat capacity so much you can’t get anywhere near the efficiency of choosing a higher boiling point which was an object lesson for me in why steam engines run at very high temperatures. If it was any other way you could use what’s called low quality heat, like that obtainable by a simple black vessel of water sitting out in full southern sun. You have to raise the temperature and pressure to many hundreds of degrees before the latent heat capacity gets big enough to see much efficiency.
In the same tinkering time frame though I did construct a very simple inexpensive room temperature distillation apparatus which could denature a bottle of wine without ever removing it from the original bottle or heating it up enough to cause chemical reactions that impart off flavors. A second initially empty bottle collected and condensed the vapors from the first bottle and that ended up with a 100+ proof (it would burn so it must be over 100 proof) grape brandy. I heated the first bottle with a hot air dryer. It didn’t have have to be hotter than room temperature you just had to replace the latent heat of vaporization being carried off by the alcohol fumes. The second bottle sat in an ice bath. A pretty good quality vacuum was obtained by an inexpensive plastic bernoulli jet driven by tap water pressure. Once the system had a good vacuum in it, which was under a minute you can shut off the water supply to the bernoulli jet after closing a valve so air coudn’t get back into the system. The total volume of liquid and pressure never changes because as the alcohol evaporates one side it is condensing on the other. It was pretty cool I’m but I’m an uber-geek/engineer who gets a kick out of odd elegant contraptions things like especially when I cobble them to together out of ordinary items like copper tubing, hair dryers, rubber corks, ice, cooking pots, tap water pressure, and so forth. The only item that wasn’t off the shelf at home depot was the bernoulli jet which was like $5 from a scientific supply store. In fact I initally did it without the bernoulli jet using a common home depot gizmo for $20 used to evacuate water from air conditioner freon lines. That was based on an air driven bernoulli jet but you need a lot of 90psi air to generate a half-assed vacuum and it’s not something cheap and quiet you can set up on a kitchen countertop when part of the apparatus is a big loud expensive two horsepower shop compressor. The water jet was awesome, cheap, safe, and easily driven anywhere near a 3/4″ garden hose tap.

Dave Springer
May 17, 2011 9:29 am

RJ says:
May 17, 2011 at 7:45 am

Dave Springer
“This so-called back radiation has been physically measured many times by infrared spectrometers looking up from the surface and down from above. It is even verifiable by anyone who beg, borrow, or steal a $50 infrared thermometer”
What does the comment below mean. From
http://www.slayingtheskydragon.com/en/blog/102-climate-follies-encore
Lord: “Back radiation can be simply demonstrated by pointing a simple infrared detector at the underside of a cloud. Try it.”
Chorus: “My IR detector only cost $60! Simple! Agreed! Agreed!”
Slayer: “Clouds do not absorb and re-radiate heat back to Earth. Clouds add THERMAL MASS which takes longer to heat and cool. Warmists ‘support’ this false hypothesis with IR thermometer readings, but the IR readings of a hot Barbie is the same from any distance; ENERGY is not. Your $60 REMOTE thermometer is not measuring the radiant energy you are receiving, it is measuring the resonance of the Barbie.”

This falls somewhere between a red herring and straw man fallacy.
I said you can point that thermometer up at clear black sky at night and it will still read much closer to ground temperature than the near absolute zero of the cosmic void. No thermal mass water droplets in clouds straw man in sight. What the instrument is recording is back radiation from water vapor, CO2, methane, and whatever trace greenhouse gases are polluting the nitrogen and oxygen.

Robert Stevenson
May 17, 2011 9:43 am

Joel Shoresays:
‘Unfortunately, your understanding of the greenhouse effect hasn’t progressed beyond what was understood around 1950.’
Did you mean to say 1850

Joel Shore
May 17, 2011 10:22 am

Dave Springer says:

The latent heat of evaporation is much lower than latent heat of vaporization (boiling)

This is not correct. For one thing, I have never seen the distinction made between latent heat of vaporization and latent heat of evaporation. They are the same thing, evaporation just being the term used to describe vaporization below the boiling point. Furthermore, the latent heat of vaporization increases with decreasing temperature for water. For example, the value is 22.6 x 10^5 J/kg at 100 C but~24 x 10^5 J/kg at 30 C. [Source: Knight, Jones, & Field, “College Physics: A Strategic Approach, 2nd ed., (2010), pp. 384-5]

Steve
May 17, 2011 10:30 am

Dave Springer says: “That’s not quite right about steam…Where you wandered off the reservation seemed to be implying that water vapor molecules must carry the energy of 212F sensible heat plus heat of vaporization which is about a thousand times the sensible heat. Water is really weird. The physics of boiling and the physics of evaporation at two seperate and distinct phenomena. The latent heat of evaporation is much lower than latent heat of vaporization (boiling) as is the sensible heat.”
So the latent heat content of a mole of water vapor that has evaporated is not exactly the same Joules/mole as water vapor that has boiled? So the latent heat per mole that evaporated water carries away from the oceans surface is much lower than the calculations state? What is it then, in Joules/mole?

Dave Springer
May 17, 2011 10:31 am

Joel Shore says:
May 17, 2011 at 8:21 am

Unfortunately, your understanding of the greenhouse effect hasn’t progressed beyond what was understood around 1950. The modern understanding is that you have to consider both radiation and convection. You are way too focused on the surface energy budget, which is more difficult to calculate than the budget at the top-of-the-atmosphere.

Ya know Joel, if we lived in the thermosphere, which is TOA, I’d be more concerned about it. Unfortunately we live in the much more cluttered world where the air is dense enough to do odd things like convect and conduct and propagate sound waves. It’s a bitch but there’s no getting around it. We don’t live on airless grey bodies made of rock nor in rarefied gases that routinely get heated beyond their ionization threshhold. It’s nice to know about for things like explaining the northern lights to your kids, but even the finicky old ionosphere we used to bounce our radio signals off of to get wireless communication past the horizon is quaint nowadays.
Tyndall demonstrated all the radiative characteristics of greenhouse gases that we need to know. There’s no need to go beyond classical physics. Going beyond that is mostly a tactic that overeducated intellectual thugs use to convince other people it’s just all too difficult to understand and rather perfectly equates to saying we need relativistic physics to land a man on the moon and return him safely to earth. Spare me.

Joel Shore
May 17, 2011 10:32 am

Dave Springer says:

The latent heat of evaporation is much lower than latent heat of vaporization (boiling)

This is not correct. For one thing, I have never seen the distinction made between latent heat of vaporization and latent heat of evaporation. They are the same thing, evaporation just being the term used to describe vaporization below the boiling point. Furthermore, the latent heat of vaporization increases with decreasing temperature for water. For example, the value is 22.6 x 10^5 J/kg at 100 C but~24 x 10^5 J/kg at 30 C; hence the latter is what is used in a simple example to calculate the rate at which humans can cool themselves by perspiration. [Source: Knight, Jones, & Field, “College Physics: A Strategic Approach, 2nd ed., (2010), pp. 384-5]
Robert Stevenson says:

Did you mean to say 1850

It might seem that way given Dave’s reference to Tyndall but I was actually referring more to the chronology of understanding discussed here: http://www.aip.org/history/climate/simple.htm#L_0623

Dave Springer
May 17, 2011 10:38 am

The atmopheric greenhouse effect from radiative charateristics of water vapor were predicted circa 1850. They were, as it turns out, correct. Tyndall’s lab gear lacked the sensitivity to identify CO2 as a greenhouse gas becuase it’s so pathetically weak compared to water vapor but the principle remains the same. The design of modern electronic CO2 are like Tyndall’s whole LWIR/gas experimental setup which took up a small building reduced to fit into a thimball and do it with orders of magnitude better precision. The underlying theory however is unchanged.

Phil.
May 17, 2011 11:07 am

Dave Springer says:
May 17, 2011 at 6:24 am
@Ira
The too many pickers problem is also part of why you can’t slow down the rate of cooling of a body of water with LWIR. The water molecules are so denesly packed in liquid water that the first micrometer absorbs it all. Brownian motion allows the molecules to mix downward another couple micrometers at most but the warmer water is lower density so it wants to fight its way back up to the surface. The end result is there’s virtually no heating beyond the first few micrometers and the molecules near the surface just keep picking up more and more energy as latent heat until they have enough energy to vaporize and then they leave the surface and quickly convect upwards because water vapor is lighter than air. At no point does the average temperature of the vapor exceed the average temperature of the water from which it sprang so the air near the surface never gets any warmer either. The latent heat only becomes sensible again when adiabatic cooling moves it below the dewpoint. This effectively carries off every scrap of downwelling energy up to very near the extinction altitude for 15um and well beyond the extinction altitude of water absorption bands.

Nice try but it’s wrong!
Consider a still ocean surface (~290K) at night with about 300W/m^2 of IR incident on it that gets absorbed in the first 10 microns. That would potentially heat up that surface microlayer at several degrees/sec! Obviously that doesn’t happen, what does happen is that the water surface radiates away at about 400W/m^2, in fact the top few microns will be cooler than the water a mm deeper because the transfer of heat from lower down (conduction, convection, diffusion etc.) is slower than the loss from the surface. Heat will be lost from the surface due to evaporation but it’s not the only mode that you suggest. Also the water vapor doesn’t convect up as you state, humid air will do so.
http://disc.sci.gsfc.nasa.gov/oceans/additional/science-focus/modis/MODIS_and_AIRS_SST_comp_fig2.i.jpg

Steve
May 17, 2011 11:08 am

RJ says: ” What does the comment below mean. From http://www.slayingtheskydragon.com/en/blog/102-climate-follies-encore
“Clouds do not absorb and re-radiate heat back to Earth.”
This is verbal jujutsu. It is twisting the use of the word heat with energy, and technically correct. Heat transfer does not occur in the “back to Earth” direction. But clouds, or any part of the atmosphere that has a temperature above absolute zero, absorb and emit energy from/to the Earth. The energy emitted in the “back to Earth” direction lowers the heat transfer that is occurring in the “away from Earth” direction.
“Warmists ‘support’ this false hypothesis with IR thermometer readings, but the IR readings of a hot Barbie is the same from any distance; ENERGY is not. Your $60 REMOTE thermometer is not measuring the radiant energy you are receiving, it is measuring the resonance of the Barbie.”
Here’s the manual for a typical IR laser thermometer: http://support.fluke.com/find-sales/download/asset/2437622_a_w.pdf
The statement is technically incorrect, as the instrument also measures distance and averages the total measured IR radiation over the area to come up with a temperature for that area. Pointing the device at a tiny heated object a mile away isn’t going to register the temperature of that heated object, but of the entire area. As you get further from the Barbie the temperature reading will go down.
Conceptually, though, I understand what they are trying to say. With an extremely accurate device we would indeed measure that the hot object has the same high temperature from either one foot or one mile away. Less energy would be radiated directly onto us as the object gets further away, despite the high temperature reading remaining the same.
But this is conceptual sleight of hand. The device is indeed measuring radiation, then combining this measurement with others to calculate a temperature for the object. The only fact that matters is that the device has measured an amount of incoming IR radiation. Whatever radiation you are emitting, which determines your rate of cooling, must now subtract the amount of incoming radiation to determine your lower rate of cooling.

Dave Springer
May 17, 2011 11:44 am

I can’t resist responding to Joel’s red herring about understanding TOA and it’s just a matter of plugging in lapse rate to get surface temperature.
Joel, the lapse rate stalls and reverses for a bit in the stratosphere, which is MIDDLE atmosphere, and completely reverses at the mesopause which is the boundary of the upper atmosphere, and by the time you reach the middle of the upper atmosphere it has reversed so hard for so long it reaches thousands of degrees.
You need to stick to your specialty of crystal growth or whatever the hell it is. You are WAY out of your element in atmospheric physics, geology, biology, oceanography, and pretty much everything else near as I can tell. I haven’t time to more than
cherry pick all the uninformed handwaving you’ve done.
For instance when I had you cornered by ocean opacity to LWIR and asked you how shortwave heating in the mixed layer was supposed to escape you started waving your hands about convective turnover and mentioned diurnal turnover in the mixed layer. I didn’t know whether to laugh or cry. Joel, FYI here buddy, diurnal turnover in the mixed layer, which is 100 meters deep, is restricted to 3 meters on a good day. If you’d actually done more than skimmed the paper I cited on Tropical Atlantic Mixed Layer Energy Budget you might perhaps have caught the text and the graph describing diurnal turnover. In fact the shortwave energy absorbed during the tropical summer at depth remains there until the winter when the humidity falls enough so that evaporative cooling sets up strong enough vertical convection to overcome the shallow diurnal reversals in the topmost one to three meters and dredge those deeper waters up to surface where some more rapid cooling can take place. And it doesn’t do it radiatively to any great degree it’s convection of latent heat that carries it up off the water surface well past the optical extinction layer in typical water vapor concentration and up to about the extinction altitude of CO2.
Moreover the tropical summer atlantic only receives 200-210w/m2 of shortwave energy. How in the hell is it supposed to receive more back radiation than it can possibly emit? You running some kind of perpetuum mobile scam here Joel? It emits 50w/m2. That’s NOT net of 390-340 or whatever ridiculous claim you parrot from Hansen’s mouth. The ocean doesn’t even absorb but half the amount of shortwave you claim it emits!
And nobody really yet understands the complexity of heat exchange between the mixed layer and thermocline nor between the end of thermocline and the vast bulk of the ocean (90% of its volume) below the 1000 meter extent of the thermocline.
You have to expend more effort here Joel than being the third or fourth name on a single critical comment to an actual published paper like Gerlich et al. Gerlich laughed at it your critical comment and it was a well deserved chuckle.
Try doing what Gerlich suggested and do a heat budget for an earth with an ocean and without an ocean. It should be easy enough for you to do. Snag one of your graduate students and tell him to strip the ocean out of a coupled ocean/atmosphere GCM and run that puppy to see what the earth looks like with all the greenhoue gasses intact but no ocean. See where your lapse rate from TOA gets you for a surface temperature in that situation. I dare you. You won’t do it of course because I suspect you know just as well I that merely changing 70% of the planet’s surface albedo from the less than 1% of the ocean to the 15% of dirt and rocks will reflect enough additional shortwave energy away from the planet that the bloody thing will be covered in snow faster than you can say Al Gore’s Momma Wears Combat Boots. The albedo of a snow covered world would be about 90% and the earth would soon be so cold that it would make the -26C degree average temperature of the moon look like sunny summer day in Miami.
I know you’re not stupid enough to think this isn’t the way how it plays out with no ocean. What I don’t know is whether you have the intellectual integrity to admit it or whether you’re going keep obfuscating until the glaciers return or hell freezes over, whichever comes last.

Joel Shore
May 17, 2011 11:59 am

O H Dahlsveen

If, as according to the plan, 324 W/m² are radiated downwards then 324 W/m² have got to be radiated upwards towards the next layer (hence the origin of the IPCC models’ “Hotspot”.

Yes, if the layer is thin enough then the amount emitted up and down will be about the same…but the ratio of the difference to the thickness approaches a constant nonzero value. As a result, there is a gradient. And, the top layer emits more both in the upward and downward directions than the bottom layer, as it must since it is colder.
The “hotspot” predicted in the tropical troposphere has absolutely nothing to do with what you are talking about. It has to do with the physics of the moist adiabatic lapse rate.
Please, O H Dahlsveen, at least preface your nonsense with some statement that this is how you imagine it works. Don’t pretend you know, because you rarely do. It is not embarrassing to admit your ignorance but it is embarrassing when you parade it around as knowledge.

Joel Shore
May 17, 2011 12:12 pm

Dave Springer says:

Ya know Joel, if we lived in the thermosphere, which is TOA, I’d be more concerned about it. Unfortunately we live in the much more cluttered world where the air is dense enough to do odd things like convect and conduct and propagate sound waves.

That is exactly the point: The whole reason you want to solve for the radiative imbalance at the top-of-the-atmosphere is that it is rather useless to solve for the surface where other energy flows are just as important.
You are confused: Saying we are solving for the radiative imbalance at the top-of-the-atmosphere doesn’t mean that we are only interested in the temperature there…What it means is that we can use that value, along with the fact that the lapse rate in the troposphere will be determined mainly by convection to figure out what will happen at the surface most easily.

Tyndall demonstrated all the radiative characteristics of greenhouse gases that we need to know. There’s no need to go beyond classical physics. Going beyond that is mostly a tactic that overeducated intellectual thugs use to convince other people it’s just all too difficult to understand and rather perfectly equates to saying we need relativistic physics to land a man on the moon and return him safely to earth. Spare me.

You know, if you weren’t so arrogant then perhaps you would actually be able to learn from the scientists who have studied this stuff a lot more than you have. You tie yourself in goofy knots trying to figure everything out at the surface without having a clue as to what these “overeducated intellectual thugs” might have learned that would make your life easier (although, disappointing, it might also not allow you to lead yourself as easily to the conclusion that you clearly desire, which is that the greenhouse effect, while real, is basically negligible).
I really have no clue why you are so harsh on jae and others given that you are basically cut from almost exactly the same cloth as he is.

Dave Springer
May 17, 2011 12:14 pm

And while you’re running that oceanless GCM in your head to see what happens be sure to look up the term “continentality” which 18th century stone age dumbasses figured out empirically. The continents experience much greater diurnal and seasonal temperature swings. That’s because there’s no ocean mixed layer there to buffer the heat between day and night, winter and summer. The atmsphere’s heat capacity is negligible and sunlight doesn’t heat rock very deep either. Absent the ocean it’s all over except the crying. Snow takes over in a vicious positive feedback the first opportunity the rocks get to dump their shallow heat storage and fall below freezing. More snow raising albedo in lower and lower latitudes fostering even more snow cover as surface temperature drops below freezing and no ocean to stop it. Just a dusting does the trick.
For crying out loud Joel, 4 stupid degrees change in axial tilt on a 26,000 year cycle on the planet as it is now, which does nothing more than slightly increase or decrease the summer/winter temperature differential is enough to tip the balance in favor of snow and the glaciers rule for a hundred thousand years until the cyclic change in the eccentricity of the planet’s orbit, driven by harmonics with Jupiter’s mass of all things, gets in phase briefly with favorable axial tilt, is enough cause a 10,000 year respite from the glaciers. Absent a global ocean that respite will never happen again.
You have SO much to learn, Joel. Our climate is driven by SO many more things than TOA budget and lapse rate and the science needed connet all the dots together span many disciplines that even 50 years of devoted cross-discipline study that I have behind me is barely adequate to start putting all the ducks in a row. You should be embarrassed at even uttering such nonsense in public where a student of yours who pays your employer a handsome fee to sip from the chalice of your knowledge might discover how shallow that chalice really is. This is a classic case for anonymity because the internet never forgets.

Joel Shore
May 17, 2011 12:31 pm

The atmosphere retards excessive heating and cooling. Moon vs. Earth CO2 does not not heat the ground.
No GHE via CO2. As I have stated before I am more in tune with the hot water bottle world theory.

So, in other words, you still believe nonsense that doesn’t even satisfy the conservation of energy. http://scienceofdoom.com/2010/06/03/lunar-madness-and-physics-basics/

Dave Springer
May 17, 2011 12:38 pm


Your numbers are so far from reality it’s ridiculous.
The troical atlantic ocean receives an average of 200w/m2 (see Tropical Atlantic Mixed Layer Heat Budget) yet you’d have me believe it gets 400w/m2 at night. Must be some really bright moon on your planet. Full moon every night that shines down with almost half the intensity of the unobstructed high noon tropical summer sun. ?That’s some trick. What’s your moon made of – radioactive cheese?
Right.
Your comment has been duly noted.
Next!

Dave Springer
May 17, 2011 12:48 pm


Hey thanks for digging up the diurnal turnover depth in that graph you linked. See how it only extends to a depth of 1 meter? Joel was trying to tell me the whole mixed layer turns over every day when I knew only the top 1% turns daily. The entire mixed layer turns seasonally.
I’m not at all sure what point you wanted to make with it. Did you accidently make the wrong link or just didn’t know what you were looking at or what?
Nonetheless I thank for linking to a graph of what I already pointed out to Joel. Very sporting of you.

Dave Springer
May 17, 2011 1:01 pm


re; “Also the water vapor doesn’t convect up as you state, humid air will do so.”
Huh?
http://en.wikipedia.org/wiki/Water_vapor#Lifting_gas

Due to its low molecular weight, water vapor is a lifting gas under STP conditions; however, it is limited by the low amount of water vapor which can exist in the air at a given point in time. High enough temperatures to maintain a theoretical “steam balloon” yield approximately 60% the lift of helium and twice that of hot air.

Of course it won’t rise if it condenses close to the surface. But then it isn’t water vapor. We call that “fog” on my planet. Again, what’s your point? As long as it remains a vapor it rises until adiabatic lapse cools it below the dew point.
Until it condenses the vast majority of the heat if lifted out of the ocean remains hidden away in latent form.
Did I lose you anywhere there? What part don’t you understand? I like you and am willing to spoon feed this to you if you’d just stop making faces and spitting it out.

Dave Springer
May 17, 2011 1:15 pm


Maybe I figured out the soruce of your confusion.
Steam is invisible. I pointed that out earlier in the thread for anyone who might not know that.
http://wiki.answers.com/Q/Is_steam_Invisible?

Answer:
Yes, if you see the steam, it is actually water droplets forming. Look at the steam from your teapot, while the water is boiling. The first few centimeters from the teapot you won’t see the steam; later you see it. That is because it has already started to cool down, forming droplets.

This is elementary school science, Phil. Literally. Steam (water vapor) is fourth grade material.
http://www.scprimesite.com/portal/page/portal/santee_cooper/communityeducation/teacher%20resources/videos/energy
How old did you say you were?

RJ
May 17, 2011 1:20 pm

Steve Ok Thanks
But what does he mean by Barbie. What is it?
So what does the slayer think the reader is reading if it is not incoming radiation because I do not know what a Barbie is.
(Unfortunately this slayer paragraph seems poorly worded)
And even if the reader is reading incoming radiation does it also measure the type of radiation and whether when the radiation arrives it will in fact have a warming impact. Because radiation from a colder atmosphere will not warm a warmer earth.
“Warmists ‘support’ this false hypothesis with IR thermometer readings, but the IR readings of a hot Barbie is the same from any distance; ENERGY is not. Your $60 REMOTE thermometer is not measuring the radiant energy you are receiving, it is measuring the resonance of the Barbie”.

May 17, 2011 1:33 pm

Mr. Springer says: “It’s also a unversal solvent, among other things as well a possibly unique property (pure water only) of the solid phase being less dense than the liquid phase.”
In regard to pure water, if I remember correctly my college chem professor said that absolutely pure water was poison to humans. I don’t remember the quanity you had to drink.

Dave Springer
May 17, 2011 1:36 pm

Anybody ever seen fog “burn off” at night?
Me neither. I’ve seen it blow away if a wind picks up but never burn away in still air.
That’s because even though it is absorbing every last joule of upwelling and downwelling LWIR at night it isn’t enough to heat it above the dew point. Only sunlight has the energy density to do that and it typically does it early in the morning in most times and places where fog forms, long before the sun has attained an elevation bringing it near full noonday intensity.
The notion that there is all this power, hundreds of watts/meter illuminating the surface night is utter bullshit. Pardon my French.

wayne
May 17, 2011 1:37 pm

Dave, your statements at May 17, 2011 at 5:44 am and May 17, 2011 at 5:56 am hit it directly on the nose. That is exactly what I visualize but I seem not able to word it, BUT YOU CAN! Now I know where to holler for help. ☺

May 17, 2011 1:40 pm

Joel Shore saysMay 17, 2011 at 7:58 am:
“———–No…Tim is absolutely right.” ——
Ok Joel, I must be getting old and forgetful, possibly senile even. So thank you for putting back on track, you’re right and I am wrong.
Well done Tim.

May 17, 2011 1:46 pm

Dave Springer says on May 17, 2011 at 1:36 pm:
“Anybody ever seen fog “burn off” at night?”
Heh-heeh-heh-h
There’s another missing “Hotspot” Dave

Joel Shore
May 17, 2011 2:04 pm

Dave Springer says:

Hey thanks for digging up the diurnal turnover depth in that graph you linked. See how it only extends to a depth of 1 meter? Joel was trying to tell me the whole mixed layer turns over every day when I knew only the top 1% turns daily. The entire mixed layer turns seasonally.

What? You must be confusing me with someone else who tries to correct your misstatements.

Joel Shore
May 17, 2011 2:07 pm

RJ says:

But what does he mean by Barbie. What is it?

It’s Aussie-speak for a barbeque. See http://www.australianbeers.com/culture/barbies.htm

May 17, 2011 2:33 pm

Joel Shore says on May 17, 2011 at 11:59 am:
“O H Dahlsveen ———— at least preface your nonsense with some statement that this is how you imagine it works. Don’t pretend you know, because you rarely do.”
So you see me as you see yourself then Joel? Are you really telling me that the so-called “Hotspot” has got nothing to do with “Greenhouse Gas Warming”? If so, please expand on your claim:
”It has to do with the physics of the moist adiabatic lapse rate.”

Joel Shore
May 17, 2011 2:44 pm

Dave Springer, I’ll try to respond to the few things left in your recent posts that are not unintelligible gobbley-gook (or are at least less-unintelligible gobbley-gook) .

Moreover the tropical summer atlantic only receives 200-210w/m2 of shortwave energy. How in the hell is it supposed to receive more back radiation than it can possibly emit? You running some kind of perpetuum mobile scam here Joel? It emits 50w/m2. That’s NOT net of 390-340 or whatever ridiculous claim you parrot from Hansen’s mouth. The ocean doesn’t even absorb but half the amount of shortwave you claim it emits!

So, now you have made a full transition to the pseudoscience side and are endorsing the notion that there is no way the earth’s surface can emit more longwave radiation than the shortwave radiation that it absorbs from the sun? Try doing the calculation for Venus and see what you come up with. I really think that jae must have taken over your keyboard or something.
And, silly me to interpret the quote from a paper that says “net longwave emission” to mean “NET longwave emission”. After all, it makes perfect sense that something that is nearly a blackbody at a temperature of about 15 C will emit only 50 W/m^2 of emission (gross)…at least once you repeal a few laws of physics that were never much use to us anyway! It’s not like we could use the Stefan-Boltzmann Equation to calculate what it actually emits (gross)!

Try doing what Gerlich suggested and do a heat budget for an earth with an ocean and without an ocean. It should be easy enough for you to do…You won’t do it of course because I suspect you know just as well I that merely changing 70% of the planet’s surface albedo from the less than 1% of the ocean to the 15% of dirt and rocks will reflect enough additional shortwave energy away from the planet that the bloody thing will be covered in snow faster than you can say Al Gore’s Momma Wears Combat Boots.

And, what does that prove exactly? That albedo affects climate? I’ll alert the media.
You really have gone off the deep end and I have no clue what points you are even trying to make anymore. That oceans affect climate? That water vapor is important? That albedo matters? I really have no clue.

Steve
May 17, 2011 2:51 pm

RJ says: “But what does he mean by Barbie. What is it? So what does the slayer think the reader is reading if it is not incoming radiation because I do not know what a Barbie is.”
Don’t ask me why they picked it as their example “hot object”, but by Barbie (capitalized) they are referring to a Barbie doll. If it had been “barbie” it would make more sense to me, being slang for a barbecue (which could quite understandably be a good example of a hot object).
In either case you can easily substitute “rock” for “Barbie/barbie”. They are saying that if you point the IR thermometer at a rock, the reading represents the device’s calculated temperature of the surface. This is correct. What is incorrect is their claim that the “thermometer is not measuring the radiant energy you are receiving, it is measuring the resonance of the [rock’s surface].” The thermometer is, in fact, measuring the radiant energy received (in the device’s IR capabilities). It then uses this measurement, along with other measurements (e.g. distance), to calculate a temperature for the object it is pointing at.

May 17, 2011 3:06 pm

Anyway Joel if you stand by with what you say on May 17, 2011 at 11:59 am i.e. :
“———. As a result, there is a gradient. And, the top layer emits more both in the upward and downward directions than the bottom layer, as it must since it is colder.”
Then I must assume the above statement is part of your own great knowledge and you can certainly say to yourself: “it is not embarrassing to admit my ignorance but it is embarrassing when I parade it around as knowledge.”

wayne
May 17, 2011 3:16 pm

RJ when you said “Your $60 REMOTE thermometer is not measuring the radiant energy you are receiving, it is measuring the resonance of the Barbie.”, I tend to agree with you even though I have never owned an IR thermometer. From what I have gathered from other commenters, it reads the maximum possible radiant power not the actual flux. Just because it reads 390 Wm-2 when aimed at the 59 ºF surface does not mean that 390 Wm-2 is actually flowing, just that it has the capability if that surface at that temperature were in the void of space with nothing else to attenuate it, THEN, and only then, would the actual flux also be 390 Wm-2.
In other words, the temperature of the Barbie (barbeque for those ‘up above’) you read with an IR thermometer is the same if you are 6 feet away or 12 feet away, when you know it should be reading 1/4th at 12 feet away than when 6 feet away.
Is that not what you are saying? That is some of the fallacy that is going on in minds, people are thinking 390 Wm-2 upward when the reality is close to 65 Wm-2 upward. The 390 is the maximum power, the 65 is the actual flux upward.
So by KT97 if you let the surface be 289K as stated in TFK09 instead of 288K you can take:
67 Wm-2 absorbed SW by the atmosphere plus
24 Wm-2 carried upward by thermals (dry conduction/convection) plus
78 Wm-2 carried upward by evaporation (convection) plus
66 Wm-2 actual LW radiation flux upward (radiation)
———
235 Wm-2 detected LW upwelling by satellites above the TOA
Also 24+78+66 equals the 168 absorbed by the surface and 168+67 is the total SW absorbed by the system. See, the only fallacy numbers are the 390, 350 and 324 maximum radiant powers injected into the IR side of the graphic. So, scratch out 390 and put 66, scratch out 350 and replace with 26, scratch out 324 and replace with zero. Now all figures on the diagram are all FLUXES, not a mixture of fluxes and maximum radiant powers as if against empty space.
Tying in the other figures you have the 67+24+78+26−30 = 195 in the atmosphere less the 30 that K&T isolated and 165+30+40 = 235 or total flux to space. It all jives now and all numbers are now fluxes.
(RJ, I just realized that after reading your comment, it is maximum radiant powers mixed with actual fluxes that is the problem thrashing so many people’s minds and I owe it to your simple comment above)
I like simplicity, physics is simplicity, AGW proponents insist on complexity when it is not even needed. The 390, 350 and 324 should have been just a footnote on their graphic.

Myrrh
May 17, 2011 3:39 pm

Joel – I’ve been thinking about a couple of things re statistical mechanics. Firstly, a comment you’ve made a few times I’ve noticed, that the equations are used, tried and tested by scientists and engineers who build things and you give this as a proof that this shows the science behind the approach is sound.
I’ve given before the example of the oximeter, that ‘uses’ the Beers-Lambert law, but then has to adjust it because the law is idealised and not of the real world, like using ideal gas laws. Because there is actually no such thing as an ideal gas, it is purely imaginary, various adjustments have to be made when making any calculations of real gases in the real world because real gases don’t obey the idea gas law at all temps and pressures. Scientists who actually build things, like engineers, know these limitations and know how to adjust the equations to include the missing parameters, so your comments on this aspect don’t actually relate back as proof that such laws are valid to describe the real world.
What I’ve seen often enough in AGW reasoning is Carbon Dioxide’s properties described as an ideal gas – which is how I came into these arguments – especially by one physics PhD who claimed that CO2 mixes thoroughly in the atmosphere because it behaves like an ideal gas; that a pool of CO2 on the floor in a room will therefore diffuse into the atmosphere of the room according to the random nature of the ideal gas without any work being done to move it, fan or open window and so on, and once thoroughly mixed cannot be unmixed without work being done. He said he would fail any of his students who said this was not how CO2 behaved in our atmosphere, as I was saying.
Secondly, in thinking about the descriptions of ‘heat as the net which is from hotter to colder because some heat is going from colder to hotter, how exactly?
What is the actual mechanism, what physical event decides that the ‘net’ will always come out as heat flowing from hotter to colder?

Joel Shore
May 17, 2011 4:23 pm

O H Dahlsveen says:

“———. As a result, there is a gradient. And, the top layer emits more both in the upward and downward directions than the bottom layer, as it must since it is colder.”
Then I must assume the above statement is part of your own great knowledge and you can certainly say to yourself: “it is not embarrassing to admit my ignorance but it is embarrassing when I parade it around as knowledge.”

Well, you’ve got me there, although I don’t so much think it is a case of ignorance in that case but rather of typing something at variance to what I was thinking. Obviously, I meant to say that the top layer emits less.

Are you really telling me that the so-called “Hotspot” has got nothing to do with “Greenhouse Gas Warming”? If so, please expand on your claim:
”It has to do with the physics of the moist adiabatic lapse rate.”

Yes…The amplification of warming in the tropical troposphere is not specific to the mechanism of the warming being due to greenhouse gases. People ranging from Gavin Schmidt ( http://www.realclimate.org/index.php/archives/2007/12/tropical-troposphere-trends/ ) to Richard Lindzen (who says “The dominant role of cumulus convection in the tropics requires that temperature approximately follow what is called a moist adiabatic profile. This requires that warming in the tropical upper troposphere be 2-3 times greater than at the surface. Indeed, all models do show this, but the data doesn’t and this means that something is wrong with the data.” http://wattsupwiththat.com/2011/01/17/richard-lindzen-a-case-against-precipitous-climate-action/ )
Basically, the picture is this: Imagine two saturated parcels of air that ascend through the atmosphere, one being 1 deg warmer when it starts out at the surface. As it ascends, it expands and cools…but as it cools, some of the water vapor must also condense out and the warmer one will contain more water vapor and have more water vapor condense out as it cools than the colder one. The water vapor condensing out releases latent heat, which means that the air parcel no longer cools as quickly as if it were not saturated. However, the warmer one has more latent heat released than the colder one; hence, as it rises, it cools less rapidly than the colder one cools. This means that as you go up in the troposphere, the temperature difference you had between these two at the surface gets amplified.

Joel Shore
May 17, 2011 4:38 pm

Myrrh says:

What is the actual mechanism, what physical event decides that the ‘net’ will always come out as heat flowing from hotter to colder?

For all forms of heat transfer, it basically has to do with the statistics of large numbers of events. For conduction, which occurs via molecular (or electronic) collisions, you will have collisions between molecules…and the molecules will have a range of velocities but ON AVERAGE, those coming from the hotter side of the object will have slightly higher speeds than those coming from the cooler side and so, in a large number of collisions, the molecules from the cooler side will gain speed on average and those from the hotter side will lose speed on average in the collisions.
For radiation, it is basically that the hotter object emits more radiation than the cooler object. That, coupled with Kirchhoff’s Law ( http://en.wikipedia.org/wiki/Kirchhoff%27s_law_of_thermal_radiation ), guarantees that the heat flow will be from the hotter object to the colder object.

May 17, 2011 4:43 pm

O H Dahlsveen says:
May 17, 2011 at 6:13 am
Do not make the mistaken assumption that the Atmosphere can be likened to a solid piece of, say steel which can easily be warmed more on one side than the other.
I agree, it is more like a pile of blankets – cold in the middle and warm on both the top and bottom. The question is – Why is it coldest in the middle? The interesting thing is that water vapor stays fairly high until it reaches the cold middle. My interpretation of the data is that water vapor is IR opaque until the tropopause and that CO2 is opaque until a bit higher. This model completely explains why the middle is colder than both the air above and below the tropopause.
Otherwise, I agree with your arguments. The problem is that they do not explain why the stratosphere gets warmer with height.

jae
May 17, 2011 6:51 pm

Dave springer:
“If you can’t accept all these things as proof that back radiation is real then you’re a cause and if this was my blog I’d throw your ass out so to reduce the dimwitted clutter that pollutes the more informed converstations.”
Nice comment. You have all the markings of a “civil, tolerant progressive,” which is just as mythological as CAGW!
BTW, you will never find a place where I state that backradiation is not real, so your comment is not only insulting, but also dishonest.

Editor
May 17, 2011 10:01 pm

Dave Springer says:


Hey thanks for digging up the diurnal turnover depth in that graph you linked. See how it only extends to a depth of 1 meter? Joel was trying to tell me the whole mixed layer turns over every day when I knew only the top 1% turns daily. The entire mixed layer turns seasonally.

What you have referenced is not an actual measurement, Dave. It is a schematic plot. And you can’t take numbers off of a schematic plot. The mixed layer can extend for anywhere from 1 metre to a hundred metres. And yes, it’s called the mixed layer because a good chunk of it mixes every day. The citation you referenced says:

The vertical structure of the skin layer SST can be generally described as in Figure 2. The interface SST, SSTint, is the temperature at the infinitely thin layer at the exact air-sea interface. This temperature cannot be measured using current technology. The skin SST, SSTskin, is the temperature measured by an infrared radiometer at a depth of order of 500 µm depending on the wavelength of the measurement. This temperature is depth (wavelength) dependent, but the differences measured by the infrared radiometers are very small (less than 0.01 K due to the small penetration depth differences). Therefore, the wavelength dependence of SSTskin is usually ignored. The subskin SST, SSTsubskin, is representative of the SST at the bottom of the skin temperature layer and is usually the value measured by a low-frequency (6-10 GHz) microwave radiometer. The SST at depth, SSTdepth, (traditionally referred to as the bulk SST) represents the temperature of the upper mixed layer produced by the turbulence associated with wind stirring and convective overturning, etc.
Within the surface mixed layer usually of a thickness on the order of 1 m to 100 m, the bulk SST has very small vertical gradient. In fact, the depth of the surface mixed layer is frequently defined as the depth where the temperature drops from the surface bulk SST by a small amount (say 0.5 K or 1 K). Occasionally, vertical gradient of SST is present in the upper mixed layer due to interleaving and overturning processes of high-frequency heating/cooling. But the gradient is quickly eliminated by the turbulence over periods of a few hours to a day. The mean skin temperature is generally several tenths of a degree colder than the mean bulk temperature (Schluessel et al., 1990). The instantaneous bulk-skin temperature differences can be as large as 1.0 K to -1.0 K (Robinson, 1985), depending on the wind and surface flux conditions. For instance, when the long wave radiation from the upper few micrometers of the ocean is upward, the skin temperature is usually cooler than the bulk SST.Latent and sensible heat fluxes can cool the sea surface further if the air is dryer or colder.

Note that the mixed layer is defined as the layer having no vertical temperature gradient, and it is defined as 1 to 100 metres deep. Now Dave, that mixed layer is being warmed at the top both day and night. In the day there’s a temperature gradient. In the night there’s no temperature gradient.
If there is no daily overturning in that mixed layer, and the mixed layer can only lose energy from the top … they why is there no temperature gradient in the night but there is a gradient in the day? The answer, of course is that the overturning actually is diurnal.
Yes, there is also a slow seasonal component to the mixed layer. It mixes deeper during one season and less during another, equalizing the entire mixed layer over the period of a year. But what that means is that some times of the year, the daily mixing is deeper on average.
So despite your claim that the diurnal mixed layer is only a metre thick, any oceanographer is going to laugh in your face about that one. For me, as an inveterate night diver, I knew your statement about the nocturnal overturning being only one metre deep was terribly wrong because I’ve regularly dived through the descending columns of cool water during the nocturnal overturning … down ten metres or so …
w.

Phil.
May 17, 2011 11:24 pm

Dave Springer says:
May 17, 2011 at 12:38 pm

Your numbers are so far from reality it’s ridiculous.
The troical atlantic ocean receives an average of 200w/m2 (see Tropical Atlantic Mixed Layer Heat Budget) yet you’d have me believe it gets 400w/m2 at night. Must be some really bright moon on your planet. Full moon every night that shines down with almost half the intensity of the unobstructed high noon tropical summer sun. ?That’s some trick. What’s your moon made of – radioactive cheese?
Right.
Your comment has been duly noted.

In order for that to be true you’d have to have read it and understood it, you apparently did neither!
I chose a downwelling IR value of 300W/m^2, a fairly modest value for planet Earth as shown by measurement (300W/m^2+ has been measured during summer at the North pole). The value of 400W/m^2 referred to the blackbody radiation leaving the surface, what would be expected for a sea surface around 290K. So my numbers are quite reasonable for planet Earth.

Phil.
May 17, 2011 11:28 pm

Dave Springer says:
May 17, 2011 at 12:48 pm

Hey thanks for digging up the diurnal turnover depth in that graph you linked. See how it only extends to a depth of 1 meter? Joel was trying to tell me the whole mixed layer turns over every day when I knew only the top 1% turns daily. The entire mixed layer turns seasonally.
I’m not at all sure what point you wanted to make with it. Did you accidently make the wrong link or just didn’t know what you were looking at or what?

Perhaps you should read it again since you evidently didn’t understand it the first time?
“what does happen is that the water surface radiates away at about 400W/m^2, in fact the top few microns will be cooler than the water a mm deeper because the transfer of heat from lower down (conduction, convection, diffusion etc.) is slower than the loss from the surface.”

Phil.
May 17, 2011 11:45 pm

Dave Springer says:
May 17, 2011 at 1:01 pm

re; “Also the water vapor doesn’t convect up as you state, humid air will do so.”
Huh?
Of course it won’t rise if it condenses close to the surface. But then it isn’t water vapor. We call that “fog” on my planet. Again, what’s your point? As long as it remains a vapor it rises until adiabatic lapse cools it below the dew point.
Until it condenses the vast majority of the heat if lifted out of the ocean remains hidden away in latent form.
Did I lose you anywhere there? What part don’t you understand? I like you and am willing to spoon feed this to you if you’d just stop making faces and spitting it out.

You apparently fail to understand that water vapor doesn’t convect up on its own but does so along with the air that it’s part of, i.e. humid air.

Phil.
May 17, 2011 11:48 pm

Dave Springer says:
May 17, 2011 at 1:15 pm

Maybe I figured out the soruce of your confusion.

The confusion is all yours!

Myrrh
May 18, 2011 2:49 am

Joel Shore says:
May 17, 2011 at 4:38 pm
Re my: “What is the actual mechanism, what physical event decides that the ‘net’ will always come out as heat flowing from hotter to colder?”
For all forms of heat transfer, it basically has to do with the statistics of large numbers of events. For conduction, which occurs via molecular (or electronic) collisions, you will have collisions between molecules..and the molecules will have a range of velocities but ON AVERAGE, those coming from the hotter side of the object will have slightly higher speeds than those coming from the cooler side and so, in a large number of collisions, the molecules from the cooler side will gain speed on average and those from the hotter side will lose speed on average in the collisions.
For radiation its basically that the hotter object emits more radiation than the cooler object.

Where in this process does it show that the radiation from the colder to the hotter is ever taking place?
As I’ve tried to explain what I mean here before, the ‘net’ in ‘statistics of large numbers’ has nothing to say about a colder’s heat flowing to the hotter.
And so can be fully true to the 2nd Law that heat ALWAYS flows from hotter to colder in that the ‘net’ can be simply explained as I described somewhere above – that the colder gaining energy becoming hotter which then in turn energises the colder, thereby losing energy and again being ready to be energised by the hotter. This is the real ‘net’ mechanism being described.
The ON AVERAGE then, is not about, or rather including, some idea of energy flowing from the colder to the hotter, but the average of the exchanges as the hotter energises the colder thereby itself losing energy in any amount where this is happening, the gains and losses in these exchanges.
In other words, the ‘net’ is irrelevant to the 2nd Law, as this refers only to the process of these exchanges depending on such things as amount of energy available and materials and their properties relative to each other.
The “statistics of large numbers” is no proof whatsoever that heat flows from the colder to the hotter, ever. That’s just been added in without any logical antecedents.
If the colder could add its energy to the hotter, then statistically in the ‘large number of events’ the colder could always be giving up its energy to the hotter, and losing what energy it has so itself getting colder; so putting something hotter on ice the hotter will get hotter not colder if there is enough colder to hotter events.
Unless, you can provide a mechanism which switches this off.

Steve
May 18, 2011 7:14 am

Myrrh says: “The “statistics of large numbers” is no proof whatsoever that heat flows from the colder to the hotter, ever. That’s just been added in without any logical antecedents.”
Heat doesn’t flow from the colder to the hotter, energy flows from the colder to the hotter. The heat flow from the hotter to the colder is the net energy flow.
“If the colder could add its energy to the hotter, then statistically in the ‘large number of events’ the colder could always be giving up its energy to the hotter, and losing what energy it has so itself getting colder…”
… and gaining what energy the hotter object transfers to it, for a net effect of getting warmer. The hotter object is transferring more energy to the cooler object than the cooler object is transferring back to it, for a net effect of getting cooler.

richard verney
May 18, 2011 7:29 am

Joel regarding yout post
Joel Shore says:May 14, 2011 at 7:48 pm richard verney: Claim 3. Some caution is required. Theoretical black body calculations do not accord with actual measurements for Venus, Mars Titan or even the moon.,,,,,,”
Your response: You consider yourself a skeptic, I assume, and yet you accept any piece of garbage that you find on the internet? Hertzberg, Schreuder, and Siddons are well-known for being purveyors of pseudo-scientific nonsense. Why would you possibly believe their nonsense when you won’t accept real science that has been tested thoroughly? It seems like your skepticism is very selective. Here is a detailed debunking of their paper: http://scienceofdoom.com/2010/06/03/lunar-madness-and-physics-basics
//////////////////////
I have tried half a dozen times to open that link (including going directly to science of doom and searching for their article). Unfortunately, it merely causes my computer to freeze.
If you had read my comments you would know that I was citing the article that I referred to merely in support of the statement that that actual temperature measurements taken on the moon do not correspond with theoretical BB calculations. I was not citing the paper in support of their conclusions that there is no GHE.
I presume that their empirical data (ie., the BB calculations performed by NASA and the actual temperature measurements performed by NASA) are correct. If you consider that empirical data to be wrong, please refer me to the correct data.
If their empirical data is correct, then it follows that the moon does not behave in accordance with BB calculations. As you are aware, I consider that it is unlikely that the Earth would behave as if it were a BB. There are many reasons for this (the composition of the atmosphere with gases that have different absoptive and emissitivity, the vast stored heat capacity of the oceans etc). The upshot of this is that the Earth’s daytime temperatures are not as warm as BB calculations suggest that they should be nor are the night time temperatures as low as BB calculations suggest that they should be. The type of lag and dampening seen with respect to the moon would be exacerbated with the Earth.
I do not consider it likely that the Earth behaves as a black body, it is not a BB but a Greybody and we do not have a proper handle on this such that and assessments based upon the Earth being a BB are likely misconceived and erroneous.

May 18, 2011 8:12 am

Steve says:
May 18, 2011 at 7:14 am
“… energy flows from the colder to the hotter. The heat flow from the hotter to the colder is the net energy flow.”
Steve this is wrong. Let me do this again basic formula q/A = e (SB)(T1^4-T2^4) if T1=T2 then zero energy flows anywhere. Zero on right side of equation means zero on left side of equations. q/A is watts per meter squared a watt is juole per second and a joule is a unit of energy. Use any heat transfer formula you want but if T1=T2 no energy flows. There must be a path and a temperature gradient for q/A to have a value.

May 18, 2011 8:49 am

Dave Springer added a new comment to the post Visualizing the “Greenhouse Effect” – Light and Heat.
Dave Springer said on Visualizing the “Greenhouse Effect” – Light and Heat
May 16, 2011 at 8:06 pm
In response to Ira Glickstein, PhD on May 7, 2011 at 7:55 pm:
Guest Post by Ira Glickstein Solar “light” radiation in = Earth “heat” radiation to Space out! That’s old news to those of us who understand all energy is fungible (may be converted to different forms of energy) and energy/mass is conserved (cannot be created nor destroyed). My Visualizing series [Physical Analogy, Atmospheric Windows, Emission Spectra, […]
JAE
Nowhere did I say anything about any net warming caused by back radiation from greenhouse gases. All I claimed was that GHGs do indeed absorb radiation and redirect a portion of the absorbed energy back towards the source. This is the basis upon which most electronic CO2 sensors used in commercial ventilation systems work. This so-called back radiation has been physically measured many times by infrared spectrometers looking up from the surface and down from above. It is even verifiable by anyone who beg, borrow, or steal a $50 infrared thermometer
http://www.amazon.com/gp/product/B0017L9Q9C/ref=pd_lpo_k2_dp_sr_2?pf_rd_p=486539851&pf_rd_s=lpo-top-stripe-1&pf_rd_t=201&pf_rd_i=B000MX5Y9C&pf_rd_m=ATVPDKIKX0DER&pf_rd_r=0A2YZ0RNVN98DZFNB1EG
and verify for themselves that when pointed upward through clear sky it still reads a temperature far closer to the 200-something K of the ground you’re standing on than the 3K temperature of empty space.
If you can’t accept all these things as proof that back radiation is real then you’re a cause and if this was my blog I’d throw your ass out so to reduce the dimwitted clutter that pollutes the more informed converstations.
Just a quick note to thank Dave Springer for providing the empirical evidence of ‘back radiation” which cemented my understanding of the subject.  I had missed your previous comments on the handheld infrared thermometer, but noticed it today while browsing previous posts.  
FWIW most cheap handheld IRTs have the emissivity set by default to 0.95, so the temperature it read of the sky and earth would need to be corrected in order to account for their different emissivities.  More expensive ones allow adjustment of the emissivity, or rough & ready calibration by measuring the temperature of masking tape (emissivity ~0.95) on the object to be measured, or by drilling a hole at least 6 times as deep as its diameter which approximates a blackbody (emissivity 0.98+).  Problem with that approach is that the FOV of most cheap ones is so broad that a fairly large hole is required 🙂
There are also FLIR cameras which use the radiated LWIR energy to image things, a technology with applications in veterinary medicine, industry & many other fields.
All these technologies use the IR radiation emitted from the objects being scanned, at least according to all the various manufacturers I am familiar with.
I have known and used these devices for years, which got me thinking about the difficulty I had accepting the concept when applied to climate, and the fact I had not considered the implications of the science upon which the technology was built.  I concluded that it had to do with semantics, at least in my case.  All the textbooks I have read and all the books I have read state categorically that heat is the manifestation of the transfer of energy, and this is true.  In the case of two objects, one colder and one hotter, there is an EXCHANGE of energy as well as a TRANSFER of energy.  The energy exchanged amounts to the total energy radiated by the cold object and the energy transfered is always from hot to cold and is simply the difference between the amount the hot object radiates and the amount the cold object radiates – ie the net difference in their energies.  This respects the 2nd law & requires no mechanism or volition, no little ‘Maxwell’s Demons” as Joel scarcastically suggested.  In effect, the colder object transfers all it can to the hotter object, but it will always have less to give than the hotter object, therefore the transfer of energy (flow of heat) must always be from hot to cold – unless external work is applied or some other change in universal entropy compensates.
So with respect to the surface and the atmosphere, the atmosphere doesn’t heat the surface, or really make it warmer, all it does is slow radiative heat loss, instead of losing 356W/m^2, with a 333W/m^2 atmosphere all it really loses is 23W/m^2.
I also wanted to say that I agree with you about CO2 being a beneficial trace gas (plants wouldn’t grow and a lot of people would die of apnea), up to a fairly high level anyway.  Witness the levels of CO2 used in commercial greenhouses…  While the oceans must represent the largest single influence on global and regional climate, Land Use & Land Cover changes also appear to have significant regional forcing effect on land.  
http://wattsupwiththat.com/2011/04/18/more-on-land-use-change-affecting-temperatures/

Steve
May 18, 2011 8:56 am

mkelly says: “Steve this is wrong. Let me do this again basic formula q/A = e (SB)(T1^4-T2^4) if T1=T2 then zero energy flows anywhere.”
Did you think that through? You are saying that, in the one case where T1=T2, the equation shows that there is no energy flow between the objects. But in all other cases where T1 is not equal to T2, you can have either a positive or negative energy flow between the objects. What would it mean to have an energy flow less than zero?
Are you positive this equation is representing individual energy flows? I think it represents net energy flow, hence it can be either zero or even negative.
Are you saying that if I surround a 500 degree ball bearing by a 600 degree sphere, the 500 degree ball bearing stops radiating energy away from itself, and the 600 degree sphere will begin radiating a less energy towards its center?

Joel Shore
May 18, 2011 9:27 am

richard verney says:

I presume that their empirical data (ie., the BB calculations performed by NASA and the actual temperature measurements performed by NASA) are correct. If you consider that empirical data to be wrong, please refer me to the correct data.

Richard: The problem is in their analysis of the temperature data. So, it is not NASA’s data that is incorrect but their analysis that led them to conclude that the moon is not, to a very good approximation, obeying the expectations of a blackbody emitter. Basically, they did not understand how to deal with the case where temperature is not constant.

I do not consider it likely that the Earth behaves as a black body, it is not a BB but a Greybody and we do not have a proper handle on this such that and assessments based upon the Earth being a BB are likely misconceived and erroneous.

Emissivities of terrestrial materials in the wavelength range of interest are extremely close to 1 ( http://en.wikipedia.org/wiki/Climate_model ), so much so that the earth not being a perfect blackbody could only account for two or three degrees at the very most. (Because of the T^4 dependence of radiative power on temperature, an emissivity of 0.99 rather than 1 leads to ~0.7 deg K of change in the predicted temperature.)
mkelly says:

Let me do this again basic formula q/A = e (SB)(T1^4-T2^4) if T1=T2 then zero energy flows anywhere.

In this equation, q is the rate of heat transfer, which is the NET rate of energy transfer. So, it being zero does not mean that there are no energy transfers back and forth, but merely that they cancel out to give no heat transfer.
Not that it matters…The equation that you wrote down gives the greenhouse effect regardless.

wayne
May 18, 2011 12:37 pm

Joel Shore says:
May 18, 2011 at 9:27 am

mkelly says:
Let me do this again basic formula q/A = e (SB)(T1^4-T2^4) if T1=T2 then zero energy flows anywhere.

In this equation, q is the rate of heat transfer, which is the NET rate of energy transfer. So, it being zero does not mean that there are no energy transfers back and forth, but merely that they cancel out to give no heat transfer.
Not that it matters…The equation that you wrote down gives the greenhouse effect regardless.

So you are a “Quantum Electrodynamics Denier”? I had that feeling. You do not think amplitudes and probabilities can never be zero? Really? See Joel, you have to prove it to all here before you thrash more minds.

wayne
May 18, 2011 1:04 pm

mkelly (and all of the other commentators that have made it this far):
CAUTION: Only for the brave.
I highly suggest all take the time to go back to school, just for a moment, and listen to these four lectures by Dr. Richard (Dick) Feynman. Really? Really! Your view of the transfer of radiant energy will never be the same again.
It requires no complex math, no huge intellect, just a curious mind.
http://vega.org.uk/video/programme/45
http://vega.org.uk/video/programme/46
http://vega.org.uk/video/programme/47
http://vega.org.uk/video/programme/48
The last one and a half you can really skip if you don’t want to see how this applies to electrons, Feynman diagrams and how it applied to the zoo of particles that physics once was experiencing, but the first two are mandatory.
Try it. You’ll like it! Then come back and prove it to me that there are always photons passing back and forth within a smooth hollow steel sphere at any temperature. (hint: by cancelling amplitudes everywhere it seems it is zero photons inside).
Seems to me Joel Shore is the one passing pseudo-science here!

May 18, 2011 1:20 pm

Steve says:
May 18, 2011 at 8:56 am
mkelly says: “Steve this is wrong. Let me do this again basic formula q/A = e (SB)(T1^4-T2^4) if T1=T2 then zero energy flows anywhere.”
Did you think that through? You are saying that, in the one case where T1=T2, the equation shows that there is no energy flow between the objects. But in all other cases where T1 is not equal to T2, you can have either a positive or negative energy flow between the objects. What would it mean to have an energy flow less than zero?
Are you positive this equation is representing individual energy flows? I think it represents net energy flow, hence it can be either zero or even negative.
Are you saying that if I surround a 500 degree ball bearing by a 600 degree sphere, the 500 degree ball bearing stops radiating energy away from itself, and the 600 degree sphere will begin radiating a less energy towards its center?
Steve a negative sign would indicate you put it in the formula wrong. You must have missed the part about watt equal joule per second and joule is unit of energy so no energy gain by either object.
I have not said that cooler objects don’t radiate at warmer objects but here is no demonstrable proof that the hotter object absorbs radiation of a lower frequency.

wayne
May 18, 2011 1:47 pm

If anyone takes me up on listening to Dr. Feynman’s lectures, pay particular attention when he teaches you that “there is only one kind or type of photon”. Did he really mean that? Really! Frequencies of radiation has to do with the “source matter”, not the photon. They just come from the “source matter” at different frequencies. There are not different frequency types of photons, all photons are identical! Now, listen to his lectures, two or three times if necessary (me? four times).
Quantum is a strange twist indeed, but all experiments seem to show in every case and to huge precision that it is what is REALLY happening everywhere in our universe, our Earth, out atmosphere, and even down to single atoms. Well, not in Joel case after reading his response to mkelly and Phil. in his response to Dave Springer just above. Not in their minds. Watch them dance all around reality to fit their “worldview” and it always ends up firmly planted in AGW pseudo-science.

Steve
May 18, 2011 2:01 pm

mkelly says: “Steve a negative sign would indicate you put it in the formula wrong. You must have missed the part about watt equal joule per second and joule is unit of energy so no energy gain by either object.”
Begin with T2=T1. Then switch on a heating element within object 2. Soon T2>T1, and q/A is negative. What does that mean? It means the equation represents a net flow, with positive or negative indicating the direction of flow. Zero indicates no net flow.
“I have not said that cooler objects don’t radiate at warmer objects but here is no demonstrable proof that the hotter object absorbs radiation of a lower frequency.”
So what happens to the radiation? Does the hotter object reflect it or transmit it? Absorption, transmission and reflection are your three options. Think it through. Propose your hypothesis and how to falsify it in terms of the competing hypothesis.

May 18, 2011 2:23 pm

Robert Clemenzi says on May 17, 2011 at 4:43 pm:
“ —————. My interpretation of the data is that water vapor is IR opaque until the tropopause and that CO2 is opaque until a bit higher. This model completely explains why the middle is colder than both the air above and below the tropopause.”
Thank you Robert for a nice reply, I shall have to take a bit longer to think about it in full, but for now let me just confirm that as far your interpretation of the data is concerned they are not too far removed from my own.
I know the data say that 99 % of all the atmospheric Water Vapor (WV) is to be found in the Troposphere, so yes it is very likely to be some in the “tropospause” and even maybe as high up as in the Stratosphere. After all, they say it takes WV for the formation of “Cirrus Cloud” or Cirro Stratus. – But, even so we must not forget that because temperatures fall to well below 0 deg. C. in the upper Throposphere, much of it will have some kind of “Up Down” circuit high up in the Troposphere, i.e. rise – freeze –sink (drop) – melt, warm –rise – freeze – etc. etc. – And yes, – it should be equally opaque all the way.
However when it comes to the “conundrum” as to why the Troposphere cools, with height, the Stratosphere warms, the Mesosphere cools and the Thermosphere warms I think we shall have to take into consideration that there are two types of “radiation” i.e. “Ionizing” and “Non Ionizing” Ionizing is “short Wave as in solar incoming radiation that has enough energy to dilsodge electrons, non ionizing (long wave) has not got enough energy for that kind of behaviour and is the kind of radiation discussed amongst the (us) global warming pundits.
Some part of the atmosphere must be warming from the top down, but instead of me telling you what I conclude from that I urge you to use your own logic and decide for yourself.
And of course it is only the very few N, O and O2 atoms & molecules in the “upper atmosphere that warm, – not the space between them, so you will still freeze to death up there.
Just remember, (I haven’t yet formulated it but, – “energy absorption must lead to heat production. – Conversely, – heat absorption must lead to energy conservation”) — Oh, maybe somebody already thought of that one when he said “X (1 J of Energy) has got to enter before X (1 J of Energy) can exit”

Steve
May 18, 2011 2:26 pm

wayne says: “Try it. You’ll like it! Then come back and prove it to me that there are always photons passing back and forth within a smooth hollow steel sphere at any temperature. (hint: by cancelling amplitudes everywhere it seems it is zero photons inside).”
The key phrase is “seems”, as in the ability for the quantum components of the system to measure a photon within the sphere can drop to zero. Now go back to the other key phrase “by cancelling amplitudes”. Amplitudes cannot be cancelled if, in fact, there are no photons traveling 180 degrees out of phase with each other. So the cancellation occurs because of the fact that photons are passing back and forth.
This is cavity quantum electrodynamics, and not something with any ghost of a chance of being observed in macroscopic objects. We are talking about extremely small (micrometer scale) spheres.

May 18, 2011 2:38 pm

Oh yes Robert, O2 becomes O3 when ionized by UV radiation. – O3 as you probably know is OZONE.

richard verney
May 18, 2011 3:02 pm

Wayne
regarding your post wayne says: May 18, 2011 at 1:04 pm
I have previously mentioned something similar, ie., not disputing that all objects above absolute zero radiate but that does not automaticlly mean that radiative energy emitted by a colder object is actually absorbed by the warmer object. I will review the links to which you refer.
I am not for one moment saying that I have the answers. I think that either Joel or Tim set out an analogy with tennis balls. I do not particularly like giving analogies of matters not well understood. As far as the tennis analogy goes, it may be that on one side of the net there is a weaker player (ie., the side of the net with ‘colder’ photons) and that player cannot consistently hit the ball over the net such that balls are generally flowing from the side with the stronger player (the ‘hotter’ photons).
Whilst a photon may not have a temperature, it would appear that it has a temperature signature. I say this since it appears common ground that photons radiating from an object at temperature T cannot warm another oject to a temperature exceeding T. If they had no temerature signature the work that could be extracted from them would simply be proportional to the watts per sq m and this would mean that back radiation from the sky could be harnessed to heat objects with similar results to that achieved by using solar radiation. A black car is noticeably warmer to touch than a white car parked nearby during the day in sunlight, but at night there is no obvious difference. The back radiation from the sky that is bombardding the black car does not (noticeably) heat it to a temperature higher than a white car parked nearby.
If photons radiating from the atmosphere at say 240K towards the ground cannot heat the ground to above 240K, whereas photons radiating from the ground at say 288K towards the sky could potential heat an object to 288K, I question whether it is fully correct that one source can simply be deducted from the other since effectively the energy (the ability to do work) is different.
Hence I question Tim’s alternative take on the Trenberth energy budget wherein he deducts the (240K) 333 W/m^2backradiating from the sky from the (288K) 399 W/m^2radiating from the ground saying effectively that the ground is radiating 66 W/m^2.

jae
May 18, 2011 6:35 pm

wayne:
“Did he really mean that? Really! Frequencies of radiation has to do with the “source matter”, not the photon. They just come from the “source matter” at different frequencies. ”
Whaaaat? Have not listened yet, but this seems very confusing, since Planck gives the energy of a photon as E=hv. So, how can all photons be equal?
http://en.wikipedia.org/wiki/Planck_constant

wayne
May 18, 2011 10:02 pm

jae says:
May 18, 2011 at 6:35 pm
Whaaaat?
—–
I agree. Caused me halt, reset, and rethink (with a headache). On the surface it seems to imply that the frequency of light is merely the frequency that identical photons are emitted by the source. Seems wrong but that does bring back words of many years back of photon packets or trains, forgot that long ago. Or maybe he just meant photons have no attatched properties that man has found that can be altered. Then I start thinking, well, how do photon particales get doppler shifteded? Is that related to his satement?
I’m totally open for help on exactly what Dr. Feynman could have meant by that statement. That is why I threw it out there. HELP.
Do watch at least the first two, may be three, videos. You should enjoy, he’s very entertaining. (BTW: there is some static in video #2 but they do correct it some twenty minutes in)

wayne
May 18, 2011 10:16 pm

jae, whoa, excuse me for not running that through a spell checker! Don’t know about you but the words are appearing two or three behind where I am typing. So slow…

May 19, 2011 12:09 am

Response to O H Dahlsveen – May 18, 2011 at 2:23 pm
There are several, somewhat conflicting, definitions of the tropopause. However, the data indicates that the temperature decreases until water vapor drops to 5 ppm. (Some sources say zero.) For the stratosphere, the water vapor stays about 5 ppm until the stratopause. Above that, there is no detectable water vapor. It is my opinion that this distribution is somehow related to the temperature profile.
O2 becomes O3 when ionized by UV radiation
Well, yes and no. Apparently, UV radiation is more likely to convert O3 into O2 and an oxygen free radical than the other way around. From what I’ve read, UV converts H2O and O2 into OH radicals. Then UV combines the OH radicals with O2 to make ozone. When an additional UV photon destroys the O3 molecule, the excess energy becomes heat. The reason that there is a hole over Antarctica is because it is too cold for the OH radicals to form. I know this disagrees with what is in the popular press, but it appears to more correct than what most sources say.
At any rate, once the water vapor goes to zero, there is no longer a hydrogen source for the OH radicals and the temperature decreases above that point. The weird thing is that there is almost no ozone where the temperature is highest, which strongly argues against my theory.

wayne
May 19, 2011 12:13 am

jae, I’m getting way OT so I’ll stop on the quantum aspect here.
I did find his insisting that all photons being identical, #3@23min, but of course he is not invalidating E=hv, seems he is only speaking of when you look at existence (amplitudes) in spacetime. Later he shows that even electrons and photons are identical except for the mass, more, even all point-like particles are identical but mass (muons, etc.). That is what experiments show really happens. That in itself is getting too way too deep for here, any further and I’ll really get myself in trouble by wording something wrong.
My only point in starting this dive into the depths is sometimes all things are not as they first appear (or how others think they must appear). That’s all.

Dave Springer
May 19, 2011 7:10 am

“Then I start thinking, well, how do photon particales get doppler shifteded?”
If photons were baseballs and spectrometers were baseball bats then the doppler effect is the difference between a bunt and a swing.

May 19, 2011 8:20 am

So I’ve been looking over the Trenberth diagram Joel pointed me to from the point of view of my new-found understanding of the bi-directionality of energy exchanges.
It has been well established here that it is the net energy flow which heats and that it always flows from hot to cold. Therefore it is only the net energy flows which need be considered when estimating the radiative heat transfers in the diagram.
Based on that there are a few things that still puzzle me about this diagram:
1- The total net radiative energy from the Sun to the Earth is approximately 239W/m^2 (16W/m^2 to the surface + 78W/m^2 to the atmosphere) and the total net radiative energy from the Earth to space is 239W/m^2 (169W/m^2 +30W/m^2 + 40W/m^2).
Therefore, as I understand it, based on the repeated assertions made here, there is no net heat and energy transfer between the Sun and the Earth.  Which means that the Earth is neither cooling nor warming.
Is this correct?
2- The surface radiates 453W/m^2 (17 + 80 + 356) to the atmosphere and the atmosphere radiates 333W/m^2 towards the surface, for a net heat transfer of 120W/m^2 from the surface to the atmosphere.  Additionaly the atmosphere receives 78W/m^2 radiative energy transfer from the sun, for a total energy gain of 198W/m^2.  The atmosphere also radiates 199W/m^2 (169 + 30) to space, for a net loss of 1W/m^2 or approximately ) 0.505% of the energy input.
So this suggests that the atmosphere is cooling very slightly, as Tim pointed out, 1 m^2 of atmosphere may contain 30,000,000 J of energy.  However, unless I am mistaken that 1W/m^2 = 1J/second/m^2/K and therefore over the course of 100 years the atmosphere would have lost 3,155,760,000 J or 3.15576x10e9J which should represent significant cooling.
Is that correct?
I am not trying to deny anything here, just trying to understand.

Dave Springer
May 19, 2011 8:41 am

wayne says:
May 18, 2011 at 1:04 pm
You seem to have missed the beginning lesson that theory must explain reality. If it fails then the theory is wrong. Reality trumps theory in all cases.
In the particular instance of so-called back radiation the reality is that if you point an IR spectrometer up at the night sky photons of far higher energy than the cosmic microwave background are hitting it. Theory must explain these. In fact theory does explain it. If your understanding of the theory denies reality in this case your understanding is flawed.
In each and every instance where someone denies the reality of back radiation they are wrong. To say this back radiation might be real but a warmer surface cannot absorb photons emitted by a colder surface is also wrong as the reality is that if you measure the rate of energy loss between two surfaces of different temperatures the rate decreases as the temperature difference decreases and increases as the temperature difference increases. Thus when the two surfaces reach the same temperature there is no loss or gain by either of them. But just because the two surfaces are the same temperature doesn’t mean they stop emitting energy. It only means they are absorbing as much energy as they are emitting. This is the reality of the situation.

May 19, 2011 8:52 am

Alleyne, you are basically correct, except that a difference of 1 W/m2 is less than what can actually be determined. As a result, it is not correct to assume that the atmosphere is actually cooling that much. Based on actual measurements, it is not currently possible to determine if the net balance is other than zero.

Dave Springer
May 19, 2011 9:09 am

@Alleyne
“Therefore, as I understand it, based on the repeated assertions made here, there is no net heat and energy transfer between the Sun and the Earth. Which means that the Earth is neither cooling nor warming.”
That is correct for a system in equilibrium. The earth/sun system is never in perfect equilibrium but it will always seek to attain equilibrium and the farther out of equilibrium the harder it tries to reach equilibrium.
The AGW thesis is that additional CO2 moves the system further out of equilibrium and that in moving back towards equilibrium the surface temperature will rise.
That thesis is fine in theory up to the point where the claim is made that surface temperature must rise. The surface temperature is hypothesis not theory. There are other possible responses the system might make to move back towards equilibrium.
Two possible responses are:
1. An increased rate of evaporation and convection will move the additional energy at the surface to a higher layer in the atmosphere and because evaporation carries energy in what’s called “latent heat” there will be no measurable rise in temperature near the surface as thermometers measure what’s called “sensible heat”.
2. An increased amout of energy at the surface may cause an increase in albedo higher in the atmosphere through change in type and/or amount of cloud cover which simply reflects more energy away before it ever reaches the surface.
The reality of the situation is that no matter how much CO2 is in the atmosphere at least up to the point of 10x today’s level surface temperature correlates poorly with it. The reality is also that anywhere up to at least 10x today’s level of CO2 there is no runaway greenhouse.
The reasonable interpretation of the reality of the situation is that there is a negative feedback loop involved with increased energy at the surface. The negative feedback is almost certainly one or both of the equilibrium responses enumerated above.

wayne
May 19, 2011 9:19 am

Dave, I accept that analogy of a bat, but it was more questioned in relation to the statement by Feynman that all photons are identical. Now I regret even mentioning it. As I clarified later, that seems to have been said by him only as to how amplitudes were being calculated in relation to time. That’s the catch that I missed. ☺

Dave Springer
May 19, 2011 9:44 am

By the way, Feynman in his dated lectures insists reality is non-deterministic. Some physicists every bit as smart believe otherwise. Indeterminism implies that information (not the layman’s concept of “information”) can be destroyed. This concept of information being non-destructable is a basic tenet of modern physics. It reaches so deeply that a rather notorious claim by Stephen Hawking was argued for years by the most highly respected theoretical living physists and recently Hawking admitted defeat.
Hawking proposed that if a set of encyclopias fell through the event horizon of a black hole that the information in them could never be recovered. He didn’t even propose that the information was destoyed but merely lost forever to the universe outside the black hole’s event horizon.
Othere physicists claimed it was not possible to remove information from the universe even in the most exotic situation imagineable – falling into a black hole – because conservation of information is as inviolable as conservation of energy and that both conservation laws are in fact different manifestations of the same phenomenon.
Hawking conceded when sufficiently convincing mathematics showed that the information in the enclopedia would reemerge as patterns in so-called Hawking radiation emitted by the blackhole. Hawking radiation is based on the well established fact of quantuum tunneling where a particle may disappear at one point in space and reappear at another point without enough energy to have moved across a barrier from point A to point B. Flash memory chips work by quantuum tunneling where an electron is raised to an energy level just short of being able to cross a barrier into a holding pen. The electron then tunnels through the barrier and its energy level is reduced such that the probability of it tunnelling back in the other direction is so small you count on it never happening. When power is removed from the flash memory device the electron stays trapped indefinitely.
So there you have it. At the very deepest level the universe still appears to be deterministic and Feyman’s dated claim that it isn’t was premature at best and probably wrong at worst.

Dave Springer
May 19, 2011 10:04 am

Not to get too much farther off-topic but the law of conservation of information reaches deep into theology. If indeed an inviolable law then it implies all the information in the universe today, which includes all the information encoded into each and every human brain, was present in the universe since the instant of the big bang. This then raises the question “where did that information come from?”. Or if the information wasn’t present at that time then “where, when, and how was the information introduced into the universe?”. Science has no answer, or rather the best current answer offered by science is the so-called multiverse which postulates a reality of an infinite or near infinite number of universes each born with different information in them and so in an infinite series there must inevitably be a universe just like ours – in fact an infinite number of them just like ours. By implication there would also be an infinite number of them that are infinetely close to the same as ours but not quite.
The multiverse hypothesis of course doesn’t pass the giggle test and theologians answer it by saying God made the universe the way it is. I don’t have a position on the matter either way but the multiverse explanation seems just as contrived, if not moreso, than the God hypothesis.

May 19, 2011 10:19 am

Mr. Springer says: “To say this back radiation might be real but a warmer surface cannot absorb photons emitted by a colder surface is also wrong …”
Harkening back to my heat transfer classes it was explained that higher temperature object does not absorb lower temperture emissions. The object at the higher temperature is already at a state higher than what the lower temerature object is or so there is no “where to go” with the lower energy.

Dave Springer
May 19, 2011 10:54 am

wayne says:
May 18, 2011 at 1:47 pm
“If anyone takes me up on listening to Dr. Feynman’s lectures, pay particular attention when he teaches you that “there is only one kind or type of photon”. Did he really mean that? Really!”
Sure he meant it. Going back to the baseball analogy where the rules sanction only one baseball so that all baseballs used in the games are as identical as possible. The only difference as it approaches the batter is thus reduced to the spin and speed imparted to it by the pitcher. Photons are pretty much like that. The rules of the universe make them all the same except for the amount of energy in them which is determined by the pitcher (the object which emitted it).
Now that I think about it Feynman got something else wrong in his lecture to the best of my knowlege. He insisted that electromagnetic radiation is entirely a particle phenomenon. I believe that’s incorrect and the modern interpretation is that EMR exhibits wave-particle duality (in fact ALL matter has wave-particle duality) and this was verified by the famous double slit experiment.
http://en.wikipedia.org/wiki/Double-slit_experiment
The truest thing Feynman said was the reason this is all so confusing to students is because the physics professors trying to explain it don’t fully understand reality either. That includes Richard Feynman. The theory of reality is woefully incomplete. We still don’t have, for instance, a quantum theory of gravity which is required to reconcile relativistic physics with quantum physics. Essentially we don’t know the properties of gravity at the quantum scale. We only know it at the bulk scale and have no quantum theory which predicts or explains gravity at the macroscopic scale.
There’s no explantion for all kinds of observations. My favorite is so-called “dark energy” which somehow, in some way, overcomes gravity across vast distances and is accelerating the expansion of the universe. The standard model offers no explanation but the observations reveal that dark energy must comprise some 70% of the “stuff” which makes up the universe. In other words 70% of what makes up you, me, and the rest of the universe is a complete mystery to modern physics. We think we know a lot but in reality we know about as much about the universe as you can know about the ocean just from observing the waves & froth at the surface.

Dave Springer
May 19, 2011 11:09 am

I lost track of who said the lapse rate & hence temperature structure of atmospheric layers can’t exceed dry adiabatic lapse rate.
This is a nonsense claim. There are three kinds of lapse rates: dry, saturated, and environmental. The environmental lapse rate brings convective processes into the picture and explains the reality of temperature inversions where a warmer layer of air is riding on top of a cooler one. Inversions don’t just exceed the dry lapse rate, they change its polarity!

Dave Springer
May 19, 2011 11:26 am

re: “dry adiabatic lapse rate can’t be exceeded”
Reality trumps theory. Write that down.
http://www.erh.noaa.gov/rah/education/eduit.html
My emphasis.

We have examined the stability of an atmospheric layer by comparing the layer’s lapse rate to the dry adiabatic lapse rate. By heating a fluid (air, of course, being a fluid) from below, we increase the lapse rate. This means temperature changes more per unit altitude because the ability of the fluid to conduct heat without motion is exceeded. Since heat conduction is a slow process, heating a fluid from below normally creates a large temperature difference across the depth of the fluid. Once the dry, adiabatic lapse rate is exceeded, the slightest push up or down on the parcel causes that parcel to rise or sink. Thus, a fluid with a lapse rate exceeding the dry adiabatic rate overturns very easily (Doswell, C.A.III, et al., 1991: Basic Convection I – A Review of Atmospheric Thermodynamics, NOAA-NWS, Norman, OK).
Rarely do we see an environmental lapse rate as great as the one in Figure 4 except near the ground on days of strong surface heating by the sun. In a shallow layer near the ground, it is possible for extremely high lapse rates to develop on a sunny day. Here, the rapid overturning of the air creates the shimmering effect you see near the ground on such days. Elsewhere in the atmosphere, sufficient random motion exists to quickly relieve the imbalance by mixing the layer of unstable air before it becomes established.

So, environmental lapse rate is typically exceeded on warm sunny days over land.
Presumably CO2 increases surface warming over land (something I’ve never disputed) which presumably tends to increase the incidence of environmental lapse rate exceeding dry adiabatic lapse rate.
QED

Dave Springer
May 19, 2011 12:13 pm

Regardless of weird environmental lapse rates in unstable situations in the lower troposphere my point was that additional CO2 can add water vapor to the lower troposphere creating a stable situation of environmental lapse rate smaller than dry adiabatic rate by decrement larger than less CO2. This effectively raises the emission altitude. While it’s true that dry adiabatic lapse rate can’t be exceeded higher in the atmosphere which does indeed set a limit on the temperature differential that can exist from top of troposhere to space there is nothing that prevents more or less mixing of layers in the lower troposphere. And of course changes in albedo make the dry adiabatic lapse of stratsosphere a moot point because high altitude clouds reject insolation before it ever gets a chance to reach the troposphere.
Toy models called GCMs are so from sophisticated enough to model everything that’s happening it isn’t even funny it’s just sad where the sad part is there are so many ostensibly smart people who don’t know a toy when they see one.
For crying out loud the gravitional mass of Jupiter makes small changes in the eccentricy of the earth’s and depending on whether eccentricity max/min is in or out of phase with earth axial tilt min/max spells the difference between glaciers a mile thick covering everything north of Washington, DC and whether grass is able to sprout in Montreal.
Note that changes in axial tilt and eccentricity do not, in and of themselves, change by one iota the amount of insolation at the top of the atmosphere over the course of year it merely changes how the insolation is delivered across the surface and across the seasons in a year.
Can these toy models predict the end of the Holocene interglacial? Do they even take into account axial precession and orbital eccentricity? If they’re supposed to keep track of climate change for us it seems the climate change represented by the contrast between glacial and interglacial periods is a pretty dramatic change and would be of utmost importance in being predictable within a 100 years or so precision.
But no. The toy models that can’t tell us when the interglacial period is going to end, or even grossly hindcast the glaciation cycle, can tell us within a degree or two what the addition of a small amount of trace greenhouse is going to do. This claim of model accuracy is made even more ludicrous when coupled with the fact that over geological periods of time there is little correlation between CO2 and temperature other than a post hoc response where temperature change precedes CO2 change (probably due to chemical and biological response to hotter/colder). Indeed all the evidence from prehistory suggests CO2 level is driven by temperature change not CO2 change drives temperature change. The climate boffins have the relationship ass backwards and all from just a correlation in the last 30 years.
Correlation DOES NOT equal causation. Write that down.

May 19, 2011 1:05 pm

Robert Clemenzi says on May 19, 2011 at 12:09 am:
“—————, which strongly argues against my theory.”
Maybe so Robert, but your theory seems, mostly, quite good to me. The only thing that can prove or disprove any theory is more data, – good hard data. I was mainly, in the case of “Ozone” going by the fact that the word “Ion” describes an atom, or groups of atoms that have lost or gained an electron and as my search to find out how ozone is formed, I mainly came across things like the following:
“In the upper atmosphere: Light from the Sun breaks apart oxygen molecules (UV wavelengths of 215nm or shorter). Some of this monatomic oxygen combines with nitrogen molecules, some with oxygen molecules to make ozone, but most with other monatomic oxygen. Some of the nitrogen+oxygen molecules can catch lower energy light (still UV, but more available) and make ozone also.”
However I do not find – anywhere – that temperature is involved in the making or breaking of ozone, but it may well be that the “Ozone Hole” (OzH) is located above the S. Pole because it is colder than the N. Pole, after all neither pole has any sunlight at all for half the year. – What I find interesting, however, is how the “OzH” manages to stay static (more or less) above the S. Pole during the whole winter period. (No northward air movements?)
As for the popular press, Al Gore, his scientific team and their claim that CFC gases are responsible for the creation of the OzH, I must say I personally have my many doubts of their validity. The main reasons, for my doubts, being that CFCs are heavier than air and are not likely to streak off to the Stratosphere and that they are mainly produced in the Northern Hemisphere by us humans and taking the Hadley, Ferrell and Polar Cells into account the North Pole is by far a more likely candidate for a hole in the Ozone.

May 19, 2011 1:54 pm

O H Dahlsveen says:
May 19, 2011 at 1:05 pm
“However I do not find – anywhere – that temperature is involved in the making or breaking of ozone, but it may well be that the “Ozone Hole” (OzH) is located above the S. Pole because it is colder than the N. Pole, after all neither pole has any sunlight at all for half the year.”
A good chemistry book or a thermodynamics book should have the requirements for energy to break bonds. Or you can apply Wiens Law to the 215nm UV and get a temperature of 13900 K but I am not sure if the law applies that high in the spectrum.

May 19, 2011 3:12 pm

Lapse Rates – the dry adiabatic lapse rate (DALR) is -9.8 K/km.
When someone says that the DALR can not be exceeded, what they really mean is that a more negative value is unstable and short lived. Thus, a lapse rate of -12 K/km would be extremely unstable because the colder air would be denser than the warm air below it.
There are two important exceptions to this. Given two almost identical parcels of air at the same temperature and pressure, the one with more water vapor will be less dense. The other is related to wind speed. In the right conditions, cold air can be blown over warm air.
However, in the long term, what matters is that more dense air masses will tend to sink, and less dense ones will rise.
Another common exception to “the rule” occurs most days. It is fairly common for the surface to be about 57C (135F) while the air 1 meter above is only 21C (70F). This produces a boundary layer lapse rate of about -36,000 K/km. This is the reason that standard temperature measurements are made a fixed height above the ground and is also the reason that satellites can not be used to measure temperatures over land.

Myrrh
May 19, 2011 3:37 pm

Steve says:
May 18, 2011 at 7:14 am
Re my: “The ‘statistics of large numbers’ is no proof whatsoever that heat flows from the colder to the hotter, ever. That’s just been added in without any logical antecedents.”
Heat doesn’t flow from the colder to the hotter, energy flows from the colder to the hotter. The heat flow from the hotter to the colder is the net energy flow.
Heat is energy on the move spontaneously from higher to lower in temperature difference. The colder does not have any heat to send to the hotter because it only exists in flowing from the hotter to the colder.
So what ‘energy’ is your colder sending to the hotter?
Re my: “If the colder could add its energy to the hotter, then statistically in the ‘large number of events’ the colder could always be giving up its energy to the hotter, and losing heat energy it has so itself getting colder..”
..and gaining what energy the hotter object transfers to it, for a net effect of getting warmer. The hotter object is transferring more energy to the cooler object than the cooler object is transferring back to it, for a net effect of getting cooler.
So when I put my hot cup of hot coffee in the middle of the Arctic, all the energy flowing from the cooler to the hotter will boil it away, or at least keep it hot until I can drink it? Because there must be so much more energy flowing from the cooler to the hotter to tip the balance in its favour to be the dominant heat source.

May 19, 2011 4:34 pm

Dave Springer said on Visualizing the “Greenhouse Effect” – Light and Heat
May 19, 2011 at 9:09 am
“The reasonable interpretation of the reality of the situation is that there is a negative feedback loop involved with increased energy at the surface. The negative feedback is almost certainly one or both of the equilibrium responses enumerated above.”
Alleyne replies:
Dave,
Thank you for the excellent explanation.   I subscribe to your hypothesis that there are one or more negative feedbacks which return the system to a state at or near equilibrium. From all that I have read of Roy Spencer’s, Roger Pielke Sr’s, Courtillot’s and others, it seems to me that the net feedback in the climate system is negative and tied to that magical substance, water.

May 19, 2011 4:35 pm

Robert Clemenzi said on Visualizing the “Greenhouse Effect” – Light and Heat
May 19, 2011 at 8:52 am
Alleyne, you are basically correct, except that a difference of 1 W/m2 is less than what can actually be determined. As a result, it is not correct to assume that the atmosphere is actually cooling that much. Based on actual measurements, it is not currently possible to determine if the net balance is other than zero.
Alleyne replies:
Thanks Robert.  I appreciate that the 1W/m^2 is less than the error in the measurements.  I would be surprised if the radiative energy measurements were much better than +/- 5% and in any case the sun’s outptut fluctuates over time, the albedo is apparently +/- 5% and so on…
I wasn’t trying to imply that the atmosphere must be cooling based on that 1W/m^2, just using the numbers available to confirm that I understood what the diagram was saying.

May 19, 2011 4:46 pm

Dr. Roy Spencer posted a very interesting article here: http://www.drroyspencer.com/2011/05/indirect-solar-forcing-of-climate-by-galactic-cosmic-rays-an-observational-estimate/
that relates to this thread in that what it presents is quite convincing that CO2 is not the only, and perhaps not even the major forcing, in the climate system – and ties in nicely with a comment Dave Springer made earlier.

Phil.
May 19, 2011 5:52 pm

mkelly says:
May 19, 2011 at 10:19 am
Mr. Springer says: “To say this back radiation might be real but a warmer surface cannot absorb photons emitted by a colder surface is also wrong …”
Harkening back to my heat transfer classes it was explained that higher temperature object does not absorb lower temperture emissions. The object at the higher temperature is already at a state higher than what the lower temerature object is or so there is no “where to go” with the lower energy.

Unfortunately you had a bad teacher, he was wrong as far as solids and liquids go. With gases only the energies that exactly match the energy level separation can be absorbed, the temperature of the source is irrelevant.

Jim Masterson
May 19, 2011 6:30 pm

Years ago we used to argue about the importance of CO₂ as a GHG. We never resolved this issue and now years later I see we are still arguing about it. The important CO₂ GHG frequency is the 15 micron band. This band is at the upper end of the atmospheric window (or lower end depending on whether you’re using wavelength or frequency). The Wien’s Displacement Law temperature for this wavelength is about -80 °C. The average planetary temperature is supposed to be far warmer so CO₂ doesn’t provide much blocking in general. In fact, part of the argument is that CO₂ by itself doesn’t do much at all, but in conjunction with water vapor we are supposed to have a serious problem. Again, more arguments ensued on this point. This is usually called the Enhanced Greenhouse Effect. Argue, argue, argue.
So along comes Kiehl and Trenberth 1997. Their figure 7 appears everywhere: the last two IPCC reports, my GCM text book (643 pages, whew!), all over the web, and in conversations like this one. What makes KT 97 important is that this is the quintessential model of the GH effect. Every GCM should average out to the same general values in KT 97. Not exactly, because some of those values are off by 10%, 50%, maybe 100%, or more. However, the basic flows are about right (for the GHG model), and GCMs are basically duplicating KT 97’s flows and feedbacks (although at a much finer detail).
I created a simple model based on KT 97, and it does stabilize at the values in KT 97. (I designed it that way. But the model is flexible enough to stabilize at many values, even the new ones in Trenberth’s update of KT 97.) This is a steady-state model. Such models are common in EE where we apply a step input and wait for the transients to die out.
My model assumes a black-body for the surface and another for the atmosphere. (I’m not going to argue about whether a colder black-body can warm a hotter black-body. Of course it can. That’s how black-bodies are defined–they absorb all radiation that falls on them. The reason why this doesn’t violate the Second Law of Thermodynamics is that warmer black-bodies radiate more energy than colder black-bodies.) Because KT 97 is a steady-state model, my version of it maintains its values until it is perturbed.
There are several ways to perturb my model. One way is to narrow the atmospheric window. This simulates adding more GHGs. The advantage with this method is that we don’t need to distinguish between GHGs. If we are modeling the Enhanced GHG Effect, then adding more CO₂ and more H₂O will essentially do the same thing–narrow the atmospheric window. There’s no need to argue.
What happens is that the model values start to fluctuate, and they need several cycles to pass before they stabilize at their new values.
To determine temperature, we need to use the Stefan-Boltzmann Law. The Stefan-Boltzmann Law requires heat flux, the Stefan-Boltzmann constant, and the emissivity of the surface (or effective emissivity of a gas). If we hold the latent and sensible heat fluxes constant, keep the atmosphere upward-downward ratio the same, and assume both surface and atmosphere emissivities at 1.0, then when we narrow the window ever so slightly, the surface temperature increases. But the atmosphere temperature increases faster–about 1.15 times as fast. If we let the sensible heat flux change according to the bulk aerodynamic equation (as KT 97 does), and let the latent heat flux increase in proportion to the surface temperature, then the atmosphere heats faster–about 1.45 times.
It doesn’t get any better for this model if we start using more realistic values for emissivity (0.95 for the surface and say 0.6 for the atmosphere). For those values of emissivities, the atmosphere warms at an even faster rate–about 1.63 times.
The current rate of atmospheric warming is much less–from 0.7 to 0.9 times the surface. At those rates, any surface warming is not due to the GHG effect. Hansen’s tweaking of the surface temperature is only making matters worse. By increasing the surface temperature faster, he is further decoupling the surface heating from the atmosphere. Tsk, tsk.
Jim

Steve
May 19, 2011 7:10 pm

Myrrh says: “So what ‘energy’ is your colder sending to the hotter?”
The same energy it is emitting if it is surrounded by objects half it’s temperature. Do you think a 500 degree object emits energy towards the walls in a 250 degree box, but stops emitting energy towards the walls in a 1,000 degree box?
“So when I put my hot cup of hot coffee in the middle of the Arctic, all the energy flowing from the cooler to the hotter will boil it away, or at least keep it hot until I can drink it?”
No, the cup will emit more energy than it absorbs, for a net effect of cooling to the temperature of the surroundings. It will cool to the temperature of the surrounding Arctic, and not to absolute zero, because the surrounding Arctic is emitting energy towards the cup that far exceeds the absolute zero of space.
“Because there must be so much more energy flowing from the cooler to the hotter to tip the balance in its favour to be the dominant heat source.”
How?

IAmDigitap
May 19, 2011 7:22 pm

I wonder how floored every one of the mathematicians will be, when it’s revealed to them that if there were any GHG THEORY effect,
optical telescopy would have to be showing it; because ANGULAR MOMENTUM in GAS is EQUIVALENT to HEAT on that GAS, and if there has been MORE HEAT,
then OPTICAL TELESCOPY would have NOTICED the VIEWING getting WORSE, and WORSE, and WORSE all these years.
What about the INFRA-RED telescopy field? WHY AREN’T THEY reporting EVER RISING ATMOSPHERIC I.R. CONTAMINATING their INSTRUMENTS?
B.E.C.A.U.S.E. kids, there never WAS
there is NOT,
and no, no matter how many times someone claims to have calculated ENTROPY failing,
there won’t be TOMORROW, a G.reen H.ouse G.as Effect.
Optical Telescopes MAGNIFY ANOMALIES in the ATMOSPHERE MANY TIMES.
They have ASSEMBLIES which FLEX the MIRRORS to OFFSET this
ATMOSPHERIC SCINTILLATION also known as the stars twinkling.
In GAS, more HEAT is MORE MOTION. More MOTION is more DISTORTION.
P.e.r.i.o.d.
If there were MORE HEAT or MORE I.R. DISTORTION, then THE PEOPLE who CONSTRUCT and MAINTAIN the ASSEMBLIES which FLEX TELESCOPE MIRRORS to ADJUST for HEAT DISTORTION
would have LONG AGO trotted out T.H.E.I.R. DOCUMENTATION that SURE ENOUGH: the ATMOSPHERIC DISTORTION was GROWING.
But of course in ALL these years, N.O.T.H.I.N.G. because THERE’S N.O.T.H.I.N.G.
there.
So no, I didn’t chase down the math to see where you flipped your polarities so the
HIGHLY TURBULENT
HEAT CONDUCTIVE
COMPRESSED FLUID
F.R.I.G.I.D.
immersion bath
comes out defying entropy.
A WARMING BLANKET EFFECT is SIMPLY not the PHYSICS a
HIGHLY TURBULENT
FRIGID
immersion bath,
has.
IF you thought they have the same characteristics, no.
If you can’t be bothered with yet another “I calculated entropy out of existence” then look to your optical telescopy, your i.r. telescopy, the REAL atmospheric radiation business, two-way electromagnetic communications:
WHY UTTER SILENCE although these fields deal DAILY with the REAL artifacts of ATMOSPHERIC RADIATION?
Why is it when you google “Electronic Engineers endorse AGW” YOU get Z.E.R.O?
Because the PEOPLE who MAKE the INSTRUMENTS these MATHEMATICIANS and GEOLOGISTS MIS-READ,
know all that ‘reversal of entropy’/’reversal of gravity’ (yes it’s necessary)/’reversal of mathematics so now, entropy no longer works’
is so much bunk by people who don’t work daily in plying the atmosphere with radiation, and recapturing, then analyzing it.
A.G.A.I.N: although I understand blogs aren’t school, I’ll LEAVE it to YOU to DETERMINE how there is A.N.Y. G.H.G. EFFECT, if even the INSTRUMENTS which MAGNIFY ATMOSPHERIC HEATING THOUSANDS of TIMES as they TRY to LOOK THROUGH IT,
pick up N.O.T.H.I.N.G.
N.ot
O.nce
N.ot
E.ver
NONE of the MANDATORY SIGNATURES of HEAT’s PRESENCE.
NONE of the MANDATORY INTERFERENCE with I.R. Telescopy.
NONE of the expected CLAIMS from COMMUNICATIONS and MILITARY RADAR people about the DIFFERENT HEAT CHARACTERISTICS, giving DIFFERENT ATMOSPHERIC TRANSMISSION and RECEPTION characteristics,
those guys, have documented M.A.N.Y. places on either side of zero.
There IS none of this EVIDENCE because there I.S. no G.H.G. FACT only G.H.G. Myth of MAGIC GAS that does MAGIC THINGS that NO INSTRUMENT
ever
detected.
It’s NOT EVEN MAKING the STARS TWINKLE more, but you’re expected to believe ENTROPY blinked, and a WELL MIXED, MILES DEEP FRIGID BATH
warmed
a rock.
Yea. Well, you go check those telescopic usage journals and WE’D BETTER SEE, FEWER VIEWING DAYS due to EXCESSIVE DISTORTION particularly AT NIGHT.
You go check those I.R. telescope journals, and see for yourself: the TOTAL LACK of A.N.Y. DOCUMENTATION of IMPEDED VIEWING at ALL.
When you have O.N.E. INSTRUMENT that DETECTS that FAIRY TALE you’ll have a FAIRY TALE with some legs.
Till then it’s just more of the “I calculated that entropy blinked, let’s dismantle civilization,” routine from people who simply
never
tested things
for money
for a living.
PARTICULARLY in the field of atmospheric radiation transmission, capture, analysis, etc.
NOBODY in the WORKING ATMOSPHERIC RADIATION FIELDS, OR in the INSTRUMENTATION FIELDS, believes in this V.O.O.D.O.O.

jae
May 19, 2011 7:28 pm

“My model assumes a black-body for the surface and another for the atmosphere. (I’m not going to argue about whether a colder black-body can warm a hotter black-body. Of course it can. That’s how black-bodies are defined–they absorb all radiation that falls on them. The reason why this doesn’t violate the Second Law of Thermodynamics is that warmer black-bodies radiate more energy than colder black-bodies.) Because KT 97 is a steady-state model, my version of it maintains its values until it is perturbed.”
Kook alert!

Jim Masterson
May 19, 2011 8:17 pm

>>
jae says:
May 19, 2011 at 7:28 pm
Kook alert!
<<
Really? You’ve cut me to the quick. Maybe you should look up the definition of a black-body.
Jim

Phil.
May 19, 2011 8:19 pm

Jim Masterson says:
May 19, 2011 at 6:30 pm
Years ago we used to argue about the importance of CO₂ as a GHG. We never resolved this issue and now years later I see we are still arguing about it. The important CO₂ GHG frequency is the 15 micron band. This band is at the upper end of the atmospheric window (or lower end depending on whether you’re using wavelength or frequency).

Middle if you’re using frequency.
The Wien’s Displacement Law temperature for this wavelength is about -80 °C.
You should use the frequency form of Wien’s Law not wavelength, we want to know where the bulk of the energy is emitted. The peak energy at ~20THz (i.e.~15micron wavelength) is ~340K.
The average planetary temperature is supposed to be far warmer so CO₂ doesn’t provide much blocking in general. In fact, part of the argument is that CO₂ by itself doesn’t do much at all, but in conjunction with water vapor we are supposed to have a serious problem. Again, more arguments ensued on this point. This is usually called the Enhanced Greenhouse Effect. Argue, argue, argue.
Predicated on a false premise, not correct.

jae
May 19, 2011 8:25 pm

Open discourse.
Measured stuff (real empirical data!):
342 wm-2 coming in from Sun, which means same going out at equilibrium. This amount of radiation corresponds to -18 C or 255 K, assuming blackbody radiation.
However, avg. surface temp. is actually about 15 C or 288 K. Avg. BB radiation at this temp. = about 390 wm-2 from surface of planet. Thus, surface emitting much more radiation than is coming from Sun. Explanation needed.
Generally Accepted Explanations for discrepancy: Greenhouse effect ADDS heat and radiation to surface by some magical mechanism explained vaguely as: backradiation; insulation ; slowing of loss of heat by backradiation; or other other claptrapcrap ™.
Problems:
Cannot explain the 390 wm-2 (15 C) from surface with any of above concepts, if the Sun is only adding 342 wm-2, despite testimony from so many “experts.” No way to explain added heat. The backradiation from the cold air cannot HEAT the surface above the -18 C equivalent provided by the Sun, according to Second Law physics. Slowdown concept OK, but doesn’t explain higher radiation from surface than from Sun .
Only possible answer and conclusion:
Greenhouse Theory is garbage, despite the fact that it is “consensus,” even among “skeptics.”
Summary:
Surface cannot be continually irradiating 390 wm-2, if Earth is receiving only 342 wm-2, EVEN IF THE HEAT LOSS IS SLOWED BY BACKRADIATION. “Slowing” just ain’t enough; heat must be added, and that violates the Second Law, since it would have to be added by a colder source.
Mathematical and Physical Checkmate!
End discourse.
LOL.
New discourse:
390 wm-2 explained by ideal gas law, as can be shown with all other planetoids with atmospheres. Heat is STORED by molecules and that heat is released when backradiation is less than forward radiation. How novel.
End new discourse.

Phil.
May 19, 2011 8:43 pm

IAmDigitap says:
May 19, 2011 at 7:22 pm
I wonder how floored every one of the mathematicians will be, when it’s revealed to them that if there were any GHG THEORY effect,
optical telescopy would have to be showing it; because ANGULAR MOMENTUM in GAS is EQUIVALENT to HEAT on that GAS, and if there has been MORE HEAT,

Now this is a real kook!

Steve
May 19, 2011 11:06 pm

jae says: “The backradiation from the cold air cannot HEAT the surface above the -18 C equivalent provided by the Sun, according to Second Law physics.”
Wrong. The 2nd Law implies that the radiative energy emitted by the sun and atmosphere combined cannot heat the surface of the earth higher than the temperature of the surface of the sun.
“Surface cannot be continually irradiating 390 wm-2, if Earth is receiving only 342 wm-2, EVEN IF THE HEAT LOSS IS SLOWED BY BACKRADIATION.”
A point on the earth’s surface must eventually emit all radiative energy it absorbs. Radiative energy is emitted by the sun and absorbed by that point on the surface of the earth for approximately half of the day. Radiative energy is emitted by the atmosphere and absorbed by that point on the surface of the earth for 24 hours of the day. For that point on the surface of the earth to stop heating up (equilibrium) the radiative energy emitted by that point in a 24 hour period must therefore exceed the radiative energy it absorbed from the sun in that 24 hour period. This would be true for any point on the earth’s surface, so it is therefore true for the entire surface.
“Mathematical and Physical Checkmate!”
En passant.

Steve
May 19, 2011 11:21 pm

IAmDigitap says: “If there were MORE HEAT or MORE I.R. DISTORTION, then THE PEOPLE who CONSTRUCT and MAINTAIN the ASSEMBLIES which FLEX TELESCOPE MIRRORS to ADJUST for HEAT DISTORTION would have LONG AGO trotted out T.H.E.I.R. DOCUMENTATION that SURE ENOUGH: the ATMOSPHERIC DISTORTION was GROWING.”
A couple of degrees would disable their instruments so? They just read the air temperature and adjust, don’t they. Are you implying that they can’t observe the stars from day to day if the temperature changes? Their telescopes work in the winter, but not in the summer?

May 20, 2011 12:27 am

Response to O H Dahlsveen on May 19, 2011 at 1:05 pm
The CFC ozone hole only occurs in the polar spring, after the dark night is over. The destruction requires ice, extremely cold temperatures, and solar UV radiation. Only the Antarctic gets that cold, therefore, the hole is only in the Southern hemisphere.
According to one reference, in the normal atmosphere,
all of the O3 is destroyed by UV photolysis every few minutes, leading to the formation of free O atoms, and all of the O atoms are immediately consumed in reactions with O2 to reform O3 in a fraction of a second. section 2.2.1
http://www.ccpo.odu.edu/~lizsmith/SEES/ozone/class/Chap_5/index.htm
This reference has a whole section on the “hole” and the physics involved.

Myrrh
May 20, 2011 12:52 am

Steve says:
May 19, 2011 at 7:10 pm
How?
That’s what I want to know.
Because you’re saying that the colder is emitting the same kind of energy to the hotter.
Joel said it is the amount of hotter’s energy emitting to the colder being more than colder’s emitting to the hotter which gives a ‘net flow of hotter to colder’.
So, a cup of hot coffee in the Arctic is surrounded by tons and tons and tons of colder emitting the same energy towards it, the net effect must be for the coffee to get hotter.
Which is the AGW back-radiation of the colder atmosphere warming the warmer Earth further.
So you tell me how.

R Stevenson
May 20, 2011 1:48 am

Phil says:
‘Unfortunately you had a bad teacher, he was wrong as far as solids and liquids go. With gases only the energies that exactly match the energy level separation can be absorbed, the temperature of the source is irrelevant.’
I think this would be very difficult to prove experimentally.

May 20, 2011 4:59 am

Robert Clemenzi says on May 20, 2011 at 12:27 am:
“—————-. According to one reference, in the normal atmosphere,
all of the O3 is destroyed by UV photolysis every few minutes, leading to the formation of free O atoms, and all of the O atoms are immediately consumed in reactions with O2 to reform O3 in a fraction of a second. section 2.2.1”
Well, yes Robert if “all of the O3 is destroyed by UV “photolysis”(whatever that may be) every few minutes, leading to the formation of free O atoms, — etcetera”, – then it seems reasonable, for me to assume that UV radiation is responsible for the making as well as for the breaking of the “ozone hole”
I am getting a feeling we have started some kind of a “circular discussion” here where we are both saying the same thing, but don’t quite realize it.
However, my main point was not so much about the “make up” of ozone but more about the possibility that as long as oxygen atoms and molecules absorb enough energy from UV radiation to alter their structure it may be that they also produce an increase in their heat content, which should be greater at any points nearest to the source – i.e. The Sun, plus there may be a few other reasons as well as to why the Statosphere warms from the top down but that is way outside of my capabilities to explain. .

Steve
May 20, 2011 7:42 am

Myrrh says: “Because you’re saying that the colder is emitting the same kind of energy to the hotter. So, a cup of hot coffee in the Arctic is surrounded by tons and tons and tons of colder emitting the same energy towards it, the net effect must be for the coffee to get hotter…So you tell me how.”
Same “kind” of energy (electromagnetic radiation), yes. Same quantity of energy per unit area, no.
You are the one proposing that all of the energy emitted by the surrounding Arctic could be concentrated toward the cup and exceed the energy emitted by the cup. How?
If you put the cup in a warmer area (somewhere in Canada), will the cup cool off as fast as it does in the Arctic? What would cause the difference in the rate of cooling?

May 20, 2011 10:54 am

At May 20, 2011 at 12:52 am, Myrrh questions
So, a cup of hot coffee in the Arctic is surrounded by tons and tons and tons of colder emitting the same energy towards it, the net effect must be for the coffee to get hotter?
Consider 2 cups of coffee in the Arctic – one has an insulating container and the other doesn’t. They both start at the same temperature. Record the temperature over 24 hours and find the 24-hour average. I think that we would all agree that the one with an insulator has the higher average temperature. Note that the back radiation from the insulator did not make its coffee warmer than it started, but it does make the over time average temperature higher than not having an insulator.

JAE
May 20, 2011 11:18 am

Steve says:
May 19, 2011 at 11:06 pm
jae says: “The backradiation from the cold air cannot HEAT the surface above the -18 C equivalent provided by the Sun, according to Second Law physics.”
Yup. You can shine 100 100-watt light bulbs on a surface, and you still will not get that surface to emit even 100 watts. And you can combine 100 propane/air torches and never get a flame temperature as hot as a single oxy-acetylene flame. You cannot heat a warmer object with radiation from a colder one. Period. (Note for any nuts that still think I don’t believe in “backradiation:” I did NOT say there is no radiation going from the colder one to the warmer one).
BTW, I think the Kiehl and Trenberth radiation cartoon is generally correct; however it does not explain where the 390 wm-2 radiation comes from, and that is the key point.
And then:
“Wrong. The 2nd Law implies that the radiative energy emitted by the sun and atmosphere combined cannot heat the surface of the earth higher than the temperature of the surface of the sun.”
Wow, that is a real tangled concoction! I guess the second law does ALSO say this, but what is your point?

May 20, 2011 11:20 am

O H Dahlsveen says:
However, my main point [is] that as long as oxygen atoms and molecules absorb enough energy from UV radiation to alter their structure it may be that they also produce an increase in their heat content, which should be greater at any points nearest to the source – i.e. The Sun,
We both agree on that. However, above the stratopause, even though there is still enough UV and oxygen to make ozone, the temperature begins to decrease again. In order to understand the Greenhouse Effect, you must first understand why the atmosphere has the temperature profile observed by measurements. Most people (on both sides of the argument) seem to assume that the atmosphere simply gets cooler with height. However, direct measurements prove this wrong. In my opinion, it is necessary to actually understand why the atmosphere has the current temperature profile before trying to determine what effect, if any, a change in CO2 will have.
Based on my analysis, the tropopause is created where the water vapor decreases to about 5 ppm. Below this point the water vapor spectrum is opaque which is why there is no diurnal temperature cycle in the troposphere.
The stratopause is where water vapor drops from 5 ppm to zero. Above this point, the amount of ozone decreases significantly because water vapor (specifically, the OH radical) is necessary to produce ozone. This line is probably where UV is able to disassociate the OH radical, producing free hydrogen, but I do not have enough data (yet) to support that.

wayne
May 20, 2011 11:30 am

Dave Springer says:
May 19, 2011 at 8:41 am
You seem to have missed the beginning lesson that theory must explain reality. If it fails then the theory is wrong. Reality trumps theory in all cases.
In the particular instance of so-called back radiation the reality is that if you point an IR spectrometer up at the night sky photons of far higher energy than the cosmic microwave background are hitting it. Theory must explain these. In fact theory does explain it. If your understanding of the theory denies reality in this case your understanding is flawed.
In each and every instance where someone denies the reality of back radiation they are wrong. …
——
Dave says: “Reality trumps theory in all cases.”
Oh, I definitely agree. Always. Never have said otherwise.
Dave, I disagree, I’m not trying to mislead anyone but it is you that might be wrong in the end. That is why I took the time to think of a definitive experiment of a Fresnel lens (here) to prove it, one way or the other, whether 324± Wm-2 of downwelling radiation is actually and always flowing from the atmosphere down on the surface, or, is there merely a much smaller portion of radiation always flowing upward that you could focus and heat something with a lens looking downward. If I had the money I would do it right now, just to satisfy my curiosity.
There is also a flop side that may very well mean you are not looking at reality. Don’t you see that possibility. It depends on what that experiment would show. I have taken two days to read about EXACTLY how IR thermometers work. I can’t find even one yet that says “back radiation” is real. I find a few manufacturers that leave you with that feeling but by going deeper and checking their construction they are never extracting energy from colder “back radation”.
You have thermopile or thermocouple versions that focus IR with a lens onto the thermocouple to generate a tiny voltage but in those versions the metal of the thermocouple must be cooler than the temperature they are trying measure. Some have internal thermometers to compensate to raise the accuracy but never greater range.
There are models that visually compare radiation but those are for only high temperatures (electron level radiation) where you make the test filament to match the measured color until it appears to disappear (the same temperature).
In all cases below the emissivities affect the readings so must be compensated for.
There are more modern versions using thermistors but once again the temperature of the device and the measured surface has to be different. That is not detecting “back radiation”. In one case it is detection NO radiation at all (when the temperature of the instrument and test surface match) and just report the room temperature.
There are thermistor versions and I have messed with these in the garage years ago. Still have one from RadioShack. In these cases you are using electric pressure in a semiconductor to separate a “cold” and “hot” side of potential and radiation that is hotter than the cold potential. Now these CAN read temperatures even lower that the ambient temperature that the instrument is at but that is because the electric pressure IS CREATING a cooler environment so that spontaneous emission from the measured surface to the thermistor flows.
Don’t just try to make me look foolish, help me prove it one way or the other but you must look even deeper than the manufacturers claims. If I were them I would not give away such details easily either. That just creates competitors.
If you have detailed schematics and the design of IR spectrometers, let’s see how they work down to where the energy is extracted from the “back radiation”.
“Back radiation” is a term being used that is not merely downward or oppositely-directed radiation. It is always being said to mean counter-flow energy being transferred in an opposite direction than the real radiation if flowing. Dave, if you want me to believe in that kind of counter-flow energy transfer show me a case when it has been used in a device or a real experiment. Prove it. Otherwise, to me, it remains merely a mental and/or mathematical non-real construct that can be used to visualize hard to imagine thoughts or to simplify calculations.
Proof, not just more words.

wayne
May 20, 2011 11:49 am

Dave Springer, one more thing. Don’t just show me “less energy” or “less energy flow”. Both are also not a real things, it is a mathematical visualization or construct of a non-real thing. Energy is a real thing. I know how devices can be manufactured to detect “less energy” but that does not prove back-radiation’s existence.
I’ll also give you a way out. One answer is that you suspect it does exist but mankind to date has never been able to perform an experiment that actually let’s us see “back-radiation”. That may very well be the case.

Tim Folkerts
May 20, 2011 12:55 pm

Wayne,
You are completely mistaken with your expectations for this experiment you propose:

Now, ready? If back-radiation proponents are correct, we should be able to use this huge lens to focus back-radiation from any warm wall within your house, at the ambient room temperature of say 20 ºC (68 ºF), in some room to boil water in seconds.

To understand what is going on, consider first a more familiar situation using a converging lens (either or regular or Fresnel — it doesn’t matter — you could even use a mirror) to heat some object.
With no lens, there is light coming from the hot sun, but it is coming from a very small part of the sky. As a consequence, the object will get slightly warm, not very hot.
To make the object hotter, there are a couple things you could do. 1) you could cover more of the sky with ‘suns’ or with objects as hot as the sun. If the sun was 10 times bigger in diameter, then 100 times as much energy would hit the object and it would get MUCH hotter! OR! You could use a lens or mirror to focus some of the light that was going to miss the object. You could focus light so that 100 times as much of the sky seemed to be covered my the sun (say by putting 100 mirrors around the sides of the object, or by putting a large lens that focuses a lot of light on the the object.)
Note two very important facts. 1) The hottest you could make the object using objects at the temperature of the sun (5700 K) would be 5700 K by surrounding the object by surfaces at 5700 K. 2) If the object WAS surrounded by a heater at 5700 K, then adding lenses or mirrors cannot heat it further (think about the 2nd law and how it is impossible to heat something with a cooler object). A mirror can only reflect light from a 5700 K surface, but it also BLOCKS light from some other part of he 5700 K surface.
NOW … simply mentally replace “sun” by “walls within your house” or by “back radiation from the sky”. No matter how you try to focus the IR energy from the walls, you can’t heat the object above room temperature using the radiation from the walls.
What you COULD do is have most of the surface of the walls at 77 K and a tiny bit at 300 K. An object in such a room would be somewhere between 77 K and 300 K (closer to 77 K). By focusing IR from the small warm part of the wall, you could warm the object up close to 300 K, because the object would “see” 300K from a much larger area. But if the focusing system is simply blocking IR from other warm areas, it is doing no good.
IN CONCLUSION, “back-radiation proponents” can be correct without violating your thought experiment.

Phil.
May 20, 2011 1:34 pm

O H Dahlsveen says:
May 20, 2011 at 4:59 am
The Sun, plus there may be a few other reasons as well as to why the Statosphere warms from the top down but that is way outside of my capabilities to explain.

The stratosphere warms from the top down because it is heated by the sun from above, the main absorber being molecular oxygen, O2, followed by products of its photodissociation, O3. The mostly UV that does the heating is completely absorbed in the stratosphere.

Jim Masterson
May 20, 2011 1:53 pm

>>
Phil. says:
May 19, 2011 at 8:19 pm
Middle if you’re using frequency.
<<
Before I became a “kook’, there used to be a one-to-one correspondence between frequency and wavelength. Silly me, I guess climate science does it differently.
>>
You should use the frequency form of Wien’s Law not wavelength, we want to know where the bulk of the energy is emitted. The peak energy at ~20THz (i.e.~15micron wavelength) is ~340K.
<<
Same comment.
>>
The average planetary temperature is supposed to be far warmer so CO₂ doesn’t provide much blocking in general. In fact, part of the argument is that CO₂ by itself doesn’t do much at all, but in conjunction with water vapor we are supposed to have a serious problem. Again, more arguments ensued on this point. This is usually called the Enhanced Greenhouse Effect. Argue, argue, argue.
Predicated on a false premise, not correct.
<<
Is it false because CO₂ can do it by itself or false because the EGE concept is wrong? I’d say that latter is false, which is one thing we can agree on. Of course CO₂ still has trouble causing much GHE.
Jim

Steve
May 20, 2011 2:00 pm

jae says: “You can shine 100 100-watt light bulbs on a surface, and you still will not get that surface to emit even 100 watts.”
Surely you do not mean that you can direct 10,000 watts in a given unit time at a surface and it will never emit 100 watts in the same unit time? It will absorb 9,900 watts of this radiation forever, yet never heat up? What happens to the energy?
“I did NOT say there is no radiation going from the colder one to the warmer one.”
So is the energy of the radiation absorbed, reflected or transmitted?
“BTW, I think the Kiehl and Trenberth radiation cartoon is generally correct; however it does not explain where the 390 wm-2 radiation comes from, and that is the key point.”
It explains it fairly simply, as described in the part of my comment you decided not to respond to. Energy emitted by the earth’s surface equals energy it absorbed from direct sunlight plus energy it absorbed from the atmosphere (also emitting light, e.g. infrared).
“Wow, that is a real tangled concoction! I guess the second law does ALSO say this, but what is your point?”
The 2nd law says nothing about whether or not a surface emitting 340 W/m^2 can absorb another incoming 50 W/m^2, or that if sources of both 340 W/m^2 and 50 W/m^2 are present that the surface can only absorb the energy of the 340 W/m^2 source.

Myrrh
May 20, 2011 6:04 pm

Robert and Steve – you’re teasing me, right? I give you an example of loads of cold atmosphere plus tons of ice of fungible cooler all radiating “the same energy” claimed in the ‘heat is net in the exchange of energy’, and I still can’t get a small cup of hot coffee without insulating it which will only slow down its cooling or by moving to Canada. Yet, I’m being told constantly that this colder atmosphere radiates energy which adds to the heat of the Earth and only needs a tiny tiny extra bit more of CO2 and the whole Earth’s temperature will go up several degrees and this will lead to runaway global warming, because in this is a net exchange of energy which includes from the colder to the hotter.
Which means, because there is a far greater amount of energy being radiated towards my cup of coffee from the cold, then the coffee radiating hot to cold and losing energy is pretty much irrelevant, it should get hotter.

jae
May 20, 2011 6:30 pm

Steve:
Do you have a degree in any way related to science?

Tim Folkerts
May 20, 2011 6:34 pm

Jim Masterson says:
May 20, 2011 at 1:53 pm
>Before I became a “kook’, there used to be a one-to-one correspondence between
>frequency and wavelength. Silly me, I guess climate science does it differently.
Apparently have not had to change variables in calculus lately. The peak in the two curves will indeed be different.
Read the section “Properties of the Planck distribution” at http://en.wikipedia.org/wiki/Planck%27s_law
It is only the silly kooks who do it differently. Climate scientists apparently do it the way ALL scientists and mathematicians do it.
PS, not to pick on Jim specifically, but this is perhaps my greatest frustration when interacting here — people who took maybe a year of freshman physics spending 2 weeks on thermodynamics and 2 week on optics, but think they are qualified to correct career scientists on the second law or how lenses could focus IR light or how Plank’s law is different when written in terms of frequency rather that wavelength.
It’s like someone who played a little HS football thinking he could coach in the NFL.
It’s like someone who plays video games thinking they could really fly a plane.
It’s like someone who took a first aid course thinking they are a doctor.
Sure, once or twice per game, a fan might make a better call than the coach, but day-in and day-out, my money is on the professional. It takes a pretty big ego to think that freshman-level musings are somehow new or unknown to practicing scientists and teachers.

jae
May 20, 2011 6:49 pm

Steve: before you respond, please read the article linked below. Truly read it, not just glance at it. Then tell us what you think:
http://www.tech-know.eu/uploads/Greenhouse_Effect_on_the_Moon.pdf
The “hide the decline” secret in the K&T cartoon is that it does not explain just where the hell the 390 wm-2 at the surface comes from. It starts out just ASSUMING this radiation!

May 20, 2011 7:17 pm

Wayne,
I read your post about IR thermometers and ran an experiment of my own. Quite a simple one actually:
I took a block of ice put it in a 6 day rated cooler in the shade, allowed 2 hours for temperatures to equilibrate.
I took my IR thermometer, a wet bulb thermometer which I know was accurate to 1 deg F as of last calibration and a thermocouple temperature probe. I checked the probe against the thermometer and they both agreed, within the reading accuracy of the thermometer. I then took my IR thermometer a fairly cheap one, and measured the temperature of the block of ice a number of times from a few inches away to about almost 3 feet away. The IR thermometer read the same temperature as both the thermometer and thermocouple probe (1C) within 0.2C. Ambient air temperature was 18C and the thermocouple probe suggested the body of the IR thermometer varied in temperature from ambient to around 23C where my hand had been holding it.
This website states specifically:
“How does an infrared thermometer work?
The most basic design consists of a lens to focus the infrared (IR) energy on to a detector, which converts the energy to an electrical signal that can be displayed in units of temperature after being compensated for ambient temperature variation.”
http://www.omega.ca/prodinfo/infraredthermometer.html
Obviously the way the sensor reacts to the incoming IR radiation varies with temperature. This is normal for sensors which detect light, think of a digital camera, there is less noise the colder the ccd. So in order to measure temperature the IR thermometer needs to compensate for ambient temperature. Just like a UT thickness meter is calibrated empirically to the object being measured.
So, if energy from a colder object cannot transfer to a warmer object, please explain how the IR thermometer measured the temperature of the block of ice, especially in light of what the manufacturer Omega states.

wayne
May 20, 2011 7:21 pm

I must now apologize to Dr. Miskolczi, I just found this in his last paper Greenhouse effect in semi-transparent planetary atmospheres stating:
“For atmospheres, where ED≈ 5ST or TA ≈ 1/6, Eq. (9) will take the form
of Eq. (8).”
That is exactly what I portrayed above as a six-sided cube with three bi-degrees of freedoms and I thought I had found something rather new. Not so it seems. Put any credit to him and rightfully so. It does shore up my current view of energy flow through atmospheres though.

IAmDigitap
May 20, 2011 8:13 pm

With all the claims of “having money on the professionals” the people who are altering the instrumental records with claims of fake math being real math, claims of need for magical washings of data, the fact is, nearly ALL the people claiming the world is warming are N.O.T. professional radiation people.
This is easily proven. The REAL radiation people in the world are those who unleash electromagnetic radiation in E.V.E.R.Y. frequency into the atmosphere, then capture it, and analyze it.
There IS no word of any G.H.G. from we who DO this, because WE don’t FIND the FALSELY CLAIMED ATMOSPHERIC RADIATION changes.
The people who are making claims of G.H.G. effect are N.O.T. working atmospheric radiation people who have to be right or be fired.
That’s obvious from the fact they think there’s no way to accurately tell the temperature of the atmosphere. It’s possible, it’s just that there’s nothing unusual there, so they’re making up FAKE math, FAKE readings, FAKE EFFECTS of a form of PHYSICS they weren’t prepared to deal in when they thought they reversed the polarity of gravity by claiming that PHOTONS dragging atoms UP, were radiating DOWNWARD.
The problem with this ludicrous claim is that in first place there’s only a T.I.N.Y. amount of carbon dioxide. We live in a proportionate physics universe so there’s no such thing as magical gas.
“Oh, yes there IS magic gas!”
If there was, one of them would have produced an instrument that recorded their mythical magic force from a mystical dimension. We even put buttons and knobs on them so they can use them.
They still can’t. It’s not the fault of the people who built those instruments, it’s AMATEUR ERROR by people who thought Mannian Statistics was real math, and Jones/Briffa extensions to that FAKE DOODLING was real math.
Go ahead: try to find an instruments engineers’ or technicians’ group that endorses A.G.W.
Try to find O.N.E. report of electromagnetic radiation communications people claiming altered electromagnetic response to the atmosphere due to G.H.G. theory.
There is none.
Try to find O.N.E. report of rising heat in the atmosphere creating more ATMOSPHERIC SCINTILLATION, or MOTION of GAS MOLECLES which is MANDATORY in GAS: creating INCREASED DISTORTION as people LOOK THROUGH the ATMOSPHERE.
T.h.e.r.e. is N.O.N.E. because HEAT on GAS means MOTION and MOTION means OPTICAL DISTORTION and NO ADDED DISTORTION means N.O. H.E.A.T.
No matter HOW many of these “I thought it was real math” people try to tell you they think they calculated entropy revoked itself.
People telling you they believe in G.H.G. Theory are ALL TRYING to TELL you a
HIGHLY TURBULENT,
WELL MIXED
F.R.I.G.I.D,
-15 degree
OCEAN
of GAS,
functions as a WARMING BLANKET.
That’s the class intellect you’re dealing with. Mannian Statistics is real math, HEAT FLOWS from a -15 BATH into the WARM ROCK submerged in it, and although the ATMOSPHERE is EVER WARMER and MORE TURBULENT with HEAT,
nobody’s noticed to the point where an instrument can measure it. Not even a telescope lens which MAGNIFIES ATMOSPHERIC DISTORTION THOUSANDS of times.
No, and – not even ‘nice try.’
People were told YEARS ago this would never fly, and here you have them on the internet telling you a TURBULENT FRIGID BATH is WARMING a rock.
Find anyone but a climatologist who’ll tell you to WARM a ROCK with FIFTEEN DEGREE BELOW ZERO GAS.
He’ll laugh in your face.
All these claims of untestability, are just that: fake claims. Like I said: the DEFINITION of HEAT on an ATOM of GAS is M.O.T.I.O.N: ANGULAR MOMENTUM.
There IS no such thing as ‘hiding heat’ in an atmosphere being peered through with optical magnification that makes the slightest heat distortion noticeable.

Steve
May 20, 2011 8:50 pm

jae says: “Do you have a degree in any way related to science?”
B.S. in Biochemistry, University of California San Diego, 1994
http://i56.tinypic.com/2126104.jpg
“…please read the article linked below. Truly read it, not just glance at it. Then tell us what you think:”
I read it last week when you first posted the link. The article points out that a blackbody model for the moon that does not take into account the specific heat capacity of the moon generates a certain amount of error in predicting the maximum and minimum temperatures. Unfortunately the article doesn’t then explain that, once the heat capacity is known, the correct heat capacity can be plugged back into the models to generate the correct temperatures. It wasn’t a surprising problem for the Apollo mission, considering they hadn’t been to the moon yet. We know the heat capacity of the surface of the earth because we’ve been here for a while.
“The “hide the decline” secret in the K&T cartoon is that it does not explain just where the hell the 390 wm-2 at the surface comes from. It starts out just ASSUMING this radiation!”
It’s assuming an average temperature, from the temperature record. Which could be wrong. That doesn’t negate the ability of CO2 to absorb more IR radiation than N2. There is no getting around a greenhouse effect. Ira’s post seeks to quell this issue so that we can get to the matter of quantifying the response of the climate for a given increase in CO2.

wayne
May 20, 2011 9:43 pm

Alleyne said:
“So, if energy from a colder object cannot transfer to a warmer object, please explain how the IR thermometer measured the temperature of the block of ice, especially in light of what the manufacturer Omega states.”
Alleyne honestly, don’t know, only guessing. The IR thermometer and the ice are different objects so this doesn’t get into the self-warming bit. The net energy would be from the device to the ice, I agree. How does it sense the differential, or, block the devices local thermal noise at the ambient temperature? Hmmm. Do you have schematics listing the devices to see how they are performing that? Seems on the top to be a type of differential circuit but I could only guess and the chances of being right are small. I would like to know too. Being a customer maybe you could get the details and share here.
Wait, of course, if it is a differential balance circuit it may swing right from positive through zero at the ambient device temperature to negative where it is then really measuring the net energy lost from the warmer thermocouple to the ice in the field being focused. That might be it but just don’t know.
My whole point is to look at each instance in detail before you convince yourself that something is reading or manipulating energy that is flowing backwards from a cold object to a warmer object, it is most likely taking the difference from an already energized point. That’s all. Really need an electronic engineer!
I’m tired of speaking of this, seems no one will get anything if they haven’t already.

wayne
May 20, 2011 9:54 pm

Alleyne, can’t help it, one more thing. With the two things I know of that device a thermocouple and a lens, if that type of circuit is being used it should take you longer to get a stable reading on a much colder that ambient temperature object than a much warmer than ambient temperature object. Is that so? It has to do with the lens.

Myrrh
May 21, 2011 1:50 am

Steve says:
May 20, 2011 at 2.00 pm
Re jae’s: “BTW, I think the Kiehl and Trenberth radiation cartoon is generally correct; however it does not explain where the 390 wm-2 radiation comes from, and that is the key point.”
It explains it fairly simply, as described in the part of my comment you decided not to respond to. Energy emitted by the earth’s surface equals energy it absorbed from direct sunlight plus energy it absorbed from the atmosphere (also emitting light, e.g. infrared.).
The real missing energy is the actual real amount of Thermal IR directly hitting the Earth and warming land and ocean.
Firstly, the cartoon energy budget excludes the real heat we feel from the Sun which comes only from the real Thermal IR arriving on Earth and which we feel as real heat, and which heats our innards as it does the land and sea.
Secondly, the Solar energy which the cartoon claims converts to heat to produce this outgoing Thermal IR is composed of UV, Visible and Near IR, none of which are actual thermal IR, heat on the move, we cannot feel them, and there is no attempt made to prove that these energies convert to enough heat of land and sea to raise the temperature of the Earth.
Moreover, the claim now as Ira consistently teaches, is that it is these “Solar” energies of the cartoon which we feel as heat. This is nonsense, a complete perversion of traditional science understanding of the difference between Light and Heat energies.
See as before the NASA page I gave and also any that still give traditional science and not AGW/Ira’s twisting of it. The heat we feel from the Sun is Thermal IR which is not included in the cartoon’s downwelling energies. We can feel this heat, we know it exists, we know it makes things hotter, yet, the AGWScience says that it does not reach Earth in any significant amount and excludes it.
So, the missing energy begins because the real amount of Thermal IR reaching Earth and which is what we feel as heat from the Sun and which is the only energy capable of actual directly heating the Earth has been excluded and in its place has been put Light and short wave energies either side, called Solar, which barely convert to any heat at all, even UV is not enough to heat the Earth to produce outgoing of so much Thermal IR.
How much Thermal IR is REALLY getting to Earth compared with the other energies? Because these “Solar” energies are incapable of raising the temperature of the Earth to any significant amount means that the greatest amount of energy capable of heating the land and sea comes directly from the Thermal IR downwelling from the Sun.
Until this AGW magic illusion is seen through, it will continue to confuse energies and confuse the two aspects needed to understand the outgoing Thermal IR of the cartoon.
The creation directly of Thermal IR heating to land and sea in raising the temperature of the Earth which then radiates out x amount of Thermal IR, and, the creation of heat from Life itself as it uses these different energies, including the Solar of the cartoon, to convert to chemical and mechanical uses which enable Life to flourish and in turn radiate out Thermal IR.
The missing energy is from excluding downwelling Thermal IR, the real heat we feel from the Sun.
The lie that Ira promotes is that it is the Solar energies we feel as heat from the Sun.
The rest is the complete garbling of science which AGW produces in its continually more tortured reasoning because its basic premises are always garbage in.

May 21, 2011 6:05 am

wayne said on Visualizing the “Greenhouse Effect” – Light and Heat
May 20, 2011 at 9:54 pm
“Alleyne, can’t help it, one more thing. With the two things I know of that device a thermocouple and a lens, if that type of circuit is being used it should take you longer to get a stable reading on a much colder that ambient temperature object than a much warmer than ambient temperature object. Is that so? It has to do with the lens.”
Alleyne replies:
OK, I have no problem asking them, I can’t guarantee I’ll get an answer, but it is no big deal to ask.  I’ll post if I get an answer.  I too would like to know.
As to the acquisition time, I can’t comment other than to say my impression is that the acquisition time was not noticeably different than that for hot objects I’ve measured.  It is so fast that I would need very fancy equipment to measure it as it could probably only be done electronically measuring circuit/processor response times.  However I’ll ask that question as well.
I don’t expect they will answer till after the week end though.
It would be interesting to know!

May 21, 2011 6:27 am

Myrrh said on Visualizing the “Greenhouse Effect” – Light and Heat
May 20, 2011 at 6:04 pm
“…… and I still can’t get a small cup of hot coffee without insulating it which will only slow down its cooling or by moving to Canada.
Which means, because there is a far greater amount of energy being radiated towards my cup of coffee from the cold, then the coffee radiating hot to cold and losing energy is pretty much irrelevant, it should get hotter.”
That’s because we have Tim Horton’s up here, which is why we have hot coffee in all sizes all year around 😉
I hesitate to jump into this, but perhaps the sticking point, which I noticed previously in the post with the 10 100W heaters is that it is a function of the temperature gradient, not power.
In the case of the coffee and the heaters, the cold or the heat isn’t really additive as far as energy transfer is concerned is it?  The 10 heaters each put out 100W and total power consumption is 1,000W, but in terms of the energy differential or gradient it is still only 100W.  With your cup of coffee, it doesn’t matter if you are in the Arctic or just have a snowbank, the temperature difference between the coffee and the snow is the same (yes of course the mass of snow will determine the final temperature of each, but the heat transfer is a function of the difference in temperatures not the mass)
At least that is the way I envision it at present.

Robert Stevenson
May 21, 2011 9:21 am

CO2 is irrelevant to atmospheric warming – after the first 120m of traverse of LWIR through the atmosphere water vapour absorbs the IR to extinction and there is nothing left for the CO2 bands to absorb. Spending trillions of $ on CO2 emission reductions is absolutely ludicrous. As far as climate change goes we are heading for the next ice age.

Steve
May 21, 2011 9:27 am

Myrrh says: “Yet, I’m being told constantly that this colder atmosphere radiates energy which adds to the heat of the Earth…”
Absolutely not. You are being told that the atmosphere radiates energy which reduces the loss of heat from the Earth. A surface beneath a 17 degree atmosphere will cool slower than a surface beneath a 15 degree atmosphere. That surface, having cooled less throughout the night, is then exposed to the sun (again) the next day. It hasn’t cooled off as much, so the same energy input from the previous day (solar) is now shining down on a warmer surface that didn’t cool as much the previous night. Same energy input into warmer surface = heating to higher temperature the next day.
And no, you don’t need the day/night cycle to prove the GHE (the earth’s surface is emitting/absorbing energy every second of the day), but I think a day/night cycle description makes the GHE more obvious.
“Which means, because there is a far greater amount of energy being radiated towards my cup of coffee from the cold… ”
You keep stating as fact that the surroundings are radiating more energy toward your cup than the cup is radiating out. Again, I ask you “How?” Tons of mass does not equate to more radiant energy than a cup of coffee. A billion tons of mass at absolute zero would radiate no electromagnetic energy.
“…then the coffee radiating hot to cold and losing energy is pretty much irrelevant, it should get hotter.”
If it is losing more energy than it is gaining it will cool, so it is quite relevant.

Phil.
May 21, 2011 9:29 am

Jim Masterson says:
May 20, 2011 at 1:53 pm
>>
Phil. says:
May 19, 2011 at 8:19 pm
Middle if you’re using frequency.
<<
Before I became a “kook’, there used to be a one-to-one correspondence between frequency and wavelength. Silly me, I guess climate science does it differently.

Science does indeed do it differently, there is a reciprocal relationship between frequency and wavelength, not a linear one!

You should use the frequency form of Wien’s Law not wavelength, we want to know where the bulk of the energy is emitted. The peak energy at ~20THz (i.e.~15micron wavelength) is ~340K.
<<
Same comment.

And still wrong!

>>
The average planetary temperature is supposed to be far warmer so CO₂ doesn’t provide much blocking in general. In fact, part of the argument is that CO₂ by itself doesn’t do much at all, but in conjunction with water vapor we are supposed to have a serious problem. Again, more arguments ensued on this point. This is usually called the Enhanced Greenhouse Effect. Argue, argue, argue.
Predicated on a false premise, not correct.
<<
Is it false because CO₂ can do it by itself or false because the EGE concept is wrong? I’d say that latter is false, which is one thing we can agree on. Of course CO₂ still has trouble causing much GHE.

False because you’re wrong about where in the energy spectrum CO2 absorbs.

Myrrh
May 21, 2011 10:18 am

Allyene – not in ‘heat is the net transfer of energy from hot to cold and cold to hot’ , and, statistically speaking or not, see Joel’s post re amount. Without giving a mechanism to stop it then statistically cold can takeover being the dominant source of heat if there’s enough of it. My cup of coffee in the Arctic should boil away.

Robert Stevenson
May 21, 2011 10:25 am

Dave Springer says:
This was disproven by expermental physicist John Tyndall over 150 years ago. Back radiation from gases that absorb IR is quite real. Suggest you read the original work here (it’s free):
http://www.archive.org/details/heatconsideredas00tyndrich
Al Gore is always quoting Tyndall’s work in 1860 as the ‘final word’, the science is settled end of discussion its all been nailed by the great Irish bogtrotter.

jae
May 21, 2011 10:28 am

Folkerts:
“It’s like someone who played a little HS football thinking he could coach in the NFL.
It’s like someone who plays video games thinking they could really fly a plane.
It’s like someone who took a first aid course thinking they are a doctor.
Sure, once or twice per game, a fan might make a better call than the coach, but day-in and day-out, my money is on the professional. It takes a pretty big ego to think that freshman-level musings are somehow new or unknown to practicing scientists and teachers.”
LOL. And just HOW are you going to reconcile these thoughts with the FACT that the whole CAGW “theory” has now been exposed as a complete scam?
I think most truck drivers probably have a better nose for reality than most climate scientists and politicians.

Myrrh
May 21, 2011 11:06 am

I think IAmDigitap has summed up the problem neatly, (/#comment-664758 May20 8:13 pm). No one who does and understands what it is they does in the applied sciences can believe any of the reasoning from the AGWScience as to how things work.
Perhaps it’s a problem of theoretical v applied, or rather, the particular theoretical we have now who think they can change the 2nd Law to fit their musings or call Light energies thermal and keep a straight face, while of course still letting the practical bods do the actual adjustments necessary, such as in the proliferation of ‘ideal’ laws, to make things work, because they know why things work.
An amusing example here: http://www.phschool.com/science/science_news/articles/hot_crystal.html

Not everyone believes the Sandia results. “I do not see how they could possibly exceed the black body limit,” comments John B. Pendry, a theoretical physicist at the Imperial College of London. He suggests that Lin and his colleagues may have erred when processing their measurements.
Some other scientists accept the work as valid, however. “The numbers are believable,” remarks Jonathan P. Dowling of NASA’s Jet Propulsion Laboratory in Pasadena, Calif. He’s now applying the same approach to solar cells. Adds Dowling, “I think highly efficient lightbulbs and solar cells far more efficient than anybody could have imagined, are a real possibility now.”

Right or wrong will come out in the testing.

jae
May 21, 2011 12:49 pm

Ah, Steve…
“I read it last week when you first posted the link. The article points out that a blackbody model for the moon that does not take into account the specific heat capacity of the moon generates a certain amount of error in predicting the maximum and minimum temperatures. Unfortunately the article doesn’t then explain that, once the heat capacity is known, the correct heat capacity can be plugged back into the models to generate the correct temperatures. It wasn’t a surprising problem for the Apollo mission, considering they hadn’t been to the moon yet. We know the heat capacity of the surface of the earth because we’ve been here for a while.”
So, it looks like you read the part about the moon. Now keep on reading for the most important parts of the article. The widely cited “blackbody temperature of Earth,” -18 C is the temperature that a satellite would “see.” It is the radiation at about 5 km above the surface–the altitude where more radiation goes up than goes down. BUT, because of the lapse rate, that means that the surface is much warmer. In fact it’s about 33 C warmer, a warmth that is attributed to a greenhouse effect. However, the laplse rate has ABSOLUTELY NOTHING to do with greenhouse effects, being dependent upon only the heat capacity of the air (Cp) and gravity (g).
Thus, one does not need any greenhouse effect to explain the warmer surface (and the 390 watts shown in the K&T cartoon).
The linked article shows that this same effect occurs on all planetoids with atmospheres, irregardless of how concentrated the greenhouse gases are.
It appears that the greenhouse effect in the atmosphere does not work for the same reason a greenhouse doesn’t work when all the windows are open–convection.
Then you state:
“It’s assuming an average temperature, from the temperature record. Which could be wrong. That doesn’t negate the ability of CO2 to absorb more IR radiation than N2. There is no getting around a greenhouse effect. Ira’s post seeks to quell this issue so that we can get to the matter of quantifying the response of the climate for a given increase in CO2.”
No, the diagram is trying to show how the greenhouse effect adds the 33 using radiation. What is ironic, to the point of being weird, is that the diagram is probably quite correct; the surface is at 15 C, the upward radiation is 390 wm-2, the backradiation is as shown, etc. But as I said above, it has to START with the 390 wm-2, and there is no way that can be somehow be “created” with backradiation.
So we will never get to the matter of quantifying the response (temperaturewise) of the climate to CO2, since there ain’t one. Which the real world data (surface temperature, SST, radiosonde, etc.) has now shown, anyway.

Steve
May 21, 2011 3:47 pm

jae says: “The widely cited “blackbody temperature of Earth,” -18 C is the temperature that a satellite would “see.” It is the radiation at about 5 km above the surface–the altitude where more radiation goes up than goes down. BUT, because of the lapse rate, that means that the surface is much warmer. In fact it’s about 33 C warmer, a warmth that is attributed to a greenhouse effect. However, the laplse rate has ABSOLUTELY NOTHING to do with greenhouse effects, being dependent upon only the heat capacity of the air (Cp) and gravity (g).”
You are reading an explanation into the article that isn’t in the article. The word “lapse rate” doesn’t appear once. The article attributes a lack of accounting for the heat capacity of the mass of the atmosphere to the higher than expected temperatures. You are confusing the explanation in this article with the one given in the “No Greenhouse Effect on Venus” article, which you also cited.
No matter, the lapse rate does not negate the ability of CO2 to absorb more infrared radiation than N2. Assuming the lapse rate stays the same, exchanging CO2 for N2 will raise that 5 km altitude higher (as the temperature at the surface goes up). Removing all CO2 from the atmosphere will bring that 5 km altitude lower (as the temperature of the surface goes down). The question should be “by how many meters per unit change in CO2?”.

May 21, 2011 4:34 pm

jae, your statement that the
lapse rate has ABSOLUTELY NOTHING to do with greenhouse effects, being dependent upon only the heat capacity of the air (Cp) and gravity (g).
is absolutely wrong. The “dry adiabatic lapse rate” is controlled by the “the heat capacity of the air (Cp) and gravity (g)“, but the actual measured lapse rate is controlled by greenhouse emissions (among other things). Without greenhouse gases, the atmosphere would actually have the same temperature at all altitudes. It is the greenhouse gases that cause the lower atmosphere to cool. It is also the greenhouse gases that help produce the morning temperature inversions where the atmosphere is much warmer than the surface.

jae
May 21, 2011 5:17 pm

Steve continues:
“You are reading an explanation into the article that isn’t in the article. The word “lapse rate” doesn’t appear once. The article attributes a lack of accounting for the heat capacity of the mass of the atmosphere to the higher than expected temperatures. You are confusing the explanation in this article with the one given in the “No Greenhouse Effect on Venus” article, which you also cited.”
It’s actually the “same difference,” since the lapse rate depends on heat capacity and gravity (lapse rate = g/Cp). And I note that you are not refuting either article, just restating the “consensus ho-hum.” Hmmm, is that because you cannot?
And you state:
“No matter, the lapse rate does not negate the ability of CO2 to absorb more infrared radiation than N2. Assuming the lapse rate stays the same, exchanging CO2 for N2 will raise that 5 km altitude higher (as the temperature at the surface goes up). Removing all CO2 from the atmosphere will bring that 5 km altitude lower (as the temperature of the surface goes down). The question should be “by how many meters per unit change in CO2?”.
Yeah, I know the theory, since I hear it from even many of the “skeptics.” Problem is, there is absolutely no empirical support for it, so it is dying a natural death. The effects you mention, as well as many others, are just not being observed. No hot spot in the upper tropical atmosphere, as all models predict. Not even a rise in surface temperature or sea surface temperature over the last 10-15 years. To the contrary, it seems likely we will see a notable cooling for the next few years, according to many climate scientists. Ice core records show that increases in CO2 LAG warming by about 800 years. If the GHE from OCO exists, it must be a very weak force, indeed!
FWIW, I doubt that I will ever win this battle, but I’ll wage it until someone shows me wrong with EMPIRICAL EVIDENCE. Radiation cartoons and climate models don’t count.

wayne
May 21, 2011 5:31 pm

Ira, bless you, for you seem to have learned not one single thing in these some 800 comment. Here you are speaking again of ½ up and ½ down and that is only correct in a one dimensional world.
We live in a three dimensional world. I had to apologize to Dr. Miskolczi for stepping on his toes on that very point. the “light-bulb” that went off in my head, he had already covered that in his last paper.
It is ideally 1/6th up and 1/6th down and you have to take the components of the 3d vectors, six possible ways to move randomly in a 3d world, viewed best as degrees of freedoms. Much the same as seen when visualizing how gases create pressure with tree degrees of freedom. See the ‘3’ in the pressure equations.
That also means that 4/6th or 2/3rd of a photon random movement will always be horizontal or tangent to the surface, and, since any atmosphere is reasonably viewed as a homogeneous horizontally, these components do absolutely nothing except maintain a constant temperature. That horizontal component of any photons random emission plus the 1/6th downward component or when totaled is 5/6th and this IS THE GREENHOUSE EFFECT. It is geometric in nature and has nothing to do with radiation except that radiation and conduction transfer energy (see: equipartition across possible states).
A photon, better just a certain amount of energy, that just jumps around in the atmosphere does absolutely but to maintain equilibrium, radiative, temperature, pressure and density.
I keep saying please, but will say it again, please, even if you don’t like certain points in his papers, read them. The atmospheric physics in the equations are correct for any atmosphere, those part of his papers are not contested. Enlighten yourself before this thread ends and please retract that mumble-jumble you just fed to Robert Stevenson.

jae
May 21, 2011 8:43 pm

Wayne says:
“Ira, bless you, for you seem to have learned not one single thing in these some 800 comment. ”
Oh, Wayne, how can you possibly question the SELF-APPOINTED EXPERT WHO IS THE HOST? LOL! You need to keep in mind that the host has a PhD, which is loudly proclaimed and which must mean something….

Steve
May 22, 2011 2:09 am

jae says: “It’s actually the “same difference,” since the lapse rate depends on heat capacity and gravity (lapse rate = g/Cp).”
The “Greenhouse Gas on the Moon” article proposes that the blackbody models that explain the GHE on the earth don’t correctly account for heat capacity at all, either rocks, oceans or atmosphere. That’s why the exposition begins with the NASA error on the moon (with no appreciable atmosphere). The article disputes using blackbody modeling, in total, for calculating planetary surface temperatures.
“And I note that you are not refuting either article, just restating the “consensus ho-hum.” Hmmm, is that because you cannot?”
I stated that the NASA error was easily adjusted for once the correct heat capacity was known. Blackbody modeling can be adjusted. That directly refutes the main premise of the article. I did refute the article, you didn’t notice.
I also refuted the notion that the lapse rate “explains” the temperature at the surface. No, the lapse rate can be used to calculate the temperature at the surface after the altitude and temperature of the tropopause is known. The lapse rate does not explain why the tropopause is at a specific altitude. It is the ability of the atmosphere to absorb and emit energy that explains the temperature of the atmosphere.
“Problem is, there is absolutely no empirical support for it, so it is dying a natural death.”
Gases like H2O and N2 absorb more infrared than N2. A 15 degree surface beneath a 9 degree gas will cool slower than a 15 degree surface beneath a 7 degree gas. This is empirical support for the GHE that cannot be refuted by hand waving about bad models getting this or that wrong about climate predictions.
What can be refuted is that (shocker) the models are correct about a given change for a given increase in CO2. But there will be a change, and it will be due to more energy within the earth atmosphere than before. Energy can be expressed as heat, but it can also be expressed as work.
“Not even a rise in surface temperature or sea surface temperature over the last 10-15 years. To the contrary, it seems likely we will see a notable cooling for the next few years, according to many climate scientists.”
We see cooling every night of every day, every winter of every season. The GHE does not trump solar forcing, it supplements it.
“Ice core records show that increases in CO2 LAG warming by about 800 years.”
Ice age cycles are due to solar forcing cycles, no doubt. It is to be expected that as the oceans warm the CO2 content of the atmosphere increases.
“If the GHE from OCO exists, it must be a very weak force, indeed!”
At present concentrations, I am in the “weak force” camp. Well, weak compared to the standard models. I believe that most of the 20th century warming was due to natural, solar variations that are not entirely understood.

R Stevenson
May 22, 2011 6:23 am

As a so called greenhouse gas, CO2 pales into insignificance when compared with water vapour. To be as effective as water vapour in absorbing land LWIR the concentration of CO2 would need to increase 900 times ie from its present 380ppm to 42000ppm. Yet governments in the EU, US, Canada etc insist on spending huge sums to reduce CO2 emissions and in the process closing down a large part of our almost wholly CO2-dependent economies. This is a ludicrous and futile waste of resources.

R Stevenson
May 22, 2011 6:50 am

Steve says:
“If the GHE from OCO exists, it must be a very weak force, indeed!”
At present concentrations, I am in the “weak force” camp. Well, weak compared to the standard models. I believe that most of the 20th century warming was due to natural, solar variations that are not entirely understood.
This is entirely true Steve and I would add that as a so called greenhouse gas, CO2 is very weak and pales into insignificance when compared with water vapour. To be as effective as water vapour in absorbing land LWIR the concentration of CO2 would need to increase 900 times ie from its present 380ppm to 42000ppm. Yet governments in the EU, US, Canada etc insist on spending huge sums to reduce CO2 emissions and in the process closing down a large part of our almost wholly CO2-dependent economies. This is a ludicrous and futile waste of resources.

Phil.
May 22, 2011 7:25 am

R Stevenson says:
May 22, 2011 at 6:23 am
As a so called greenhouse gas, CO2 pales into insignificance when compared with water vapour. To be as effective as water vapour in absorbing land LWIR the concentration of CO2 would need to increase 900 times ie from its present 380ppm to 42000ppm.

Not true, what do you base that on?

R Stevenson
May 22, 2011 7:43 am

jae says:
May 21, 2011 at 8:43 pm
Wayne says:
“Ira, bless you, for you seem to have learned not one single thing in these some 800 comment. ”
Oh, Wayne, how can you possibly question the SELF-APPOINTED EXPERT WHO IS THE HOST? LOL! You need to keep in mind that the host has a PhD, which is loudly proclaimed and which must mean something….
Patronise as much as you like but Ira talks a lot more sense than either of you two.

R Stevenson
May 22, 2011 8:31 am

Heat is transferred by conduction, convection and radiation and heat losses are often computed using a combined convection and radiation coefficient hc + hr. Heat loss:
Q = (hc+hr)*A*deltaT
Heat from the land surface is transferred by conduction through a boundary layer and also by radiation to the immediate atmosphere. Considerable heat is then transferred by natural convection from the lower reaches to the upper reaches clearly radiating in three dimensions as well before finally leaving for space. The transfer of heat from the surface is not just dependent on the mechanism of radiation and re-radiation to cool the heated thin (120m) boundary layer as so many contributors have pointed out.

R Stevenson
May 22, 2011 9:07 am

Phil says:
R Stevenson says:
May 22, 2011 at 6:23 am
‘As a so called greenhouse gas, CO2 pales into insignificance when compared with water vapour. To be as effective as water vapour in absorbing land LWIR the concentration of CO2 would need to increase 900 times ie from its present 380ppm to 42000ppm.’
Not true, what do you base that on?
This is based on the increase in CO2 concentration needed to make the gas as effective as water vapour in absorbing LWIR ie based on equivalence:
For a product term PwL of 0.0231*(4*3.281) ft.atm a water vapour absorptivity of o.2 absorbs 84Wm-2 of land LWIR in 4m.
The equivalent for CO2 requires a product term PcL of 0.0004*900*(4*3.281) ft.atm giving a CO2 absorptivity of 0.2 absorbing 84Wm-2 of land LWIR in 4m.
NB a partial pressure of o.ooo4*900 atm is 380ppm *900.

R Stevenson
May 22, 2011 10:17 am

Phil says:
‘ False because you’re wrong about where in the energy spectrum CO2 absorbs.’
It is apparent from whatyou say that you are not familiar with ‘Wien’s law of displacement’ judging by your comment that the spectral band from 12 to 18 microns includes the peak emission of the Earth’s emission spectrum.
Recourse to Wien’s law shows that a wavelength of maximal emission λ_max of 15μm yields a temperature of minus 80C; whilst 15C gives a maximal emission λ_max of 10μm.
CO2 absorbs infrared emissions from the Earth’s surface only minimally in the range 7 to 13μm and it is within this range where the greatest proportion of radiation emitted by the Earth is found. This range is called ‘open radiation window’, because it is here that the least amount of absorption by water vapour and CO2 takes place

wayne
May 22, 2011 12:38 pm

Ira, I apologize. didn’t mean to use to strong of words and your example of the integration using 1/4 + 1/8 + 1/16 and so on … was basically correct, but it has become apparent that this only applies to the vertical movement, the vertical axis of radiation transfer, where you also have two more horizontal axises involved in three dimensions. So, the 390 Wm-2 up and down should always be spoken properly as 130 Wm-2 going up and down just as you described it. I just thought you should have pointed out that only one-third of the radiation participates in this vertical dance. That was my only point. I’ll try to not get so emotional when seeing misleading expalnations are made.
Five or six of your posts ago I also thought it was exactly as you were explaining it above, explaining that this happens to all radiative movement from the surface and in the lower atmosphere, but I have learned that is not so

Joel Shore
May 22, 2011 2:10 pm

R Stevenson says:

This is based on the increase in CO2 concentration needed to make the gas as effective as water vapour in absorbing LWIR ie based on equivalence:

In other words, it is based on extremely naive approximations and yet you expect us to believe it over actual line-by-line radiative calculations?!? The magnitude of the effect produced by CO2 is well-understood and accepted by even skeptical scientists like Richard Lindzen and Roy Spencer.
Stop talking nonsense.

jae
May 22, 2011 7:11 pm

Steve continues with more arm waving:
“I also refuted the notion that the lapse rate “explains” the temperature at the surface. No, the lapse rate can be used to calculate the temperature at the surface after the altitude and temperature of the tropopause is known. The lapse rate does not explain why the tropopause is at a specific altitude. It is the ability of the atmosphere to absorb and emit energy that explains the temperature of the atmosphere. ”
LOL, you refuted nothing, you just waved your arms AGAIN (do you know the diff?). It seems that you don’t even understand my comments or the article I linked! Please explain why all of the planets with atmospheres (including Earth) have a surface temperature that is much higher than their blackbody temperatures. Is it the greenhouse effect, or a more simple physical explanation, like the lapse rate and pV=RT?
I don’t think that you radiation freaks truly understand Local Thermodynamic Equilibrium. You seem to simply ignore heat capacity and thermalization–and prefer to look at radiation, in vacuuo. That is simplistic, to say the least.
It boggles my mind that actual physicists and other scients are not noticing the misleading crap portrayed in the K&T radiation cartoon. But then it took me about 5 years to finally discover the problem, so maybe there is hope for the others, too. While all the numbers in the cartoon may well be close to reality, it is interpreted (probably even by the authors) as demonstrating that a magic “greenhouse effect” keeps Mother Earth at an average temperature of 15 C, which would irradiate at a rate of 390 Wm-2. The cartoon only works IF the surface is at 15 C due to factors OTHER THAN the GHE (which Siddons, et. al. have adequately explained). Most people looking at that cartoon will be led to believe that the 324 wm-2 “backradiation” is SUSTAINING the 390 wm-2 surface radiation by ADDING energy to the surface. THAT, folks, is pure baloney, because it is suggesting that a cold atmosphere (about 5C average) can warm a warmer surface which is really, really bad physics (yeah, yeah, there are still some folks out there that believe that, but they will eventually learn….). The diagram is correct ONLY if the surface is maintained at 15 C by phenomenon other than some impossible backradiation magic. Which, of course, shows that the GHE theory is nonsense.
Don’t know how to express this more clearly….
The GHG theory is baloney. No wonder it cannot be demonstrated empirically.

wayne
May 22, 2011 7:47 pm

Ira Glickstein, PhD says:
May 22, 2011 at 6:41 pm
I have claimed that, despite the above fact, the layers above the 120 meter distance also contribute to the downwelling radiation that ultimately reaches the Surface such that, if about 1/2 are due to the first

I do agree with you on most points. If talking of absorption in a given length, that never means all is absorbed in that distance. It is a logarithmic function. it just means most is absorbed in that length and the ratio is normally termed as the mean free path and that seems to be 1-1/e portion or 63.2% is absorbed, 36.8% being transmitted, each mean free path length. For the next length 63% of what is left from the first length will be absorbed, and so on. Just take 1−0.368^lengths to get the statistical chance of that happening, in any direction. This is not the same as total extinction.
So radiation has a chance if directed downward to reach the surface no matter how high in the atmosphere it is emitted, it’s just that the chance fades quickly as you count lengths or shells. Same applies no matter which direction you are speaking of. That is why it is so easy to be misconstrued and sure hope it hasn’t been done to you.
Probably numerically a photon ever has a chance to pass from the surface of the sun down to the core without absorption but you probably would require arbitrary precision math to calculate the answer, for 10^327 might not be enough exponent.
The only problem as you keep speaking of this photon logic is the temperature lower is always warmer so the reverse always trumps and will have even more flowing upward. Many find it easier to accept that SB with a delta T properly used makes tracing individual photons rather irrelevant and of course so much easier for all to follow.

jae
May 22, 2011 7:48 pm

I hasten to add that the infrared active gases DO play an important role in the atmosphere: they facilitate rapid thermalization and radiation to space. It is possible that LTE would not exist without a certain amount of those “GHE gases.” However, it appears that only a certain amount of these gases is necessary to effect thermalization and LTE. So, unfortunately, we cannot test the concept of an atmosphere without “GHEs,” since we have none to observe. Even Jupiter, with its massive predominance of Helium and Hydrogen contain plenty of IR-active gases to facilitate LTE through thermalization reactions (collisions).
So, the “what if the atmosphere had no GHEs and contained only N2” concept cannot be addressed with empirical data. Still mere conjecture…

Phil.
May 22, 2011 8:00 pm

R Stevenson says:
May 22, 2011 at 10:17 am
Phil says:
‘ False because you’re wrong about where in the energy spectrum CO2 absorbs.’
It is apparent from whatyou say that you are not familiar with ‘Wien’s law of displacement’ judging by your comment that the spectral band from 12 to 18 microns includes the peak emission of the Earth’s emission spectrum.

On the contrary I am very familiar with all forms of Wien’s Law, I suggest you reread my comment, to help you I’ve highlighted the pertinent part:
“False because you’re wrong about where in the energy spectrum CO2 absorbs.
Recourse to Wien’s law shows that a wavelength of maximal emission λ_max of 15μm yields a temperature of minus 80C; whilst 15C gives a maximal emission λ_max of 10μm.
Frankly the wavelength of maximum emission doesn’t mean anything, we’re concerned with what’s happening to the energy so it’s the frequency or wavenumber form that’s relevant as I told you before.
CO2 absorbs infrared emissions from the Earth’s surface only minimally in the range 7 to 13μm and it is within this range where the greatest proportion of radiation emitted by the Earth is found.
No it isn’t, it accounts for about one third of the energy emitted at 288K. (45W/m^2/sr vs 124W/m^2/sr)

Phil.
May 22, 2011 8:42 pm

R Stevenson says:
May 22, 2011 at 8:31 am
Heat is transferred by conduction, convection and radiation and heat losses are often computed using a combined convection and radiation coefficient hc + hr. Heat loss:
Q = (hc+hr)*A*deltaT

No way, qr= fn((T+Δ)^4-T^4)

Phil.
May 22, 2011 9:33 pm

R Stevenson says:
May 22, 2011 at 9:07 am
This is based on the increase in CO2 concentration needed to make the gas as effective as water vapour in absorbing LWIR ie based on equivalence:
For a product term PwL of 0.0231*(4*3.281) ft.atm a water vapour absorptivity of o.2 absorbs 84Wm-2 of land LWIR in 4m.
The equivalent for CO2 requires a product term PcL of 0.0004*900*(4*3.281) ft.atm giving a CO2 absorptivity of 0.2 absorbing 84Wm-2 of land LWIR in 4m.
NB a partial pressure of o.ooo4*900 atm is 380ppm *900.

Where do you get your value of 0.2 emissivity from, what range of wavenumber is it for, why haven’t you accounted for the variation of energy over the wavenumber range or the different absorption bands of H2O and CO2?

Phil.
May 23, 2011 4:42 am

wayne says:
May 22, 2011 at 12:38 pm
Ira, I apologize. didn’t mean to use to strong of words and your example of the integration using 1/4 + 1/8 + 1/16 and so on … was basically correct, but it has become apparent that this only applies to the vertical movement, the vertical axis of radiation transfer, where you also have two more horizontal axises involved in three dimensions. So, the 390 Wm-2 up and down should always be spoken properly as 130 Wm-2 going up and down just as you described it. I just thought you should have pointed out that only one-third of the radiation participates in this vertical dance. That was my only point. I’ll try to not get so emotional when seeing misleading expalnations are made.

This is a misunderstanding, it isn’t an ‘either/or’ situation, the photons have a component in each of the three directions. Every one has a component in the vertical direction which may be up or down.

Steve
May 23, 2011 9:15 am

jae says: “THAT, folks, is pure baloney, because it is suggesting that a cold atmosphere (about 5C average) can warm a warmer surface which is really, really bad physics (yeah, yeah, there are still some folks out there that believe that, but they will eventually learn….).”
Again, a misrepresentation of the explanation. No one is proposing that the colder atmosphere warms the surface. It emits energy towards the surface, which lowers the rate of cooling of the (warmer) surface.
Please explain how I could raise the temperature of that cold atmosphere to 7C average (still much colder than the surface) in such a way that it would not lead to a rise in average surface temperature. Please explain how I could exchange a percentage of the molecules in the atmosphere that don’t absorb outgoing surface radiation for molecules that do absorb outgoing surface radiation without raising the temperature of the atmosphere.
“The diagram is correct ONLY if the surface is maintained at 15 C by phenomenon other than some impossible backradiation magic. Which, of course, shows that the GHE theory is nonsense.”
The fact that you have been unable to respond to the above, and insist on (back) radiation being “magic”, shows that your thought process is nonsense.

Robert Stevenson
May 23, 2011 12:27 pm

Phil says:
‘Where do you get your value of 0.2 emissivity from, what range of wavenumber is it for, why haven’t you accounted for the variation of energy over the wavenumber range or the different absorption bands of H2O and CO2?’
The precise method of calculating the effective absorptivity or emissivity for a gas body of finite dimensions is quite complex but for engineering calculations an approximate method developed by Hottel (1) yields results of satisfactory accuracy.
(1)Trans. Am. Inst. Chem. Engrs. 31, 517-549 (1935)

Robert Stevenson
May 23, 2011 12:36 pm

Phil says:
Heat loss:
Q = (hc+hr)*A*deltaT
No way, qr= fn((T+Δ)^4-T^4)
Will reply fully manana I am surprised you are not familiar with this notation for combined convection and radiation heat loss coefficient from horizontal and vertical surfaces. Reflects on modern technical education methods.

wayne
May 23, 2011 2:09 pm

“Phil. says:
May 23, 2011 at 4:42 am
This is a misunderstanding, it isn’t an ‘either/or’ situation, the photons have a component in each of the three directions. Every one has a component in the vertical direction which may be up or down.”
No misunderstanding, and the up-down component is 1/3. But you are right, the chances of it ever being dead horizontal with a vertical component of zero is astronomically tiny, same if it were vertical axis aligned. You are also right, it is never an either/or situation and should not be spoken as if it is. All components need to be mentioned and their influence accounted for.

Phil.
May 23, 2011 3:43 pm

Robert Stevenson says:
May 23, 2011 at 12:36 pm
Phil says:
Heat loss:
Q = (hc+hr)*A*deltaT
No way, qr= fn((T+Δ)^4-T^4)
Will reply fully manana I am surprised you are not familiar with this notation for combined convection and radiation heat loss coefficient from horizontal and vertical surfaces. Reflects on modern technical education methods.

It does indeed, it’s flat out wrong! If they’re teaching junk like this these days it does reflect poorly on modern education.

Joel Shore
May 23, 2011 7:13 pm

jae says:

The diagram is correct ONLY if the surface is maintained at 15 C by phenomenon other than some impossible backradiation magic.

You are the one who believes in magic, not us. You believe that there exists some magical version of the 2nd Law of Thermodynamics that prevents colder objects from radiating toward warmer objects…or the warmer objects from absorbing the radiation…or this absorbed radiation from affecting the energy balance, or some other total nonsense that violates the laws of physics.
You also seem to believe that invoking the lapse rate or the ideal gas law or gravity or some magical combination of them allows one to violate conservation of energy.
It is really embarrassing that you continue to argue such complete and utter nonsense. If I ever want to convince any of my physicist colleagues how ignorant skeptics are of actual science, all that I have to do is show them the sort of nonsense you argue in this thread. (Admittedly, that is unfair to the skeptics like Ira and Willis who at least try to fight against this sort of clear nonsense in violation of basic laws of physics.)

Robert Stevenson
May 24, 2011 2:28 am

Phil. says:
May 23, 2011 at 3:43 pm
Robert Stevenson says:
May 23, 2011 at 12:36 pm
Phil says:
Heat loss:
Q = (hc+hr)*A*deltaT (1)
No way, qr= fn((T+Δ)^4-T^4)
Let me enlighten you Phil in (1) above
hc=0.3*(T1-T2)^0.25
hr=[0.173*0.8[(T1/100)^4-(T2/100)^4]]/(T1-T2)
You add the two together to give a combined convection and radiation coefficient for a combined loss in (1) where deltaT of course is T1-T2. Im surprised you have not seen this before – so you’ve learned something new today

Robert Stevenson
May 24, 2011 2:37 am

Robert Stevenson says:
May 23, 2011 at 12:27 pm
Phil says:
‘Where do you get your value of 0.2 emissivity from, ‘
Further to my previous post concerning above and Hottel; O.2 (or 0.193) is the emissivity/absorptivity of CO2 in the atmosphere to the extinction point of 3600m for dry air.

Robert Stevenson
May 24, 2011 3:14 am

Joel says:
‘The magnitude of the effect produced by CO2 is well-understood and accepted by even skeptical scientists like Richard Lindzen and Roy Spencer.’
The more I read of what Ira is saying the more I realise what a negligible GHE CO2 has. Water vapour absorbs over 3 x more heat in the first 120m than the total potential of CO2 to its extinction point. Increasing CO2 does not affect or increase absorption in this 120m distance as it is dominated by water vapour. Subsequent reradiation and absorption above the 120m level is also dominated by water vapour . CO2 is a very weak GHG – it is but a walking shadow, a poor player, that struts and frets his hour upon the stage, and then is heard no more, it is tale told by an idiot, full of sound and fury, signifying nothing.
Spending trillions of $ on CO2 emission reduction is moronic to the nth degree

Joel Shore
May 24, 2011 8:00 am

Robert,
The relative magnitudes of the radiative effect of water vapor and CO2 are well-understood. I don’t see how your naive reasoning (and basic confusion about how the greenhouse effect works anyway) adds or changes anything in regards to that.

Phil.
May 24, 2011 8:46 am

Robert Stevenson says:
May 24, 2011 at 2:37 am
Robert Stevenson says:
May 23, 2011 at 12:27 pm
Phil says:
‘Where do you get your value of 0.2 emissivity from, ‘
Further to my previous post concerning above and Hottel; O.2 (or 0.193) is the emissivity/absorptivity of CO2 in the atmosphere to the extinction point of 3600m for dry air.

Over what range of wavelengths?

Robert Stevenson
May 24, 2011 10:34 am

Phil says:
‘Where do you get your value of 0.2 emissivity from, ‘
Further to my previous post concerning above and Hottel; O.2 (or 0.193) is the emissivity/absorptivity of CO2 in the atmosphere to the extinction point of 3600m for dry air.
Over what range of wavelengths?
The principal emission bands are at about 2.64 to 2.84, 4.13 to 4.5 and 13 to 17 microns

Robert Stevenson
May 24, 2011 10:53 am

Joel Shore says:
May 24, 2011 at 8:00 am
Robert,
The relative magnitudes of the radiative effect of water vapor and CO2 are well-understood. I don’t see how your naive reasoning (and basic confusion about how the greenhouse effect works anyway) adds or changes anything in regards to that.
Retired Engineer says:
May 8, 2011 at 8:22 am
This makes sense. Absent CO2 and water vapor, we would freeze. How much warming does each contribute? If CO2 did it all, and mankind produces about 3% of the CO2, then we are responsible for 3% of the 33C warming or about 1C. Thus AGW is real. Except for water vapor which may cause 90% of the 33C, in which case we cause 0.1C warming. Big deal. Given the daily and seasonal variations in temperature, I find it very hard to believe that 0.1C will have any measurable effect. Or be measurable at all.
This makes a great deal of sense to me. What does Joel the scientist say?

Joel Shore
May 24, 2011 11:21 am

Robert Stevenson (quoting Retired Engineer) says:

Thus AGW is real. Except for water vapor which may cause 90% of the 33C, in which case we cause 0.1C warming. Big deal. Given the daily and seasonal variations in temperature, I find it very hard to believe that 0.1C will have any measurable effect. Or be measurable at all.
This makes a great deal of sense to me. What does Joel the scientist say?

Nope…No sense at all. Four big problems with this estimate:
(1) CO2 is responsible for somewhat more than 10% of the greenhouse effect. How much depends on how you measure since the effects are not additive. (So if you took an atmosphere devoid of greenhouse gases and added CO2, you’d get a different answer than if you took the current atmosphere and just subtracted out the CO2.) A rough valuer is 20% (http://pubs.giss.nasa.gov/cgi-bin/abstract.cgi?id=sc05400j)
(2) The claim that only 3% is attributable to humans is basically pure fiction. It is true that there are large exchanges back-and-forth between the atmosphere and the biosphere and ocean mixed layer. If you count only the additions due to the biosphere and ocean mixed layer to the atmosphere and compare it to our additions to the atmosphere, it is true that ours is about 3% of the total. However, this ignores the fact that the biosphere and ocean mixed layer are also absorbing CO2 from the atmosphere. What is important is not the exchanges between these different subsystems but rather the additions to the overall system consisting of the atmosphere + biosphere + ocean mixed layer. By adding carbon to this system, we are responsible for essentially all of the increase in the level of atmospheric CO2 that has occurred since the start of the industrial revolution, which is at present about a 40% increase. In fact, if about half of what we emitted didn’t rapidly partition into the biosphere and ocean mixed layer, the CO2 levels would have gone up about twice as much. Whenever you see someone making this 3% claim, they are basically trying to deceive you (or have been deceived themselves).
(3) To calculate the total warming due to an increase in CO2, you have to consider feedbacks, for example, the water vapor feedback whereby the increase in CO2 leads to an increase in water vapor too which then causes further warming.
(4) The dependence of forcing on CO2 levels is logarithmic, not linear. (This is the only one of the 4 major problems that actually is in the direction of causing an overestimate of the contribution due to increasing CO2 levels.)

May 24, 2011 11:39 am

Steve says:
May 23, 2011 at 9:15 am “… It emits energy towards the surface, which lowers the rate of cooling of the (warmer) surface.”
Steve I will ask you to show the radiative heat transfer equation in which you input an emission from another body, gas/solid or fluid and show where it lowers the rate of cooling. It must be an emmission.
Mr. Stevenson keep up the good work.

Robert Stevenson
May 24, 2011 1:33 pm

R Stevenson says: Joel Shore have you read this post by John of Kent? From what you say you appear to believe a greenhouse theory and in AGW. You should read the paper contained within the post by physicist Joe Postma and put asde your own naive thinking.
John of Kent says:
May 8, 2011 at 3:18 am
“I modified the Carleton spreadsheet to compute the mean Solar radiation per square meter absorbed by the Earth System, which turns out to be 240 Watts/m^2. I then used the spreadsheet to determine the effective mean temperature of the Earth System that would emit an equal amount of energy to Space, and that turned out to be 255 Kelvins (-18ºC which is 1ºF).
Since the mean temperature at the surface of the Earth is 288 Kelvins (+15ºC which is 59ºF), that leaves 33 Kelvins (33ºC which is 58ºF) to be accounted for. Guess how we acount for it?”
We can account for the 33C difference without any need for an imaginary greenhouse effect. I urge everyone at WUWT to read this brilliant paper by physicist Joe Postma ;
http://www.globalwarmingskeptics.info/forums/attachment.php?aid=327
In this paper, Joe accounts for the 33C. The truth is that the radiative temperature of the Earth is measured from space IS -18C. And it is true that the average air temperature near the surface of the earth is 15C. However, if one includes the whole of the atmosphere right up to the edge of space and the surface of the earth in calculating the average radiative temperature of the Earths surface [i]and[/i] atmosphere, then we get to the -18C figure.
There is no discrepancy, there is no need for an imaginary “greenhouse effect”.
Surely the Phlogiston of our times!

JAE
May 24, 2011 1:52 pm

Joel Shore again waves arms rapidly:
“You believe that there exists some magical version of the 2nd Law of Thermodynamics that prevents colder objects from radiating toward warmer objects…or the warmer objects from absorbing the radiation…or this absorbed radiation from affecting the energy balance, or some other total nonsense that violates the laws of physics.”
You will not find anywhere that I said any of those things. Are you making this up or just misunderstanding me?
How about those laws of physics relating to the lapse rate and the ideal gas equation?
Anyway, you better tether yourself to an anchor, because if you wave your arms much harder you are going to take off like a Harrier Jet. I guess some folks just have to resort to insults when they cannot produce substantive arguments.
WHERE ARE THE ACTUAL EMPIRICAL DATA, JOEL? And please don’t tell me that the numbers in the radiation cartoons or the output of GCMs are data. Hell, I can’t even get the K&T numbers with Modtran (maybe I’m doing something wrong??).
Here are some numbers to consider: Let’s use the “radiation budget” in the K&T diagram. It presumes a surface temperature of 15 C. And at that temperature we have backradiation at 324 wm-2. But at that temperature, assuming a humid environment (which must be so, since there is plenty of water on most of the earth’s surface), saturated air can contain 12.75 g/m3 water vapor. The actual average amount is about 9 g. Now, let’s pretend we go to 30 C, where the amount of water vapor at saturation is over double (29.77) and actual is about 24 g, or 2.7 times as much. If we (only) double the amount of backradiation to adjust for all that extra water vapor, we would have 648 wm-2 backradiation. If we add that to the high-noon direct solar radiation in July–about 900 wm-2 in tropical and temperate zones– we would have a total amount of radiation of 1548 wm-2 at noon. That is equivalent to a blackbody temperature of 133 C (272 F). I dare say that doesn’t make sense to me. Even if the backradiation stays at only 324 wm-2, the noon bb temp. would be 110 C (230 F). Something seems wrong with this picture when you start looking at actual numbers, that’s all.
(Amt. vapor at saturation = 5.465 e^0.0565 t; actual amounts are taken from a regression of 30-year average data for July and December from 156 locations = 3.422 e^0.064 t; R^2 = 0.98)

Robert Stevenson
May 24, 2011 2:57 pm

Another much hyped GHG is methane; there is only 1 ppm in the atmosphere!

May 24, 2011 4:01 pm

Robert Stevenson says on May 24, 2011 at 1:33 pm:
“Joel ————————————————–times!”
All the words between those two above make perfect sense to me. – If IRA and others who “believe” there must be some truth in “The AGW Theory” because they have worked out a surface temperature of -18 °C by using good mathematics and undisputable physical laws should read up on what Joseph E. Postma has got to say about it. —-(All 31 pages of it)

Phil.
May 24, 2011 6:14 pm

Robert Stevenson says:
May 24, 2011 at 10:34 am
Phil says:
‘Where do you get your value of 0.2 emissivity from, ‘
Further to my previous post concerning above and Hottel; O.2 (or 0.193) is the emissivity/absorptivity of CO2 in the atmosphere to the extinction point of 3600m for dry air.
“Over what range of wavelengths?”
The principal emission bands are at about 2.64 to 2.84, 4.13 to 4.5 and 13 to 17 microns

Yes I know that, the only one of which is relevant is 13-17, so I’ll ask again, over what range of wavelengths does your value of 0.2 apply? From the above I’ll accept that it’s zero from 2.84 to 4.13, 4.5 to 13 and over 17.

jae
May 24, 2011 6:22 pm

“All the words between those two above make perfect sense to me. – If IRA and others who “believe” there must be some truth in “The AGW Theory” because they have worked out a surface temperature of -18 °C by using good mathematics and undisputable physical laws should read up on what Joseph E. Postma has got to say about it. —-(All 31 pages of it)”
Yeah. We can hope that Joel Shore and his CAGW-lovin buddies will address this, using logic and facts, as opposed to armwaving and insults. I still have not seen a refutation of Postma’s ideas, as well as the related ideas by Siddons, et. al. that I linked. Are the “experts” too busy to bother? Ira, would you care to “pitch in,” here?

Phil.
May 24, 2011 6:31 pm

Robert Stevenson says:
May 24, 2011 at 2:57 pm
Another much hyped GHG is methane; there is only 1 ppm in the atmosphere!

More like 1.7ppmv, absorbs about 2W/m^2 at about 1200-1300 cm-1

Joel Shore
May 25, 2011 10:16 am

Robert Stevenson says:

We can account for the 33C difference without any need for an imaginary greenhouse effect. I urge everyone at WUWT to read this brilliant paper by physicist Joe Postma ;
http://www.globalwarmingskeptics.info/forums/attachment.php?aid=327

jae says:

I still have not seen a refutation of Postma’s ideas, as well as the related ideas by Siddons, et. al. that I linked. Are the “experts” too busy to bother?

Good God, are you guys blind? How many times do we have to debunk Postma’s nonsense?
http://wattsupwiththat.com/2011/05/07/visualizing-the-greenhouse-effect-light-and-heat/#comment-655686
http://wattsupwiththat.com/2011/05/07/visualizing-the-greenhouse-effect-light-and-heat/#comment-655880
In this thread http://wattsupwiththat.com/2011/05/14/life-is-like-a-black-box-of-chocolates , Postma even came by to try to defend his nonsense after Willis and I tore into it, although he didn’t stick around long.

May 25, 2011 10:29 am

Further to our discussion concerning IR thermometers, Omega was kind enough to reply and provide links to some of their technical documents. The document which I think most relevant to our discussion is the following one:
http://www.omega.com/temperature/Z/pdf/z063-066.pdf
which describes the design and operation of such as unit. There is also a summary of radiative physics and the equations upon which the technology rests.
I quote from the document as follows:
“A basic infrared thermometer (IRT) design, comprises a lens to collect the energy emitted by the target; a detector to convert the energy to an electrical signal; an emissivity adjustment to match the IRT calibration to the emitting characteristics of the object being measured; and an ambient temperature compensation circuit to ensure that temperature variations within the IRT, due to ambient changes, are not transferred to the final output.”
So when I measure the temperature of a block of ice while standing in the sun at 23C my IR Thermometer is collecting energy emitted from a colder object to a hotter object.

Joel Shore
May 25, 2011 10:36 am

jae says:

“You believe that there exists some magical version of the 2nd Law of Thermodynamics that prevents colder objects from radiating toward warmer objects…or the warmer objects from absorbing the radiation…or this absorbed radiation from affecting the energy balance, or some other total nonsense that violates the laws of physics.”
You will not find anywhere that I said any of those things. Are you making this up or just misunderstanding me?
How about those laws of physics relating to the lapse rate and the ideal gas equation?

(1) Just saying the words “lapse rate” and “ideal gas equation” doesn’t get you around having to explain why the surface temperature of the earth is not affected by having the atmosphere radiating at it.
(2) Just saying the words “lapse rate” and “ideal gas equation” doesn’t get you around having to explain how one could conserve energy if one considers the current surface temperature of the earth and imagines that one had an atmosphere transparent to IR radiation from the earth’s surface.
(3) Just saying the words “lapse rate” and “ideal gas equation” doesn’t get you around having to explain what sets the temperature somewhere in the earth’s atmosphere or at the surface. The lapse rate only determines the slope of the temperature vs. altitude line. You still need to set the value of the temperature at some point along that line in order to determine the temperature at the surface (or anywhere). [Also, the lapse rate only holds in part of the atmosphere…the troposphere…because the adiabatic lapse rate sets a stability limit on the temperature decrease with height. Where the heating of the atmosphere occurs is also relevant in determining what part of the atmosphere has a lapse rate approximately at that stability limit vs what parts have a lower lapse rate or even an increase in surface temperature with height.]

WHERE ARE THE ACTUAL EMPIRICAL DATA, JOEL? And please don’t tell me that the numbers in the radiation cartoons or the output of GCMs are data. Hell, I can’t even get the K&T numbers with Modtran (maybe I’m doing something wrong??).

K&T’s numbers aren’t output of models. They are empirical values obtained mainly from satellite data. And, there is plenty of empirical data at every level: There is empirical data on the basic absorption lines of the various atmospheric constituents, there is a wealth of empirical data backing up the basic equations of radiative transfer that are applied in calculating the greenhouse effect in just the same way that engineers and scientists use these equations everyday in other calculations, there is empirical spectra looking both up from the surface of the earth and down from satellites.

Here are some numbers to consider: Let’s use the “radiation budget” in the K&T diagram. It presumes a surface temperature of 15 C. And at that temperature we have backradiation at 324 wm-2. But at that temperature, assuming a humid environment (which must be so, since there is plenty of water on most of the earth’s surface), saturated air can contain 12.75 g/m3 water vapor. The actual average amount is about 9 g. Now, let’s pretend we go to 30 C, where the amount of water vapor at saturation is over double (29.77) and actual is about 24 g, or 2.7 times as much. If we (only) double the amount of backradiation to adjust for all that extra water vapor, we would have 648 wm-2 backradiation. If we add that to the high-noon direct solar radiation in July–about 900 wm-2 in tropical and temperate zones– we would have a total amount of radiation of 1548 wm-2 at noon. That is equivalent to a blackbody temperature of 133 C (272 F). I dare say that doesn’t make sense to me. Even if the backradiation stays at only 324 wm-2, the noon bb temp. would be 110 C (230 F). Something seems wrong with this picture when you start looking at actual numbers, that’s all.

(1) You don’t double the backradiation when you double the water vapor. You increase it by some amount but definitely don’t double it.
(2) I don’t know where you get your intuition as to what is a reasonable amount of radiation for the earth to receive at high noon in the tropics. Even with no back-radiation, the “bb temp.” calculated would be 82 C. However, I put “bb. temp.” in quotes because this temperature has very little real meaning: It would mean something in the limit of an earth with no heat capacity and no advection, convection, etc. For the real earth, with a significant heat capacity and significant atmospheric and ocean transport, the one summary number that has meaning is the average of T^4 over the surface of the earth…That is what is going to go into determination of the global surface radiative balance. The local balance will tend to be dominated by these other issues.

Robert Stevenson
May 25, 2011 10:41 am

Phil says:
‘Where do you get your value of 0.2 emissivity from, ‘
Work it out for yourself Phil its not rocket science.

Robert Stevenson
May 25, 2011 10:44 am

Another much hyped GHG is methane; there is only 1 ppm in the atmosphere!
More like 1.7ppmv, absorbs about 2W/m^2 at about 1200-1300 cm-1
A new definition of clutching at straws.

gnomish
May 25, 2011 11:54 am

jae says:
I don’t think that you radiation freaks truly understand Local Thermodynamic Equilibrium. You seem to simply ignore heat capacity and thermalization–and prefer to look at radiation, in vacuuo. That is simplistic, to say the least.
yah, and they’re still saying it and they won’t stop saying it. it’s the climate catechism.
they won’t say what the phase change of water looks like on IR, tho…lol
50,000 times more heat moved by phase change effects of 1% h20 gas compared to 500ppm co2 in any volume of atmosphere.
and it’s well known that any improvement in heat capacity improves coupling from heat source to sink, not the opposite.
and they wish to imagine that every co2 molecule never touches a neighbor – no kinetic transfer, so they must imagine that the co2 gets hotter than the surroundings?
otherwise, it would be the same temperature as the surroundings and contribute its tiny tiny share of thermal capacity to the transfer of heat from source to sink – which is virtually an infinite sink.
lol – there’s a reason why houses are not insulated with nothing but mirrors, eh?

Robert Stevenson
May 25, 2011 12:40 pm

Phil says:
… over what range of wavelengths does your value of 0.2 apply?
13 to 17 microns.

May 25, 2011 12:57 pm

All this talk about “radiation and back-radiation” has confused my cat. But she is Siamese and may not fully understand the language. Having said that, she is – still pretty good at mathematics.
Now then, – she has worked out, – mathematically of course – that once x units of energy have been transported away from the surface, the surface must cool. – And, furthermore – that if then those x units of energy are transported (by radiation) to atmospheric GHGs that eagerly consume or absorb them, those GHGs must warm. – Well, so far, so good and I agree with her there.
However then her mathematical model shows that if those same x units of energy are radiated back to the “Surface” the surface must warm. – Yes, of course it does, her model shows – but not to a higher temperature than what it was before it lost the energy in the first place.
Shall I give my cat a “Cat-treat” today?

May 25, 2011 1:33 pm

Joel Shore says:
May 25, 2011 at 10:36 am
(2) I don’t know where you get your intuition as to what is a reasonable amount of radiation for the earth to receive at high noon in the tropics.
My heat transfer book shows 1061 W/m^2 for sun over head. SOD has an example from a book showing 700 W/m^2 in Saskatchewan (sp) in July at the surface. That is a measured 700 which would indicate a T of 60 C, but actual T reading was 40 C ,I think. Gases dissipate heat.
http://scienceofdoom.files.wordpress.com/2011/02/hartmann-ch4-extract.png
So day time solar W/m^2 are quite large even at mid latitudes. I though you knew this stuff.

JAE
May 25, 2011 1:49 pm

Oh, Joel, where to begin:
“(3) Just saying the words “lapse rate” and “ideal gas equation” doesn’t get you around having to explain what sets the temperature somewhere in the earth’s atmosphere or at the surface. The lapse rate only determines the slope of the temperature vs. altitude line. You still need to set the value of the temperature at some point along that line in order to determine the temperature at the surface (or anywhere). ”
That’s WHY I and some others here are trying to get you to actually read the links we posted that explain the “ghe” without using radiative balances (have you even bothered to LOOK at them, let alone tell us why they are incorrect???). As they explain so much better than I can, the temperature of the Earth’s surface is “set” by the radiation signal at about 5 km, where it is about -18 C (i.e., the so-called bb temp. of Earth). One then “works out” the surface temperature using lapse rate, and lo and behold it is about 15 C–without having to do radiation balances.
Please note that I DO believe the radiation exists; I just don’t think it is the CAUSE of much; but rather an EFFECT.
“(1) You don’t double the backradiation when you double the water vapor. You increase it by some amount but definitely don’t double it.”
Yes, I know. Probably some logarithmic function, eh? But this gets very funny to me when I hear “climate scientists” suggest that there will be all this “positive water vapor feedback” from a putative increase of 3 wm-2 from doubling OCO! You can’t have it both ways, bud.
“(2) I don’t know where you get your intuition as to what is a reasonable amount of radiation for the earth to receive at high noon in the tropics. Even with no back-radiation, the “bb temp.” calculated would be 82 C. However, I put “bb. temp.” in quotes because this temperature has very little real meaning: It would mean something in the limit of an earth with no heat capacity and no advection, convection, etc. For the real earth, with a significant heat capacity and significant atmospheric and ocean transport, the one summary number that has meaning is the average of T^4 over the surface of the earth…That is what is going to go into determination of the global surface radiative balance. The local balance will tend to be dominated by these other issues.”
This makes absolutely no sense to me. My “intuition” on solar radiation comes from here:
http://rredc.nrel.gov/solar/old_data/nsrdb/redbook/sum2/state.html
What would be your guess, looking at this data. 900 wm-2 is probably quite conservative for high noon!
You folks are telling us that you can ADD wattages to arrive at a total, as in the K&T diagram. Now you are saying I cannot add the power from the sun to the power from backradiation to get a new total–and then back-calculate the temperature of a blackbody which would irradiate at that temperature. Again, sir, you cannot have it both ways.
“K&T’s numbers aren’t output of models. They are empirical values obtained mainly from satellite data. And, there is plenty of empirical data at every level: There is empirical data on the basic absorption lines of the various atmospheric constituents, there is a wealth of empirical data backing up the basic equations of radiative transfer that are applied in calculating the greenhouse effect in just the same way that engineers and scientists use these equations everyday in other calculations, there is empirical spectra looking both up from the surface of the earth and down from satellites.”
Baloney. K&T’s numbers are guestimates. You cannot get anywhere near their values using Modtran (averages of various conditions or any single conditions). Is MODTRAN empirical?
The rest of the data you mention are, indeed, empirical, but they do NOT demonstrate the magnitude of any greenhouse effect. They demonstrate that IR-active substances emit IR in very predictable amounts. This information, indeed, does allow many calculations, but not one that proves, from first-principles, that there is a purely radiative greenhouse effect that makes the surface 33 C warmer than it “should be.”
“(2) Just saying the words “lapse rate” and “ideal gas equation” doesn’t get you around having to explain how one could conserve energy if one considers the current surface temperature of the earth and imagines that one had an atmosphere transparent to IR radiation from the earth’s surface.”
If we had an atmosphere completely transparent to IR, we would not likely have LTE, either, so I don’t know what would happen–and it doesn’t matter for the purposes of this discussion. Again, you guys seem to be ignoring all other heat transfer mechanisms other than radiation in your “ghe” explanations. It think that is incorrect.

May 25, 2011 2:01 pm

O.H. Dahlsveen said:
Shall I give my cat a “Cat-treat” today?
Alleyne replies:
Lucky for her your name is not Schroedinger 🙂 The way I understand it, she deserves a Cat-treat. The surface won’t warm up to where it was before, but also the heat loss slows as a result of only the net difference between the energy radiated from the surface and the energy “returned” from the atmosphere being lost from the surface.

JAE
May 25, 2011 2:31 pm

Alleyne:
“The surface won’t warm up to where it was before, but also the heat loss slows as a result of only the net difference between the energy radiated from the surface and the energy “returned” from the atmosphere being lost from the surface.”
This is true. However, picture this scenario: If you use a flame to heat the end of a metal rod, the heat will at first travel toward the other end of the rod at a very fast rate. As equilibrium conditions are approached, however, the heat will flow towards the other end at a slower and slower rate. But we don’t call this effect a “greenhouse metal effect.” It is the “delta T effect” present in all equations of thermal change. Of course, in the air heat flows slower from the surface to the layer above as that above layer becomes warmer. But not just due to radiation, but due to “back -conduction” (i.e., kinetic energy transfer between molecules), also. This total focus on radiation is keeping folks from looking at other heat transfer mechanisms.

May 25, 2011 2:46 pm

885 responses to this article, – and we still cannot agree. (or maybe we do agree to disagree) I cannot agree with the GHgT (Greenhouse gas theory) simply because it defies logic.
Once upon a time, when I was the chief engineer on a ship that had the misfortune of suffering “a breakage of a camshaft in one of it’s engines” I reported to the shipping company that no further usage of that engine was possible until ’a (new) renewed camshaft had been installed – I was, promptly treated to a visit from “The Company’s Doctor of Mechanical Engineering” who informed me that my report showed that no fractures of “External Engine Panels” had taken place. He therefore drew the conclusions that none of the parts that formerly made up the composition of the cam-shaft had left the engine and that it would be a (maybe tedious but not impossible) task for my crew to search out, reassemble and reinstall the said camshaft.
Well, I never attained a doctorate, so I could not fix it. – nor can I – to this date, fix it.

May 25, 2011 2:58 pm

Alleyne says on May 25, 2011 at 2:01 pm:
“O.H. Dahlsveen said:
Shall I give my cat a “Cat-treat” today?
Alleyne replies:
Lucky for her your name is not Schroedinger :-)”
What! – You mean Schroedinger had a cat as well?

May 25, 2011 3:28 pm

Alleyne says on May 25, 2011 at 2:01 pm:
“Lucky for her your name is not Schroedinger 🙂 The way I understand it, she deserves a Cat-treat. The surface won’t warm up to where it was before, but also the heat loss slows as a result of only the net difference between the energy radiated from the surface and the energy “returned” from the atmosphere being lost from the surface.”
So Alleyne does that mean that “as far as you can understand it” the surface temperature and my cat’s explanation thereof coincide?
Well, the lucky little beastie is in for two treats tonight

ferd berple
May 25, 2011 4:19 pm

“lol – there’s a reason why houses are not insulated with nothing but mirrors, eh?”
That is a good observation. Why do we not see someone selling highly IR reflective paint to coat houses inside and out if back radiation is so significant? We should be able to focus this with IR mirrors and heat our houses at night, given that it is calculated to be a significant percentage of solar radiation.
So, why do we spend so much to heat our cities at night? Why not just focus the IR back radiation from the atmosphere and boil water to produce steam/electricity, as we do with solar radiation during the day.
We could place mirrors in farms just outside the cities and boil water at night using back radiation and use this steam to produce electricity and heat our cities without any need for batteries. How come engineers are so stupid that they cannot figure out how to do this? Why haven’t climate scientists shown then how it is done? Why are we trying instead to generate power with windmills when there is all this back radiation?

May 25, 2011 5:08 pm

O H Dahlsveen said on May 25, 2011 at 2:58 pm:
“What! – You mean Schroedinger had a cat as well?”
Alleyne replies:
Yes he did, or does, we can’t be sure 🙂 BTW, I am all too familiar with the opinions people with no field experience come up with, to the point where in the fabrication company I used to work for we had to developed a new material, ‘unatanium’, in order to implement some of the wonderful ideas and designs these people came up with 🙂
It seems to me that so far, having determined that radiative energy can slow the heat loss from the surface, the questions which remain unresolved are:
To what extent does CO2 account for warming? Now I am not denying that it does, but how much warming is the result of the lapse rate and adiabatic heating resulting from the depth of the atmosphere and how much from CO2 and other GHGs “trapping” heat?
Is increased water vapour a positive or negative feedback? More water vapour = more clouds = more albedo seems to be the dominant effect, and therefore a negative feedback.
Is the climate as sensitive as the AGW theory theorizes? It most likely isn’t based on the paleoclimatic record (as I understand it).
Is it worth spending trillions of dollars “decarbonizing” our society based on a very unlikely probability that AGW will have a significant impact on the climate, moreover one that is adverse to our interests? Obviously not given the significant risks such a policy poses.

Joel Shore
May 25, 2011 6:15 pm

jae says:

That’s WHY I and some others here are trying to get you to actually read the links we posted that explain the “ghe” without using radiative balances

Not only have I read them but I have actually critically analyzed them, which is clearly more than you have.

(have you even bothered to LOOK at them, let alone tell us why they are incorrect???).

Yes and yes http://wattsupwiththat.com/2011/05/07/visualizing-the-greenhouse-effect-light-and-heat/#comment-667513 …but it doesn’t seem to matter because you just ignore the explanations and pretend that I haven’t given them.

As they explain so much better than I can, the temperature of the Earth’s surface is “set” by the radiation signal at about 5 km, where it is about -18 C (i.e., the so-called bb temp. of Earth). One then “works out” the surface temperature using lapse rate, and lo and behold it is about 15 C–without having to do radiation balances.

Why do you repeat this again and again when I have again and again told you what is wrong with it. The reason that the temperature is set by the the radiation from about 5 km is because of the current level of IR-absorbing matter in the atmosphere. If there were more greenhouse gases, the level would be lower and the ground temperature lower; if there were less greenhouse gases, the level would be higher and the ground temperature would be higher. Which part of this is so difficult for you to understand?

Please note that I DO believe the radiation exists; I just don’t think it is the CAUSE of much; but rather an EFFECT.

What the heck does that mean? Does it mean that we can just ignore conservation of energy because you find it inconvenient to what you want to believe?

Yes, I know. Probably some logarithmic function, eh? But this gets very funny to me when I hear “climate scientists” suggest that there will be all this “positive water vapor feedback” from a putative increase of 3 wm-2 from doubling OCO! You can’t have it both ways, bud.

There’s no “both ways”. There is nothing inconsistent in this picture.

This makes absolutely no sense to me. My “intuition” on solar radiation comes from here:
http://rredc.nrel.gov/solar/old_data/nsrdb/redbook/sum2/state.html
What would be your guess, looking at this data. 900 wm-2 is probably quite conservative for high noon!

My point about intuition was not applying to that. It was from saying (to paraphrase), “Oh, my god, the amount of radiation hitting the surface sounds so high. It corresponds to a blackbody temperature of X.” My point was that you have no good intuition as to what constitutes an excessively large amount of radiation hitting the surface. The blackbody temperature isn’t particularly relevant at a single point at the surface because there are lots of different heat transport mechanisms that affect the local surface energy balance and there’s lots of thermal inertia at the surface, particularly the oceans.

You folks are telling us that you can ADD wattages to arrive at a total, as in the K&T diagram. Now you are saying I cannot add the power from the sun to the power from backradiation to get a new total–and then back-calculate the temperature of a blackbody which would irradiate at that temperature. Again, sir, you cannot have it both ways.

Is this really all so difficult for you to understand? I am not saying you can’t add powers. I am just saying that this “blackbody temperature” that you derive from it doesn’t mean a whole lot because it is at one particular point on the surface. At another point on the opposite side of the planet, there is only the radiation from the atmosphere and the “blackbody temperature” you’d calculate is much lower. What is actually relevant in figuring out how much the earth is going to absorb is averaging the powers over the entire planet and what is relevant in figuring out how much it is going to radiate is averaging the temperature over the entire surface (actually T^4…or, most technically, the emissivity*T^4 but the emissivity in the mid- and far-IR is very close to 1 for most surfaces).

Baloney. K&T’s numbers are guestimates. You cannot get anywhere near their values using Modtran (averages of various conditions or any single conditions). Is MODTRAN empirical?

No…MODTRAN is a model…which you are likely misapplying if you are getting nowhere close. The K&T numbers are derived from empirical data, mainly satellite data. Yes, there is data analysis involved and there are uncertainties…but if you want to call them “guestimates” for this reason, you’d have to call basically every measurement in the universe a guestimate.

The rest of the data you mention are, indeed, empirical, but they do NOT demonstrate the magnitude of any greenhouse effect. They demonstrate that IR-active substances emit IR in very predictable amounts. This information, indeed, does allow many calculations, but not one that proves, from first-principles, that there is a purely radiative greenhouse effect that makes the surface 33 C warmer than it “should be.”

Yes…The 33 C number is in fact derived from empirical values: One compares the actual surface temperature to the surface temperature that would be required to satisfy conservation of energy in the absence of absorption of emitted surface radiation.

If we had an atmosphere completely transparent to IR, we would not likely have LTE, either, so I don’t know what would happen–and it doesn’t matter for the purposes of this discussion. Again, you guys seem to be ignoring all other heat transfer mechanisms other than radiation in your “ghe” explanations. It think that is incorrect.

We’re not ignoring all other heat transfer mechanisms. We are just saying that radiation alone from the surface would violate energy balance in the absence of IR-absorbing elements in the atmosphere; all that convection and conduction could do is make the violation worse. Between the earth system and space, radiation is all there is. You could try to argue that convection and conduction from the atmosphere to the surface adds enough heat to the surface close the energy balance…but claiming that convection and conduction lead to heat transfer from the colder atmosphere to the warmer surface would REALLY violate the 2nd Law.

jae
May 25, 2011 6:28 pm

Ira:
“The part of your posting that I highlighted above has generated an example that may convince you that there is, indeed, what you call “back-conduction” equivalent to what we have been calling “back-radiation”.”
No argument at all! My point is only that the air also “back-conducts,” not just “back-radiates.” Water is another example where the reasons for a “delta T” involves conduction and no radiation. IMHO, there is way too much emphasis on radiation, to the virtual exclusion of convection and conduction and the simple storage of heat by matter. And this “obsession” with radiative-only explanations tends to focus emphasis just on GHGs and even tiny additions of them (CO2) which cannot possibly have any significant effect, to the exclusion of the immense importance of LTE (thermalization) and lapse rates.
BTW, you are just as quiet as Joel Shore on those articles which explain the 15 C average surface temperature without considering GHGs at all. Do you have no opinion? If these articles are correct, we don’t need a ghe to explain what is going on, so they are worth refuting, if it can be done.

jae
May 25, 2011 6:49 pm

And, Ira:
If you look closely at the K&T diagram and the whole ghg theory, it is not just saying that the ghgs are “slowing the cooling,” which indeed they are. THE THEORY is SAYING that the Sun is providing only 342 wm-2 into the planet, only 168 wm-2 of which is being absorbed by the surface, but SOMEHOW, through backradiation, the surface is capable of emitting 390 wm-2 and increasing in temperature to 15 C. I simply CANNOT accept this, even after about 5 years staring at that diagram. I still think it is nonsense. And I’m not exactly alone.
I admit to being basicly a B student, so maybe I simply cannot think and understand like the pros. But I can still keep asking “the pros” to explain it at a level that I can understand. And to provide some type of empirical demonstration for their theory (no empirical evidence, NO SCIENCE!). They have not done that, IMHO.

wayne
May 25, 2011 6:53 pm

Ira:

“By the way, I accept that, in the case of both soldering irons, the direction of net heat transfer is from the very hot heating element to the less hot iron to the still less hot penny. But, if the energy flow is only going one way, how does the first tip get the “message” back that it has to be warmer and the second tip get the “message” it has to be cooler?”
Through your new “back-conduction”? LOL! ROTF! I could feel you moving that way all along. /sarc
Seriously, the “message” is through delta T, that simple, just as jae clearly explained, as I and others all up this thread have explained to you. Open a good physics book.
Your example is lacking. If you are “maintaining” the pennies’ temperatures; that could be a fixed energy sinks on the pennies, or, a variable ones. Specify. If fixed, B’s sink must be greater that A’s sink to keep B at 0 ºC and A at 15º Cif the room is warm. See, your example needs no “message” at all. If you turn off the cooling that “maintains” the lower temperatures then they will equalize in the end to the SAME temperature, all calculated by delta Ts.
This is the same problem you are having when viewing the atmosphere. Toss the “back-conduction”, toss the “back-radiation” and look at the problem frontwards from the standpoint of what the energy “sees”, always warm to cooler, always from higher energy density to lower energy density, if Ts are the same, all energy transfer ceases numerically and really, literally. It will do your mind nothing but some good.

May 25, 2011 7:37 pm

jae says on May 25, 2011 at 6:28 pm

IMHO, there is way too much emphasis on radiation,

It is (emphasized) because it is TRUE.
You have obviously never sat out on a car hood till the late hours (with a member of the opposite sex) have you? And witnessed TRUE radiative cooling of the car’s hood, roof, until they eventually ‘dew up’ … the reason why FOG FORMS (surface radiative cooling).
All your posts re: ‘radiative cooling’ show this deficiency.
.

Joel Shore
May 26, 2011 6:06 am

wayne says:

This is the same problem you are having when viewing the atmosphere. Toss the “back-conduction”, toss the “back-radiation” and look at the problem frontwards from the standpoint of what the energy “sees”, always warm to cooler, always from higher energy density to lower energy density, if Ts are the same, all energy transfer ceases numerically and really, literally. It will do your mind nothing but some good.

Really? So let’s put the atmosphere and the earth at the same temperature. I agree there will be no net heat flow from the earth to the atmosphere. Will there be no temperature change of the earth? If so, what happens to the energy that the earth is receiving from the sun? Does it just magically vanish?
jae says:

BTW, you are just as quiet as Joel Shore on those articles which explain the 15 C average surface temperature without considering GHGs at all. Do you have no opinion? If these articles are correct, we don’t need a ghe to explain what is going on, so they are worth refuting, if it can be done.

Every time you say this I am going to link to the most recent post I have that discusses it…or links back to a previous post discussing it: http://wattsupwiththat.com/2011/05/07/visualizing-the-greenhouse-effect-light-and-heat/#comment-667782
(Eventually people will have to jump back several posts to get to the actual substance, but it will serve to show how you are just saying this despite the fact that we have already refuted it…multiple times.)

jae
May 26, 2011 7:18 am

Jim”
“You have obviously never sat out on a car hood till the late hours (with a member of the opposite sex) have you? And witnessed TRUE radiative cooling of the car’s hood, roof, until they eventually ‘dew up’ … the reason why FOG FORMS (surface radiative cooling).”
Oh, yes I have! And nothing you say is wrong. But I don’t see what that has to do with the subjects at hand.
Why do so many folks here think that I somehow deny the existence of radiation? Hell, I’ve used IR innumerable times to identify chemical compounds!
I just don’t think the GHE, however you want to define it, is caused by a radiative effect. It’ s essentially a simple heat storage phenomenon and is explained well by Siddons and some other links provided above. The radiation FOLLOWS from all this; it doesn’t CAUSE it.
You simply cannot have the sun furnishing an average of 342 wm-2 into the Earth system and then say that the system somehow “magnifies” that amount of energy to create a surface that emits at at an average of 390 wm-2. And it seems to me that that is exactly what is being proposed.
Folks, like Ira, keep saying that the GHE is just a result of “slowing down” the loss of radiation. If that was all that was being proposed, I would have no objection (although I would again say that radiation is only part of the mechanism). But that is NOT all that is being proposed!

May 26, 2011 7:31 am

So far, none of our experts out here in WUWT land – or anywhere else I have looked have made what I can call “a good job of explaining” as to how it is possible for energy that is transported away from the surface and back again via GHGs manages to warm the surface. (It can warm GHGs, yes but the atmosphere does not warm the surface, it cools it.)
Energy can neither be destroyed nor created and that’s the natural law that matters most in this case. – In other words if the total “wattage” exchange at the top of the Atmosphere (TOA) say 340 W/m² incoming radiation matches any outgoing W/m² radiation + reflection, as it does in K&T’s plan, then I cannot see why we must not (all of us) accept that there is neither warming nor cooling. The only thing that any radiation therefore can do is similar to what conduction, convection and advection do.
The difference is that radiation transports energy (that can facilitate work which creates motion which can create friction which does create heat or heat-energy, as it is sometime called, the strength of which can be measured by its temperature.

I know Ira has “disconnected” because I once said I do not “believe” that W/m² IR radiation contains heat, but even so he did a good job of explaining what “Kinetic energy” is. And I still do not believe “molecules in motion” can be transported by radiation.
Oh, – and Jim did you ever try to drive your “passion wagon” with the cooling water drained from the engine? If you believe LWIR radiation is anything but a minor player inside the earth system then, go ahead; drain away any cooling-fluid and let radiation do the job. – Or maybe perhaps rethink your “radiating away from the hood” idea.

Robert Stevenson
May 26, 2011 7:49 am

Phil says:
‘Where do you get your value of 0.2 emissivity from, ‘
Further to our recent posts concerning the above and Hottel; 0.2 (or 0.193) is the emissivity/absorptivity of CO2 in the atmosphere to the extinction point of 3600m for dry air.
Phil, what do your calculations give for the distance in which the first-generation photons from the Surface up into the atmosphere are absorbed to extinction in CO2’s case?

Robert Stevenson
May 26, 2011 9:34 am

Joel says:
‘Good God, are you guys blind? How many times do we have to debunk Postma’s nonsense?
http://wattsupwiththat.com/2011/05/07/visualizing-the-greenhouse-effect-light-and-heat/#comment-655686
Powerful arguments Joel, but the GHE involving back radiation is just a theory right and I’m sure you could demolish it if you put your mind to it. Incidentally what is the magnitude of the emissive power of the atmosphere at 5km altitude?

Bryan
May 26, 2011 10:45 am

Joel says:
‘Good God, are you guys blind? How many times do we have to debunk Postma’s nonsense?
Now pay attention!
For Joel is an expert in nonsense.
He is a co author of an embarrassing collection of nonsense.
Read some earlier posts.
Halpern et al sets a new low standard by which all previous and subsequent nonsense will be judged.

JAE
May 26, 2011 1:16 pm

Joel waves arms again:
“Good God, are you guys blind? How many times do we have to debunk Postma’s nonsense? ”
And then provides links to….more armwaving… and more armwaving….and….
ROFLMAO.
Joel, you not only didn’t “debunk” anything, it appears you simply do not EVEN understand the issue. All you did is mumble about what if the planet had no ghgs, or something. I could not even understand what your point was.
OF COURSE, the radiation at 5 km is from what you are calling “greenhouse gases” (i.e. IR-active gases). So what?? The so called “black body temperature of the earth” (-18 C) is actually the temperature at 5 km (both measured and calculated). Because of the LAPSE RATE (NOT GHGs!), that means that the surface HAS to be about 15 C. The lapse rate has nothing to do with GHGs, depending only on heat capacity of the air and gravity. That is about as basic as physics gets.
Good God, did you say somewhere that you are a physicist?

JAE
May 26, 2011 1:24 pm

Perhaps another way to state the obvious is: Planet Earth stores just enough energy, as measured by an average temperature at the surface of 15 C, to support a column of air about 30 km deep.

Joel Shore
May 26, 2011 4:06 pm

Jae says:

OF COURSE, the radiation at 5 km is from what you are calling “greenhouse gases” (i.e. IR-active gases). So what??

So, the fact that the greenhouse gases raise the effective radiating level from 0 to 5km, together with the lapse rate in the troposphere, is what gives the greenhouse effect.
Do you learn in getting a PhD in chemistry how the equation for a line involves both a slope and a constant “intercept” term and how both are needed to determine y for a given x?

That is about as basic as physics gets.
Good God, did you say somewhere that you are a physicist?

Yes…which is apparently why I understand this stuff and you seem incapable of doing so (although most other chemists are intelligent enough to too). And, it is why all but a few crackpots or active deceivers in the physics community accept the greenhouse effect as reality. This includes even hard-core skeptics like Fred Singer and Will Happer.

Joel Shore
May 26, 2011 5:21 pm

Just a point of clarification on the last post: It is ALL of the IR-absorbing atmospheric constituents, i.e., greenhouse gases and clouds, that raise the effective radiating level from 0 to 5km.

wayne
May 26, 2011 7:30 pm

I just learned that even though the 390 Wm-2 shown on K&T graphics may be correct in a one-dimensional world, in an actual three-dimensional world, that is really only 130 Wm-2 radiating upward in the purest sense. The other 260 Wm-2 comprises of the horizontal two dimensions. When you integrate the three components across the hemisphere and account for random radiation in all directions from a flat surface like an ocean what you end up with is just that amount. In addition, due to horizontal homogeneity and hungry absorbers present in our atmosphere, all horizontal energy transfer cancels leaving the 130 Wm-2 upward. In this light the downward “back-radiation” would be 324-260 or 64 Wm-2 for the same reason (think cubic) leaving the NET upward to be 130-64 or 66 Wm-2.
The cancellation is the same thing that occurs between two opposing walls right there in the room you are sitting. Each wall has the power to transfer a great amount of energy if the other wall was 3 K, but that power to transfer never manifests due to the equal temperature and symmetry.
Of course, if the atmosphere could not absorb any radiation at all as a nitrogen atmosphere, the horizontal radiation would proceed right through the entire atmosphere unimpeded to exit the atmosphere in all directions. There would be no cancelling of transfer for there would be no absorption.
Finally, it is all falling in place and the now corrected numbers are actually making some sense.
We keep ignoring the other two dimensions in radiative transfer, nary a mention of them!
BTW: Does anyone know roughly how much energy can be radiated by 1.225 kg of air at 288 K as a cubic meter at sea level and at average relative humidity to empty space? If you have that, how much if much higher in altitude (like ~100 grams in the cubic meter as it would be near the 65-mbar level at 217 K, little humidity). What I’m getting at is it takes mass to radieate so how does the mass itself affect radiation rates.

Jim D
May 26, 2011 8:31 pm

JAE seems to agree that without greenhouse gases the surface would be -18 C, and with them it is +15 C. This, by definition, is the greenhouse effect, however you arrived at it. Postma, too.
wayne, your cube analogy is not correct. 390 W/m2 is the average outward component from a spherical surface. There is no sideways component to this number. It is equivalent to a surface radiative temperature of +15 C, which as you see from above is completely consistent with the greenhouse effect that JAE and Joel explain.

Phil.
May 27, 2011 6:27 am

Robert Stevenson says:
May 26, 2011 at 7:49 am
Phil says:
‘Where do you get your value of 0.2 emissivity from, ‘
Further to our recent posts concerning the above and Hottel; 0.2 (or 0.193) is the emissivity/absorptivity of CO2 in the atmosphere to the extinction point of 3600m for dry air.
Phil, what do your calculations give for the distance in which the first-generation photons from the Surface up into the atmosphere are absorbed to extinction in CO2′s case?

Yesterday’s post didn’t seem to make it so I’ll try again.
It depends on the wavelength, for the core of the 15 micron band less than 1000m but for the outer bands clearly more. The Q branch less than 100m.
http://i302.photobucket.com/albums/nn107/Sprintstar400/CO2spectra-1.gif
[As Phil demonstrates, it is best to always save a copy of your comments until they appear. WordPress is not always 100% reliable. On occasion I have also lost comments. ~dbs, mod.]

JAE
May 27, 2011 8:59 am

Well, Joel:
” And, it is why all but a few crackpots or active deceivers in the physics community accept the greenhouse effect as reality.”
IMHO, the jury’s still out regarding just who the crackpots are. You would have made a good pope back in Copernicus’ time.
But, to move beyond your insults, arm-waving, and fiats. There must be some other factors besides GHGs which determine the “effective radiating level,” like maybe convection. First, it is true that all radiation would have to come from the surface if there were no ghgs, but it is also likely that the atmosphere would be extremely hot, due to conduction/convection. The only way the gases could get rid of heat would be to conduct it back to the surface, but convection would try to prevent that. It would certainly be weird.
Second, there must be something else that affects the “effective radiating level” besides the types and quantities of ghgs. Maybe it’s density. As Alan Siddons has demonstrated in his paper, forget exact title, but something about moon ghe, almost all planets with atmospheres have an “effective radiating level” at about 0.1 bar, irregardless of the type(s) and amounts of ghgs present. The Earth’s level is lower, probably because of the unique properties of water.
Despite your convienient theory, you have no proof for your conjectures about the ghe. And the empirical evidence regarding increasing CO2 levels is not helping you any, either (no warming).
BTW, you didn’t respond to my reasoning for why the “positive feedback” part of the CAGW nonsense doesn’t make any sense.

JAE
May 27, 2011 9:02 am

Jim D:
“JAE seems to agree that without greenhouse gases the surface would be -18 C, and with them it is +15 C. This, by definition, is the greenhouse effect, however you arrived at it. Postma, too.”
Yes, but it’s still fun (for me, at least) to try to figure out just how it works. It’s unfortunate that I seem to be raising the blood pressure for certain folks, though. 🙂

May 27, 2011 9:13 am

The Hottel analysis uses an average “grey gas” emissivity. That is, it ignores the spectral lines and simply assumes that it absorbs (or emits) the same at all frequencies. As a result, it should not be used for more than 100 meters or so. Using the Hottel graph to try and find a distance where *all* the radiation is removed will give the wrong result because, even at large values, some of the radiation won’t be absorbed. This is because the radiation is completely absorbed at some frequencies, while CO2 remains mostly transparent at others.
To be clear, the Hottel average is across the entire blackbody emission for that temperature. As a result, trying to compute the power emitted (or absorbed) for a narrow band of frequencies will give wrong results.

May 27, 2011 9:28 am

Joel and others here is a paper on atmospheric radiative heat transfer. Please note his statements concerning CO2 on page 23.
http://www.scribd.com/doc/34962513/Elsasser1942

Phil.
May 27, 2011 9:31 am

JAE says:
May 27, 2011 at 8:59 am
But, to move beyond your insults, arm-waving, and fiats. There must be some other factors besides GHGs which determine the “effective radiating level,” like maybe convection.

No, it’s related to the opacity of the atmosphere at altitude and is a function of wavelength depending on the atmospheric composition.
First, it is true that all radiation would have to come from the surface if there were no ghgs, but it is also likely that the atmosphere would be extremely hot, due to conduction/convection.
No the surface would be cold, say -18ºC, and the incoming solar radiation would be balanced by reflection and IR emissions, the atmosphere would be colder. Heat transfer would be slow between surface and atmosphere (mainly conduction), any convection would give rise to a profile determined by the lapse rate (dry).
The only way the gases could get rid of heat would be to conduct it back to the surface, but convection would try to prevent that. It would certainly be weird.
Convection wouldn’t prevent conduction, the highest temperature would be next to the surface (lapse rate).
Second, there must be something else that affects the “effective radiating level” besides the types and quantities of ghgs. Maybe it’s density.
It is related to density, it’s the altitude where thermalization via collision starts to not be as effective a means for the excited molecules to lose energy compared with radiation, it varies with altitude depending on the gas.

May 27, 2011 9:35 am

“effective radiating level” is nonsense. The atmosphere is about -18C at 5km, 45.5km, 56.5km, and another higher up. Which of the 4 is closer to the surface? Which is closer to space?
At 49km, the atmosphere is about -2.5C. Get it, above the “effective” 5km level, the atmosphere is warmer?
This is one of the reasons that I started questioning the consensus. When the consensus claims that the “effective radiating level” is below 2 levels of atmosphere that are much warmer, then I think that a good explanation is required.

May 27, 2011 10:34 am

Phil. says:
No the surface would be cold, say -18ºC,
No, that is simply the average temperature. The daily fluctuation would be from hot enough to boil water to extremely cold.
the atmosphere would be colder. Heat transfer would be slow between surface and atmosphere (mainly conduction), any convection would give rise to a profile determined by the lapse rate (dry).
No – The atmosphere would be the same temperature as the peak surface temperature. I agree that the main method of heating would be convection until the temperature profile matched the dry adiabatic lapse rate. After that, you are right that slow conduction would take over. However, at some point, the atmosphere would have a single constant temperature, from top to bottom, equal to the daily peak value.

Robert Stevenson
May 27, 2011 1:49 pm

Joel says:
‘Good God, are you guys blind? How many times do we have to debunk Postma’s nonsense?
I haven’t studied Joseph Postma’s paper in detail (all 32 pages), but I noted that there is no mention of the LWIR radiant heat which is supposed to be absorbed by water vapour, CO2 etc. Most contributors accept that first generation photons from the surface are absorbed to extinction by water vapour in the first 120m of traverse through the atmosphere. This means that 63.3% (248Wm-2) of the 391Wm-2 emitted as LWIR radiant energy from the surface is absorbed by water vapour in 120m of traverse. On page 3 Postma states that anthropogenic global warming means a general warming of the atmosphere theorized to be human emission of carbon dioxide (CO2 ), which is then theorized to cause a strengthening of the effect of the Greenhouse Theory, which actually causes said warming. It should be noted however that CO2 absorbs very little heat in the first 120m of traverse and as has been mentioned by many contributors before is a very weak GHG. So, AGW or ‘any global warming’ as it should be known is therefore down to water vapour and governments could save millions of $ expenditure by reducing water vapour emissions instead of CO2 emissions by merely paying lip service (10$) to it (water vapour that is).

Robert Stevenson
May 27, 2011 2:38 pm

Robert Clemenzi says:
The Hottel analysis uses an average “grey gas” emissivity. That is, it ignores the spectral lines and simply assumes that it absorbs (or emits) the same at all frequencies. As a result, it should not be used for more than 100 meters or so.
Have you read Hottel and Sarofim, “Radiative Transfer,” Chap 11, McGraw Hill, New York 1967. Data is presented on CO2 band emissions for use in calclations.

May 27, 2011 5:00 pm

Robert Stevenson says:
Have you read Hottel and Sarofim, “Radiative Transfer,” Chap 11, McGraw Hill, New York 1967. Data is presented on CO2 band emissions for use in calculations.
No. I am using “Principles of Heat Transfer” by Krieth, 1967, which provides only 5 pages on Hottel’s methods. Since the years are the same, and based on your comment, I assume that I am missing a more advanced analysis. Unfortunately, the book is $110 to $160.

wayne
May 27, 2011 5:32 pm

mkelly, thanks for the link to Elsasser (1942), he explains much of what I have harping on (even my 3d examples) above in the first few pages. He has a good way to explain the full (or even partial equalibrium with residual transfer) radiative equalibrium that most of our atmosphere is in at all times. Thanks, his words will help me explain myself better.

jae
May 27, 2011 5:56 pm

Phil:
I’m having trouble putting these two statements together:
“No, it’s related to the opacity of the atmosphere at altitude and is a function of wavelength depending on the atmospheric composition.”
and
“It is related to density, it’s the altitude where thermalization via collision starts to not be as effective a means for the excited molecules to lose energy compared with radiation, it varies with altitude depending on the gas.”
Please help.
Also,
“No the surface would be cold, say -18ºC,”
Robert is on the right track, I think. The surface on most of the planet would warm during the day to at least 90 C (probably higher), which would warm the “no-ghg-atmosphere” by conduction/convection. I don’t know (and you probably don’t, either) how much of that energy would be lost at night, especially since there would be an inversion. If not all the energy was re-irradiated at night, the next day would add heat. An equilibrium would be reached, and it just might be very warm. I don’t know how to determine what it would be, and neither do you, I suspect.
Maybe Joel knows….

Jim D
May 27, 2011 7:03 pm

The effective level concept is a simplified way of looking at all the wavelengths together. In reality, each wavelength of IR emission has a different altitude which is up in the stratosphere for some where GHGs have strong lines, and down at the surface for others in the “window” region. 5 km just happens to be the weighted average over the emitted wavelengths.

May 28, 2011 7:02 am

How do scientists determine what the average temperature for the whole globe is anyway? – It seems to me to be pure guesswork, or maybe it is a “comparison-standard to work by”, i.e. one standard set by taking the temperatures at x numbers of stations y number of times each year.
I just took some quite random temperatures, – or rather – I found them on my favourite weather site Yr.no
Tomorrow the temperature on the North Pole is supposed to be minus (-) 2 °C, and on the South Pole – 61 °C. In Death Valley + 35 °C and in a place called Azebe (in the Gabon – very close to the Equator and at an elevation of 18 m.) it was to be +28 °C.
Now then if you take the average of those few temperatures you are not going to find yourself too far away from zero °C. However if I substitute The Death Valley + 35 °C with another one, this time from Quito in Ecuador which – just like Azebe is very close to the Equator but this time the elevation is 2108 meter (m). The Quito temperature however is to be 17 °C. So, why is an average temperature of – 18 °C unsuitable for us mammals?
And of course “My Average” is obviously not right but that could equally go for anybody else’s. – Pray tell me who has got a record of every temperature for every spot on Earth for a constant period of the past 150 years.
(Hands up those who have heard Azebe and Quito before)

Robert Stevenson
May 28, 2011 8:40 am

Robert Clemenzi:
Hottel’s paper with Mangelsdorf was entitled ‘Heat transmission by radiation from non-luminous gases. Experimental study of carbon dioxide and water vapour’. Emissivities were evaluated experimentally based on direct measurement of total emission for a number of gases including CO2 and water vapour; these gases absorbed in certain regions of the IR spectrum and this was the data which was used.

Robert Stevenson
May 29, 2011 5:37 am

Robert Clemenzi:
Robert Stevenson says: I use “Principles of Heat Transfer” by Krieth, 1958 (4th printing June 1961) which has 6 pages on Hottel’s methods. Perry’s Chemical Engineers’ Handbook (7th Edition 1997) has a section 5 ‘Heat and Mass Transfer’ written among others by Hoyt C Hottel and Adel F Sarofim. I don’t know much this costs but you could probably get it out of the library. You can also write to McGraw Hill on technical queries, in the past I have had some very helpful replies after spotting some errors in texts.

May 29, 2011 1:37 pm

It looks like even Joel Shore has given up n these pages now, which means life – for me – is getting a bit less interesting.
I, of course, – have no ill feelings or aggravated “Blood Pressure problems” just because I disagree with those who “believe” in what is called “The Greenhouse Theory”. –
After all the theory is good and convincing – which it should be – as it has been “honed and fitted” to suit all new climate evidence – as it emerges.
However as I am not “The Naturally Born Orator” I cannot convince you all that there is something wrong with any statements from the state of Denmark or from the “Copenhagen Treaty 2009”
Suffice to say that if “all objects that have a temperature higher than 0 Kelvin (K) must emit thermal radiation – (Basic Stuff)- . Then, – if it is basic stuff – why do Nitrogen, Oxygen and Argon not radiate anything at all towards the Earth’s surface?

May 30, 2011 12:39 am

Concerning Postma – http://www.globalwarmingskeptics.info/forums/attachment.php?aid=327
It should be pointed out immediately that the “Greenhouse Effect” is indeed a theory – it is not a benign empirical fact, such as the existence of the Sun, for example.
Baloney – By definition, the Greenhouse Effect is what makes the surface of the Earth different than the surface of the Moon. Immediately below that statement, he provides a *gravity* analogy – gravity is an observed fact, the theory is in how it works. I suggest that the Greenhouse Effect is identical, the difference between the Earth and the Moon is an observed fact – the theory is in how it works.
As a result, he makes the mistake of assuming that there is only one Greenhouse Effect theory. There are, in fact, many.
On page 6, he misses the point when describing blackbodies.
the blackbody is already in radiative thermal equilibrium with a hotter source of energy
That is simply wrong – That might be true if the blackbody was surrounded by a perfect insulator but is definitely not true for planets in space. In that case, the blackbody is in radiative thermal equilibrium with a hotter source of energy and with the cold microwave background radiation.
Next he suggests that adding a mirror can not make the blackbody hotter. Again, he misses the point that the mirror shields the blackbody from the cold of space. This is even more important when a planet is rotating.
You cannot make something warmer by introducing to it something colder
Absolutely true. However, the mirror is effectively warmer than what is available without the mirror.
Kirchhoff’s Law of Thermal Radiation … states: “At radiative thermal equilibrium, the emissivity of a body equals its absorptivity.”
Wrong – that statement is true for each individual frequency, but not for the body as a whole.
For Ira – He completely messes up the bit about lapse rates. As you said, he has the DALR correct, but the rest makes no sense. He obviously missed the fact that the troposphere is much warmer than predicted by the DALR.
Additionally, the direction of heat flow is always only from hot (the ground), to cold (upper atmosphere),
Everyone should look at the lapse rate over Antarctica – during the winter (night), the atmosphere is ALWAYS warmer than the surface. The only reason the surface stays so “warm” at night is because of back radiation from the warm atmosphere to the cold ground.
Infrared energy leaves the atmosphere in only a few milliseconds
If that were true, the atmosphere would drop significantly below freezing every night. Everywhere.
There are numerous additional errors in the last few pages .. such as the repeat of his bucket analogy (p29). I agree that adding a 5C bucket of water will not warm a 40C tub of water. However, to complete the analogy, he needs to compare the results to adding a -273C bucket of water. In that case, there is a real difference in the final average temperature of the two experiments.

Robert Stevenson
May 30, 2011 4:39 am

Ira says:
If anyone more knowledgeable than I (such as Joel or Dave or Tim, etc.) is still following this thread, now at over 930 comments which …
Didn’t think Joel was very knowledgeable, he hasn’t said anything sensible yet.

wayne
May 30, 2011 5:14 am

Robert Clemenzi seems to have a clear mind on the physical world. Here are some comments on Roberts analysis of Postma’s paper:
Greenhouse Effect – check! There are many theories.
A blackbody in equilibrium – check! Does not imply equilibrium at all, overall radiative equilibrium is but one possible state.
On the mirror – check, but, in the case with space on the backside of the mirror the planet side would be warmer of course, BUT, the space side would also be equally cooler, shielded from the warm planet. It’s called radiative insulation, like aluminum radiative barrier in homes. Both sides of this effect must always be accounted for.
You cannot make something warmer by introducing to it something colder – check!
Kirchhoff’s Law of Thermal Radiation – check on each frequency, but I would add to the statement: “but not for the body as a whole UNLESS ALL individual frequencies are also in radiative thermal equilibrium”. I’m sure that was just a slip.
Lapse rates – can’t compare yet – hasn’t jelled for me.
Infrared energy leaves the atmosphere in only a few milliseconds – check, I agree, no! Five-sixth by simple geometry is colloquially ‘trapped’ in either the case of conduction OR radiation if the mean free path is rather small in comparison to the scale of the entire system.
Follow Robert Ira. I concur with him point by point.

May 30, 2011 12:27 pm

Ira Glickstein, PhD said on Visualizing the “Greenhouse Effect” – Light and Heat
May 29, 2011 at 8:04 pm
“…. I would appreciate it if they would properly debunk Postma’s explanation, which seems to me to be “putting the cart before the horse”.”
Alleyne replies:
Good to know you don’t have any pre-conceived ideas about this Ira 🙂  I too would like to hear some rational, factual, scientific discussion about it, but I have no vested interest in the outcome.  This is not meant as a personnal attack, Ira, just an observation.  I expect that it will turn out that both CO2 and adiabatic heating will prove to be mexhanisms in the atmosphere’s temperature profile.  In systems as dynamic and complex as the atmosphere it rarely turns out that a single parameter accounts for everything.
Ira Glickstein, PhD said on Visualizing the “Greenhouse Effect” – Light and Heat
May 29, 2011 at 8:04 pm
Earlier in his paper, Postma (page 5) states: “If the source of light is constant, meaning it shines with the same unchanging brightness all the time, then the blackbody absorbing that light will warm up to some maximum temperature corresponding to the energy in the light, and then warm up no further. When this state is reached it is called ‘radiative thermal equilibrium’, which means that the object has reached a stable and constant temperature equilibrated with the amount of radiation it is absorbing from the source of light.” While true, this seems to ignore the reason for the maximum temperature, which is that the black body has been radiating , according to T^4, all along as its temperature had been rising, and its temperature levels off at the point that the incoming radiation equals the outgoing. In other words Postma “puts the cart before the horse” ignoring the direction of causality.
Alleyne replies:
I must disagree, though it may just be semantics getting in the way again, the reason for the maximum temperature is not that it has been radiating all along, but that it has reached radiative equilibrium.  His statement makes no comment one way or the other as to the radiative state of the BB before it reaches equilibrium.  Furthermore, perhaps I am misunderstanding, but the conclusion in your last sentence makes no sense to me.  I really don’t see how he ignores the direction of causality which surely is from the source to the BB.  The BB radiation doesn’t determine the energy level of the source, though it will determine the rate at which the source’s energy level will decrease.  The cause of the BB radiation is obviously the energy from the source mediated by the BB Absorbtivity and Emissivity (which at certain wavelengths will be equal once radiative equilibrium is reached)
Ira Glickstein, PhD said on Visualizing the “Greenhouse Effect” – Light and Heat
May 29, 2011 at 8:04 pm
“This is confirmed when he later says (also page 5): “When a blackbody has reached thermal equilibrium, it can no longer absorb more light for heating and therefore has to re-emit just as much light-energy as it is absorbing. Because the blackbody can‟t just reflect the light, it has to re-emit it as thermal radiation.”
Again, I don’t understand what your point is or how you see this as putting the cart before the horse.
Ira Glickstein, PhD said on Visualizing the “Greenhouse Effect” – Light and Heat
May 29, 2011 at 8:04 pm
“Later in his paper (pages 27 to 28) he raises total misconception of the role of CO2 and other so-called “greenhouse gases” in the Atmospheric “greenhouse effect”. He says (again correctly) that, if the vacuum of a thermos is replaced by CO2, the hot liquid in the thermos will cool faster. That, he thinks (incorrectly) proves that “CO2 does not ‘trap’ any radiative heat” in the Atmosphere. Of course, a Thermos bottle works by blocking both convection and conduction (by the vacuum) and radiation (by metalic coating), so the comparison is totally misleading.”
I agree that this is a very poor example to use, though it would be an interesting experiment to run.  Replacing the vaccuum with CO2 would no doubt reduce the conductive and convective insulation (ie increase conductive & convective heat loss) theoretically radiative heat loss should remain the same.  An interesting question is whether the heat loss would be less than if the vaccuum were replaced with Nitrogen.
Ira Glickstein, PhD said on Visualizing the “Greenhouse Effect” – Light and Heat
May 29, 2011 at 8:04 pm
“Still later (page 29) he gives the also misleading example that adding even a large quantity of 5ºC water (which, since it is way above the absolute zero of -273ºC contains a great deal of thermal energy) to a bathtub of water that is at 40ºC, will not raise its temperature. On the other hand, he notes that even a tiny bucket of water at a bit over 40ºC will raise the temperature of the tub, at least a little bit. Again correct, but absolutely no evidence against the ability of downwelling radiation from the Atmosphere to the Surface to cause the Surface to be warmer than it would be absent the “greenhouse gases”.”
Agreed.  But given that the presence of any gas will cause the surface to be warmer than it would otherwise be, all we are talking about is the slight amount of endogenous (if I can put it that way) warming from CO2, most of which has already occured.
The real question is whether climate feedbacks are positive or negative.  In other words will the small amount of warming cause the powerful GHGs (like water vapour) to increase the amount warmed or is the climate system stable such that there are feedback mechanisms which works to dampen and/maintain the climate in a ‘steady state’.  So far the evidence, as I understand it, seems to indicate that this is the case.
I would like to second your thanks to all those who have contributed positively to the discussion and specifically to Joel, Tim and Dave Springer who all helped me with my understanding of the way in which “back radiation” works.
I still have some reservations and thinking to do about that Trenberth diagram and the whole Postma thesis, but that is for another day.
As a person who spends quite a bit of time commenting online, this is my first experience with WordPress and I must say I find it very unwieldly to use. Firstly it is very difficult to search and find anything so even if one were inclined to try, it would be laborious and probably fruitless trying to find and read previous explanations. Secondly I find it awckward trying to reply online instead of from my inbox, which is more immeadiate and flexible – but then I guess that is just the way it is and no reflection on WUWT or the people here.

May 30, 2011 1:58 pm

Ira Glickstein and others – there are many things about this I frankly do not understand.
1) As I mentioned on May 29, 2011 at 1:37 pm: “If it is correct that all objects that have a temperature above Zero Kelvin (0K) must emit energy by radiation, then why do Nitrogen, Oxygen and Argon not radiate anything at all towards the Earth’s surface?” And also why do these other 3 gases not seek equilibrium with GHGs?
2) I have no problem understanding that energy radiation from a warmer source can warm a cooler object (one that receives and absorbs that energy). Nor have I any problem with the idea that a colder object is radiating its energy towards a warmer one, – but, of course, – any reduction in cooling of that warmer object can then only take place at the side facing the cooler object.
Therefore I understand perfectly well that radiation from the surface can warm the atmospheric GHGs that absorb it. – What I can not understand however is this: “How can it even be considered possible that energy that was removed from the surface, by radiation in the first place, thus causing the surface to cool can ‘warm the original source’, i.e. the surface – just because some gas or other sends it all back.” – It seems perfectly reasonable to me that if we imagine the surface never emits that energy in the first place,- energy that is stored in the surface and just below, i.e. oceans, lakes, rivers, ground, and air, – just to mention a few, then any surface temperature change would be completely reliant on variations in Solar irradiation and advection mainly by Water Vapor (WV) but also by other GHGs that have the ability to contain more heat than the rest of the atmospheric gases. –
To my way of thinking “atmospheric heat can only be trapped, or slowed down, by a reduction in convection. –
In other words the atmosphere’s energy radiation back to space – is delayed.
The two lapse rates should smack you in the face – DALR is fixed – The Moist Adiabatic Lapse (MALR) rate is not. It varies (a scale is used) depending on the atmosphere’s moisture content.
The much belowed CO2, by the way, has not got a lapse rate of its own. (The MALR (Moist Adiabatic Lapse Rate) is also called the wet or saturated adiabatic lapse rate. It is the temperature trajectory a parcel of saturated air takes. The dry adiabatic lapse rate is a near constant of 9.8 C/km, however, the wet adiabatic lapse rate is much less of a constant. The wet adiabatic lapse rate varies from about 4 C/km to nearly 9.8 C/km.)
But, having said all that, I would like an answer from those who believe that GHGs supply the surface with additional heat, to explain why I am wrong.
I am now begging all you AGW enthusiasts to explain where I do go wrong. Because if you are right – then – I am WRONG.

May 30, 2011 2:21 pm

Ira, having said all that, it does not matter much whether we agree or disagree on the correctness of the GHG theory.
However, I do not think it is impossible that if you agree with the theory that says CO2 rules our world’s well being,, then it may not be long before you admit that the melting of the North Pole will not liberate Siberian and other frozen landmasses but instead fry us all (on the top of the surface this time), unlike down below – as stokers of hell fires – as envisaged it just a few years ago

Phil.
May 30, 2011 9:27 pm

O H Dahlsveen says:
May 30, 2011 at 1:58 pm
Ira Glickstein and others – there are many things about this I frankly do not understand.
1) As I mentioned on May 29, 2011 at 1:37 pm: “If it is correct that all objects that have a temperature above Zero Kelvin (0K) must emit energy by radiation, then why do Nitrogen, Oxygen and Argon not radiate anything at all towards the Earth’s surface?” And also why do these other 3 gases not seek equilibrium with GHGs?

Your premise isn’t true, gases are not ‘objects’ and N2, O2 & Ar don’t emit in the IR due to the lack of a dipole.

Robert Stevenson
May 31, 2011 2:07 am

Ira Glickstein says:
‘Joel’s mistaken belief in the dangers of modest increases in mean temperatures and CO2 levels. He, along with others whose opinions are closer to mine, have taught me much on this Blog and I thank them, one and ALL.’
This absurd fixation with CO2 as the sole driver of global warming is my main reason for blogging; even the BBC in their bulletins now speak of too much carbon in the atmosphere hoping that listeners will not realise the link with CO2 (the considerable benefits to plant growth are well documented).
The principal emission bands of CO2 and water vapour are as follows:
CO2 2.64 to 2.84 4.13 to 4.5 13 to 17 microns
Water vapour 2.55 to 2.84 5.6 to 7.6 12 to 25 microns
Because of its high concentration, water vapour absorbs to extinction first generation photons (hv) in the main waveband in 120m of traverse (as many contributors have pointed out) and CO2 absorbs very little, due to its small concentration.
This means that 63.3% (248Wm-2) of the 391Wm-2 emitted as LWIR radiant energy from the surface is absorbed by water vapour in 120m of traverse and the remainder 140Wm-2 leaves the atmosphere as it is transparent to it. Increases in CO2 have very little effect in this water vapour dominated environment as it is a very weak absorber.
Opposition to CO2AGW does not mean we do not care about the environment, we do and hate to see the rural landscape despoiled by ugly noisy wind farms for instance. These installations cost enormous sums but generate only a fraction of their installed capacity.

Robert Stevenson
May 31, 2011 2:20 am

Ira,
Further to previous post – in the table I wanted to high light the spectral overlap between CO2 and water vapour in the main band thus:
Carbon dioxide – 13 to 17 microns
Water vapour – 12 to 25 microns
but it didn’t come out too well

Phil.
May 31, 2011 5:36 am

Robert Stevenson says:
May 31, 2011 at 2:20 am
Ira,
Further to previous post – in the table I wanted to high light the spectral overlap between CO2 and water vapour in the main band thus:
Carbon dioxide – 13 to 17 microns
Water vapour – 12 to 25 microns
but it didn’t come out too well

Yeah, not particularly accurate either, a few weak spread out lines hardly constitutes ‘overlap’!
Your calculation of H2O absorption is way too high.
http://i302.photobucket.com/albums/nn107/Sprintstar400/H2OCO2.gif

May 31, 2011 8:00 am

Robert Stevenson said on Visualizing the “Greenhouse Effect” – Light and Heat
May 31, 2011 at 2:07 am
“Opposition to CO2AGW does not mean we do not care about the environment, we do and hate to see the rural landscape despoiled by ugly noisy wind farms for instance. These installations cost enormous sums but generate only a fraction of their installed capacity.”
Alleyne replies:
Not to get off topic, but I whole heartedly agree with you Robert.  The damage to endangered species, habitat and biodiversity done in the name of ‘saving the planet’ by proponents of ‘Greed’ Energy is tragic.
They plan on installing almost 800 IWT in this area I live in which is in large part unspoiled habitat for Peregrines, Golden Eagles, Bald Eagles, Sand Hill Cranes as well as various species of endangered and threatened plants and reptiles.  Some of these are recognized as species at risk and supposedly protected by any number of Provincial, Federal and International laws.  Yet the Ministry is turning a blind eye in the name of so-called green energy (only thing green about it are the profits being made)

May 31, 2011 9:22 am

Phil, your spectracalc graphs are meaningless because they lack the
pressure, molar concentration, and path length
Graphs of a typical atmosphere show a very different picture. For one thing, the CO2 fine structure disappears because more than 95% (3-sigma) of the photons from 650 to 690 cm-1 are absorbed in the first 80 meters. Based on what I see, I feel safe assuming that the spectracalc CO2 path length was about 1 meter. It also appears that the water vapor concentration was about 500 ppm, where the actual value in the lower troposphere is about 10,000 ppm (yes, ten thousand ppm, aka 1% or 62% RH at 15C/59F).

Joel Shore
May 31, 2011 11:19 am

Robert Stevenson says:

This means that 63.3% (248Wm-2) of the 391Wm-2 emitted as LWIR radiant energy from the surface is absorbed by water vapour in 120m of traverse and the remainder 140Wm-2 leaves the atmosphere as it is transparent to it. Increases in CO2 have very little effect in this water vapour dominated environment as it is a very weak absorber.

The reason why the radiative forcing of ~4 W/m^2 due to doubling CO2 is-what-it-is is well-understood: : http://www.realclimate.org/index.php/archives/2007/06/a-saturated-gassy-argument/ and is again something that is not disputed by any serious scientist (i.e., even Spencer and Lindzen accept it).

Joel Shore
May 31, 2011 11:47 am

O H Dahlsveen says:

1) As I mentioned on May 29, 2011 at 1:37 pm: “If it is correct that all objects that have a temperature above Zero Kelvin (0K) must emit energy by radiation, then why do Nitrogen, Oxygen and Argon not radiate anything at all towards the Earth’s surface?”

Solids and liquids have essentially continuous emission and absorption spectra. However, gases are different: An individual molecule of nitrogen or oxygen is non-polar and even its modes of oscillation are non-polar. As a result, oscillation and rotation modes of the molecule do not “couple” with electromagnetic radiation, which means the molecule can not emit or absorb radiation over these (IR) wavelengths. The only way that O_2 and N_2 can couple at all with IR radiation is through molecular collisions and, while these can become important at high pressures, at atmospheric pressures, any such absorption or emission of IR radiation through such processes is essentially negligible.

It seems perfectly reasonable to me that if we imagine the surface never emits that energy in the first place,- energy that is stored in the surface and just below, i.e. oceans, lakes, rivers, ground, and air, – just to mention a few, then any surface temperature change would be completely reliant on variations in Solar irradiation and advection mainly by Water Vapor (WV) but also by other GHGs that have the ability to contain more heat than the rest of the atmospheric gases. –
To my way of thinking “atmospheric heat can only be trapped, or slowed down, by a reduction in convection. –
In other words the atmosphere’s energy radiation back to space – is delayed.

I think you are getting confused about what the comparison case is: Yes, the surface of the earth is not warmer than it would be if it never emitted radiation in the first place. However, if that were the case, it would be absorbing radiation from the sun and not emitting radiation and its temperature would be constantly rising. In fact, as Ray Pierrehumbert has pointed out, under such a counterfactual situation, the amount of energy that the earth receives from the sun over the time of a million years is enough to raise the temperature of the ENTIRE EARTH (fr0m core to skin) by ~800 K!
Your statement about a delay is true but is simply not the most useful way to look at it because it doesn’t really lead to intuition about what happens as a result. The best way to look at it is to consider the fact that the rate at which the earth emits radiation is reduced and that the earth will then increase its temperature until it reaches the point where the amount of radiation it emits back out into space matches the amount it receives from the sun.
To make an analogy: It would be wrong to argue that a beaver dam can’t produce a pond because all the dam does is delay slightly the flow of the water down the stream. The fact is that such a dam causes a pond to form until such a point that the additional water flow produced by the pressure from the pond causes the amount of water flowing out to once again balance the amount coming in.

But, having said all that, I would like an answer from those who believe that GHGs supply the surface with additional heat, to explain why I am wrong.

I don’t understand what you are even trying to say in regards to the lapse rate. To the first approximation, additional CO2 does not change the lapse rate. What it does do, however, is raise the effective radiating level in the atmosphere…i.e., the level at which the temperature has to average about 255 K. As a result, if you use the lapse rate to extrapolate the temperature to the surface, you find that the surface temperature is now higher.
(In the second approximation, additional CO2 does actually change the lapse rate…because of the fact that the temperature profile in the tropics follows the moist adiabatic lapse rate. This actually turns out to be a negative feedback…i.e., the lapse rate is reduced slightly so the temperature at the surface does not have to rise quite as much as would be predicted by just considering the change in the effective radiating level and a fixed lapse rate.)

Joel Shore
May 31, 2011 12:01 pm

mkelly says:

Joel and others here is a paper on atmospheric radiative heat transfer. Please note his statements concerning CO2 on page 23.
http://www.scribd.com/doc/34962513/Elsasser1942

I am not sure exactly what we are supposed to take from this 1942 book. Note that this book was written before the 1950s, which is when the importance of the wings of the absorption band and of where in the atmosphere the absorption is significant was first widely understood. Again, see http://www.realclimate.org/index.php/archives/2007/06/a-saturated-gassy-argument/ See also the second, more technical part of the discussion: http://www.realclimate.org/index.php/archives/2007/06/a-saturated-gassy-argument-part-ii/ The physics of radiative transfer in the atmosphere has advanced over the last 70 years.

May 31, 2011 2:51 pm

Joel, I enjoyed reading
http://www.realclimate.org/index.php/archives/2007/06/a-saturated-gassy-argument-part-ii/
It is almost correct.
First problem – it focuses on the energy going thru the atmosphere. However, that is totally irrelevant. The focus should be on the energy returning to the surface.
Second problem – it assumes that adding CO2 will increase the temperature of the atmosphere. If radiation was the only source of energy, then that would be a valid assumption. However, because of conduction/convection and the latent heat of vaporization (to evaporate water), the paper’s assumption is wrong. In fact, adding CO2 (or any other IR active gas) to the lower atmosphere will cause it to cool. Those few gases that do cause the atmosphere to get hotter (such as ozone in the stratosphere) also cause the surface to be cooler.
Third problem – the paper focuses only on the wings of a single gas spectrum. When a second gas is added, the analysis changes. If the CO2 only eats red MM’s while water vapor eats all colors, once the total number of MM’s is gone, adding more of either gas will have no effect. Once it is realized that greenhouse gases cool the atmosphere, then it becomes obvious that the atmosphere is already emission saturated, that the “wing” argument does not apply, and that the system is (nearly) stable.
Fourth – The paper does not indicated that the CO2/water vapor overlap reduces the effect by about half. Adding clouds reduces it by another half. Thus, the net effect of increasing CO2 would be about one fourth the amount suggested in the paper .. except that the emissive saturation reduces it to almost zero.

Joel Shore
May 31, 2011 4:39 pm

Robert Clemenzi says:

Joel, I enjoyed reading
http://www.realclimate.org/index.php/archives/2007/06/a-saturated-gassy-argument-part-ii/
It is almost correct.

Or, perhaps, the scientists who have studied this much more than you ever have are in fact basically completely correct and it is you who suffers from misconceptions. It continues to amaze me how many people on this website seem to be so much more knowledgeable than all the experts in the field!

First problem – it focuses on the energy going thru the atmosphere. However, that is totally irrelevant. The focus should be on the energy returning to the surface.
Second problem – it assumes that adding CO2 will increase the temperature of the atmosphere. If radiation was the only source of energy, then that would be a valid assumption. However, because of conduction/convection and the latent heat of vaporization (to evaporate water), the paper’s assumption is wrong. In fact, adding CO2 (or any other IR active gas) to the lower atmosphere will cause it to cool.

Oh boy…What you fail to understand is that your first problem and second problem are in fact problems with your view of things and that the two taken together represent the solution to the problem of each viewed in isolation. There is a good reason why scientists have focused on the energy that is emitted back to space (i.e., the energy budget at the top of the atmosphere) and that is precisely the issue that you bring up: the energy budget at the surface is strongly influenced by convection and latent heat transfer. Hence, the easiest way to figure out what is going to happen to the surface temperature is actually to look at the energy budget at the top of the atmosphere (where we know the only energy transfers are via radiation) and then to incorporate convection and latent heat transfer essentially through understanding the constraints that they set on the lapse rate.
As for adding CO2 causing the atmosphere to cool: this is only true if the amount of energy that is absorbed from the surface by this CO2 is less than the amount that is emitted into space, which is not going to be the case. (A more complicated version of your argument is used to explain why the addition of greenhouse gases does lead to the prediction that the stratosphere will cool.)

Third problem – the paper focuses only on the wings of a single gas spectrum. When a second gas is added, the analysis changes. If the CO2 only eats red MM’s while water vapor eats all colors, once the total number of MM’s is gone, adding more of either gas will have no effect. Once it is realized that greenhouse gases cool the atmosphere, then it becomes obvious that the atmosphere is already emission saturated, that the “wing” argument does not apply, and that the system is (nearly) stable.

Water vapor and CO2’s absorption lines overlap less than you think, especially once those lines are well-resolved. And, as is explained, as you go up in the troposphere water vapor concentration rapidly decreases and so CO2 becomes more important. What you don’t realize is that behind the description that is given are actual calculations. You can criticize the description for explaining things in an over-simplified way; however, ultimately what it comes down to is who is solving the equations correctly. Can you tell me what is wrong with the standard solution of the equations for radiative transfer in the atmosphere and how you know this and where you are planning to publish it? Otherwise, you are just talking nonsense.

Fourth – The paper does not indicated that the CO2/water vapor overlap reduces the effect by about half. Adding clouds reduces it by another half. Thus, the net effect of increasing CO2 would be about one fourth the amount suggested in the paper .. except that the emissive saturation reduces it to almost zero.

Nonsense. The radiative forcing for additional CO2 is already calculated taking into account the effects of which you speak. Your 1/4 number might be correct about the amount that the effect is reduced relative to a hypothetical calculation that assumed that CO2 was the only IR-absorber in the atmosphere. But, such a calculation is not what is actually made to determine the radiative forcing due to additional CO2.

jae
May 31, 2011 7:42 pm

[snip – multiple violations of policy – rephrase and resubmit – Anthony]

jae
May 31, 2011 8:21 pm

That prior rant should read, of course, that Mr. Shore is NOT restrained at all. He seems to me to be the perfect example of a raving Ehrlich-style “progressive” idiot. But, thank God, such folk are losing this battle, and they know it. And that makes them very angry and prone to insults. Losing folks resort to insults, every time!

Phil.
May 31, 2011 8:52 pm

Robert Clemenzi says:
May 31, 2011 at 9:22 am
Phil, your spectracalc graphs are meaningless because they lack the
pressure, molar concentration, and path length
Graphs of a typical atmosphere show a very different picture. For one thing, the CO2 fine structure disappears because more than 95% (3-sigma) of the photons from 650 to 690 cm-1 are absorbed in the first 80 meters. Based on what I see, I feel safe assuming that the spectracalc CO2 path length was about 1 meter.

Your assumption is wrong, the pathlength was 10m. The fine structure is still clearly visible at 80m, and less than 95% of the photons between 650 and 690 cm-1 are absorbed in 80m.
It also appears that the water vapor concentration was about 500 ppm, where the actual value in the lower troposphere is about 10,000 ppm (yes, ten thousand ppm, aka 1% or 62% RH at 15C/59F).
No, the H2O concentration was 1% over the same path length of 10m.

June 1, 2011 12:19 am

Phil, thanks for the response. It appears that my program at 3 meters produces the same graphs as spectracalc at 10 meters. Most likely my error. That would also make the 95% absorbance around 250 meters (not 80m). Perhaps you could add plots for 250m and 1,000m.

jae
June 1, 2011 7:57 pm

Hmmmm.
“snip – multiple violations of policy – rephrase and resubmit – Anthony]”
Well, it is YOUR blog, but I fail to see any “multiple violations” (you did not elaborate, friend, and you really need to do so, to be fair!). And I don’t see why the word “piss” is any worse than your calling BULLSHIT on an article today.
I will repeat, hopefully without bullshit censorship, that Ira is being a biased “host” when he posts the following obvious baloney:
” Joel has been remarkably restrained in the face of IMHO unjustified personal attacks and postings by commenters who have not read his previous explanations and simply -thoughtlessly- repeat their Disbeliever mantra. I congratulate Joel for not hectoring us here on his (again IMHO) mistaken belief in the dangers of modest increases in mean temperatures and CO2 levels. He, along with others whose opinions are closer to mine, have taught me much on this Blog and I thank them, one and ALL.”
Mr. Shore is ANYTHING than a “restrained” person. He has insulted me many times here, just because I call his undocumented CRAP! And Ira-idealogue-PhD, you can repeat the “disbeliever” crap as much as you want, but it adds not one thing to the debate. It just shows that you have made your mind up, which is ALREADY TOTALLY UNSCIENTIFIC!

Robert Stevenson
June 2, 2011 4:00 am

Simplifying the argument and considering CO2 alone in dry air, the first generation photons absorbed to extinction in a distance Lm of traverse of LWIR radiation are the ones ultimately responsible for any warming as they subsequently rattle around the atmosphere; the remainder shoot off to space, untouched, at the speed of light, as the transmittance of air for the remainder is 1.
A doubling of CO2 would not absorb any more (extra) first generation photons but the distance to the extinction point would reduce to Lm/2.
In an atmosphere well mixed by natural convection the subsequent mean retention time of these photons rattling around the atmosphere may be slightly increased (considering a Poisson distribution for a well mixed system this increase is probable). However as no additional heat or energy is absorbed (only that from the first generation photons) any temperature change would be very marginal.
The same argument applies to water vapour but with a larger absorption number for first generation photons (say about 200W/m^2 absorbed in a shorter extinction distance and about the same in the remainder shooting off to space; compared with about 80W/m^2 absorbed for CO2 etc).
I have carried out complex analyses in heat transfer equipment designs in the past involving series of thermal resistances and associated temperature drops – starting with changes of phase, boiling heat transfer to liquid metals, conduction through silicon carbide walls, convection and radiation from non-luminous gases and radiation from refractory walls all on a single spreadsheet. In all cases the subsequent energy audit balanced, predicted operating costs were reliable and correct (also capital costs) in the finished process plant. I didn’t whinge subsequently about any ‘lost heat’ from the balance as Trembarth does; what sort of analysis is he carrying out when he can’t even get a simple balance right.

Joel Shore
June 2, 2011 6:41 am

Robert Stevenson says:

Simplifying the argument and considering CO2 alone in dry air, the first generation photons absorbed to extinction in a distance Lm of traverse of LWIR radiation are the ones ultimately responsible for any warming as they subsequently rattle around the atmosphere; the remainder shoot off to space, untouched, at the speed of light, as the transmittance of air for the remainder is 1.
A doubling of CO2 would not absorb any more (extra) first generation photons but the distance to the extinction point would reduce to Lm/2.

Again, your thinking on the issue is about where most scientists were in the mid-20th century. To see how and why they got beyond that, read here http://www.realclimate.org/index.php/archives/2007/06/a-saturated-gassy-argument/ and here http://www.realclimate.org/index.php/archives/2007/06/a-saturated-gassy-argument-part-ii/
The basic points, in a nutshell, is that one has to go beyond the first absorption and subsequent emission to consider the full problem of multiple absorptions and emissions and one has to consider the “wings” of the absorption band rather than it just being a binary problem of absorption or no absorption at a given wavelength.

I didn’t whinge subsequently about any ‘lost heat’ from the balance as Trembarth does; what sort of analysis is he carrying out when he can’t even get a simple balance right.

As I understand it, Trenberth is complaining about the fact that, with the available satellites, we don’t currently have the capability to measure things down to the better than 1% level that would be necessary to see the fraction-of-a-W/m^2 changes in various quantities.

Robert Stevenson
June 12, 2011 6:25 am

Joel Shore says:
‘Robert Stevenson says:
Simplifying the argument and considering CO2 alone in dry air, the first generation photons absorbed to extinction in a distance Lm of traverse of LWIR radiation are the ones ultimately responsible for any warming as they subsequently rattle around the atmosphere; the remainder shoot off to space, untouched, at the speed of light, as the transmittance of air for the remainder is 1.
A doubling of CO2 would not absorb any more (extra) first generation photons but the distance to the extinction point would reduce to Lm/2.’
Again, your thinking on the issue is about where most scientists were in the mid-20th century. To see how and why they got beyond that, read here:
In 1900, shortly after Svante Arrhenius published his pathbreaking argument that our use of fossil fuels will eventually warm the planet, another scientist, Knut Ångström, asked an assistant, Herr J. Koch, to do a simple experiment. He sent infrared radiation through a tube filled with carbon dioxide, containing somewhat less gas in total then would be found in a column of air reaching to the top of the atmosphere. That’s not much, since the concentration in air is only a few hundred parts per million. Herr Koch did his experiments in a 30cm long tube, though 250cm would have been closer to the right length to use to represent the amount of CO2 in the atmosphere. Herr Koch reported that when he cut the amount of gas in the tube by one-third, the amount of radiation that got through scarcely changed. The American meteorological community was alerted to Ångström’s result in a commentary appearing in the June, 1901 issue of Monthly Weather Review, which used the result to caution “geologists” against adhering to Arrhenius’ wild ideas.
Still more persuasive to scientists of the day was the fact that water vapor, which is far more abundant in the air than carbon dioxide, also intercepts infrared radiation. In the infrared spectrum, the main bands where each gas blocked radiation overlapped one another. How could adding CO2 affect radiation in bands of the spectrum that H2O (not to mention CO2 itself) already made opaque? As these ideas spread, even scientists who had been enthusiastic about Arrhenius’s work decided it was in error. Work on the question stagnated. If there was ever an “establishment” view about the greenhouse effect, it was confidence that the CO2 emitted by humans could not affect anything so grand as the Earth’s climate.
I had heard of Knut Ångström, but hadn’t read of his experiments which match my spreadsheet analysis. The idea of a layered atmosphere as though it were a laminar boundary layer extension absorbing and re-emitting LWIR isn’t at all convincing but Knut Ångström’s work certainly is.